Doctrines - Consti

You might also like

Download as docx, pdf, or txt
Download as docx, pdf, or txt
You are on page 1of 201

POLITICAL LAW

ARTICLE I NATIONAL TERRITORY

SECTION 1

Magallona v. Ermita LB: RA 9522 is a statutory tool to demarcate the country’s maritime zones and
continental shelf under UNCLOS III, not to delineate Philippine territory.

UNCLOS III has nothing to do with the acquisition (or loss) of territory. It is a multilateral treaty
regulating, among others, sea-use rights over maritime zones (i.e., the territorial waters [12
nautical miles from the baselines], contiguous zone [24 nautical miles from the baselines],
exclusive economic zone [200 nautical miles from the baselines]), and continental shelves that
UNCLOS III delimits. Baselines laws are nothing but statutory mechanisms for UNCLOS III States
parties to delimit with precision the extent of their maritime zones and continental shelves.
UNCLOS III and its ancillary baselines laws play no role in the acquisition, enlargement or, as
petitioners claim, diminution of territory. Under traditional international law typology, States
acquire (or conversely, lose) territory through occupation, accretion, cession and prescription, not
by executing multilateral treaties on the regulations of use of sea rights or enacting statutes to
comply with the treaty’s terms to delimit maritime zones and continental shelves.

RA 9522 use of the framework of regime of islands to determine the maritime zones of
the kig and the scarborough shoal, not inconsistent with the Philippine’s claim of
sovereignty over these areas.

Far from surrendering the Philippines claim over the KIG and the Scarborough Shoal, Congress
decision to classify the KIG and the Scarborough Shoal as Regimes of Islands under the Republic
of the Philippines consistent with Article 121 of UNCLOS III manifests the Philippine States
responsible observance of its pacta sunt servanda obligation under UNCLOS III. Under Article 121
of UNCLOS III, any naturally formed area of land, surrounded by water, which is above water at
high tide, such as portions of the KIG, qualifies under the category of regime of islands, whose
islands generate their own applicable maritime zones.

UNCLOS III AND RA 9522 not incompatible with the constitution’s delineation of
internal waters.

Whether referred to as Philippine internal waters under Article I of the Constitution or as


archipelagic waters under UNCLOS III (Article 49 [1]), the Philippines exercises sovereignty
over the body of water lying landward of the baselines, including the airspace over it
and the submarine areas underneath. UNCLOS III affirms this.

ARTICLE II DECLARATION OF PRINCIPLES AND STATE POLICIES

SECTION 1

A. Sovereignty

People v. Gozo LB: Principle of Auto-Limitation

Any state may, by its consent, express or implied, submit to a restriction of its sovereign rights.
The concept of sovereignty as autolimitation is the property of a state-force due to which it has
the exclusive capacity of legal self-determination and self-restriction. A state, then, if it chooses
to, may refrain from the exercise of what otherwise is illimitable competence.

AP: By the agreement, the Philippine Government merely consents that the US exercise
jurisdiction in certain cases. This consent was given purely as a matter of comity, courtesy and
expediency. The Philippine Government has not abdicated its sovereignty over the bases as part
of the Philippine territory or divested itself completely of jurisdiction over offenses committed
therein. Under the terms of treaty, the US Government has prior or preferential but not exclusive
jurisdiction of such offenses. The Philippine Government retains not only jurisdictional rights not
granted, but also such ceded rights as the US Military authorities for reasons of their own decline
to make use of.

ACCFA v. CUGCO LB: Government functions: constituent and ministrant

Governmental functions are classified into constituent and ministrant. The former are those which

1
constitute the very bonds of society and are compulsory in nature; the latter are those that are
undertaken only by way of advancing general interests of society, and are merely optional.

The following are CONSTITUENT FUNCTIONS:


1. The keeping of order and providing for the protection of persons and properties from
violence and robbery;
2. The fixing of the legal relations between man and wife and between parents and
children; and
3. The regulation of the holding, transmission, and interchange of property, and the
determination of its liabilities for debt or for crime;
4. The determination of contractual rights between individuals;
5. The definition and punishment of crime;
6. The administration of justice in civil cases;
7. The determination of the political duties, privileges, and relations of citizens;
8. Dealings of the state with foreign powers – the preservation of state from external
danger or encroachment and the advancement of its international interests.

The most important of the MINISTRANT FUNCTIONS are public works, public education, public
charity, health and safety regulations, and regulations of trade and industry. The principles
determining whether or not a government shall exercise certain of these
optional functions are that:
1. a government should do for the public welfare those things which private capital would
not naturally undertake; and
2. a government should do these things which by its nature is better equipped to
administer for the public welfare than is any private individuals or group of individuals.

AP: ACCFA performs ministrant functions.

The ACA is a government office engaged in governmental, not proprietary functions. There can be
no dispute as to the fact that the land reform program contemplated in the Land Reform Code is
beyond the capabilities of any private enterprise to translate into reality. It is a purely
governmental function, no less than, say, the establishment and maintenance of public schools
and public hospitals. And when, aside from the governmental objectives, of the ACA, geared as
they are to the implementation of the land reform program of the State, the law itself declares
that the ACA is a government office, with the formulation of policies, plans and programs vested
no longer in a Board of Governors, as in the case of the ACCFA, but in the National Land Reform
Council, itself a government instrumentality; and that its personnel are subject to Civil Service
Laws and to rules of standardization with respect to positions and salaries, any vestige of doubt
as to the governmental character of its functions disappears.

B. Government and de facto

Co Kim Cham v. Valdez LB: Kinds of de facto governments

There are several kinds of de facto governments.


1. Government that gets possession and control of, or usurps, by force or by the voice of
the majority, the rightful legal government and maintains itself against the will of the
latter;
2. That which established and maintained by military forces who invade and occupy a
territory of the enemy in the course of war, and which is denominated a government of
paramount force; and
3. That established as an independent government by the inhabitants of a country who rise
in insurrection against the parent state.

Second kind of de facto government

The distinguishing characteristics of the second kind of de facto government, more


aptly denominated a government of paramount force, are:
(1) that its existence is maintained by active military power within the territories,
and against the rightful authority of an established and lawful government; and
(2) that while it exists it must necessarily be obeyed in civil matters by private
citizens who, by acts of obedience rendered in submission to such force, do not become
responsible, as wrongdoers, for those acts, though not warranted by the laws of the
rightful government.
Actual governments of this sort are established over districts differing greatly in extent and
conditions. They are usually administered by military authority, but they may be administered,
also, by civil authority, supported more or less directly by military force.

AP: Here, the PEC (Philippine Executive Commission) which was organized by the Commander of
Japanese Forces, was a civil government established by the military forces of occupation and
therefore a de facto government of the second kind.

LB: Principle of Postliminy

Under the principle of postliminy, which provides "a state or other governmental entity, upon the
removal of a foreign military force, resumes its old place with its right and duties
substantially unimpaired. . . . Such political resurrection is the result of a law analogous to
that which enables elastic bodies to regain their original shape upon removal of the external
force, — and subject to the same exception in case of absolute crushing of the whole fibre and
content."
2
AP: Here, the present courts are as the same courts which had been functioning during the
Japanese regime and, therefore, can continue the proceedings in cases pending therein prior to
the restoration of the Commonwealth of the Philippines, is confirmed by EO 37. Thus, the present
courts have jurisdiction to continue, to final judgment, the proceedings in cases, not of political
complexion, pending therein at the time of the restoration of the Commonwealth Government.

In re Letter of LB: Revolution; Revolutionary Government – Definition


Associate Justice Puno
A revolution has been defined as "the complete overthrow of the established government
in any country or state by those who were previously subject to it" or as "a sudden,
radical and fundamental change in the government or political system, usually effected with
violence or at least some acts of violence.”

AP: The Aquino administration was a de jure government, established by authority of the
legitimate sovereign, the people, at the time of the issuance of EO 33 . Then, she was still
exercising revolutionary powers of a government, which encompasses both executive and
legislative powers. Thus, she may create a new court and ranking of justices for reasons
only known to her and such is a valid exercise of her revolutionary powers.

Republic vs. LB/AP: Revolutionary Government/ De Jure Government


Sandiganbayan
The EDSA Revolution was done in defiance of the provisions of the 1973 Constitution. The
resulting government was indisputably a revolutionary government bound by no constitution or
legal limitations except treaty obligations that the revolutionary government, as the de jure
government in the Philippines, assumed under international law.

The protection accorded to individuals under the ICCPR and the UDHR remained in
effect during the interregnum.

The Bill of Rights under 1973 Constitution was not operative during the interregnum. However,
the protection accorded to individuals under the ICCPR and the UDHR remained in effect during
the interregnum. During that time, the directives and orders of the revolutionary government
were the supreme law because no constitution limited the extent and scope of such directives and
orders. With the abrogation of 1973 Constitution by the successful revolution, there was no
municipal law higher than the directives and orders of the revolutionary government. Thus,
during the interregnum, a person could not invoke any exclusionary right under Bill of Rights
because there was neither a constitution nor a Bill of Rights during that time.

As the de jure government, the revolutionary government could not escape


responsibility for the State’s good faith compliance with its treaty obligations under
international law.

The revolutionary government, after installing itself as the de jure government, assumed
responsibility for the State’s good faith compliance with the Covenant to which the Philippines is a
signatory no one shall be subjected to arbitrary or unlawful interference with his privacy, family,
home or correspondence; no one shall be arbitrarily deprived of his property. Thus, warrantless
seizure is still void even if at that time, there was no Bill of Rights.

SECTION 12 RIGHT TO LIFE OF THE UNBORN

Imbong v. Ochoa LB/AP: The moment of conception begins when the ovum is fertilized by the sperm.

[!] Contraceptives that kill or destroy the fertilized ovum should be deemed an abortive
and thus prohibited.

Conversely, contraceptives that actually prevent the union of the sperm and the ovum,
and those that similarly take action prior to fertilization should be deemed nonabortive
and thus constitutionally permissible.

Note:

NOT ABORTIFACIENTS:
1. Tubal litigation;
2. Vasectomy.

ABORTIFACIENTS:
1. Any drug or device that induces abortion (one which kills or destroys the fertilized
ovum);
2. One that prevents the fertilized ovum to reach and be implanted in the mother’s womb.

SECTION 16 RIGHT TO A BALANCED AND HEALTHFUL ECOLOGY

Oposa v. Factoran, Jr. LB: While the right to a balanced and healthful ecology is to be found under the
Declaration of Principles and State Policies and not under the Bill of Rights, it does not
follow that it is less important than any of the civil and political rights enumerated in
the latter.
3
Such a right belongs to a different category of rights altogether for it concerns nothing less than
self-preservation and self-perpetuation — aptly and fittingly stressed by the petitioners — the
advancement of which may even be said to predate all governments and constitutions.

AP: All licenses (timber license agreements in this case), is only a privilege, may thus be revoked
or rescinded by executive action. It is not a contract, property or a property right protected by
due process clause. Thus, non-impairment clause cannot be invoked.

Concerned Residents LB: The right to a balance and healthful ecology need not even be written in the
of Constitution for it is assumed, like other civil and political rights guaranteed by the Bill
Manila Bay v. of Rights, to exist from the inception of mankind and it is an issue of transcendental
MMDA importance with intergenerational implications. Even assuming the absence of a categorical
legal provision specifically prodding petitioners to clean up the bay, they and the men and women
representing them cannot escape their obligations to future generations of Filipinos to keep the
waters of the Manila Bay clean and clear as humanly as possible. Anything less would be a
betrayal of the trust repost in them.

AP: Here, RA 9003 (Ecological Solid Waste Management Act) is a sweeping piece of legislation
enacted to radically transform and improve waste management. It implements Art. 2, Sec. 16 of
the Constitution.

SECTION 19 INDEPENDENT NATIONAL ECONOMY/FILIPINO FIRST POLICY

Garcia v. Board of LB: An approval of business intent to move to another location without any cogent
Investments advantage to the government is a repudiation of the independent policy of the
government expressed in numerous laws and the Constitution to run its own affairs the way it
deems best for the national interest.

AP: Here, a petrochemical industry is not an ordinary investment opportunity, it is one essential
to the national interest. Thus, the foreign investor has no right of final choice to move the plant
site.

Manila Prince Hotel v. LB: When the Constitution mandates that in the grant of rights, privileges, and
GSIS concessions covering national economy and patrimony, the State shall give preference
to qualified Filipinos, it means that – qualified Filipinos shall be preferred.

In its plain and ordinary meaning, the term patrimony pertains to heritage. When the
Constitution speaks of national patrimony, it refers not only to the natural resources of the
Philippines, as the Constitution could have very well used the term natural resources, but also to
the cultural heritage of the Filipinos.

AP: Manila Hotel has become a landmark – a living testimonial of Philippine heritage. It becomes
part of our national economy and patrimony. Hence, its 51% shares cannot be sold to Renong
Berhad, a malaysian corporation.

SECTION 26 EQUAL ACCESS TO OPPORTUNITIES FOR PUBLIC OFFICE

Pamatong v. Comelec LB: There is no constitutional right to run for or hold public office.

What is recognized is merely a privilege subject to limitations imposed by law. Section


26, Article II of the Constitution neither bestow such a right or elevates the privilege to the level
of an enforceable right. There is nothing in the plain language of the provision which suggests
such a thrust or justifies an interpretation of the sort.

The provisions under this article are generally considered not self-executing, and there is no
plausible reason for according a different treatment to the “equal access” provision. The provision
is not intended to compel the State to enact positive measures that would accommodate as many
people as possible into public office.

Privilege of equal access to opportunities to public office may be subjected to limitations. Some
valid limitations specifically on the privilege to seek elective office are found in Omnibus Election
Code (nuisance candidates) and COMELEC Resolutions. As long as the limitations apply to
everybody equally without discrimination, the equal access clause is not violated.

ARTICLE IV CITIZENSHIP

SECTION 1

A. Citizens by Election

4
Co v. House Electoral LB: Chinese Father, Filipino Mother; Born 1948
Tribunal
The Court interprets Sec. 1(3) "Those born before Jan. 17, 1973, of Filipino mothers, who elect
Philippine citizenship upon reaching the age of majority” as applying not only to those who
elect Philippine citizenship after Feb. 2, 1987 but also to those who, having been born
of Filipino mothers, elected citizenship before that date.

The provision in question was enacted to correct the anomalous situation where one born of a
Filipino father and an alien mother was automatically granted the status of a natural-born citizen
while one born of a Filipino mother and an alien father would still have to elect Philippine
citizenship. If one so elected, he was not, under earlier laws, conferred the status of a natural-
born. Under the 1973 Constitution, those born of Filipino fathers and those born of Filipino
mothers with an alien father were placed on equal footing. They were both considered as natural-
born citizens.

AP: Here, to expect the respondent to have formally or in writing elected citizenship when he
came of age is to ask for the unnatural and unnecessary because he was already a citizen.
Not only was his mother a natural born citizen but his father had been naturalized when
respondent was only 9 years old. He could not have divined when he came of age that in 1973
and 1987 the Constitution would be amended to require him to have filed a sworn statement in
1969 electing citizenship inspite of his already having been a citizen since 1957. In 1969, election
through a sworn statement would have been an unusual and unnecessary procedure for one who
had been a citizen since he was 9 years old. Concededly, it was the law itself that had already
elected Philippine citizenship for Ong by declaring him as such.”

In re Ching LB: Procedure that should be followed in order to make a valid election of Philippine
citizenship.

CA 625 which was enacted pursuant to Sec. 1(3), Art. 4 of the 1935 Constitution, prescribes the
procedure that should be followed in order to make a valid election of Philippine citizenship. Under
Sec. 1 thereof, legitimate children born of Filipino mothers may elect Philippine citizenship by
expressing such intention “in a statement to be signed and sworn to by the party concerned
before any officer authorized to administer oaths, and shall be filed with the nearest civil registry.
The said party shall accompany the aforesaid statement with the oath of allegiance to the
Constitution and the Government of the Philippines.”

Time period within which the election of Philippine citizenship – “Reasonable Time”

The 1935 Constitution and C.A. No. 625 did not prescribe a time period within which the election
of Philippine citizenship should be made upon reaching the age of majority (21 years old). The
phrase “reasonable time” has been interpreted to mean that the election should be
made within 3 years from reaching the age of majority. However, the Court held in
Cuenco vs. Secretary of Justice, that the 3 year period is not an inflexible rule.

AP: Ching, having been born on 11 April 1964, was already 35 years old when he complied with
the requirements of CA 625 on 15 June 1999, or over 14 years after he had reached the age of
majority. Based on the interpretation of the phrase “upon reaching the age of majority,” Ching’s
election was clearly beyond, by any reasonable yardstick, the allowable period within which to
exercise the privilege.

Philippine citizenship can never be treated like a commodity that can be claimed when needed
and suppressed when convenient. One who is privileged to elect Philippine citizenship has only an
inchoate right to such citizenship. As such, he should avail of the right with fervor, enthusiasm
and promptitude. Sadly, in this case, Ching slept on his opportunity to elect Philippine citizenship
and, as a result, this golden privilege slipped away from his grasp. Thus, he cannot be admitted
to the Philippine Bar.

Vilando v. HRET LB: Art. 4, Sec. 1 (3) and (4) of the 1935 Constitution provides that: “The following are citizens
of the Philippines:
(3) Those whose fathers are citizens of the Philippines;
(4) Those whose mothers are citizens of the Philippines and, upon reaching the age of majority,
elect Philippine citizenship.”

AP: Here, HRET correctly relied on the presumption of validity of the July 9, 1957 and Sep. 21,
1959 Orders of the CFI Negros Oriental, which granted the petition and declared Julio Sy a
naturalized Filipino absent any evidence to the contrary. Indubitably, with Limkaichong’s father
having been conferred the status as a naturalized Filipino, it follows that she is a Filipino citizen
born to a Filipino father.

Even on the assumption that the naturalization proceedings and the subsequent issuance of
certificate of naturalization were invalid, Limkaichong can still be considered a natural-born
Filipino citizen having been born to a Filipino mother and having impliedly elected Filipino
citizenship when she reached majority age.

SECTION 2

5
A. Natural-born citizen

David v. Senate LB: There are only 2 categories of Filipino citizens: natural-born and naturalized.
Electoral Tribunal
A NATURAL-BORN CITIZEN is defined in Art. 4, Sec. 2 as one who is a citizen of the
Philippines "from birth without having to perform any act to acquire or perfect
Philippine citizenship." By necessary implication, a NATURALIZED CITIZEN is one who is not
natural-born.

AP: Here, Private respondent was a Filipino citizen at birth. This status' commencement from
birth means that private respondent never had to do anything to consummate this status. By
definition, she is natural-born. At no point has it been substantiated that private respondent went
through the actual naturalization process.

LB: Natural-born Philippine citizens who, after RA 9225 took effect, are naturalized in
foreign countries "retain," that is, keep, their Philippine citizenship, although the
effectivity of this retention and the ability to exercise the rights and capacities
attendant to this status are subject to certain solemnities (i.e., oath of allegiance and other
requirements for specific rights and/or acts, as enumerated in Sec. 5).

On the other hand, those who became citizens of another country before the effectivity of RA
9225 "reacquire" their Philippine citizenship and may exercise attendant rights and
capacities, also upon compliance with certain solemnities. Read in conjunction with Sec.
2's declaration of a policy of immutability, this reacquisition is not a mere restoration that leaves
a vacuum in the intervening period. Rather, this reacquisition works to restore natural-born status
as though it was never lost at all.

A natural-born Filipinos who have been naturalized elsewhere and wish to run for
elective public office must comply with all of the following requirements:
1. First, taking the oath of allegiance to the Republic.
2. Second, compliance with Article V, Section 1 of the 1987 Constitution, Republic Act No.
9189, otherwise known as the Overseas Absentee Voting Act of 2003, and other existing
laws.
3. Third, "making a personal and sworn renunciation of any and all foreign citizenship
before any public officer authorized to administer an oath."

AP: Private respondent has, therefore, not only fully reacquired natural-born citizenship; she has
also complied with all of the other requirements for eligibility to elective public office, as
stipulated in RA 9225.

1. First, on July 7, 2006, she took the Oath of Allegiance to the Republic of the Philippines.
2. Second, on Aug. 31, 2006, she became a registered voter of San Juan. This evidences
her compliance with Art. 5, Sec. 1 of the 1987 Constitution. Since she was to vote within
the country, this dispensed with the need to comply with the Overseas Absentee Voting
Act of 2003.
3. Lastly, she executed an Affidavit of Renunciation of Allegiance to the USA and
Renunciation of American Citizenship. This was complemented by her execution of an
Oath/Affirmation of Renunciation of Nationality of the US before Vice Consul Somer E.
Bessire-Briers, which was, in turn, followed by Vice Consul Jason Galian’s issuance of a
Certificate of Loss of Nationality, and the approval of this certificate by the Overseas
Citizen Service, Department of State.

SECTION 3

A. Loss of Citizenship

Yu v. Defensor LB/AP: Despite his naturalization as a Philippine citizen on Feb. 10, 1978, on July 1981,
Santiago petitioner applied for and was issued Portuguese passport by Portuguese Embassy in
Tokyo. While he was still a citizen of the Philippines who had renounced, upon his naturalization ,
he declared his nationality as Portuguese in commercial documents he signed,
specifically, the Companies registry of Tai Shun Estate Ltd. filed in HK sometime in April 1980.

Thus, the foregoing acts considered together constitute an express renunciation of


petitioner’s Philippine citizenship acquired through naturalization. Petitioner, with full
knowledge, and legal capacity, after having renounced Portuguese citizenship upon naturalization
as a Philippine citizen resumed or reacquired his prior status as a Portuguese citizen, applied for a
renewal of his Portuguese passport and represented himself as such in official documents even
after he had become a naturalized Philippine citizen. Such resumption or reacquisition of
Portuguese citizenship is grossly inconsistent with his maintenance of Philippine citizenship.

Frivaldo v. Comelec LB: Forfeiture of foreign citizenship did not and could not have the effect of
automatically restoring his citizenship in the Philippines that petitioner had earlier
renounced.

AP: Here, if he really wanted to disavow his American citizenship and reacquire Philippine
citizenship, the petitioner should have done so in accordance with the laws of our country. Under
6
CA 63 as amended by CA 473 and PD 725, Philippine citizenship may be reacquired by direct act
of Congress, by naturalization, or by repatriation.

While Frivaldo does not invoke either of the first 2 methods, he nevertheless claims he has
reacquired Philippine citizenship by virtue of a valid repatriation. He claims that by actively
participating in the elections in this country, he automatically forfeited American citizenship under
the laws of the United States. Such laws do not concern us here. The alleged forfeiture is between
him and the US as his adopted country. It should be obvious that even if he did lose his
naturalized American citizenship, such forfeiture did not and could not have the effect of
automatically restoring his citizenship in the Philippines that he had earlier renounced. At best,
what might have happened as a result of the loss of his naturalized citizenship was that he
became a stateless individual.

Labo, Jr. v. Comelec LB: Modes by which Philippine citizenship may be lost

CA 63, which enumerates the modes by which Philippine citizenship may be lost. Among these
are:
(1) naturalization in a foreign country;
(2) express renunciation of citizenship; and
(3) subscribing to an oath of allegiance to support the Constitution or laws of a foreign
country, all of which are applicable to the petitioner.

In this connection under Art. 4, Sec. 5, of the present Constitution, “Dual allegiance of citizens is
inimical to the national interest and shall be dealt with by law.”

AP: Even if it be assumed that, as the petitioner asserts, his naturalization in Australia was
annulled after it was found that his marriage to the Australian citizen was bigamous, that
circumstance alone did not automatically restore his Philippine citizenship. His divestiture of
Australian citizenship does not concern us here. That is a matter between him and his adopted
country. What we must consider is the fact that he voluntarily and freely rejected Philippine
citizenship and willingly and knowingly embraced the citizenship of a foreign country. The
possibility that he may have been subsequently rejected by Australia, as he claims, does not
mean that he has been automatically reinstated as a citizen of the Philippines.

LB: Under CA 63 as amended by PD 725, Philippine citizenship may be reacquired by direct act of
Congress, by naturalization, or by repatriation.

AP: It does not appear in he record, nor does the petitioner claim, that he has reacquired
Philippine citizenship by any of these methods. He does not point to any judicial decree of
naturalization as to any statute directly conferring Philippine citizenship upon him. Neither has he
shown that he has complied with PD 725, providing that: x x x (2) natural-born Filipinos who
have lost their Philippine citizenship may reacquire Philippine citizenship through repatriation by
applying with the Special Committee on Naturalization created by LOI 270, and, if their
applications are approved, taking the necessary oath of allegiance to the Republic of the
Philippines, after which they shall be deemed to have reacquired Philippine citizenship. The
Commission on Immigration and Deportation shall thereupon cancel their certificate of
registration.

The petitioner is not now, nor was he on the day of the local elections on Jan. 18, 1988, a citizen
of the Philippines. In fact, he was not even a qualified voter under the Constitution itself because
of his alienage. He was therefore ineligible as a candidate for mayor of Baguio City under Sec. 42
of the LGC.

Aznar v. Osmeña LB: An application for alien certificate does not amount to an express renunciation or
repudiation of one’s citizenship.

AP: Petitioner failed to present direct proof that private respondent had lost his Filipino citizenship
by any of the modes provided for under CA 63. Among others, these are:
(1) by naturalization in a foreign country;
(2) by express renunciation of citizenship; and
(3) by subscribing to an oath of allegiance to support the Constitution or laws of a foreign
country.

From the evidence, it is clear that Osmena did not lose his Philippine citizenship by any of the 3
mentioned hereinabove or by any other mode of losing Philippine citizenship.

In concluding that private respondent had been naturalized as a citizen of the USA, the petitioner
merely relied on the fact that Osmena was issued alien certificate of registration and was given
clearance and permit to re-enter the Philippines by the Commission on Immigration and
Deportation. Aznar assumed that because of the foregoing, Osmena is an American and “being an
American”, he “must have taken and sworn to the Oath of Allegiance required by the U.S.
Naturalization Laws.” Philippine courts are only allowed to determine who are Filipino citizens and
who are not. Whether or not a person is considered an American under the laws of the US does
not concern us here.

By virtue of his being the son of a Filipino father (Former Pres. Osmena), the presumption that
private respondent is a Filipino remains. It was incumbent upon the petitioner to prove that
private respondent had lost his Philippine citizenship. As earlier stated, however, the petitioner
failed to positively establish this fact.

7
Valles v. Comelec LB: For renunciation to effectively result in the loss of citizenship, the same must be
express.

AP: The application of the herein private respondent for an alien certificate of registration, and
her holding of an Australian passport, as in the case of Mercado vs. Manzano, were mere acts of
assertion of her Australian citizenship before she effectively renounced the same. Thus, at the
most, private respondent had dual citizenship—she was an Australian and a Filipino, as well. Dual
citizenship as a disqualification must refer to citizens with dual allegiance.

SECTION 5

Mercado v. Manzano LB: Dual citizenship is different from dual allegiance.

The Dual citizenship arises when, as a result of the concurrent application of the different laws
of 2 or more states, a person is simultaneously considered a national by the said states. For
instance, such a situation may arise when a person whose parents are citizens of a state which
adheres to the principle of jus sanguinis is born in a state which follows the doctrine of jus soli.
Such a person, ipso facto and without any voluntary act on his part, is concurrently considered a
citizen of both states.

Considering the citizenship clause (Art. IV) of our Constitution, it is possible for the following
classes of citizens of the Philippines to possess dual citizenship: (1) Those born of Filipino fathers
and/or mothers in foreign countries which follow the principle of jus soli; (2) Those born in the
Philippines of Filipino mothers and alien fathers if by the laws of their fathers’ country such
children are citizens of that country; (3) Those who marry aliens if by the laws of the latter’s
country the former are considered citizens, unless by their act or omission they are deemed to
have renounced Philippine citizenship.

Dual allegiance, on the other hand, refers to the situation in which a person simultaneously
owes, by some positive act, loyalty to 2 or more states. While dual citizenship is involuntary, dual
allegiance is the result of an individual’s volition.

AP: Here, the record shows that private respondent was born in San Francisco, California on
Sept. 4, 1955, of Filipino parents. Since the Philippines adheres to the principle of jus sanguinis,
while the US follows the doctrine of jus soli, the parties agree that, at birth at least, he was a
national both of the Philippines and of the US. By declaring in his certificate of candidacy that he
is a Filipino citizen; that he is not a permanent resident or immigrant of another country; that he
will defend and support the Constitution of the Philippines and bear true faith and allegiance
thereto and that he does so without mental reservation, private respondent has, as far as the
laws of this country are concerned, effectively repudiated his American citizenship and anything
which he may have said before as a dual citizen.

Calilung vs. LB/AP: What RA 9225 does is allow dual citizenship to natural born Filipino citizens
Datumanong who have lost Philippine citizenship by reason of their naturalization as citizens of a
foreign country. On its face, it does not recognize dual allegiance. By swearing to the supreme
authority of the Republic, the person implicitly renounces his foreign citizenship. Plainly, from
Sec. 3, RA 9225 stayed clear out of the problem of dual allegiance and shifted the burden of
confronting the issue of whether or not there is dual allegiance to the concerned foreign country.
What happens to the other citizenship was not made a concern of RA 9225.

Also, Sec. 5, Art. 4 of the Constitution is a declaration of a policy and it is not a self-executing
provision. The legislature still has to enact the law on dual allegiance. In Sec.s 2 and 3 of RA
9225, the framers were not concerned with dual citizenship per se, but with the status of
naturalized citizens who maintain their allegiance to their countries of origin even after their
naturalization. Congress was given a mandate to draft a law that would set specific parameters of
what really constitutes dual allegiance. Until this is done, it would be premature for the judicial
department, including this Court, to rule on issues pertaining to dual allegiance.

Kilosbayan Foundation LB: In Labayo-Rowe v. Republic, this Court held that: Changes which affect the civil status or
v. Ermita citizenship of a party are substantial in character and should be threshed out in a
proper action depending upon the nature of the issues in controversy, and wherein all the
parties who may be affected by the entries are notified or represented and evidence is submitted
to prove the allegations of the complaint, and proof to the contrary admitted.

RA 9048 provides in Sec. 2 (3) that a summary administrative proceeding to correct clerical or
typographical errors in a birth certificate cannot apply to a change in nationality. Substantial
corrections to the nationality or citizenship of persons recorded in the civil registry should,
therefore, be effected through a petition filed in court under Rule 108 of the Rules of Court.

AP: It is clear that from the records of this Court, Ong is a naturalized Filipino citizen. The alleged
subsequent recognition of his natural-born status by the Bureau of Immigration and the DOJ
cannot amend the final decision of the trial court stating that Ong and his mother were
naturalized along with his father.

Furthermore, as petitioners correctly submit, no substantial change or correction in an entry in a


civil register can be made without a judicial order, and, under the law, a change in citizenship
status is a substantial change. The series of events and long string of alleged changes in the
nationalities of Ong’s ancestors, by various births, marriages and deaths, all entail factual

8
assertions that need to be threshed out in proper judicial proceedings so as to correct the existing
records on his birth and citizenship.

The chain of evidence would have to show that Dy Guiok Santos, Ong’s mother, was a Filipino
citizen, contrary to what still appears in the records of this Court. Ong has the burden of proving
in court his alleged ancestral tree as well as his citizenship under the time-line of three
Constitutions. Until this is done, respondent Ong cannot accept an appointment to this Court as
that would be a violation of the Constitution. For this reason, he can be prevented by injunction
from doing so.

Nicolas-Lewis v. LB/AP: There is no provision in the dual citizenship law—RA 9225—requiring “duals” to
Comelec actually establish residence and physically stay in the Philippines first before they can
exercise their right to vote.

On the contrary, RA 9225, in implicit acknowledgment that “duals” are most likely non-residents,
grants under its Sec. 5(1) the same right of suffrage as that granted an absentee voter under RA
9189. It cannot be overemphasized that RA 9189 aims, in essence, to enfranchise as much as
possible all overseas Filipinos who, save for the residency requirements exacted of an ordinary
voter under ordinary conditions, are qualified to vote.

Considering the unison intent of the Constitution and RA 9189 and the expansion of the scope of
that law with the passage of RA 9225, “duals” may now exercise the right of suffrage thru the
absentee voting scheme and as overseas absentee voters.

RA 9189 defines the terms adverted to in the following wise: “Absentee Voting” refers to the
process by which qualified citizens of the Philippines abroad exercise their right to
vote; “Overseas Absentee Voter” refers to a citizen of the Philippines who is qualified to
register and vote under this Act, not otherwise disqualified by law, who is abroad on
the day of elections.

Petition for Leave to LB: A Filipino lawyer who becomes a citizen of another country is deemed never to have
Resume the Practice lost his Philippine citizenship if he reacquires it in accordance with RA 9225. Although
of Law, Benjamin M. he is also deemed never to have terminated his membership in the Philippine bar, no automatic
Dacanay right to resume law practice accrues.

Requirements for a lawyer who reacquired Filipino citizenship to practice law:

Before a lawyer who reacquires Filipino citizenship pursuant to RA 9225 can resume his law
practice, he must first secure from this Court the authority to do so, conditioned on:
a. the updating and payment in full of the annual membership dues in the IBP;
b. the payment of professional tax;
c. the completion of at least 36 credit hours of mandatory continuing legal education; this
is specially significant to refresh the applicant/petitioner’s knowledge of Philippine laws
and update him of legal developments and
d. the retaking of the lawyer’s oath which will not only remind him of his duties and
responsibilities as a lawyer and as an officer of the Court, but also renew his pledge to
maintain allegiance to the Republic of the Philippines.

AP: Here, compliance with these conditions will restore his good standing as a member of the
Philippine bar.

Corodora v. COMELEC LB: Dual citizenship is not a ground for disqualification from running for any elective
local position.

The twin requirements of swearing to an Oath of Allegiance and executing a


Renunciation of Foreign Citizenship.

Sec. 5(3) of RA 9225 states that naturalized citizens who reacquire Filipino citizenship and desire
to run for elective public office in the Philippines shall meet the qualifications for holding such
public office as required by the Constitution and existing laws and, at the time of filing the
certificate of candidacy, make a personal and sworn renunciation of any and all foreign citizenship
before any public officer authorized to administer an oath aside from the oath of allegiance
prescribed in Sec. 3 of RA 9225

AP: Here, Tambunting, a natural-born Filipino, did not subsequently become a naturalized citizen
of another country. Hence, the twin requirements in RA 9225 do not apply to him. Residency, for
the purpose of election laws, includes the twin elements of the fact of residing in a fixed place and
the intention to return there permanently, and is not dependent upon citizenship. Thus,
Tambunting is eligible for the office which he sought to be elected and fulfilled the citizenship and
residency requirements prescribed by law.

Japzon vs. Comelec LB: RA 9225 governs the manner in which a natural-born Filipino may reacquire or retain his
Philippine citizenship despite acquiring a foreign citizenship, and provides for his rights and
liabilities under such circumstances. A close scrutiny of said statute would reveal that it does not
at all touch on the matter of residence of the natural-born Filipino taking advantage of
its provisions.

Residency in the Philippines only becomes relevant when the natural-born Filipino with

9
dual citizenship decides to run for public office. Breaking down the afore-quoted provision,
for a natural born Filipino, who reacquired or retained his Philippine citizenship under RA 9225, to
run for public office, he must:

1. meet the qualifications for holding such public office as required by the Constitution and
existing laws; and
2. make a personal and sworn renunciation of any and all foreign citizenships before any
public officer authorized to administer an oath.
3.
As to the 1st requirement, particularly the 1-year residency requirement, it is the fact of
residence that is the decisive factor in determining whether or not an individual has
satisfied the residency qualification requirement.

AP: Here, while Ty left for the US twice during the 1-year period, he actually resided in the place
for 9 of the 12 months required. The residency requirement does not require that the candidate
stay in the locality and never leave the place for the full 1 year period. Also, the COMELEC found
that Ty was a resident of the Municipality of General Macarthur, Eastern Samar, one year prior to
the 14 May 2007 local elections. It is axiomatic that factual findings of administrative agencies,
such as the COMELEC, which have acquired expertise in their field are binding and conclusive on
the Court.

Sobejana-Condon v. LB: “Sworn renunciation of foreign citizenship” in Sec. 5(2) of RA 9225 is NOT a mere
Comelec pro-forma requirement.

AP: Here, the petitioner has validly re-acquired her Filipino citizenship when she took an Oath of
Allegiance to the Republic of the Philippines on Dec. 5, 2005. At that point, she held dual
citizenship, i.e., Australian and Philippine. On Sept. 18, 2006, or a year before she initially sought
elective public office, she filed a renunciation of Australian citizenship in Canberra, Australia.
Admittedly, however, the same was not under oath contrary to the exact mandate of Sec. 5(2)
that the renunciation of foreign citizenship must be sworn before an officer authorized to
administer oath.

The petitioner’s act of running for public office does not suffice to serve as an effective
renunciation of her Australian citizenship. While this Court has previously declared that the filing
by a person with dual citizenship of a certificate of candidacy is already considered a renunciation
of foreign citizenship, such ruling was already adjudged superseded by the enactment of RA 9225
on Aug. 29, 2003 which provides for the additional condition of a personal and sworn renunciation
of foreign citizenship. The fact that petitioner won the elections cannot cure the defect of her
candidacy.

The petitioner's failure to comply therewith in accordance with the exact tenor of the law,
rendered ineffectual the Declaration of Renunciation of Australian Citizenship she executed. As
such, she is yet to regain her political right to seek elective office. Unless she executes a sworn
renunciation of her Australian citizenship, she is ineligible to run for and hold any elective office in
the Philippines.

Poe-Llamanzares v. LB: A foundling is presumed born of citizens of the country where he is found.
Comelec
The principles found in two conventions, while yet unratified by the Philippines, are generally
accepted principles of international law. The first is Article 14 of the 1930 Hague Convention on
Certain Questions Relating to the Conflict of Nationality Laws under which a foundling is presumed
to have the "nationality of the country of birth." The second is the principle that a foundling is
presumed born of citizens of the country where he is found.

Adopting these legal principles from the 1930 Hague Convention and the 1961 Convention on
Statelessness is rational and reasonable and consistent with the jus sanguinis regime in our
Constitution. The presumption of natural-born citizenship of foundlings stems from the
presumption that their parents are nationals of the Philippines. As the empirical data provided by
the PSA show, that presumption is at more than 99% and is a virtual certainty.

AP: The fact is that petitioner's blood relationship with a Filipino citizen is DEMONSTRABLE. The
Solicitor General offered official statistics from (PSA) that from 1965 to 1975, the total number of
foreigners born in the Philippines was 15,986 while the total number of Filipinos born in the
country was 10,558,278. The statistical probability that any child born in the Philippines in that
decade is natural-born Filipino was 99.83%. For her part, petitioner presented census statistics
for Iloilo Province for 1960 and 1970, also from the PSA. In 1960, there were 962,532 Filipinos
and 4,734 foreigners in the province; 99.62% of the population were Filipinos.

In 1970, the figures were 1,162,669 Filipinos and 5,304 foreigners, or 99.55%. Also presented
were figures for the child producing ages (15-49). In 1960, there were 230,528 female Filipinos
as against 730 female foreigners or 99.68%. In the same year, there were 210,349 Filipino males
and 886 male aliens, or 99.58%. In 1970, there were 270,299 Filipino females versus 1, 190
female aliens, or 99.56%. That same year, there were 245,740 Filipino males as against only
1,165 male aliens or 99.53%.

She also has typical Filipino features: height, flat nasal bridge, straight black hair, almond shaped
eyes and an oval face. All of the foregoing evidence, that a person with typical Filipino features is
abandoned in Catholic Church in a municipality where the population of the Philippines is
overwhelmingly Filipinos such that there would be more than a 99% chance that a child born in

10
the province would be a Filipino, would indicate more than ample probability if not statistical
certainty, that petitioner's parents are Filipinos. That probability and the evidence on which it is
based are admissible under Rule 128, Section 4 of the Revised Rules on Evidence.

David v. Agbay LB: Sec. 3 of RA No. 9225 lays down conditions for 2 categories of natural-born Filipinos
referred to in the first and second paragraphs.

Under the first paragraph are those natural-born Filipinos who have lost their citizenship
by naturalization in a foreign country who shall re-acquire their Philippine citizenship
upon taking the oath of allegiance to the Republic of the Philippines.

The second paragraph covers those natural-born Filipinos who became foreign citizens
after RA 9225 took effect, who shall retain their Philippine citizenship upon taking the
same oath. The taking of oath of allegiance is required for both categories of natural-born
Filipino citizens who became citizens of a foreign country, but the terminology used is different,
“re-acquired” for the first group, and “retain” for the second group.

The law thus makes a distinction between those natural-born Filipinos who became foreign
citizens before and after the effectivity of RA 9225. Although the heading of Sec. 3 is “Retention
of Philippine Citizenship”, the authors of the law intentionally employed the terms “re-acquire”
and “retain” to describe the legal effect of taking the oath of allegiance to the Republic of the
Philippines. This is also evident from the title of the law using both re-acquisition and retention.

In fine, for those who were naturalized in a foreign country, they shall be deemed to have re-
acquired their Philippine citizenship which was lost pursuant to CA 63, under which naturalization
in a foreign country is one of the ways by which Philippine citizenship may be lost. As its title
declares, RA 9225 amends CA 63 by doing away with the provision in the old law which takes
away Philippine citizenship from naturalborn Filipinos who become naturalized citizens of other
countries and allowing dual citizenship, and also provides for the procedure for re-acquiring and
retaining Philippine citizenship.

In the case of those who became foreign citizens after R.A. 9225 took effect, they shall retain
Philippine citizenship despite having acquired foreign citizenship provided they took the oath of
allegiance under the new law.

AP: Here, considering that petitioner was naturalized as a Canadian citizen prior to the effectivity
of RA 9225, he belongs to the first category of natural-born Filipinos under Sec. 3 (1) who lost
Philippine citizenship by naturalization in a foreign country. As the new law allows dual
citizenship, he was able to re-acquire his Philippine citizenship by taking the required oath of
allegiance. However, petitioner made the untruthful statement in the MLA, a public document,
that he is a Filipino citizen at the time of the filing of said application, when in fact he was then
still a Canadian citizen. Under CA 63, the governing law at the time he was naturalized as
Canadian citizen, naturalization in a foreign country was among those ways by which a natural-
born citizen loses his Philippine citizenship. While he reacquired Philippine citizenship under R.A.
9225 six months later, the falsification was already a consummated act, the said law having no
retroactive effect insofar as his dual citizenship status is concerned. The MTC therefore did not err
in finding probable cause for falsification of public document under Article 172 (1).

ARTICLE VI THE LEGISLATIVE DEPARTMENT

SECTION 1

A. Non-Delegability

Rubi v. Provincial LB: The maxim of Constitutional Law forbidding the delegation of legislative power should be
Board zealously protected. The distinction between the delegation of power to make the law,
which necessarily involves a discretion as to which it shall be, and conferring an
authority or discretion as to what it shall be, and conferring an authority as to its
execution, to be exercised under and in pursuance of the law.

The first cannot be done; to the latter, no valid objection can be made . An EXCEPTION to the
general rule permits the central body to delegate legislative powers to local authorities.
Section 2145 of the Administrative Code of 1917 is not unlawful delegation of
legislative power by the Philippine Legislature to provincial officials and a department
head.

AP: Here, the delegation is one stemming from necessity — the section in the Administrative
Code allowed provincial governors, with the approval of the provincial board and the Secretary of
Interior to direct Mangyans to live in areas they deem favourable to their improvement.

Pelaez v. Auditor LB: The power to create a municipal corporation is strictly a legislative function.
General
Although Congress may delegate to another branch of the Government the power to fill in the
details in the execution, enforcement or administration of a law, it is essential, to forestall a
violation of the principle of separation of powers, that said law:

1. Be complete in itself — it must set forth therein the policy to be executed, carried
11
out or implemented by the delegate; and
2. Fix a standard — the limits of which are sufficiently determinate or determinable —
to which the delegate must conform in the performance of his functions.

Without a statutory declaration of policy, the delegate would in effect, make or


formulate such policy, which is the essence of every law; and, without the
aforementioned standard, there would be no means to determine, with reasonable
certainty, whether the delegate has acted within or beyond the scope of his authority.

AP: Section 68 of the Revised Administrative Code does not meet these well settled requirements
for a valid delegation of the power to fix the details in the enforcement of a law.
● It does not enunciate any policy to be carried out or implemented by the President.
● Neither does it give a standard sufficiently precise to avoid the evil effects above referred to.

Cebu Oxygen v. Drilon LB: Implementing rules cannot add or detract from the provisions of law it is designed
to implement.

Administrative regulations adopted under legislative authority by a particular


department must be in harmony with the provisions of the law, and should be for the
sole purpose of carrying into effect its general provisions. The law itself cannot be
expanded by such regulations. An administrative agency cannot amend an act of Congress.

AP: The provisions of Republic Act No. 6640, do not prohibit the crediting of CBA anniversary
wage increases for purposes of compliance with Republic Act No. 6640. The implementing rules
cannot provide for such a prohibition not contemplated by the law.

Chiongbian v. Orbos LB: The power to “merge” administrative regions is executive in character.

o While the power to merge administrative regions is not expressly provided for in the
Constitution, it is a power that has been traditionally lodged with the President to
facilitate the exercise of the power of general supervision over local governments
[see Art. X, §4 of the Constitution]."
o Regions are not territorial and political divisions like provinces, cities, municipalities and
barangays.
✔ They are "mere groupings of contiguous provinces for administrative purposes."
✔ "Administrative in nature"

Legislative standard may be expressed or implied.

It need not be found in the law challenged because it may be embodied in other statutes
on the same subject as that of the challenged legislation.

AP: The creation and subsequent reorganization of administrative regions have been by the
President pursuant to authority granted to him by law. In conferring on the President the power
“to merge by administrative determination the existing regions” following the establishment of
the ARMM, Congress merely followed the pattern set in previous legislation dating back to the
initial organization of administrative regions in 1972. The choice of the President as delegate is
logical because the division of the country into regions is intended to facilitate not only the
administration of local governments but also the direction of executive departments which the law
requires should have regional offices.

Therefore, there is no abdication by Congress of its legislative power in conferring on the


President the power to merge administrative regions.

Therefore, there is no abdication by Congress of its legislative power in conferring on the


President the power to merge administrative regions.

U.S. v. Ang Tang Ho LB: A law must be complete, in all its terms and provisions, when it leaves the
legislative branch of the government, and nothing must be left to the judgment of the
electors or other appointee or delegate of the legislature, so that, in form and substance, it
is a law in all its details in presenti, but which may be left to take effect in futuro, if necessary,
upon the ascertainment of any prescribed fact or event.

AP: The Legislature left it to the sole discretion of the Governor-General to say
(1) what was and what was not “any cause” for enforcing the act,
(2) and what was and what was not “an extraordinary rise in the price of palay, rice or
corn,”
(3) and under certain undefined conditions to fix the price at which rice should be sold,
without regard to grade or quality, also to say whether a proclamation should be issued,
if so,
(4) when, and whether or not the law should be enforced,
(5) how long it should be enforced, and when the law should be suspended. T

The Legislature did not specify or define what was “any cause,” or what was “an extraordinary
rise in the price of rice, palay or corn,” Neither did it specify or define the conditions upon which
the proclamation should be issued. In the absence of the proclamation no crime was committed.

Solicitor General v. LB: The Court holds that there is a valid delegation of legislative power to promulgate such

12
Metropolitan Manila measures, it appearing that the requisites of such delegation are present.
Authority 1) Completeness of the statute making the delegation; and
● Statute must leave the legislature complete in all its terms and provisions such
that all the delegate will have to do when the statute reaches it is to implement
it.
● What only can be delegated is not the discretion to determine what the law shall
be but the discretion to determine how the law shall be enforced.
2) Presence of a sufficient standard.
● Enforcement may be effected only in accordance with a sufficient standard, the
function of which is to map out the boundaries of the delegate's authority and
thus "prevent the delegation from running riot."
● It is settled that the "convenience and welfare" of the public, particularly the
motorists and passengers in the case at bar, is an acceptable sufficient standard
to delimit the delegate's authority.

But the problem before us is not the validity of the delegation of legislative power. The
question we must resolve is the validity of the exercise of such delegated power.

The measures in question are enactments of local governments acting only as agents of the
national legislature. Necessarily, the acts of these agents must reflect and conform to the will of
their principal. To test the validity of such acts in the specific case now before us, we
apply the particular requisites of a valid ordinance as laid down by the accepted
principles governing municipal corporations.
1) It must not contravene the Constitution or any statute;
2) It must not be unfair or oppressive;
3) It must not be partial or discriminatory;
4) It must not prohibit but may regulate trade;
5) It must not be unreasonable;
6) It must be general and consistent with public policy.

AP: Measures under consideration do not pass the first criterion.


They do not conform to existing law, PD 1605.
● Nothing in the provisions of the decree authorizing the MMA to impose such sanctions.
● In fact, it prohibits the imposition of such sanctions in Metropolitan Manila.

As delegates of the Congress, the local government unit cannot contravene but must
obey at all times the will of their principal.

People v. Dacuycuy LB: It is not for the courts to fix the term of imprisonment where no points of reference
have been provided by the legislature.

What valid delegation presupposes and sanctions is an exercise of discretion to fix the length of
service of a term of imprisonment that must be encompassed within specific or designated limits
provided by law.

The absence of designated limits will constitute an undue delegation, if not an outright intrusion
into or assumption, of legislative power.

AP: Section 32 of Republic Act No. 4670 provides for an indeterminable period of imprisonment,
with neither a minimum nor a maximum duration having been set by the legislative authority.
The courts are thus given too much latitude in its discretion to fix the term of imprisonment.
Without the benefit of any sufficient standard, a judge may be allowed to fix the duration within
the range of one minute to the life span of the accused.

Thus, Section 32 of the Magna Carta for Public School Teachers was declared unconstitutional
(without prejudice to other constitutional provisions).

Abakada v. Executive LB: While the power to tax cannot be delegated to executive agencies, details as to the
Secretary enforcement and administration of an exercise of such power may be left to them,
including the power to determine the existence of facts on which its operation depends.

The powers which Congress is prohibited from delegating are those which are strictly,
or inherently and exclusively, legislative.
● Purely legislative power which can never be delegated is the authority to make a
complete law- complete as to the time when it shall take effect and as to whom it shall
be applicable, and to determine the expediency of its enactment.
● It is the nature of the power and not the liability of its use or the manner of its exercise
which determines the validity of its delegation.

Exceptions are:
1) delegation of tariff powers to President under Constitution
2) delegation of emergency powers to President under Constitution
3) delegation to the people at large
4) delegation to local governments
5) delegation to administrative bodies

For the delegation to be valid, it must be complete and it must fix a standard.
● A sufficient standard is one which defines legislative policy, marks its limits, maps out its
boundaries and specifies the public agency to apply it.

AP: In this case, it is not a delegation of legislative power BUT a delegation of ascertainment of
facts upon which enforcement and administration of the increased rate under the law is
13
contingent.

● The legislature has made the operation of the 12% rate effective January 1, 2006,
contingent upon a specified fact or condition.
● It leaves the entire operation or non-operation of the 12% rate upon factual matters
outside of the control of the executive.
● No discretion would be exercised by the President.

Belgica v. Executive LB: The power of appropriation is lodged in Congress.


Secretary
The fundamental rule, as categorically articulated in Abakada, cannot be overstated – from the
moment the law becomes effective, any provision of law that empowers Congress or
any of its members to play any role in the implementation or enforcement of the law
violates the principle of separation of powers and is thus unconstitutional.

AP: Post-enactment measures which govern the areas of project identification, fund release and
fund realignment are not related to functions of congressional oversight and, hence, allow
legislators to intervene and/or assume duties that properly belong to the sphere of budget
execution.

SECTION 3

A. Imposition of other qualifications

Social Justice Society LB: The COMELEC cannot, in the guise of enforcing and administering elections laws or
vs. Dangerous Drugs promulgating rules and regulations to implement Sec. 36 (g) of RA 9165, validly impose
Board qualifications on candidates for senator in addition to what the Constitution prescribes.

AP: Here, COMELEC rules required candidates for public office to submit to a drug test and be
certified ‘drug-free’. The SC declared the same as an additional qualification that violates the
Constitution, which enumerates the exclusive list of qualifications for senators.

SECTION 5

A. Apportionment

Mariano, Jr. v. Comelec LB: In the earlier case of Tobias v. Abalos, it has already been ruled that reapportionment of
legislative districts may be made through a special law, such as in the charter of a new
city.
● The Constitution clearly provides that Congress shall be composed of not more
than two hundred fifty (250) members, unless otherwise fixed by law.
○ The Constitution did not preclude Congress from increasing its
membership by passing a law, other than a general reapportionment of
the law.

The Constitution provides that a city with a population of at least two hundred fifty thousand
(250,000) shall have at least one representative.

AP: Petitioners cannot insist that the addition of another legislative district in Makati is not in
accord with Sect. 5(3), Art. 6 of the Constitution for as of the latest survey (1990 census), the
population of Makati stands at only 450,000. Said section provides, inter alia, that a city with a
population of at least 250,000 shall have at least one representative.

Even granting that the population of Makati as of the 1990 census stood at four hundred fifty
thousand (450,000), its legislative district may still be increased since it has met the minimum
population requirement of two hundred fifty thousand (250,000).

Montejo v. Comelec LB: COMELEC does not have the power of legislative reapportionment. It may only make minor
adjustments of reapportionment, which do not entail a change in the allocations of the districts or
any other substantial change.

AP: Thus, the COMELEC Resolution transferring the municipality of Capoocan of the 2nd District
and the municipality of Palompon of the 4th District to the 3rd District of Leyte is null and void.

Aldaba v. Comelec LB: A city which has reached 250,000 is entitled to have a legislative district only in the
immediately following election after the attainment of the 250,000 population.
Therefore, not as soon as it attains the 250K population.

AP: The Certification of Regional Director Miranda, which is based on demographic projections , is
without legal effect because Regional Director Miranda has no basis and no authority to issue the

14
Certification.

The certifications on demographic projections can be issued only if such projections are
declared official by the National Statistics Coordination Board (NSCB).
● Can be issued only by the NSO Administrator or his designated certifying
officer.

Aquino v. Comelec LB/AP: There is no specific provision in the Constitution that fixes a 250,000 minimum
population that must compose a legislative district. As already mentioned, the petitioners rely on
the second sentence of Section 5(3), Article VI of the 1987 Constitution, coupled with what they
perceive to be the intent of the framers of the Constitution to adopt a minimum population of
250,000 for each legislative district. The second sentence of Section 5(3), Article VI of the
Constitution, succinctly provides: “Each city with a population of at least two hundred fifty
thousand, or each province, shall have at least one representative.” The provision draws a plain
and clear distinction between the entitlement of a city to a district on one hand, and the
entitlement of a province to a district on the other.

The Mariano case limited the application of the 250,000 minimum population
requirement for cities only to its initial legislative district. In other words, while Section
5(3), Article VI of the Constitution requires a city to have a minimum population of
250,000 to be entitled to a representative, it does not have to increase its population by
another 250,000 to be entitled to an additional district. There is no reason why the
Mariano case, which involves the creation of an additional district within a city, should
not be applied to additional districts in provinces. Indeed, if an additional legislative district
created within a city is not required to represent a population of at least 250,000 in order to be
valid, neither should such be needed for an additional district in a province, considering moreover
that a province is entitled to an initial seat by the mere fact of its creation and regardless of its
population.

B. Party List System

BANAT vs. COMELEC LB: The 80-20 rule is observed in the following manner:
● For every 5 seats allotted for legislative districts, there shall be one seat
allotted for a party-list representative.
● Originally, the 1987 Constitution provides that there shall be not more than 250
members of the lower house. Using the 80-20 rule, 200 of that will be from
legislative districts, and 50 would be from party-list representatives.
● However, the Constitution also allowed Congress to fix the number of the
membership of the lower house as in fact, it can create additional legislative
districts as it may deem appropriate.
o In the May 2007 elections, there were 220 district representatives, hence
applying the 80-20 rule or the 5:1 ratio, there should be 55 seats allotted for
party-list representatives.
● Formula:
Number of seats available to party-list representatives =

( Current number of legislative


0.80
district representatives
) x (0.20)
● Hence,
220
● x ( 0.20 )=55
0.80

2) The 20% allocation for party-list representatives is merely a ceiling – meaning, the
number of party-list representatives shall not exceed 20% of the total number of the
members of the lower house. However, it is not mandatory that the 20% shall be filled.
3) No. Section 11(b) of RA 7941 is unconstitutional.
● No constitutional basis to allow that only party-lists which garnered 2% of the votes
cast are qualified for a seat and those which garnered less than 2% are disqualified.
● Further, the 2% threshold creates a mathematical impossibility to attain the ideal 80-20
apportionment. The Supreme Court explained:
o To illustrate: There are 55 available party-list seats. Suppose there are 50
million votes cast for the 100 participants in the party-list elections. A party that
has two percent of the votes cast, or one million votes, gets a guaranteed seat.
Let us further assume that the first 50 parties all get one million votes. Only 50
parties get a seat despite the availability of 55 seats. Because of the operation
of the two percent threshold, this situation will repeat itself even if we increase
the available party-list seats to 60 seats and even if we increase the votes cast
to 100 million. Thus, even if the maximum number of parties get two percent of
the votes for every party, it is always impossible for the number of occupied
party-list seats to exceed 50 seats as long as the two percent threshold is
present.
● It is therefore clear that the two percent threshold presents an unwarranted obstacle to
the full implementation of Section 5(2), Article VI of the Constitution and prevents the
attainment of “the broadest possible representation of party, sectoral or group interests
in the House of Representatives.”
4) Instead, the 2% rule should mean that if a party-list garners 2% of the votes cast, then it is
guaranteed a seat, and not “qualified”.
15
● This allows those party-lists garnering less than 2% to also get a seat.
● But how? The Supreme Court laid down the following rules:
1) The parties, organizations, and coalitions shall be ranked from the highest to
the lowest based on the number of votes they garnered during the elections.
2) The parties, organizations, and coalitions receiving at least two percent (2%) of
the total votes cast for the party-list system shall be entitled to one guaranteed
seat each.
3) Those garnering sufficient number of votes, according to the ranking in
paragraph 1, shall be entitled to additional seats in proportion to their total
number of votes until all the additional seats are allocated.
4) Each party, organization, or coalition shall be entitled to not more than three
(3) seats.
● In computing the additional seats, the guaranteed seats shall no longer be included
because they have already been allocated, at one seat each, to every two-percenter.
Thus, the remaining available seats for allocation as “additional seats” are the maximum
seats reserved under the Party List System less the guaranteed seats. Fractional seats
are disregarded in the absence of a provision in R.A. No. 7941 allowing for a rounding off
of fractional seats.
● In short, there shall be two rounds in determining the allocation of the seats.
1) In the first round, all party-lists which garnered at least 2% of the votes cast
(called the two-percenters) are given their one seat each. The total number of
seats given to these two-percenters are then deducted from the total available
seats for party-lists.
o In this case, 17 party-lists were able to garner 2% each. There are a
total 55 seats available for party-lists hence, 55 minus 17 = 38
remaining seats. (Please refer to the full text of the case for the
tabulation).
2) The number of remaining seats, in this case 38, shall be used in the second
round, particularly, in determining, first, the additional seats for the two-
percenters, and second, in determining seats for the party-lists that did not
garner at least 2% of the votes cast, and in the process filling up the 20%
allocation for party-list representatives.
o Get the total percentage of votes garnered by the party and multiply it
against the remaining number of seats. The product, which shall not be
rounded off, will be the additional number of seats allotted for the
party list – but the 3 seat limit rule shall still be observed.
o Example:

In this case, the BUHAY party-list garnered the highest total vote of 1,169,234
which is 7.33% of the total votes cast for the party-list elections
(15,950,900).

Applying the formula above: (Percentage of vote garnered) x (remaining seats)


= number of additional seat

Hence, 7.33% x 38 = 2.79

Rounding off to the next higher number is not allowed so 2.79 remains 2.
BUHAY is a two-percenter which means it has a guaranteed one seat
PLUS additional 2 seats or a total of 3 seats.

If it happens that BUHAY got 20% of the votes cast, it will still get 3 seats
because the 3 seat limit rule prohibits it from having more than 3
seats.
o If after all the tw0-percenters were given their guaranteed and
additional seats, and there are still unoccupied seats, those seats shall
be distributed to the remaining party-lists and those higher in rank in
the voting shall be prioritized until all the seats are occupied.
5) No. By a vote of 8-7, the Supreme Court continued to disallow major political parties (the likes
of UNIDO, LABAN, etc) from participating in the party-list elections.
● Although the ponencia (Justice Carpio) did point out that there is no prohibition either
from the Constitution or from RA 7941 against major political parties from participating
in the party-list elections as the word “party” was not qualified and that even the
framers of the Constitution in their deliberations deliberately allowed major political
parties to participate in the party-list elections provided that they establish a sectoral
wing which represents the marginalized (indirect participation), Justice Puno, in his
separate opinion, concurred by 7 other justices, explained that the will of the people
defeats the will of the framers of the Constitution precisely because it is the people who
ultimately ratified the Constitution – and the will of the people is that only the
marginalized sections of the country shall participate in the party-list elections. Hence,
major political parties cannot participate in the party-list elections, directly or indirectly.
6) Yes, the 3 seat limit rule is valid.
● This is one way to ensure that no one party shall dominate the party-list system.

Atong Paglaum v. LB: In determining who may participate in the coming 13 May 2013 and subsequent party-list
COMELEC elections, the COMELEC shall adhere to the following PARAMETERS:

1. Three different groups may participate in the party-list system:


(1) national parties or organizations,
(2) regional parties or organizations, and
(3) sectoral parties or organizations.
16
2. National parties or organizations and regional parties or organizations do not need to
organize along sectoral lines and do not need to represent any "marginalized and
underrepresented" sector.
3. Political parties can participate in party-list elections provided they register under the
party-list system and do not field candidates in legislative district elections. A political
party, whether major or not, that fields candidates in legislative district elections can
participate in party-list elections only through its sectoral wing that can separately
register under the party-list system. The sectoral wing is by itself an independent
sectoral party, and is linked to a political party through a coalition.
4. Sectoral parties or organizations may either be "marginalized and underrepresented" or
lacking in "well-defined political constituencies." It is enough that their principal
advocacy pertains to the special interest and concerns of their sector. The sectors that
are "marginalized and underrepresented" include labor, peasant, fisherfolk, urban poor,
indigenous cultural communities, handicapped, veterans, and overseas workers. The
sectors that lack "well-defined political constituencies" include professionals, the elderly,
women, and the youth.
5. A majority of the members of sectoral parties or organizations that represent the
"marginalized and underrepresented" must belong to the "marginalized and
underrepresented" sector they represent. Similarly, a majority of the members of
sectoral parties or organizations that lack "well-defined political constituencies" must
belong to the sector they represent. The nominees of sectoral parties or organizations
that represent the "marginalized and underrepresented," or that represent those who
lack "well-defined political constituencies," either must belong to their respective sectors,
or must have a track record of advocacy for their respective sectors. The nominees of
national and regional parties or organizations must be bona-fide members of such
parties or organizations.
6. National, regional, and sectoral parties or organizations shall not be disqualified if some
of their nominees are disqualified, provided that they have at least one nominee who
remains qualified.

SECTION 6

A. Residence Qualification

Romualdez-Marcos v. LB: Residence, for election purposes, is used synonymously with domicile. Domicile
Comelec includes the twin elements of “the fact of residing or physical presence in a fixed place”
and animus manendi, or the intention of returning there permanently.

Residence, in its ordinary conception, implies the factual relationship of an individual to a certain
place. It is the physical presence of a person in a given area, community or country.

If a person’s intent is to remain, it becomes his domicile; if his intent is to leave as soon as his
purpose is established it is residence.

Thus, it is perfectly normal for an individual to have different residences in various places.
However, a person can only have a single domicile, unless, for various reasons, he successfully
abandons his domicile in favor of another domicile of choice. It is the fact of residence, not a
statement in a certificate of candidacy which ought to be decisive in determining whether or not
an individual has satisfied the constitution’s residency qualification requirement. The Supreme
Court reiterated the following from Larena vs. Teves: First, a minor follows the domicile of his
parents. Second, domicile of origin is not easily lost.

To successfully effect a change of domicile, one must demonstrate:


1. an actual removal or an actual change of domicile;
2. a bona fide intention of abandoning the former place of residence and
establishing a new one; and
3. acts which correspond with the purpose.

In the absence of clear and positive proof based on these criteria, the residence of origin should
be deemed to continue.

AP: In this case, the Supreme Court held that the following facts did not constitute intent to
abandon her original domicile or at least proved that she had chosen her original domicile as her
present domicile:
1) having been a registered voter for several years in the past in San Juan, Manila;
2) having different residences in Metro Manila, Ilocos, etc.; and
3) Mrs. Marcos’ marriage to Mr. Marcos.

Aquino v. Comelec LB: The Constitution requires that a person seeking election to the House of Representatives
should be a resident of the district in which he seeks election for a period of not less than 1 year
prior to the elections. For election purposes, residence has always been understood as
synonymous with domicile.

Clearly, the place “where a party actually or constructively has his permanent home,” where he,
no matter where he may be found at any given time, eventually intends to return and remain,
i.e., his domicile, is that to which the Constitution refers when it speaks of residence for the
purposes of election law.

17
AP: Here, Aquino failed to prove that he had established not just residence but domicile of choice
in Makati. In his CoC for 1992 elections, he indicated that he was a resident of Concepcion, Tarlac
for 52 years; he was a registered voter of the same district; his birth certificate places
Concepcion, Tarlac as sir birthplace; and his assertion that he has transferred his domicile from
Tarlac to Makati is a bare assertion which is hardly supported by the facts in the case at bench.

Domino v. Comelec LB/AP: A person’s domicile, once established, is considered to continue and will not be
deemed lost until a new one is established. To successfully effect a change of domicile, one
must demonstrate an actual removal or an actual change of domicile; a bona fide intention of
abandoning the former place of residence and establishing a new one and definite acts which
correspond with the purpose.

The contract of lease of a house and lot does not adequately support a change of
domicile. The lease contract may be indicative of Domino’s intention to reside in Sarangani, but
it does not engender the kind of permanency required to prove abandonment of one’s original
domicile.

The mere absence of individual from his permanent residence, no matter how long,
without the intention to abandon it does not result in loss or change of domicile . Thus,
the date of the contract of lease of a house and lot in Sarangani cannot be used, in the absence
of other circumstances, as the reckoning period of the one-year residence requirement.

AP: Domino’s lack of intention to abandon his residence in Quezon City is strengthened by his act
of registering as voter in Quezon City. While voting is not conclusive of residence, it does give rise
to a strong presumption of residence especially in this case where Domino registered in his
former barangay.

B. Citizenship Qualification

Co v. House Electoral LB: Under Article IV, Section 1 of the Constitution, the following are citizens of the Philippines:
Tribunal (1) Those who are citizens of the Philippines at the time of the adoption of the Constitution;
(2) Those whose fathers or mothers are citizens of the Philippines;
(3) Those born before January 17, 1973, of Filipino mothers, who elect Philippine citizenship
upon reaching the age of majority; and
(4) Those who are naturalized in accordance with law.

The Court interprets Section 1, Paragraph 3 above as applying not only to those who elect
Philippine citizenship after February 2, 1987 but also to those who, having been born of Filipino
mothers, elected citizenship before that date. The provision in Paragraph 3 was intended to
correct an unfair position which discriminates against Filipino women.

AP: There is no dispute that the respondent’s mother was a natural born Filipina at the time of
her marriage. Crucial to this case is the issue of whether or not the respondent elected or chose
to be a Filipino citizen. Election becomes material because Section 2 of Article IV of the
Constitution accords natural born status to children born of Filipino mothers before January 17,
1973, if they elect citizenship upon reaching the age of majority.

To expect the respondent to have formally or in writing elected citizenship when he came of age
is to ask for the unnatural and unnecessary. The reason is obvious. He was already a citizen. Not
only was his mother a natural born citizen but his father had been naturalized when the
respondent was only nine (9) years old. He could not have divined when he came of age that in
1973 and 1987 the Constitution would be amended to require him to have filed a sworn
statement in 1969 electing citizenship in spite of his already having been a citizen since 1957. In
1969, election through a sworn statement would have been an unusual and unnecessary
procedure for one who had been a citizen since he was nine years old.

The filing of sworn statement or formal declaration is a requirement for those who still have to
elect citizenship. For those already Filipinos when the time to elect came up, there are acts of
deliberate choice which cannot be less binding. Entering a profession open only to Filipinos,
serving in public office where citizenship is a qualification, voting during election time, running for
public office, and other categorical acts of similar nature are themselves formal manifestations of
choice for these persons. An election of Philippine citizenship presupposes that the person electing
is an alien. Or his status is doubtful because he is a national of two countries. There is no doubt in
this case about Mr. Ong’s being a Filipino when he turned 21.

SECTION 7

A. Term and Tenure

Dimaporo v. Mitra, Jr LB: “Term” is different from “tenure”.


○ Term of office - prescribed by the Constitution, may not be extended or
shortened by the legislature.
○ Tenure - the period during which an officer actually holds office, may be

18
affected by circumstances within or beyond the power of the said officer.
■ It may be shorter than the tern or it may not exist at all. But these
situations will not change the duration of the term of office.

There are 4 grounds in Art. VI, by which the tenure of a Congressman may be shortened:
1. Section 13, Article VI: Forfeiture of his seat by holding any other office or
employment in the government or any subdivision, agency or instrumentality
thereof, including government-owned or controlled corporations or subsidiaries;
2. Section 16 (3): Expulsion as a disciplinary action for disorderly behavior;
3. Section 17: Disqualification as determined by resolution of the Electoral Tribunal in
an election contest; and,
4. Section 7, par. 2: Voluntary renunciation of office.
○ This list of grounds is not exclusive. It does not preclude the legislature from
prescribing other grounds.

The filing of a certificate of candidacy for another office constitutes an overt, concrete
act of voluntary renunciation of the elective office presently being held.

AP: Congressman Dimaporo filed a certificate of candidacy for Regional Governor of ARMM. Thus,
his removal from Congress is valid.

SECTION 10

A. Salaries

Philconsa v. Mathay In LB/AP: Length of time that has to elapse before an increase becomes effective

It is admitted that the purpose of the Section 10, Article VI of the Constitution is to place
"a legal bar to the legislators yielding to the natural temptation to increase their
salaries. Not that the power to provide for higher compensation is lacking, but with the
length of time that has to elapse before an increase becomes effective, there is a
deterrent factor to any such measure unless the need for it is clearly felt."

The use of the word "term" in the singular, when combined with the following phrase "all the
members of the Senate and of the House", underscores that in the application of Article VI,
Section 14, the fundamental consideration is that the terms of office of ALL MEMBERS
OF THE LEGISLATURE that enacted the measure (whether Senators or Representatives)
must have expired before the increase in compensation can become operative. Such
disregard of the separate houses, in favor of the whole, accords in turn with the fact that the
enactment of laws rests on the shoulders of the entire Legislative body; responsibility therefor is
not apportionable between the two chambers.

SECTION 11

A. Privilege of speech
and debate

Jimenez v. LB: Privilege of speech - Coverage


Cabangbang
The privilege speech covered by Section 11, Article VI of the Constitution refers to utterances
made by Congressmen in the performance of their official functions, such as speeches
delivered, statements made, or votes cast in the halls of Congress, while the same is in
session, as well as bills introduced in Congress, whether the same is in session or not,
and other acts performed by Congressmen, either in Congress or outside the premises
housing its offices, in the official discharge of their duties as members of Congress and
of Congressional Committees duly authorized to perform its functions as such, at the
time of the performance of the acts in question.

AP: Here, an open letter to the President of the Philippines, when Congress presumably
was not in session, and defendant Congressman caused said letter to be published in several
newspapers of general circulation in the Philippines, on or about said date. It is obvious that, in
thus causing the communication to be so published, he was not performing his official duty, either
as a member of Congress or as officer of any Committee thereof. Hence, said communication is
not absolutely privileged.

SECTION 14

A. Prohibitions

19
Puyat v. De Guzman, LB/AP: An intervention of Assemblyman in SEC proceeding falls within the ambit of the
Jr. prohibition contained in Section 14, Article VI of the Constitution.

Under those facts and circumstances, the Court are constrained to find that there has been an
indirect “appearance as counsel before an administrative body” and, in its opinion, that is a
circumvention of the Constitutional prohibition. The “intervention” was an afterthought to
enable him to appear actively in the proceedings in some other capacity. To believe the
avowed purpose, that is, to enable him eventually to vote and to be elected as Director in the
event of an unfavorable outcome of the SEC Case would be pure naivete. He would still appear as
counsel indirectly.

SECTION 16

A. Congressional Officers

Avelino v. Cuenco LB: The basis of determining the existence of a quorum in the Senate shall be the total
number of Senators who are in the country and within the coercive jurisdiction of the
Senate.

When the Constitution declares that a majority of "each House'' shall constitute a quorum, "the
House" does not mean "all" the members. Even a majority of all the members constitute "the
House". There is a difference between a majority of "all the members of the House" and a
majority of "the House", the latter requiring less number than the first. Therefore an absolute
majority (12) of all the members of the Senate less one (23), constitutes constitutional majority
of the Senate for the purpose of a quorum.

AP: Here, there was a quorum in the session of the Senate since 12 senators were present, and 1
senator being in the US.

Santiago v. Guingona LB/AP: While the Constitution is explicit on the manner of electing a Senate President
and a House Speaker, it is, however, dead silent on the manner of selecting the other
officers in both chambers of Congress.

All that the Charter says is that “[e]ach House shall choose such other officers as it may deem
necessary.” To our mind, the method of choosing who will be such other officers is merely a
derivative of the exercise of the prerogative conferred by the aforequoted constitutional provision.
Therefore, such method must be prescribed by the Senate itself, not by this Court.

Notably, the Rules of the Senate do not provide for the positions of majority and minority leaders.
Neither is there an open clause providing specifically for such offices and prescribing the manner
of creating them or of choosing the holders thereof.

In the absence of constitutional or statutory guidelines or specific rules, the Court is devoid of any
basis upon which to determine the legality of the acts of the Senate relative thereto. On grounds
of respect for the basic concept of separation of powers, courts may not intervene in the internal
affairs of the legislature; it is not within the province of courts to direct Congress how to do its
work.

AP: Here, absent any clear-cut guideline, in no way can it be said that illegality or irregularity
tainted Respondent Guingona’s assumption and exercise of the powers of the office of Senate
minority leader. Furthermore, no grave abuse of discretion has been shown to characterize any of
his specific acts as minority leader.

B. Internal discipline

Osmena v. Pendatun LB: The members of Congress may be questioned in Congress itself.

Section 15, Article VI of our Constitution provides that "for any speech or debate" in Congress,
the Senators or Members of the House of Representative "shall not be questioned in any other
place." The provision has always been understood to mean that although exempt from
prosecution or civil actions for their words uttered in Congress, the members of
Congress may, nevertheless, be questioned in Congress itself.

The determination of the acts which constitute disorderly behavior is within the full discretionary
authority of the House concerned, and the Court will not review such determination, the same
being a political question.

AP: Here, the Rules of the House which petitioner himself has invoked (Rule XVII, sec. 7),
recognize the House's power to hold a member responsible "for words spoken in debate.”

Defensor Santiago v. LB: The order of suspension provided in RA 3019 is distinct from the power of Congress
Sandigangbayan to discipline its own ranks.

Sec. 13 of RA 3019 (where it appears to be a ministerial duty of the court to issue the order of
suspension upon a determination of the validity of the criminal information filed before it) does
20
not state that the public officer should be suspended only in the office where he is
alleged to have committed the acts charged.

The order of suspension prescribed in RA 3019 is different from that of Section 16(3) of
Article VI of the 1987 Constitution because the former is preventive (not a penalty),
and the latter is punitive imposed by either House of Congress upon its members. RA
3019 does not exclude from its coverage the members of Congress. The doctrine of separation of
powers simply recognized that each of the 3 co-equal branches of government has exclusive
prerogatives and effectively prevents one branch from unduly intruding into the internal affairs of
another.

Pimentel, Jr. v. Senate LB: The Constitutional right of the Senate to promulgate its own rules of proceeding
Committee has been recognized and affirmed by this Court. Section 16(3), Article VI of the Philippine
Constitution states: "Each House shall determine the rules of its proceedings."

The only limitation to the power of Congress to promulgate its own rules is the
observance of quorum, voting, and publication when required. As long as these
requirements are complied with, the Court will not interfere with the right of Congress to amend
its own rules.

AP: In this case, the proceedings before the Senate Committee of the Whole affect only members
of the Senate since the proceedings involve the Senate’s exercise of its disciplinary power over
one of its members. Clearly, the Rules of the Senate Committee of the Whole are internal to the
Senate. However, Section 81, Rule 15 of the Rules of the Senate Committee of the Whole
provides:
o Sec. 81. EFFECTIVITY. These Rules shall be effective after publication in the Official
Gazette or in a newspaper of general circulation.

C. Journals

United States v. Pons LB: When the legislative journals show with certainty the time of adjournment of the
Legislature and are clear and unambiguous respecting the same, they are conclusive;
and extraneous evidence cannot be admitted to show a different date of adjournment.

The courts in the Philippine Islands are bound, judicially, to take notice of what the law is and, to
enable them to determine whether the legal requisites to the validity of a statute have been
complied with, it is their right, as well as their duty, to take notice of the legislative journals.

AP: Here, the legislative journal indicated that the statute was passed before midnight when the
legislature adjourned sine die. Thus, Pons’ argument that the legislature’s clock had been stopped
at midnight on the last day of session and that it was after midnight that the statute was passed
is without merit.

D. Enrolled Bill

Casco v. Gimenez LB: It is well settled that the enrolled bill is conclusive upon the courts as regards the
tenor of the measure passed by Congress and approved by the President.

If there has been any mistake in the printing of the bill before it was certified by the
officers of Congress and approved by the Executive — on which we cannot speculate,
without jeopardizing the principle of separation of powers and undermining one of the
cornerstones of our democratic system — the remedy is by amendment or curative
legislation, not by judicial decree.

AP: Here, Casco is claiming exemption from payment of 25% on the “urea formaldehyde”.
However, the enrolled bill provides “urea and formaldehyde” not “urea formaldehyde”. Thus,
Casco is not covered by the exemption.

Astorga v. Villegas LB/AP: ENROLLED BILL THEORY

That in the case of acts of the Philippine Commission or the Philippine Legislature , when there is
in existence a copy signed by the presiding officers and secretaries of said bodies, it
shall be conclusive proof of the provisions of such acts and of the due enactment
thereof” that is the very basis of the ENROLLED BILL THEORY.

This Court is merely asked to inquire whether the text of HB 9266 signed by the Chief Executive
was the same text passed by both Houses of Congress. Under the specific facts and
circumstances of this case, this Court can do this and resort to the Senate journal for the
purpose.

And since the journal discloses that substantial and lengthy amendments were introduced on the
floor and approved by the Senate but were not incorporated in the printed text sent to the
President and signed by him. And also since both the President of the Senate and the Chief
Executive withdrew their signatures therein.Thus, the court denied the petition for ruling
otherwise would be to sacrifice truth to fiction and bring about mischievous consequences not

21
intended by the law-making body.

E. Quorum

Datu Abas Kida v. LB/AP: The SUPERMAJORITY VOTE (2/3) requirement set forth in Sec. 1, Art.17 of RA
Senate No. 9054 is unconstitutional for violating the principle that Congress cannot pass
irrepealable laws.

Under our Constitution, each House of Congress has the power to approve bills by a mere
majority vote, provided there is quorum. In requiring all laws which amend RA No. 9054 to
comply with a higher voting requirement than the Constitution provides (2/3 vote), Congress,
which enacted RA No. 9054, clearly violated the principle which it cannot bind itself or its
successors by enacting irrepealable laws. To reiterate, the act of one legislature is not binding
upon, and cannot tie the hands of, future legislatures.

Section 1, Article XVII of RA 9054 erects a high vote threshold for each House of Congress to
surmount, effectively and unconstitutionally, taking RA 9054 beyond the reach of Congress’
amendatory powers. One Congress cannot limit or reduce the plenary legislative power of
succeeding Congresses by requiring a higher vote threshold than what the Constitution
requires to enact, amend or repeal laws. No law can be passed fixing such a higher vote
threshold because Congress has no power, by ordinary legislation, to amend the Constitution.

SECTION 17

A. The Electoral Tribunal

Robles v. House LB: The mere filing of the motion to withdraw protest on the remaining uncontested precincts,
Electoral Tribunal without any action on the part of respondent tribunal, does not by itself divest the tribunal of its
jurisdiction over the case. Jurisdiction, once acquired, is not lost upon the instance of the
parties but continues until the case is terminated.

AP: Here, the Court cannot agree with Robles’ contention that Santos’ ‘Motion to Withdraw
Protest on Unrevised Precincts’ effectively withdrew the precincts referred to therein from the
protest even before the Tribunal has acted thereon. Certainly, the Tribunal retains the authority
to grant or deny the Motion, and the withdrawal becomes effective only when the Motion is
granted. To hold otherwise would permit a party to deprive the Tribunal of jurisdiction already
acquired.

Therefore, the Tribunal retains the power and the authority to grant or deny Protestant’s Motion
to Withdraw, if only to insure that the Tribunal retains sufficient authority to see to it that the will
of the electorate is ascertained.

Abbas v. Senate LB: In the light of the Constitution, the Senate Electoral Tribunal cannot legally function as
Electoral Tribunal such, absent its entire membership of Senators and no amendment of its Rules can confer on
the three (3) SC Justices-Members alone the power of valid adjudication of a senatorial
election contest.

Every Member of the Tribunal may, as his conscience dictates, refrain from participating
in the resolution of a case where he sincerely feels that his personal interests or biases
would stand in the way of an objective and impartial judgment.

AP: Here, Petitoners filed a motion for disqualification or inhibition against all the 6 members of
the SET on the ground that all of them are interested parties in the election contest against the
22 candidates of LABAN Coalition who were proclaimed as Senators. Hence, the SET cannot
legally function as such.

Lazatin v. House LB: It is well within the power of the HRET to prescribe the period within which protests
Electoral Tribunal may be filed before it

The power of the HRET, as the sole judge of all contests relating to the election, returns and
qualifications of the Members of the House of Representatives, to promulgate rules and
regulations relative to matters within its jurisdiction, including the period for filing election
protests before it, is beyond dispute. Its rule-making power necessarily flows from the general
power granted it by the Constitution.

AP: Here, Lazatin’s reliance on Sec. 250 of the Omnibus Election Code, as far as contests
regarding the election, returns and qualifications of Members of the Batasang Pambansa is
concerned, had ceased to be effective under the 1987 Constitution is readily apparent. First, the
Batasang Pambansa has already been abolished and the legislative power is now vested in a
bicameral Congress. Second, the Constitution vests exclusive jurisdiction over all contests relating
to the election, returns and qualifications of the Members of the Senate and the House of
Representative in the respective Electoral Tribunals. The exclusive original jurisdiction of the
COMELEC is limited by constitutional fiat to election contests pertaining to election
regional, provincial and city offices and its appellate jurisdiction to those involving

22
municipal and barangay offices.

The 1987 Constitution, while lodging in the COMELEC exclusive original jurisdiction over all
contests relating to the elections, returns and qualifications of all elective regional, provincial and
city officials and appellate jurisdiction over contests relating to the election of municipal and
barangay, expressly makes the Electoral Tribunals of the Senate and the House of
Representatives the sole judge of all contests relating to the election, returns and qualifications of
their respective Members.

Bondoc v. Pineda LB: The HRET of the Senate and Congress were created by the Constitution as special tribunals to
be the SOLE judge of all contests relating to returns and qualifications of members of the
legislative houses, and as such, are independent bodies which must be permitted to select their
own employees, and to supervise and control them, without legislative interference. To be able
to exercise exclusive jurisdiction, the HRET must be independent. Its jurisdiction to
hear and decide congressional election contests is not to be shared with the Legislature
NOR the Courts.

AP: The independence of the HRET would be a myth and its proceedings a farce if the HR, or the
majority party therein, may shuffle and manipulate the political component of the Tribunal, to
serve the interests of the party in power.

The Resolution of HR removing Camasura from the Tribunal for disloyalty is a clear impairment of
the constitutional prerogative of the HRET, to be the sole judge of the election contest between
Pineda and Bondoc. To sanction such interference by the HR in the work of the HRET would
reduce the Tribunal to a mere tool for the aggrandizement of the party in power (LDP).

Disloyalty to party is not a valid cause for termination of membership in the HRET. The
Tribunal must be non-partisan, in that members discharge their functions with complete
detachment, impartiality and independence (especially from the political party they
belong to). Membership in the HRET may not be terminated except for just causes, such as, the
o expiration of congressional term,
o death,
o permanent disability,
o resignation from the political party he represents in the Tribunal,
o formal affiliation with another political party, or
o removal for other valid cause.

Guerrero v. Comelec LB: Once a winning candidate has been proclaimed, taken his oath, and assumed office
as a member of the House of Representatives, COMELEC’s jurisdiction over election
contests relating to his election, returns, and qualifications ENDS , and the HRET’s own
jurisdiction begins.

AP: Here, we find no grave abuse of discretion on the part of the COMELEC when it ruled that its
jurisdiction over the case had ceased with the assumption of office of Fariñas as Representative
for the 1st district of Ilocos Norte. While the COMELEC is vested with the power to declare valid or
invalid a certificate of candidacy, its refusal to exercise that power following the proclamation and
assumption of the position by Fariñas is a recognition of the jurisdictional boundaries separating
the COMELEC and the HRET.

Layug v. Comelec LB: HRET has no jurisdiction over the qualification of a party list representatives as it is
vested by law, specifically, the Party-List System Act, upon the COMELEC.

Section 6 of said Act states that the COMELEC may motu proprio or upon verified complaint of
any interested party, remove or cancel, after due notice and hearing, the registration of any
national, regional or sectoral party, organization or coalition.

AP: In this case, Buhay Party-List was entitled to two seats in the House that went to its first two
nominees, Mariano Michael DM. Velarde, Jr. and William Irwin C. Tieng. On the other hand,
Brother Mike, being the fifth nominee, did not get a seat and thus had not become a member of
the House of Representatives. Indubitably, the HRET has no jurisdiction over the issue of Brother
Mike's qualifications. Neither does the HRET have jurisdiction over the qualifications of Buhay
Party-List, as it is vested by law, specifically, the Party-List System Act, upon the COMELEC.

SECTION 18

A. Composition of the Commission on Appointments

Daza v. Singson LB: The House of Representatives has the authority to change its representation in the
Commission on Appointments to reflect at any time the changes that may transpire in
the political alignments of its membership.

It is understood that such changes must be permanent and do not include the temporary alliances
or factional divisions not involving severance of political loyalties or formal disaffiliation and
permanent shifts of allegiance from one political party to another.

AP: Here, the Commission’s reapportionments of seats is valid. The HR revised its representation
in the COA by withdrawing the seat occupied by Daza and giving this the newly-formed LDP. The
23
LDP has been in existence for more than 1 year now. It now has 157 members in the HR and 6
members in the Senate. Its titular head is no less than the President of the Philippines and its
President is Senator Gonzales, who took over recently from Speaker Mitra. It is true that there
have been, and there still are, some internal disagreements among its members, but these are to
be expected in any political organization, especially if it is democratic in structure. In fact, even
the monolithic Communist Party in a number of socialist states has undergone similar dissension,
and even upheavals. But it surely cannot be considered still temporary because of such discord.

Coseteng v. Mitra, Jr. LB/AP: The composition of the House membership in the Commission on Appointments
was based on proportional representation of the political parties in the House.

There are 160 members of the LDP in the House. They represent 79% of the House membership
(which may be rounded out to 80%). Eighty percent (80%) of 12 members in the Commission on
Appointments would equal 9.6 members, which may be rounded out to ten (10) members from
the LDP.

The remaining two seats were apportioned to the LP (respondent Lorna Verano-Yap) as the next
largest party in the Coalesced Majority and the KBL (respondent Roque Ablan) as the principal
opposition party in the House.

There is no doubt that this apportionment of the House membership in the Commission on
Appointments was done “on the basis of proportional representation of the political parties
therein.”

LB: To be able to claim proportional membership in the Commission on Appointments, a political


party should represent at least 8.4% of the House membership, i.e., it should have been able to
elect at least 17 congressmen or congresswomen.

AP: The other political parties or groups in the House, such as petitioner’s KAIBA (which is
presumably a member also of the Coalesced Majority), are bound by the majority’s choices. Even
if KAIBA were to be considered as an opposition party, its lone member (petitioner Coseteng)
represents only .4% or less than 1% of the House membership, hence, she is not entitled to one
of the 12 House seats in the Commission on Appointments.

Guingona, Jr. v. LB: The Constitution does not contemplate that the Commission on Appointments must
Gonzales necessarily include twelve (12) senators and twelve (12) members of the House of
Representatives. What the Constitution requires is that there be at least a majority of
the entire membership.

A political party must have at least two senators in the Senate to be able to have a
representatives in the Commission on Appointments, so that any number less than 2 will not
entitle such a party a membership in the Commission on Appointments. A political
party/coalition with a single senator in the Senate cannot constitutionally claims seat
in the Commission.

AP: Here, respondents’ claim to membership in the COA by nomination and election of the LDP
majority in the Senate as not in accordance with Section 18 of Article VI of the 1987 Constitution
and therefore violative of the same because it is not in compliance with the requirement that 12
senators shall be elected on the basis of proportional representation of the political parties
represented therein.

To disturb the resulting fractional membership of the political parties in the COA by adding
together two halves to make a whole is a breach of the rule on proportional representation
because it will give the LDP an added member in the Commission by utilizing the fractional
membership of the minority political party, who is deprived of half a representation.

The COA may perform its functions and transact its business even if only 10 senators are elected
thereto as long as a quorum exists.

SECTION 21

A. Legislative Investigation

Arnault v. Nazareno LB: Failure or refusal to attend a legitimate legislative investigation or contumacy of
the witness may be punished as legislative contempt. It may include imprisonment for the
duration of the session.

The questions that may be raised in a legislative investigation do not necessarily have
to be relevant to any pending legislation provided that they are relevant to the subject
matter of the investigation being conducted.

AP: Here, the question for the refusal to answer which the petitioner was held in contempt by the
Senate is pertinent to the matter under inquiry. Senate Resolution No. 8, the validity of which is
not challenged by the petitioner, requires the Special Committee, among other things, to

24
determine the parties responsible for the Buenavista and Tambobong estates deal, and it is
obvious that the name of the person to whom the witness gave the P440,000 involved in said
deal is pertinent to that determination — it is in fact the very thing sought to be determined. The
contention is not that the question is impertinent to the subject of the inquiry but that it has no
relation or materiality to any proposed legislation. We have already indicated that it is not
necessary for the legislative body to show that every question propounded to a witness is
material to any proposed or possible legislation; what is required is that it be pertinent to the
matter under inquiry.

LB: There is no limit as to time [with] the Senate's power to punish for contempt in
cases where that power may constitutionally be exerted xxx.” It was ruled therein that
had contempt been exercised by the House of Representatives, the contempt could be enforced
until the final adjournment of the last session of the said Congress.

Senate vs. HoR’s Power to Contempt

In Senate, there is no time limit in the exercise of its power to punish for contempt; on the other
hand, the House of Representatives, as it is not a continuing body, has a limit in the
exercise of its power to punish for contempt, which is on the final adjournment of its
last session.

Bengzon, Jr. v. Senate LB: Inquiries in aid of legislation


Blue Ribbon
Committee The power of both houses of Congress to conduct inquiries in aid of legislation is not,
absolute or unlimited. Thus, the investigation must be “in aid of legislation in
accordance with its duly published rules of procedure” and that “the rights of persons
appearing in or affected by such inquiries shall be respected”. It follows then that the
rights of persons under the Bill of Rights must be respected, including the right to due process
and the right not to be compelled to testify against one’s self.

AP: Here, the inquiry of the Committee was not in aid of legislation since there appears to be no
intended legislation involved. The purpose of the inquiry to be conducted is not related to a
purpose within the jurisdiction of Congress, it was conducted to find out whether or not the
relatives of Pres. Aquino, particularly Mr. Lopa had violated RA 3019 in connection with the
alleged sale of the 36 or 39 corporations belonging to Benjamin "Kokoy" Romualdez to the Lopa
Group.

Negros Oriental v. LB: Sangguniang Panlungsod of Dumaguete has ordinance-making power, may conduct
Sangguniang investigation similar to but not the same as the legislative investigations conducted by
Panglungsod the national legislative.

The difference lies in the lack of subpoena power and of the power to punish for
contempt on the part of the local legislative bodies. They may only invite resource
persons who are willing to supply information which may be relevant to the proposed
ordinance. The type of investigation which may be conducted by the Sangguniang Panlungsod
DOES NOT INCLUDE within its ambit an inquiry into any suspected violation by an electric
cooperative of the conditions of its electric franchise.

The contempt power of the legislature is, therefore, sui generis; local legislative bodies cannot
correctly claim to possess them for the same reasons that the national legislature does. The
power attaches not to the discharge of legislative functions per se but to the character of the
legislature as one of the three independent and coordinate branches of government. The
contempt power and the subpoena power partake of a judicial nature. They cannot be implied in
the grant of legislative power. Neither can they exist as mere incidents of the performance of
legislative functions. To allow local legislative bodies or administrative agencies to exercise these
powers without express statutory basis would run afoul of the doctrine of separation of powers.

AP: Here, the power to inquire into the efficiency of the service supplied by electric cooperatives
is within the franchising powers of the NEA under Sec. 43 of PD 269. In the exercise of this
power, the NEA may conduct hearings and investigations, issue subpoenas and invoke the aid of
the courts in case of disobedience to its subpoenas (Sec. 47 & Sec. 54, P.D. 269). Clearly, then,
the Sangguniang Panlungsod of Dumaguete cannot look into any suspected failure of NORECO II
to comply with the standards of electric service prescribed by law and in its franchise. The proper
recourse is to file a complaint with the NEA against NORECO II if there be sufficient basis
therefor.

Senate v. Ermita LB: Section 21 vs. Section 22 of Art 6 of the 1987 Constitution

To determine the validity of the provisions of EO 464, the SC sought to distinguish Section 21
from Section 22 of Art 6 of the 1987 Constitution.

The Congress’ power of inquiry is expressly recognized in Section 21 of Article VI of the


Constitution. Although there is no provision in the Constitution expressly investing either House of
Congress with power to make investigations and exact testimony to the end that it may exercise
its legislative functions advisedly and effectively, such power is so far incidental to the legislative
function as to be implied. In other words, the power of inquiry – with process to enforce it

25
– is an essential and appropriate auxiliary to the legislative function. A legislative body
cannot legislate wisely or effectively in the absence of information respecting the conditions which
the legislation is intended to affect or change; and where the legislative body does not itself
possess the requisite information – which is not infrequently true – recourse must be had to
others who do possess it.

Section 22 on the other hand provides for the Question Hour. The Question Hour is closely
related with the legislative power, and it is precisely as a complement to or a
supplement of the Legislative Inquiry. The appearance of the members of Cabinet would be
very, very essential not only in the application of check and balance but also, in effect, in aid of
legislation. Section 22 refers only to Question Hour, whereas, Section 21 would refer
specifically to inquiries in aid of legislation, under which anybody for that matter, may
be summoned and if he refuses, he can be held in contempt of the House. A distinction
was thus made between inquiries in aid of legislation and the question hour.
 While attendance was meant to be discretionary in the question hour, it was compulsory
in inquiries in aid of legislation.
 Sections 21 and 22, therefore, while closely related and complementary to each
other, should not be considered as pertaining to the same power of Congress.
 One specifically relates to the power to conduct inquiries in aid of legislation, the aim of
which is to elicit information that may be used for legislation, while the other pertains to
the power to conduct a question hour, the objective of which is to obtain information in
pursuit of Congress’ oversight function.
 Ultimately, the power of Congress to compel the appearance of executive officials under
Section 21 and the lack of it under Section 22 find their basis in the principle of
separation of powers.
While the executive branch is a co-equal branch of the legislature, it cannot frustrate the power of
Congress to legislate by refusing to comply with its demands for information.
When Congress exercises its power of inquiry, the only way for department heads to exempt
themselves therefrom is by a valid claim of privilege. They are not exempt by the mere fact that
they are department heads. Only one executive official may be exempted from this power
— the President on whom executive power is vested, hence, beyond the reach of
Congress except through the power of impeachment. It is based on her being the highest
official of the executive branch, and the due respect accorded to a co-equal branch of government
which is sanctioned by a long-standing custom. The requirement then to secure presidential
consent under Section 1, limited as it is only to appearances in the question hour, is valid on its
face. For under Section 22, Article VI of the Constitution, the appearance of department heads in
the question hour is discretionary on their part. Section 1 cannot, however, be applied to
appearances of department heads in inquiries in aid of legislation. Congress is not
bound in such instances to respect the refusal of the department head to appear in such
inquiry, unless a valid claim of privilege is subsequently made, either by the President
herself or by the Executive Secretary.
When Congress merely seeks to be informed on how department heads are implementing the
statutes which it has issued, its right to such information is not as imperative as that of the
President to whom, as Chief Executive, such department heads must give a report of their
performance as a matter of duty. In such instances, Section 22, in keeping with the
separation of powers, states that Congress may only request their appearance.
Nonetheless, when the inquiry in which Congress requires their appearance is ‘in aid of
legislation’ under Section 21, the appearance is mandatory for the same reasons stated
in Arnault.

Philcomsat Holdings v. LB: The SC noted the wide latitude given to the Houses of Congress in the conduct of
Senate legislative inquiries, and would not fault the Senate for approving the resolution on the
very same day that it was submitted , and that petitioners were invited as resource
persons at the inquiry, and as such, they do not have the constitutional right to counsel.

AP: Here, the petitioners charged the Senate with GADALEJ in approving Committee Resolution
312, and sought its nullification because it did not propose any piece of legislation and it had been
hastily approved by the senate, and that they were deprived of the right to counsel during the
conduct of the legislative investigation.

Neri v. Senate LB: The phrase ‘duly published rules of procedure’ requires the Senate of every
Congress to publish its rules of procedure governing inquiries in aid of legislation
because every Senate is distinct from the one before it or after it. Since Senatorial
elections are held every 3 years for 1/2 of the Senate membership, the composition of the Senate
also changes by the end of each term. Each Senate may thus enact a different set of rules
as it may deem fit.

The Senate is a continuing institution. However, in the conduct of its day-to-day business, the
Senate of each Congress acts separately and independently of the Senate of the Congress before
it. Due to the termination of the business of the Senate during the expiration of one (1) Congress,
all pending matters and proceedings, such as unpassed bills and even legislative investigations, of
the Senate are considered terminated upon the expiration of that Congress and it is merely
optional on the Senate of the succeeding Congress to take up such unfinished matters, not in the
same status, but as if presented for the first time. The termination of the Senate's business and
proceedings after the expiration of Congress was utilized by the Court in ruling that the Senate
needs to publish its rules for its legislative inquiries in each Congress.

AP: Here, not having published its Rules of Procedures, the subject hearing sin aid of legislation

26
conducted by the Senate, are therefore, procedurally infirm.

Balag v. Senate LB: Period of imprisonment under the inherent power of contempt

Facts: When the The period of imprisonment under the inherent power of contempt by the Senate during
petitioner attended the inquiries in aid of legislation should only last until the termination of the legislative
hearing Sen. Grace Poe inquiry under which the said power is invoked.
asked the petitioner if he
was the president of Termination of a legislative inquiry
Aegis Juris Fraternity
however, the petitioner
refused to answer and The legislative inquiry of the Senate terminates on 2 instances:
invoked his right to self- (1) upon the approval or disapproval of the Committee Report; and
incrimination. Sen. (2) upon the expiration of one (1) Congress.
Panfilo Lacson reminded
that it was just a “simple AP: Senate acted with GADALEJ. But the case was dismissed for being moot and academic.
question” to invoke self-
incrimination and warned
the petitioner that he
may be cited in
contempt, but the
petitioner still refused to
answer.

SECTION 24

Tolentino v. Secretary LB: It is not the law—but the revenue bill—which is required by the Constitution to
of Finance “originate exclusively” in the House of Representatives.

It is important to emphasize this, because a bill originating in the House may undergo such
extensive changes in the Senate that the result may be a rewriting of the whole. As a result of
the Senate action, a distinct bill may be produced. To insist that a revenue statute—and not only
the bill which initiated the legislative process culminating in the enactment of the law —must
substantially be the same as the House bill would be to deny the Senate’s power not only to
“concur with amendments” but also to “propose amendments.” It would be to violate the
coequality of legislative power of the two houses of Congress and in fact make the House superior
to the Senate.

AP: Here, Petitioners’ contention is that RA 7716 did not “originate exclusively” in the House of
Representatives as required by Art. VI, § 24 of the Constitution, because it is in fact the result of
the consolidation of two distinct bills, H. No. 11197 and S. No. 1630 is without merit.

SECTION 25

A. Riders

Garcia v. Mata LB: The constitution provides that “No Bill which may be enacted to law shall embrace more than
1 subject which shall be expressed in the title of the Bill”
 This provision nullifies and rendered inoperative any provision contained in the body of
an act that was not fairly included in the subject expressed in the title or not germane or
properly connected to that subject.
 This prohibition is intended to preclude the insertion of riders.
 RIDER
o A provision NOT GERMANE to the subject matter of the bill.

In determining whether a provision contained in an act is embraced in the subject and is properly
connected therewith, the subject to be considered is the one expressed in the title of the
act and when an act contains provisions which are clearly not embraced in the subject
of such act, as expressed in the title, such provisions are inoperative and without
effect. Also, it would be sufficient if the title expressed the general subject and all the provisions
of the statute are germane to that general subject.

AP: Sec 11 is a rider.


While RA 1600 appropriated money for the appropriation of the government for the fiscal year
1956-1957, Sec 11 refers to the fundamental governmental policy matters of the calling to active
duty and reversion to inactive status of the reserve officers of AFP

Thus, SEC 11 is considered as a non-appropriation item inserted in an appropriation measure


which is considered as a violation of the constitutional inhibition against “riders” to the general
appropriation act. It was indeed a new and completely unrelated provision attached to the
Appropriation Act.

B. Transfer of funds

Demetria v. Alba LB: The purpose and conditions for which funds may be transferred were specified, i.e.

27
transfer may be allowed for the purpose of augmenting an item and such transfer may
be made only if there are savings from another item in the appropriation of the
government branch or constitutional body.

AP: Paragraph 1 of Section 44 of Presidential Decree No. 1177 is hereby declared null and void
for being unconstitutional.

It empowers the President to indiscriminately transfer funds from one department, bureau, office
or agency of the Executive Department to any program, project or activity of any department,
bureau or office included in the General Appropriations Act or approved after its enactment,
without regard as to whether or not the funds to be transferred are actually savings in
the item from which the same are to be taken, or whether or not the transfer is for the
purpose of augmenting the item to which said transfer is to be made.

It does not only completely disregard the standards set in the fundamental law, thereby
amounting to an undue delegation of legislative powers, but likewise goes beyond the tenor
thereof. Indeed, such constitutional infirmities render the provision in question null and void.

Philconsa v. Enriquez LB/AP: The spending power also referred to as “the power of the purse,” belongs to
Congress, subject only to the veto power of the President.

The President may propose the budget, but still the final say on the matter of
appropriations is lodged in the Congress. The power of appropriation carries with it the
power to specify the project or activity to be funded under the appropriation law. It can be as
detailed and as broad as Congress wants it to be.

On the constitutionality of a Special Provision in the 1994 GAA which allows a member of the
Congress to realign his allocation for operation expenses to any other expense categoty, the SC
held that the members of Congress only determine the necessity of the realignment of
the savings in the allotments for their operating expenses because they are in the best
position to do so being knowledgeable of the savings available in some items of the
operating expenses and which items need augmentation.

However, it is the Senate President and the Speaker of the House of Representatives, as
the case may be, who shall approve the realignment. Before giving their stamp of approval,
these two officials will have to see to it that:
(1) the funds to be realigned or transferred are actually savings in the items of
expenditures from which the same are to be taken; and
(2) the transfer of realignment is for the purpose of augmenting the items of
expenditure to which said transfer or realignment is to be made. Hence, the
special provision adverted to is not unconstitutional.

However, the Special Provision, which allows the Chief of Staff to use savings to augment the
pension fund for the AFP being managed by the AFP Retirement and Separation Benefits System
is violative of Sections 25(5) and 29(1) of the Article VI of the Constitution.

While Section 25(5) allows as an exception the realignment of savings to augment


items in the general appropriations law for the executive branch, such right must and
can be exercised only by the President pursuant to a specific law.

Araullo vs. Aquino LB: Requisites of a valid transfer of appropriated funds

The transfer of appropriated funds, to be valid under Section 25(5), supra, must be made upon a
concurrence of the following requisites, namely:
1. There is a law authorizing the President, the President of the Senate, the Speaker of the
House of Representatives, the Chief Justice of the Supreme Court, and the heads of the
Constitutional Commissions to transfer funds within their respective offices;
2. The funds to be transferred are savings generated from the appropriations for their
respective offices; and
3. The purpose of the transfer is to augment an item in the general appropriations law for
their respective offices.

AP: Here, for the 1st element, the GAAs of 2011 and 2012 provisions carried a different phrase
("to augment any item in this Act"), and the effect was that it literally allowed the transfer of
funds from savings to augment any item in the GAAs even if the item belonged to an office
outside the Executive.

For the 2nd element, there were no savings from which funds could be sourced for the DAP.
Savings refer to portions or balances of any programmed appropriation in this Act free from any
obligation or encumbrance which are:
(i) still available after the completion or final discontinuance or
abandonment of the work, activity or purpose for which the
appropriation is authorized;
(ii) from appropriations balances arising from unpaid compensation and
related costs pertaining to vacant positions and leaves of absence
without pay; and
(iii) from appropriations balances realized from the implementation of
measures resulting in improved systems and efficiencies and thus
enabled agencies to meet and deliver the required or planned targets,

28
programs and services approved in this Act at a lesser cost.

For the 3rd element, no funds from savings could be transferred under the DAP to augment
deficient items not provided in the GAA. Also, cross-border augmentations from savings were
prohibited by the Constitution.

SECTION 26

A. Subject and title of bills

Tio v. Videogram LB: The Constitutional requirement that "every bill shall embrace only one subject
Regulatory Board which shall be expressed in the title thereof" is sufficiently complied with if the title be
comprehensive enough to include the general purpose which a statute seeks to achieve.

 It is not necessary that the title express each and every end that the statute wishes to
accomplish.
 The requirement is satisfied if all the parts of the statute are related, and are germane to the
subject matter expressed in the title, or as long as they are not inconsistent with or foreign
to the general subject and title.
 An act having a single general subject, indicated in the title, may contain any number of
provisions, no matter how diverse they may be, so long as they are not inconsistent with or
foreign to the general subject, and may be considered in furtherance of such subject by
providing for the method and means of carrying out the general object."
 The rule also is that the constitutional requirement as to the title of a bill should not be so
narrowly construed as to cripple or impede the power of legislation.
 It should be given practical rather than technical construction.

AP: Tested by the foregoing criteria, petitioner's contention that the tax provision of the DECREE
is a rider is without merit.

The foregoing provision is allied and germane to, and is reasonably necessary for the
accomplishment of, the general object of the DECREE, which is the regulation of the video
industry through the Videogram Regulatory Board as expressed in its title.
 The tax provision is not inconsistent with, nor foreign to that general subject and title.
 As a tool for regulation it is simply one of the regulatory and control mechanisms
scattered throughout the DECREE.
 The express purpose of the DECREE to include taxation of the video industry in order to
regulate and rationalize the uncontrolled distribution of videograms is evident from
Preambles 2 and 5.
 Those preambles explain the motives of the lawmaker in presenting the measure.
 The title of the DECREE, which is the creation of the Videogram Regulatory Board, is
comprehensive enough to include the purposes expressed in its Preamble and reasonably
covers all its provisions.
 It is unnecessary to express all those objectives in the title or that the latter be an index
to the body of the DECREE

Phil. Judges LB: Purposes of one subject-one title rule


Association v. Prado
The purposes of one subject-one title rule are:
1) To prevent hodge-podge or “log-rolling” legislation;
2) To prevent surprise or fraud upon the legislature by means of provisions in bills of which
the title gives no intimation, and which might therefore be overlooked and carelessly and
unintentionally adopted; and
3) To fairly apprise the people, through such publication of legislative proceedings as is
usually made, of the subject of legislation that is being considered, in order that they
may have opportunity of being heard thereon, by petition or otherwise, if they shall so
desire.

The title of the bill is not required to be an index to the body of the act, or to be comprehensive
as to cover every single detail of the measure. It has been held that if the title fairly indicates the
general subject, and reasonably covers all the provisions of the act, and is not calculated to
mislead the legislature or the people, there is sufficient compliance with the constitutional
requirement.To require every end and means necessary for the accomplishment of the general
objectives of the statute to be expressed in its title would not only be unreasonable but would
actually render legislation impossible.

AP: Here, the Title “An Act Creating the Philippine Postal Corporation, Defining Its Power,
Functions and Responsibilities, Providing for the Regulation of the Industry and for Other
Purposes Connected Therewith” was found to be sufficiently broad to cover the removal of the
franking privileges of the judiciary. However, the said provision was declared unconstitutional on
equal protection grounds.

Tobias v. Abalos LB: The Constitution does not require Congress to employ in the title of an enactment, language
of such precision as to mirror, fully index or catalogue all the contents and the minute details
therein. It suffices if the title should serve the purpose of the constitutional demand that
it inform the legislators, the persons interested in the subject of the bill and the public,

29
of the nature, scope and consequences of the proposed law and its operation.

AP: Thus, the creation of a separate congressional district for Mandaluyong is not a subject
separate and distinct from the subject of its conversion into a highly urbanized city but is a
natural and logical consequence of its conversion into a highly urbanized city. Verily, the title of
R.A. No. 7675, “An Act Converting the Municipality of Mandaluyong Into a Highly Urbanized City
of Mandaluyong” necessarily includes and contemplates the subject treated under Section 49
regarding the creation of a separate congressional district for Mandaluyong in compliance with
Art. 6, Sec. 5 (3) of the Constitution.

Philippine Constitution LB: The Constitutional requirement with respect to titles of statutes as sufficient to
Association v. Gimenez reflect their contents is satisfied if all parts of a law relate to the subject expressed in
its title, and it is not necessary that the title be a complete index of the content.

It should be reasonably construed so as not to interfere unduly with the enactment of necessary
legislation. It should be given a practical, rather than technical construction. It should be a
sufficient compliance with such requirement if the title expresses the general subject and all the
provisions of the statute are germane to that general subject. The provision of the
Constitution expressing the subject matter of an Act in its title is not a mere rule of
legislative procedure, directory to Congress, but it is mandatory. It is the duty of the Court
to declare void any statute not conforming to this constitutional provision.

AP: Here, under RA 3836, amending paragraph (1) of sec. 12, subsection (c) of CA 186, as
amended by RAs 660 and 3096, the retirement benefits are granted to members of the GSIS,
who have rendered at least 20 years of service regardless of age. This paragraph is related and
germane to the subject of CA 186.

On the other hand, the succeeding paragraph of RA 3836 refers to members of Congress and to
elective officers thereof who are not members of the GSIS. To provide retirement benefits,
therefore, for these officials, would relate to subject matter which is not germane to CA 186. In
other words, this portion of the amendment (re retirement benefits for Members of Congress and
elected officers, such as the Secretary and Sergeants-at-arms for each House) is not related in
any manner to the subject of CA 186 establishing the GSIS and which provides for both
retirement and insurance benefits to its members.

Insular Lumber LB: The purpose of the constitutional provision requiring unity of content and expression of the
Company v. Court of content in the title is to prevent duplicity of subject and surprise upon the legislators and the
Tax Appeals - public.

AP: Here, the title of RA 1435 is “An Act to Provide Means of Increasing the Highway Special
Fund.” It is contended that Sec. 5 creates an exemption and thus does not contribute to an
increase is alien to the subject of the law and is therefore unconstitutional. However, the SC ruled
that the provision for exemption comes under the general subject expressed in the title.

SECTION 27

A. Item veto

Commissioner v. Court LB: An “item” in a revenue bill does not refer to an entire section imposing a particular
of Tax Appeal kind of tax, but rather to the subject of the tax and the tax rate.

To construe the word “item” as referring to the whole section would tie the President’s hand in
choosing either to approve the whole section at the expense of also approving a provision therein
which he deems unacceptable or veto the entire section at the expense of foregoing the collection
of the kind of tax altogether.

The evil which was sought to be prevented in giving the President the power to disapprove items
in a revenue bill would be perpetrated rendering that power inutile.

AP: The presidential veto referred merely to the inclusion of hotels, motels and rest houses in the
20% caterer’s tax bracket but not to the whole section. The inclusion of hotels, motels and rest
houses in the 20% caterer’s tax bracket are “items.”

GONZALES V. LB: The Veto Power of the President is expressed in Sec 27 Article VI:
MACARAIG  1st paragraph: General veto power of the President and if exercised would result in the
veto of entire Bill as a general rule
 2nd paragraph: Item veto or line veto power which allows the President to veto over a
particular item or items in an appropriation, revenue or tariff bill.
o BUT the President may not veto less than all of an item of an Appropriation Bill
 Which means that “The power to disapprove any item in Appropriation Bill
does not grant the authority to veto a part of an item and to approve the
remaining portion of the same item”

“Item” ≠ “Provision” : The terms item and provision in budgetary legislation and

30
practice are concededly different.

o Item - refers to particulars, the details, the distinct and severable parts, it
is an indivisible sum of money dedicated to a stated purpose. It also means
specific appropriation of money, not some general provision of law.
o Provision in appropriation bill relate specifically to some “particular appropriation”
 As to the argument that when the President objects a provision of an appropriation bill, item
veto cannot be exercised instead the President should veto the entire bill
 SC ruled that this argument does not only disregard the basic principle that a distinct and
several part of the bill may be subject of a separate veto but also looks the Constitutional
mandate that any provision in GAA shall relate specifically to some particular appropriation
therein and that such provision shall be limited in its operation to the appropriation to which
it relates
 BUT, even if assuming that provisions are beyond the executive power to veto, Sec 55 and its
counterpart Sec 16 are not provisions in the budgetary sense of term because as mentioned
above, provision in appropriation bill relate specifically to some “particular appropriation”

AP: The challenged provision in this case falls short of this requirement because:
o The vetoed provision does not relate to any particular or distinctive appropriation.
Instead, they apply generally to all items disapproved or reduced by Congress in the
Appropriations bill.
o The disapproved or reduced items are nowhere to be found on the face of the bill
o The vetoed provision is more of an expression of Congressional policy in respect of
augmentation from savings rather than a budgetary appropriation

Thus, Sec 55 (FY ’89) and Sec 16 (FY’90) although labelled as provision are actually inappropriate
provisions that should be treated as items for the purpose of the President’s power.

Philconsa v. Enriquez LB: “Inappropriate provisions”

“Inappropriate provisions” are any provision which does not relate to any particular
item, or which extends in its operation beyond an item of appropriation, which can be
vetoed separately from an item.

Any provision blocking an administrative action in implementing a law or requiring legislative


approval for executive acts must be incorporated in a separate and substantive bill.

AP: Here, the Special Provision No. 2 on the “Use of Funds” in the appropriation for the
modernisation of the AFP, GAA of 1994, which requires prior approval of Congress for the release
of the corresponding modernisation funds was found unconstitutional since Special Provision No. 2
is an “inappropriate provision”. Hence, the President properly vetoed the same.

LB: GR: Under his general veto power, the President has to veto the entire bill, not merely parts
thereof.

EXC: the power given to the President to veto any particular item or items in a general
appropriations bill. In so doing, the President must veto the entire item.

AP: The President vetoed the entire paragraph one of the Special Provision of the item on debt
service, including the provisos that the appropriation authorized in said item “shall be used for
payment of the principal and interest of foreign and domestic indebtedness” and that “in no case
shall this fund be used to pay for the liabilities of the Central Bank Board of Liquidators.” These
provisos are germane to and have a direct connection with the item on debt service. Inherent in
the power of appropriation is the power to specify how the money shall be spent. The said
provisos, being appropriate provisions, cannot be vetoed separately. Hence the item veto of said
provisions is void.

Bengzon v. Drilon LB: The President cannot veto part of an item in an APPROPRIATION BILL while
approving the remaining portion of the item.

No President may veto the provisions of a law enacted thirty-five (35) years before his
or her term of office. Neither may the President set aside or reverse a final and
executory judgment of this Court through the exercise of the veto power.

AP: Here, Pres. Aquino vetoed appropriations intended for the adjustments of pensions of retired
justices under RA 910, as amended by RA 1797, which is not an item veto.

The veto impairs the power of the Chief Justice to augment other items in the Judiciary's
appropriation, in contravention of the constitutional provision on "fiscal autonomy."

SECTION 28

A. Uniform and Equitable

31
Tan v. del Rosario LB: The legislative intent to shift the income tax system towards the schedular approach in the
income taxation of individual taxpayers and to maintain a global treatment on corporations does
not violate the constitutional requirement that taxation shall be uniform and equitable.
UNIFORMITY OF TAXATION merely requires that all subjects or objects of taxation, similarly
situated, are treated alike both in privileges and liabilities. UNIFORMITY DOES NOT FOREFEND
CLASSIFICATION AS LONG AS:

(1) the standards that are used therefor are substantial and not arbitrary,
(2) the categorization is germane to achieve the legislative purpose,
(3) the law applies, all things being equal, to both present and future conditions,
and
(4) the classification applies equally well to all those belonging to the same class.

AP: Here, what may instead be perceived to be apparent from the amendatory law is the
legislative intent to increasingly shift the income tax system towards the schedular approach in
the income taxation of individual taxpayers and to maintain, by and large, the present global
treatment on taxable corporations. We certainly do not view this classification to be arbitrary and
inappropriate.

Commissioner v. LB: A tax is uniform when it operates with the same force and effect in every place
Lingayen Gulf Electric where the subject of it is found. Uniformity means that all property belonging to the
Power Co., Inc. same class shall be taxed alike.

The Legislature has the inherent power not only to select the subjects of taxation but to grant
exemptions. Tax exemptions have never been deemed violative of the equal protection clause.

AP: It is true that the private respondents municipal franchises were obtained under Act No. 667
of the Philippine Commission, but these original franchises have been replaced by a new
legislative franchise, i.e. R.A. No. 3843.
 As correctly held by the respondent court, the latter was granted subject to the terms
and conditions established in Act No. 3636, 3 as amended by C.A. No. 132.
 These conditions Identify the private respondent's power plant as falling within that class
of power plants created by Act No. 3636, as amended.
 The benefits of the tax reduction provided by law (Act No. 3636 as amended by C.A. No.
132 and R.A. No. 3843) apply to the respondent's power plant and others circumscribed
within this class.
 R.A-No. 3843 merely transferred the petitioner's power plant from that class provided for
in Act No. 667, as amended, to which it belonged until the approval of R.A- No. 3843,
and placed it within the class falling under Act No. 3636, as amended.
 Thus, it only effected the transfer of a taxable property from one class to another.

B. Exclusive Educational Use

Abra Valley College, LB: The exemption in favor of property used exclusively for charitable or educational
Inc. v. Aquino purposes is ‘not limited to property actually indispensable’ therefor, but extends to
facilities which are incidental to and reasonably necessary for the accomplishment of
said purposes.

TEST OF EXEMPTION
The test of exemption from taxation is the use of the property for purposes mentioned in the
Constitution.

AP: Here, the use of the school building or lot for commercial purposes is neither
contemplated by law, nor by jurisprudence. Thus, while the use of the second floor of the
main building in the case at bar for residential purposes of the Director and his family may find
justification under the concept of incidental use, which is complementary to the main or primary
purpose—educational,, the lease of the first floor thereof to the Northern Marketing Corporation
cannot by any stretch of the imagination be considered incidental to the purpose of education.
However, since only a portion is used for purposes of commerce, the Court said that it was only
fair that half of the assessed tax be returned to the school involved.

CIR v. Court of Appeals LB: “Educational Institution” or “Institution of Learning” - Meaning

The term “educational institution” or “institution of learning” has acquired a wellknown technical
meaning, of which the members of the Constitutional Commission are deemed cognizant. Under
the Education Act of 1982, such term refers to schools. The school system is synonymous with
formal education, which “refers to the hierarchically structured and chronologically graded
learnings organized and provided by the formal school system and for which certification is
required in order for the learner to progress through the grades or move to the higher levels.”

APP: The Court has examined the “Amended Articles of Incorporation” and “By-Laws” of the
YMCA, but found nothing in them that even hints that it is a school or an educational institution.

YMCA is exempt from the payment of property tax, but not income tax on the rentals
from its property. The bare allegation alone that it is a non-stock,non-profit
educational institution is insufficient to justify its exemption from the payment of
income tax.A claim of statutory exemption from taxation should be manifest, and unmistakable
from the language of the law on which it is based.

32
Lung Center v. Q. C. LB: Test to determine w whether an enterprise is a charitable institution

To determine whether an enterprise is a charitable institution the ff are the elements:


1) Statute creating the enterprise
2) Its corporate purpose
3) Its constitution and by-laws
4) The methods of administration
5) The nature of the actual work performed
6) The character of the services rendered
7) The indefiniteness of the beneficiaries
8) Use and occupation of the properties

AP: The Lung Center was organized for the welfare and benefit of the Filipino people principally to
help combat the high incidence of luncg and pulmonary diseases in the Philippines (Thus, anyone
could be subject of charity)

LB: As a general principle, a charitable institution does not lose its character as such
and its exemption from taxes simply because it derives income from paying patients or
receives subsidies from the government AS LONG as the money received is devoted and
used altogether to the charitable object which it is intended to achieve and no money
inures to the private benefit of the persons managing or operating the institution.

AP: BUT those portions of Lung Center’s real property that are leased to private entities are NOT
exempt from real property taxes as these are not actually, directly and exclusively used for
charitable purposes

Supported by PD 1823 provides that The Lung Center does not enjoy any property tax exemption
privileges for its real property as well as the buildings constructed therein. Sec 28 (3) covers
property taxes only.

SECTION 29

A. Expenditure of public funds

Pascual v. Secretary of LB: It is a general rule that the legislature is without power to appropriate public
Public Works revenues for anything but a public purpose.

The test of the constitutionality of a statute requiring the use of public funds is whether
the statute is designed to promote the public interests, as opposed to the furtherance
of the advantage of individuals, although such advantage to individuals might
incidentally serve the public.

AP: Here, the sum of P85,000 is appropriated by Congress for a feeder road running through a
private subdivision and over property owned by a private individual. Hence, the appropriation is
not for a public purpose. The subsequent donation of the road to the government did not validate
the law because the validity of a statute depends upon the power of Congress at the time of its
approval, and not upon events occurring or acts performed subsequently.

Gaston v. Republic LB: Revenues derived from taxes cannot be used for purely private purposes or for the
Planters Bank exclusive benefit of private persons. The stabilization fund is to be utilized for the
benefit of the entire sugar industry, "and all its components, stabilization of the
domestic market including the foreign market," the industry being of vital importance
to the country's economy and to national interest.

AP: The fact that fees were collected from sugar producers, planters and millers, and that those
funds were channeled to purchase the shares of stock in Republic Planters Bank does not convert
the funds into a trust fund for their benefit nor make them the beneficial owners of the shares.
The stabilization fees collected are in the nature of a tax, which is within the power of the State to
impose for the promotion of the sugar industry. The collections made accrue to a "Special Fund,"
a "Development and Stabilization Fund.

The stabilization fees in question are levied by the State upon sugar millers, planters and
producers for a special purpose — that of "financing the growth and development of the sugar
industry and all its components, stabilization of the domestic market including the foreign market
the fact that the State has taken possession of moneys pursuant to law is sufficient to constitute
them state funds, even though they are held for a special purpose. Having been levied for a
special purpose, the revenues collected are to be treated as a special fund, to be, in the language
of the statute, "administered in trust' for the purpose intended. Once the purpose has been
fulfilled or abandoned, the balance, if any, is to be transferred to the general funds of the
Government.

Guingona, Jr. v. LB: The sum authorized to be released must be determinate, or at least determinable.
Carague
AP: Here, the questioned laws are complete in all their essential terms and conditions and
sufficient standards are indicated therein. The legislative intention in R.A. No. 4860, as amended,

33
Section 31 of P.D. No. 1177 and P.D. No. 1967 is that the amount needed should be
automatically set aside in order to enable the Republic of the Philippines to pay the principal,
interest, taxes and other normal banking charges on the loans, credits or indebtedness incurred
as guaranteed by it when they shall become due without the need to enact a separate law
appropriating funds therefor as the need arises. The purpose of these laws is to enable the
government to make prompt payment and/ or advances for all loans to protect and maintain the
credit standing of the country. Although the subject presidential decrees do not state specific
amounts to be paid, necessitated by the very nature of the problem being addressed, the
amounts nevertheless are made certain by the legislative parameters provided in the decrees.

The Executive is not of unlimited discretion as to the amounts to be disbursed for debt servicing.
The mandate is to pay only the principal, interest, taxes and other normal banking charges on the
loans, credits or indebtedness, or on the bonds, debentures or security or other evidences of
indebtedness sold in international markets incurred by virtue of the law, as and when they shall
become due. No uncertainty arises in executive implementation as the limit will be the exact
amounts as shown by the books of the Treasury.

Petitioner- LB/AP: The coco-levy funds are in the nature of taxes and can only be used for public
Organizations v. purpose.
Executive Secretary
They were raised with the use of the police and taxing powers of the State for the benefit of the
coconut industry and its farmers in general. The COA reviewed the use of the funds. The Bureau
of Internal Revenue (BIR) treated them as public funds and the very laws governing coconut
levies recognize their public character. The Court has also recently declared that the coco-levy
funds are in the nature of taxes and can only be used for public purpose. Taxes are enforced
proportional contributions from persons and property, levied by the State by virtue of its
sovereignty for the support of the government and for all its public needs.

Here, the coco-levy funds were imposed pursuant to law, namely, R.A. 6260 and P.D. 276. The
funds were collected and managed by the PCA, an independent government corporation directly
under the President. And, as the respondent public officials pointed out, the pertinent laws used
the term levy, which means to tax, in describing the exaction.

Since taxes could be exacted only for a public purpose, they cannot be declared private
properties of individuals although such individuals fall within a distinct group of
persons.

The coco-levy funds are evidently special funds. The State collected stabilization fees from
sugar millers, planters, and producers for a special purpose: to finance the growth and
development of the sugar industry and all its components. The fees were levied for a
special purpose and, therefore, constituted special fund when collected. Its character as such fund
was made clear by the fact that they were deposited in the PNB (then a wholly owned
government bank) and not in the Philippine Treasury.

B. Special Fund

John Osmeña v. Oscar LB/AP: The Oil Price Stabilization Fund (OPSF) is a "Trust Account" which was
Orbos established "for the purpose of minimizing the frequent price changes brought about by
exchange rate adjustment and/or changes in world market prices of crude oil and
imported petroleum products."

It was established precisely to protect local consumers from the adverse consequences that such
frequent oil price adjustments may have upon the economy. Thus, the OPSF serves as a pocket,
as it were, into which a portion of the purchase price of oil and petroleum products paid by
consumers as well as some tax revenues are inputted and from which amounts are drawn from
time to time to reimburse oil companies, when appropriate situations arise, for increases in, as
well as underrecovery of, costs of crude importation.

Hence, it seems clear that while the funds collected may be referred to as taxes, they are
exacted in the exercise of the police power of the State. Moreover, that the OPSF is a
special fund is plain from the special treatment given it by E.O. 137.

It is segregated from the general fund; and while it is placed in what the law refers to as a "trust
liability account," the fund nonetheless remains subject to the scrutiny and review of the COA.
The Court is satisfied that these measures comply with the constitutional description of a "special
fund." Indeed, the practice is not without precedent.

Confederation of LB: Sec. 29 provides for TWO CLASSIFICATION OF APPROPRIATION MEASURES—


Coconut Farmers general and special appropriation.
Organizations of the
Philippines Inc. vs.
A GENERAL APPROPRIATION LAW is one passed annually to provide for the financial
Aquino
operations of the entire government during one fiscal period, whereas a SPECIAL
APPROPRIATION is designed for a specific purpose. The revenue collected for a special purpose
shall be treated as a special fund to be used exclusively for the stated purpose. This serves as a
deterrent for abuse in the disposition of special funds.

The coconut levy funds are special funds allocated for a specific purpose and can never

34
be used for purposes other than for the benefit of the coconut farmers or the
development of the coconut industry. Any attempt to appropriate the said funds for another
reason, no matter how noble or beneficial, would be struck down as unconstitutional.

AP: Here, the assailed issuances, however, did not create a new special fund. They were issued
pursuant to previous laws and jurisprudence which declared coconut levy funds such as the CCSF
and the CIDF as public funds for a special purpose. In fact, PD 1234 recognized that all funds
collected and accruing to the SAGF shall be considered automatically appropriated for purposes
authorized by law creating such fund.

Thus, E.O. No. 179 does not create a new special fund but merely reiterates that revenues arising
out of or in connection with the privatization of coconut levy funds shall be deposited in the SAGF.

SECTION 30

Fabian v. Desierto LB/AP: Sec. 27 of RA 6770 cannot validly authorize an appeal to this Court from
decisions of the Office of the Ombudsman in administrative disciplinary cases.

It consequently violates the proscription in Section 30, Article VI of the Constitution against a law
which increases the appellate jurisdiction of the Supreme Court. No countervailing argument has
been cogently presented to justify such disregard of the constitutional prohibition which was
intended to give the Supreme Court a measure of control over cases placed under its appellate
jurisdiction. Otherwise, the indiscriminate enactment of legislation enlarging its appellate
jurisdiction would unnecessarily burden the Court. As a consequence of our ratiocination that
Section 27 of RA 6770 should be struck down as unconstitutional, and in line with the regulatory
philosophy adopted in appeals from quasi-judicial agencies in the 1997 Revised Rules of Civil
Procedure, appeals from decisions of the Office of the Ombudsman in administrative disciplinary
cases should be taken to the Court of Appeals under the provisions of Rule 43.

SECTION 32

A. Initiative and Referendum

Garcia v. Comelec LB: The Constitution clearly includes not only ordinances but RESOLUTIONS as
appropriate subjects of a local initiative.

Sec. 32 of Art. 6 provides in luminous language: “The Congress shall, as early as possible,
provide for a system of initiative and referendum, and the exceptions therefrom, whereby the
people can directly propose and enact laws or approve or reject any act or law or part thereof
passed by the Congress, or local legislative body”.

An act includes a resolution. An act is defined as “an expression of will or purpose it may
denote something done, as a legislature, including not merely physical acts, but also decrees,
edicts, laws, judgments, resolves, awards, and determinations.

The constitutional command to include acts (i.e., resolutions) as appropriate subjects of initiative
was implemented by Congress when it enacted Republic Act No. 6735 entitled "An Act Providing
for a System of Initiative and Referendum and Appropriating Funds Therefor.” Thus, its section
3(a) expressly includes resolutions as subjects of initiatives on local legislations. There can hardly
be any doubt that when Congress enacted Republic Act No. 6735 it intended resolutions to be
proper subjects of local initiatives. The debates confirm this intent.

Also, Sec. 125 of the LGC provides that, “Any proposition or ordinance approved through the
system of initiative and referendum as herein provided shall not be repealed, modified or
amended by the sanggunian concerned within six (6) months from the date of the approval
thereof x x x.” Certainly, the inclusion of the word proposition is inconsistent with respondents’
thesis that only ordinances can be the subject of local initiatives.

Article VII The Executive Department

SECTION 1

A. Executive Power

Marcos v. Manglapus LB: Executive power is not limited to the power to enforce the laws, because the
president is head of state as well as head of government and whatever powers in here
in such positions pertain to the office unless the constitution itself withholds it.

Furthermore, the Constitution itself provides that the execution of the laws is only one of the

35
powers of the President. It also grants the President other powers that do not involve the
execution of any provision of law, e.g. his power over the country’s foreign relations.

On these premises, the Court held that although the 1987 Constitution imposes limitations on the
exercise of specific powers of the President, it maintains intact what is traditionally considered as
within the scope of “executive power.” Having sworn to defend and uphold the Constitution, the
president has the inherent obligation under the constitution to protect the people, promote their
welfare and advance the national interest.

AP: Here, the power involved (or exercised in prohibiting the Marcoses from returning to the
Philippines) is the President’s residual power to protect the general welfare of the people. More
particularly, this case calls for the exercise of the president’s powers as protector of the peace.
Thus, the President cannot be said to have acted arbitrarily and capriciously and whimsically in
determining that the return of the Marcoses poses a serious threat to the national interest and
welfare and in prohibiting their return.

Webb v. De Leon LB: Prosecution of crimes appertains to the executive department of government whose
principal power and responsibility is to see that our laws are faithfully executed. A
necessary component of this power to execute our laws is the right to prosecute their
violators.

The right to prosecute vests the prosecutor with a wide range of discretion — the discretion of
whether, what and whom to charge, the exercise of which depends on a smorgasbord of factors
which are best appreciated by prosecutors.

AP: The SC held that it is constitutionally permissible for Congress to enact R.A. No. 6981 vesting
in the Department of Justice the power to determine who can qualify as a witness in the program
and who shall be granted immunity from prosecution. Petitioner Webb’s challenge to the validity
of R.A. No. 6981 cannot therefore succeed.

Philconsa v. Enriquez LB/AP: Executive function under the Countrywide Development Fund involves implementation of
the priority projects specified in the law.

The authority given to the members of Congress is only to propose and identify projects
to be implemented by the President. Under Art. XLI of the GAA of 1994, the President must
perforce examine whether the proposals submitted by the members of Congress fall within the
specific items of expenditures for which the Fund was set up, and if qualified, he next determines
whether they are in line with other projects planned for the locality. Thereafter, if the proposed
projects qualify for funding under the fund, it is the president who shall implement them. In
short, the proposals and identifications made by the members of congress are merely
recommendatory.

There is less basis to complain when the President said that the expenditures shall be subject to
guidelines he will issue. Until the guidelines are issued, it cannot be determined whether they are
proper or inappropriate. The issuance of administrative guidelines on the use of public
funds authorized by Congress is simply an exercise by the President of his
constitutional duty to see that the laws are faithfully executed .
FAITHFUL EXECUTION CLAUSE

Under the faithful execution clause, the president has the power to take “necessary and
proper steps” to carry into execution the law. These steps are the ones to be embodied
in the guidelines.

Laurel v. Garcia LB: It is not for the President to convey valuable real property of the government on his
or her own sole will. Any such conveyance must be authorized and approved by a law
enacted by the Congress. It requires executive and legislative concurrence.

Whether or not the Roppongi and related properties will eventually be sold is a policy
determination where both the President and Congress must concur. Considering the properties'
importance and value, the laws on conversion and disposition of property of public dominion must
be faithfully followed.

AP: Here, the President issued EO 296 making the Roponggi Property available for sale to non-
Filipino citizens or entities. However, the property remains to be of public dominion and hence,
not available for sale.

B. Executive Immunity

Estrada v. Desierto LB: Non-sitting President does not enjoy immunity from suit, even for acts committed
during his tenure because the immunity of the president from suit is concurrent only
with his tenure and not his term.

Thereafter, no suit whatsoever shall lie for official acts done by him or by others pursuant to his
specific orders during his tenure. Only incumbent presidents are immune from suit or from being
brought to court during the period of their incumbency and tenure, but beyond their incumbency,
immunity is lost.

AP: Here, Estrada argues that his immunity from suit covers his entire term, and not only his
tenure. However, his impeachment was considered as functus officio due to the walkout of the
36
prosecutors and by the events that led to his loss of the presidency. To be sure, the debates in
the Constitutional Commission make it clear that when impeachment proceedings have become
moot due to the resignation of the President, the proper criminal and civil cases may already be
filed against him.

Also, the cases filed against Estrada are criminal in character. They involve plunder, bribery and
graft and corruption. By no stretch of the imagination can these crimes, especially plunder which
carries the death penalty, be covered by the allege mantle of immunity of a non-sitting president.
Estrada cannot cite any decision of this Court licensing the President to commit criminal acts and
wrapping him with post-tenure immunity from liability. It will be anomalous to hold that immunity
is an inoculation from liability for unlawful acts and omissions.

LB: The rule is that unlawful acts of public officials are not acts of the State and the
officer who acts illegally is not acting as such but stands in the same footing as any
other trespasser. Indeed, a critical reading of current literature on executive immunity will
reveal a judicial disinclination to expand the privilege especially when it impedes the search for
truth or impairs the vindication of a right.

David v. Arroyo LB: The President, during his tenure of office or actual incumbency, may not be sued in
any civil or criminal case, and there is no need to provide for it in the Constitution or
law.

It will degrade the dignity of the high office of the President, the Head of State, if he can be
dragged into court litigations while serving as such. Furthermore, it is important that he be freed
from any form of harassment, hindrance or distraction to enable him to fully attend to the
performance of his official duties and functions. Unlike the legislative and judicial branch, only one
constitutes the executive branch and anything which impairs his usefulness in the discharge of
the many great and important duties imposed upon him by the Constitution necessarily impairs
the operation of the Government. However, this does not mean that the President is not
accountable to anyone. Like any other official, he remains accountable to the people but he may
be removed from office only in the mode provided by law and that is by impeachment.

AP: Here, 3 out of 7 petitions impleaded PGMA as respondent during her incumbency as
President. As such, PGMA may not be sued.

Rodriguez v. Arroyo LB/AP: Former President Arroyo cannot use the presidential immunity from suit to shield herself
from judicial scrutiny that would assess whether, within the context of amparo proceedings, she
was responsible or accountable for the abduction of Rodriguez.

Given the intent of the 1987 Constitution to breathe life to the policy that a public office is a
public trust, A NON-SITTING PRESIDENT CANNOT CLAIM EXECUTIVE IMMUNITY FOR HIS
ALLEGED CRIMINAL ACTS COMMITTED WHILE A SITTING PRESIDENT. Also, because her
tenure or term is now over, she no longer enjoys immunity from suit. The President only
enjoys immunity from suit during his tenure. The intent of the framers is clear that the
immunity of the president from suit is concurrent only with his tenure and not his term. In the
case at bar, PGMA’s term ended already when the suit was filed by Rodriguez.
Term vs. Tenure

The TERM means the time during which the officer may claim to hold the office as of right, and
fixes the interval after which the several incumbents shall succeed one another. The TENURE
represents the term during which the incumbent actually holds office. The tenure may be shorter
than the term for reasons within or beyond the power of the incumbent.

SECTION 4

Macalintal v. Comelec LB/AP: Sec. 18.5 of RA 9189 (Overseas Absentee Voting Act of 2003) insofar as it grants
sweeping authority to COMELEC to proclaim all winning candidates is unconstitutional
as it is repugnant to Sec. 4, Art. 7 of the Constitution vesting in Congress the authority to
proclaim the winning candidates for the positions of president and vice-president.

Congress could not have allowed the COMELEC to usurp a power that constitutionally belongs to it
or, as aptly stated by petitioner, to encroach on the power of Congress to canvass the votes for
president and vice-president and the power to proclaim the winners for the said positions. The
provisions of the Constitution as the fundamental law of the land should be read as part
of The Overseas Absentee Voting Act of 2003 and hence, the canvassing of the votes
and the proclamation of the winning candidates for president and vice-president for the
entire nation must remain in the hands of Congress

Lopez v. Senate LB: The Congress may validly delegate the initial determination of the authenticity and
due execution of the certificates of canvass the votes for president and vice president
to a Joint Congressional Committee, composed of members of both Houses.

AP: Here, the creation of the Joint Committee does not constitute grave abuse and cannot be said
to have deprived petitioner and the other members of Congress of their congressional
prerogatives, because under the very Rules under attack, the decisions and final report of the
said Committee shall be subject to the approval of the joint session of both Houses of Congress,
voting separately.

37
Pimentel v. Joint LB: Even after Congress has adjourned its regular session, it may continue to perform
Committee this constitutional duty of canvassing the presidential and vice-presidential election
results without need of any call for a special session by the President.

The joint public session of both Houses of Congress convened by express directive of Sec. 4, Art.
7 of the Constitution to canvass the votes for and to proclaim the newly elected President and
Vice-President has not, and cannot, adjourn sine die until it has accomplished its
constitutionally mandated tasks. For only when a board of canvassers has completed its
functions is it rendered functus officio. Its membership may change, but it retains its
authority as a board until it has accomplished its purposes.

AP: Here, since the 12th Congress has not yet completed its non-legislative duty to canvass the
votes and proclaim the duly elected President and Vice-President, its existence as the National
Board of Canvassers, as well as that of the Joint Committee to which it referred the preliminary
tasks of authenticating and canvassing the certificates of canvass, has not become functus officio.
In sum, despite the adjournment sine die of Congress, there is no legal impediment to the Joint
Committee completing the tasks assigned to it and transmitting its report for the approval of the
joint public session of both Houses of Congress, which may reconvene without need of call by the
President to a special session.

Macalintal vs. LB: PET is not a separate and distinct entity from the Supreme Court, albeit it has
Presidential Electoral functions peculiar only to the Tribunal.
Tribunal
It is obvious that the PET was constituted in implementation of Section 4, Article VII of the
Constitution, and it faithfully complies – not unlawfully defies – the constitutional directive. The
adoption of a separate seal, as well as the change in the nomenclature of the Chief Justice and
the Associate Justices into Chairman and Members of the Tribunal, respectively, was designed
simply to highlight the singularity and exclusivity of the Tribunal’s functions as a special electoral
court.

The set up embodied in the Constitution and statutes characterizes the resolution of electoral
contests as essentially an exercise of judicial power xxx With the explicit provision in Art. 8, the
present Constitution has allocated to the SC, in conjunction with latter’s exercise of judicial power
inherent in all courts, the task of deciding presidential and vice-presidential election contests, with
full authority in the exercise thereof. The power wielded by PET is a derivative of the plenary
judicial power allocated to courts of law xxx On the whole, the Constitution draws a thin, but,
nevertheless, distinct line between the PET and the Supreme Court.

AP: Here, Macalintal questioned the creation of a purportedly “separate tribunal” with its own
chairman and seal complemented by a budget allocation for its own employees and staff. He also
pointed out that the PET’s power is quasi-judicial in nature, and that Sec. 12 of Art. 8 of the
Constitution prohibits the designation of the Members of the SC and of other courts to any
government agency performing quasi-judicial powers.

SECTION 8

Estrada v. Arroyo (353 LB: Elements Of Resignation


452)
The elements of resignation are:
(1) intent to resign; and
(2) the intent must be coupled by acts of relinquishment. The validity of a resignation is not
governed by any formal requirement as to form. It can be oral. It can be written. It can
be express. It can be implied. As long as the resignation is clear, it must be given legal
effect.

AP: Here, the Court hold that the resignation of the Estrada cannot be doubted as confirmed by
his leaving Malacañang. In the press release containing his final statement,
(1) he acknowledged the oath-taking of the respondent as President of the Republic albeit
with reservation about its legality;
(2) he emphasized he was leaving the Palace, the seat of the presidency, for the sake of
peace and in order to begin the healing process of our nation. He did not say he was
leaving the Palace due to any kind of inability and that he was going to re-assume the
presidency as soon as the disability disappears;
(3) he expressed his gratitude to the people for the opportunity to serve them. (without
doubt, he was referring to the past opportunity given him to serve the people as
President);
(4) he assured that he will not shirk from any future challenge that may come ahead in the
same service of our country; and
(5) he called on his supporters to join him in the promotion of a constructive national spirit
of reconciliation and solidarity.

Certainly, the national spirit of reconciliation and solidarity could not be attained if he did not give
up the presidency. The press release was petitioner’s valedictory, his final act of farewell. His
presidency is now in the past tense.

SECTION 13 PROHIBITIONS

38
Civil Liberties Union LB: The prohibition imposed on the President and his official family is therefore all-
vs. Executive Secretary embracing and covers both public and private office or employment.

The all-embracing prohibition imposed on the President and his official family are proof
of the intent of the 1987 Constitution to treat them as a class by itself and to impose
upon said class stricter prohibitions.

AP: EO 284 is unconstitutional. Executive Order No. 284 actually allows members of the cabinet,
their secretaries, and undersecretaries and any other appointive officials to hold multiple offices
or employment in direct contravention of the express mandate of Section 13, Article VII of the
1987 Constitution prohibiting them from doing so, unless otherwise provided in the 1987
Constitution itself.

SECTION 15 MIDNIGHT APPOINTMENTS

De Rama vs. CA LB: Prohibition on midnight appointments applies only to presidential appointments.
There is no law that prohibits local elective officials from making appointments during
the last days of his or her tenure.

AP: Here, De Rama sought to recall the appointments of 14 municipal employees on the ground
that said employees were midnight appointments of Mayor Abeja which is a local elective official
and not a presidential appointments. Hence, the prohibition on midnight appointments does not
apply.

De Castro v. JBC (317) LB: Prohibition on midnight appointments provided in Sec. 15 of Art. 7 does not extend
to appointments in judiciary.

The prohibition is confined only to appointments made in the Executive Department. The framers
did not need to extend the prohibition to appointments in the Judiciary, because their
establishment of the JBC and their subjecting the nomination and screening of candidates for
judicial positions to the unhurried and deliberate prior process of the JBC ensured that there
would no longer be midnight appointments to the Judiciary. The intervention of the JBC eliminates
the danger that appointments to the Judiciary can be made for the purpose of buying votes in a
coming presidential election, or of satisfying partisan considerations.

AP: Here, the compulsory retirement of CJ Puno occurs just days after the coming Presidential
elections on May 2010. Under Sec. 8 of Art. 8, vacancy shall be filled within 90 days from the
occurrence thereof from a list of at least 3 nominees prepared by JBC for every vacancy. They
argue that the PGMA cannot appoint the successor of CJ Puno because of the ban on midnight
appointments.

SECTION 16

A. POWER OF APPOINTMENT

Bermudez v. Torres LB: “Appointment”- Meaning

An “appointment” to a public office is the unequivocal act of designating or selecting by


one having the authority thereof of an individual to discharge and perform the duties
and functions of an office or trust. The appointment is deemed complete once the last act
required of the appointing authority has been complied with and its acceptance thereafter by the
appointee in order to render it effective. Appointment necessarily calls for an exercise of
discretion on the part of the appointing authority.

“All provincial and city prosecutors and their assistants shall be appointed by the
President upon the recommendation of the Secretary” – Meaning

The proper understanding of the provision “All provincial and city prosecutors and their assistants
shall be appointed by the President upon the recommendation of the Secretary” means that
appointment calls for discretion on the part of the appointing authority. The power to
appoint prosecutors is given to the President. The Secretary of Justice is under the
control of the President. Hence, the law must be read simply as allowing the Secretary
of Justice to advice the President.

AP: Here, the absence of recommendation of the Secretary of Justice to the President is not fatal
to the appointment of Quiaoit for Prosecutor since the phrase “upon recommendation of the
Secretary” should be interpreted to be a mere advise, exhortation or indorsement, which is
essentially persuasive in character and not binding or obligatory upon the party to
whom it is made. The President, being the head of the Executive Department, could be very
well disregard or do away with the action of the departments, bureaus or offices even in the
exercise of discretionary authority, and in so opting, he cannot be said as having acted beyond
the scope of his authority.

Sarmiento v. Mison LB: Four groups of officers whom the President shall appoint

39
There are four groups of officers whom the President shall appoint, to wit:
(1) The heads of the executive departments, ambassadors, other public ministers and
consuls, officers of the armed forces from the rank of colonel or naval captain, and other
officers whose appointments are vested in him in this Constitution;
(2) All other officers of the Government whose appointments are not otherwise provided for
by law;
(3) Those whom the President may be authorized by law to appoint; and
(4) Officers lower in rank whose appointments the Congress may by law vest in the
President alone.

The 1st group of officers is clearly appointed with the consent of the COA. Appointments of such
officers are initiated by nomination and, if the nomination is confirmed by the COA, the President
appoints.

Appointments to the 2nd and 3rd groups of officers can be made by the President without the
consent (confirmation) of the COA.

Only those enumerated in the 1st group that needs confirmation by the COA.

AP: Here, the appointment of the Commissioner of Customs need no confirmation since the office
is not one of those enumerated in the 1st group, nor is it specified elsewhere that such
appointment needs consent of the COA.

Bautista v. Salonga LB: The appointment of the Chairman of the CHR does not require confirmation from the
COA.

Since the position of Chairman of the Commission on Human Rights is not among the positions
mentioned in the first sentence of Sec. 16, Art. VII of the 1987 Constitution , appointments to
which are to be made with the confirmation of the Commission on Appointments, it follows that
the appointment by the President of the Chairman of the CHR is to be made without the review or
participation of the Commission on Appointments.

To be more precise, the appointment of the Chairman and Members of the Commission on Human
Rights is not specifically provided for in the Constitution itself, unlike the Chairmen and Members
of the CSC, the COMELEC and the Commission on Audit whose appointments are expressly vested
by the Constitution in the President with the consent of the COA.

AP: The President appoints the Chairman and Members of the Commission on Human Rights
pursuant to the second sentence in Section 16, Art. VII, that is, without the confirmation of the
Commission on Appointments because they are among the officers of government “whom he (the
President) may be authorized by law to appoint.” And Section 2(c), Executive Order No. 163,
authorizes the President to appoint the Chairman and Members of the Commission on Human
Rights.

Calderon v. Carale LB: The Congress may not expand the list of appointments needing confirmation.

AP: Here, RA 6715 which requires that the Chairman and Commissioners of NLRC be appointed
by the President with the confirmation of the COA was declared unconstitutional since the said
positions are not among the officers mentioned in the 1st sentence of Section 16, Article VII
whose appointments requires confirmation by the COA. Thus, the Congress exceeded its power to
make laws. As such, the said positions does not need COA’s confirmation. The NLRC Chairman
and Commissioners fall within the second sentence of Section 16, Article VII of the Constitution,
more specifically under the “third groups” of appointees, i.e. those whom the President may be
authorized by law to appoint.

Pimentel v. Ermita LB: Acting Appointments

The essence of an appointment in an acting capacity is its temporary nature. In case of vacancy
in an office occupied by an alter ego of the President, such as the office of a department
secretary, the President must necessarily appoint an alter ego of her choice as acting
secretary before the permanent appointee of her choice could assume office.

Congress, through a law, cannot impose on the President the obligation to appoint
automatically the undersecretary as her temporary alter ego. An alter ego, whether
temporary or permanent, holds a position of great trust and confidence.

Acting appointments are a way of temporarily filling important offices but, if abused, they can
also be a way of circumventing the need for confirmation by the COA.

AP: Here, the Senators, including the members of COA questioned the constitutionality of the
appointments issued by the President to respondents as Acting Secretaries of their respective
departments, and to prohibit them from performing the duties of Department Secretaries.
However, the Court find no abuse in this case since it is readily apparent from PGMA’s issuance of
ad interim appointments to respondents immediately upon the recess of Congress, way before the
lapse of 1 year.

The office of a Department Secretary may become vacant while Congress is in session. Since a

40
department secretary is the alter ego of the President, the acting appointee to the office must
necessarily have the President’s confidence. Thus, by the very nature of the office of a
department secretary, the President must appoint in an acting capacity a person of her choice
while the Congress is in session. That person may or may not be the permanent appointee, but
practical reasons may make it expedient that the acting appointee will also be the permanent
appointee.

LB: The law expressly allows the President to make such acting appointment. Section 17, Chapter
5, Title 1, Book 3 of EO 292 states that “the President may temporarily designate an officer
already in the government service or any other competent person to perform the functions of an
office in the executive branch.”

COA is an independent of the Senate, senators who are not members of the COA may
not act in their behalf.

AP: Here, the President made appointments of Acting Department Secretaries while Congress
was in session. The appointments were challenged on the grounds that: (1) the administrative
Code says, “in the absence of a Secretary, the Undersecretary performs his functions; (2)
appointments of acting secretaries need confirmation; (3) Respondent says that since the
petitioner-senators are not members of the COA, they have no standing to challenge the act of
the president.

B. Ad Interim Appointments

Matibag v. Benipayo LB: An ad interim appointment is a permanent appointment because it takes effect
immediately and can no longer be withdrawn by the President once the appointee has
qualified into office. The fact that it is subject to confirmation by the COA does not alter its
permanent character.

2 Grounds for Termination

The Constitution provides that ad interim appointment can be terminated for 2 grounds as
specified in the Constitution:
(1) disapproval by the Commission on Appointments; or
(2) until the next adjournment of Congress.

AP: Here, the President did in fact appoint permanent Commissioners to fill the vacancies in the
COMELEC, subject only to confirmation by the COA. Benipayo, Borra and Tuason were extended
permanent appointments during the recess of Congress. They were not appointed or designated
in a temporary or acting capacity. The ad interim appointments of Benipayo, Borra and Tuason
are expressly allowed by the Constitution which authorizes the President, during the recess of
Congress, to make appointments that take effect immediately.

Evidently, the exercise by the President in the instant case of her constitutional power to make ad
interim appointments prevented the occurrence of the very evil sought to be avoided by the
second paragraph of Section 16, Article VII of the Constitution. This power to make ad interim
appointments is lodged in the President to be exercised by her in her sound judgment. Under the
second paragraph of Section 16, Article VII of the Constitution, the President can choose either of
two modes in appointing officials who are subject to confirmation by the Commission on
Appointments. First, while Congress is in session, the President may nominate the prospective
appointee, and pending consent of the Commission on Appointments, the nominee cannot qualify
and assume office. Second, during the recess of Congress, the President may extend an ad
interim appointment which allows the appointee to immediately qualify and assume office.

SECTION 17

A. Control

Lacson- Magallanes v. LB: The Executive Secretary, acting by authority of the President, may reverse a
Pano decision made by the Director of Lands that had been affirmed by the Executive
Secretary of Agricultural and National Resources.

The President’s duty to execute the law is of constitutional origin. So, too, is his control of all
executive departments. Thus it is, that department heads are men of his confidence. His is the
power to appoint them; his, too, is the privilege to dismiss them at pleasure. Naturally, he
controls and directs their acts. Implicit then is his authority to go over, confirm, modify or reverse
the action taken by his department secretaries. In this context, it may not be said that the
President cannot rule on the correctness of a decision of a department secretary.

Control – Meaning

Control simply means “the power of an officer to alter or modify or nullify or set aside
what a subordinate officer had done in the performance of his duties and to substitute
the judgment of the former for that of the latter.” This unquestionably negates the assertion
that the President cannot undo an act of his department secretary.

AP: Here, particularly in reference to the decisions of the Director of Lands, as affirmed by the
41
Secretary of Agriculture and Natural Resources, the standard practice is to allow appeals from
such decisions to the Office of the President. This Court has recognized this practice in several
cases. In one, the decision of the Lands Director as approved by the Secretary was considered
superseded by that of the President’s appeal. In other cases, failure to pursue or resort to this
last remedy of appeal was considered a fatal defect, warranting dismissal of the case, for non-
exhaustion of all administrative remedies.

LB: The President is not expected to perform in person all the multifarious executive and
administrative functions. The Office of the Executive Secretary is an auxiliary unit which assists
the President.

AP: The rule which has thus gained recognition is that “under our constitutional setup the
Executive Secretary who acts for and in behalf and by authority of the President has an
undisputed jurisdiction to affirm, modify, or even reverse any order” that the Secretary of
Agriculture and Natural Resources, including the Director of Lands, may issue.

Ang-angco v. Castillo LB: President has no power to take direct action of removal of classified civil service
official.

Under Section 16(1) of the Civil Service Act of 1959 it is the Commissioner of Civil Service who
has original and exclusive jurisdiction to decide administrative cases of all officers and employees
in the classified service. The only limitation to this power is that the decision of the Commissioner
may be appealed to the Civil Service Board of Appeals, whose decision in such cases shall be
final.

The power of control of the President may extend to the power to investigate, suspend or
remove officers and employees who belong to the executive department if they are
presidential appointees or do not belong to the classified service, for such can be justified under
the principle that the power to remove is inherent in the power to appoint, BUT NOT WITH
REGARD TO THOSE OFFICERS OR EMPLOYEES WHO BELONG TO THE CLASSIFIED
SERVICE FOR AS TO THEM THAT INHERENT POWER CANNOT BE EXERCISED.

AP: The action taken by respondent Executive Secretary, even with the authority of the
President, in taking direct action by considering petitioner resigned with prejudice to
reinstatement in the same bureau in the administrative case of petitioner, without submitting the
same to the Commissioner of Civil Service, is contrary to law and should be set aside.

De Leon v. Carpio LB: An order of reinstatement directed by the Executive Secretary cannot be disobeyed
by the Director of NBI.

All executive departments, bureaus and offices are under the control of the President of the
Philippines. The President’s power of control is directly exercised by him over the
members of the Cabinet who, in turn and by his authority, control the bureaus and
other offices under their respective jurisdictions in the executive department. The
constitutional vesture of this power in the President is self-executing and does not require
statutory implementation, nor may its exercise be limited, much less withdrawn, by the
legislature.

AP: Here, there is no question that when he directed the respondent to reinstate the petitioners,
Secretary Ordoñez was acting in the regular discharge of his functions as an alter ego of the
President. His acts should therefore have been respected by the respondent Director of the
National Bureau of Investigation, which is in the Department of Justice under the direct control of
its Secretary. As a subordinate in this department, the respondent was bound to obey the
Secretary’s directives, which are presumptively the acts of the President of the Philippines.

LB: Control over Cabinet Members

The President has full control of all the members of his Cabinet and may appoint them
as he sees fit or shuffle them at pleasure, subject only to confirmation by the
Commission on Appointments, and replace them in his discretion. Once in place, they are
at all times under the disposition of the President as their immediate superior. Without minimizing
the importance of the heads of the various departments, their personality is in reality but the
projection of that of the President. Hence, their acts, performed and promulgated in the regular
course of business are, unless disapproved or reprobated by the Chief Executive, presumptively
the acts of the Chief Executive.

Blaquera v. Alcala LB: The President’s administrative order which regulates the grants and amounts of
incentives given to government employees and officers are within its power of control.

The President is the head of the government. Governmental power and authority are exercised
and implemented through him. His power includes the control over executive departments.

AP: Here, when President Ramos issued AO 29 limiting the amount of incentive benefits,
enjoining heads of government agencies from granting incentive benefits without prior approval
from him, and directing the refund of the excess over the prescribed amount, the President was
just exercising his power of control over executive departments.

The President issued subject Administrative Orders to regulate the grant of productivity incentive
benefits and to prevent discontentment, dissatisfaction and demoralization among government
42
personnel by committing limited resources of government for the equal payment of incentives and
awards. The President was only exercising his power of control by modifying the acts of the
respondents who granted incentive benefits to their employees without appropriate clearance
from the Office of the President, thereby resulting in the uneven distribution of government
resources. In the view of the President, respondents did a mistake which had to be corrected. In
so acting, the President exercised a constitutionally-protected prerogative.

Malaria Employees v. LB: The President’s power of control includes the power to reorganize its department.
Exec Sec.
The President has the authority to carry out a reorganization of the Department of Health under
the Constitution and statutory laws. This authority is an adjunct of his power of control under
Article VII, Sections 1 and 17 of the 1987 Constitution.

Reorganization- Meaning

Reorganization “involves the reduction of personnel, consolidation of offices, or


abolition thereof by reason of economy or redundancy of functions.” It alters the existing
structure of government offices or units therein, including the lines of control, authority and
responsibility between them. While the power to abolish an office is generally lodged with the
legislature, the authority of the President to reorganize the executive branch, which may include
such abolition, is permissible under our present laws.

The President must exercise good faith in carrying out the reorganization of any branch or agency
of the executive department. Reorganization is effected in good faith if it is for the purpose of
economy or to make bureaucracy more efficient.

AP: Here, President Estrada issued EO 102 which provided for structural changes and redirected
the functions and operations of the Department of Health pursuant to his power of control.
Because of EO 102, DOH issued Department Memo 157 which restructured the organization of
DOH.

Ocampo v. Enriquez LB: Faithful Execution Clause


(Marcos and the libingan
ng mga bayani case) Under the Faithful Execution Clause, the President has the power to take "necessary and
proper steps" to carry into execution the law. The mandate is self-executory by virtue of its
being inherently executive in nature and is intimately related to the other executive functions. It
is best construed as an imposed obligation, not a separate grant of power. The provision simply
underscores the rule of law and, corollarily, the cardinal principle that the President is not above
the laws but is obliged to obey and execute them.

AP: Here, consistent with President Duterte’s mandate user Sec. 17 of Art. 7, the burial of Marcos
at the LNMB does not contravene RA 289 and 10368, and the international human rights law cited
by petitioners.

LB: The presidential power of control over Executive Branch of Government is self-
executing provision of the Constitution and does not require statutory implementation,
nor may its exercise be limited, much less withdrawn, by the legislative.

AP: This is why President Duterte is not bound by the alleged 1992 Agreement between former
President Ramos and the Marcos family to have the remains of Marcos interred in Batac, Ilocos
Norte. As the incumbent President, he is free to amend, revoke or rescind political agreements
entered into by his predecessors, and to determine policies which he considers, based on
informed judgment and presumed wisdom, will be most effective in carrying out his mandate.

LB: Under the Administrative Code, the President has the power to reserve for public use and for
specific public purposes any of the lands of the public domain and that the reserved land shall
remain subject to the specific public purpose indicated until otherwise provided by law or
proclamation.

AP: At present, there is no law or executive issuance specifically excluding the land in which the
LNMB is located from the use it was originally intended by the past Presidents. The allotment of a
cemetery plot at the LNMB for Marcos as a former President and Commander-in-Chief, a
legislator, a Secretary of National Defense, a military personnel, a veteran, and a Medal of Valor
awardee, whether recognizing his contributions or simply his status as such, satisfies the public
use requirement.

SECTION 18

A. Commander-in-chief

IBP v. Zamora LB: The President may call out the armed forces EVEN THOUGH THERE IS NO
EMERGENCY EXISTS.

When the President calls the armed forces to prevent or suppress lawless violence, invasion or
rebellion, he necessarily exercises a discretionary power solely vested in his wisdom.

43
Power of the President to keep the peace is NOT LIMITED merely to exercising the
commander-in-chief powers in times of emergency or to leading the State against
external and internal threats to its existence.

The President is not only clothed with extraordinary powers in times of emergency, but is also
tasked with attending to the day-to-day problems of maintaining peace and order and ensuring
domestic tranquility in times when no foreign foe appears on the horizon.

AP: Here, the President called out the Marines to join PNP to conduct joint visibility patrols for the
purpose of crime prevention and suppression due to the alarming increase in violent crimes in
Metro Manila, like robberies, kidnappings and carnappings.

LB: Moreover, in the exercise of the power to suspend the privilege of the writ of habeas corpus
or to impose martial law, two conditions must concur: (1) there must be an actual invasion or
rebellion and, (2) public safety must require it. These conditions are not required in the case of
the power to call out the armed forces. The only criterion is that “whenever it becomes
necessary,” the President may call the armed forces “to prevent or suppress lawless
violence, invasion or rebellion.” The implication is that the President is given FULL
DISCRETION AND WIDE LATITUDE IN THE EXERCISE OF THE POWER to call as
compared to the two other powers.

Lacson v. Perez LB: The President has discretionary authority to declare a “state of rebellion.” The Court
may only look into the sufficiency of the factual basis for the exercise of the power. However, this
is no longer feasible because Proc. 38 has already been lifted.

AP: Here, PGMA issued Proc. 38 declaring that there was a state of rebellion in the NCR because
of the angry mob assaulting and attempting to break into Malacañang. She likewise issued
General Order No. 1 directing the AFP and the PNP to suppress the rebellion in the NCR.
Warrantless arrests of several alleged leaders and promoters of the “rebellion” were thereafter
effected. The argument that the declaration of a “state of rebellion” is violative of the doctrine of
separation of powers, being an encroachment on the domain of the judiciary which has the
constitutional prerogative to “determine or interpret” what took place on May 1, 2001, and that
the declaration of a state of rebellion cannot be an exception to the general rule on the allocation
of the governmental powers is without merit.

LB: The President as Commander-in-Chief has a vast intelligence network to gather


information, some of which may be classified as highly confidential or affecting the
security of the state. In the exercise of the power to call, on-the-spot decisions may be
imperatively necessary in emergency situations to avert great loss of human lives and mass
destruction of property.

AP: Petitioners cannot be arrested without the required judicial warrant for all acts committed in
relation to or in connection with the May 1, 2001 siege.

31. David v. Arroyo LB: “Sequence” Of Graduated Powers

Section 18, Article VII of the Constitution grants the President, as Commander-inChief, a
“sequence” of graduated powers. From the most to the least benign, these are:
(1) the calling-out power,
(2) the power to suspend the privilege of the writ of habeas corpus, and
(3) the power to declare Martial Law.

Criterion For The Exercise Of The Calling-out Power

The only criterion for the exercise of the calling-out power is that “ whenever it becomes
necessary,” the President may call the armed forces “to prevent or suppress lawless
violence, invasion or rebellion.”

AP: Considering the circumstances then prevailing, President Arroyo found it necessary to issue
PP 1017 declaring a state of national emergency. Owing to her Office’s vast intelligence network,
she is in the best position to determine the actual condition of the country.

LB: Under the calling-out power, the President may summon the armed forces to aid him in
suppressing lawless violence, invasion and rebellion. This involves ordinary police action. But
every act that goes beyond the President’s calling-out power is considered illegal or ultra vires.
For this reason, a President must be careful in the exercise of his powers. He cannot invoke a
greater power when he wishes to act under a lesser power. There lies the wisdom of our
Constitution, the greater the power, the greater are the limitations.

AP: Here, PP 1017 is CONSTITUTIONAL insofar as it constitutes a call by PGMA on the AFP to
prevent or suppress lawless violence. However, the provisions of PP 1017 commanding the AFP to
enforce laws not related to lawless violence, as well as decrees promulgated by the President, are
declared UNCONSTITUTIONAL. In addition, the provision in PP 1017 declaring national emergency
under Section 17, Article VII of the Constitution is CONSTITUTIONAL, but such declaration does
not authorize the President to take over privately-owned public utility or business affected with
public interest without prior legislation.

G.O. No. 5 is CONSTITUTIONAL since it provides a standard by which the AFP and the PNP should
implement PP 1017, i.e. whatever is “necessary and appropriate actions and measures to
44
suppress and prevent acts of lawless violence.” Considering that “acts of terrorism” have not yet
been defined and made punishable by the Legislature, such portion of G.O. No. 5 is declared
UNCONSTITUTIONAL. The warrantless arrest of Randolf S. David and Ronald Llamas; the
dispersal and warrantless arrest of the KMU and NAFLU-KMU members during their rallies, in the
absence of proof that these petitioners were committing acts constituting lawless violence,
invasion or rebellion and violating BP 880; the imposition of standards on media or any form of
prior restraint on the press, as well as the warrantless search of the Tribune offices and whimsical
seizure of its articles for publication and other materials, are declared UNCONSTITUTIONAL. Also,
Petitioners failed to show that President Arroyo’s exercise of the calling-out power, by issuing PP
1017, is totally bereft of factual basis.

LB/AP: As to the declaration of a “state of rebellion” vs. declaration of Martial Law

Here, President Arroyo’s declaration of a “STATE OF REBELLION” was merely an act declaring a
status or condition of public moment or interest. Such declaration is harmless, without legal
significance, and deemed not written. In these cases, PP 1017 is more than that. In declaring a
state of national emergency, President Arroyo did not only rely on Section 18, Article VII of the
Constitution, a provision calling on the AFP to prevent or suppress lawless violence, invasion or
rebellion. She also relied on Section 17, Article XII, a provision on the State’s extraordinary power
to take over privately-owned public utility and business affected with public interest. Indeed, PP
1017 calls for the exercise of an awesome power.

The declaration of MARTIAL LAW is a “warning to citizens that the military power has been
called upon by the executive to assist in the maintenance of law and order, and that, while the
emergency lasts, they must, upon pain of arrest and punishment, not commit any acts which will
in any way render more difficult the restoration of order and the enforcement of law.”

PP 1017 is not a declaration of Martial Law. It is no more than a call by the President to the
armed forces to prevent or suppress lawless violence. As such, it cannot be used to justify acts
that only under a valid declaration of Martial Law can be done. Its use for any other purpose is a
perversion of its nature and scope, and any act done contrary to its command is ultra vires.

B. Martial Law

Lagman v. Medialdea LB: Limits In The Exercise Of Power To Declare Martial Law
(duterte’s martial law on
mindanao case) The extraordinary powers of suspending the privilege of the writ of habeas corpus and/ or
declaring martial law may be exercised only when there is actual invasion or rebellion, and public
safety requires it. The 1987 Constitution imposed the following limits in the exercise of these
powers:
(1) a time limit of 60 days;
(2) review and possible revocation by Congress; and
(3) review and possible nullification by the Supreme Court.

A state of martial law is peculiar because the President, at such a time, exercises police power,
which is normally a function of the Legislature. In particular, the President exercises police power,
with the military’s assistance, to ensure public safety and in place of government agencies which
for the time being are unable to cope with the condition in a locality, which remains under the
control of the State.

These extraordinary powers are conferred by the Constitution with the President as Commander-
in-Chief; it therefore necessarily follows that the power and prerogative to determine whether the
situation warrants a mere exercise of the calling out power; or whether the situation demands
suspension of the privilege of the writ of habeas corpus; or whether it calls for the declaration of
martial law, also lies, at least initially, with the President. The power to choose, initially, which
among these extraordinary powers to wield in a given set of conditions is a judgment call on the
part of the President. As Commander-in-Chief, his powers are broad enough to include his
prerogative to address exigencies or threats that endanger the government, and the
very integrity of the State.

Even the recommendation of, or consultation with, the Secretary of National Defense,
or other high-ranking military officials, is not a condition for the President to declare
martial law. A plain reading of Section 18, Article VII of the Constitution shows that the
President's power to declare martial law is not subject to any condition except for the
requirements of actual invasion or rebellion and that public safety requires it. Besides, it would be
contrary to common sense if the decision of the President is made dependent on the
recommendation of his mere alter ego. Rightly so, it is only on the President and no other that
the exercise of the powers of the Commander-in-Chief under Section 18, Article VII of the
Constitution is bestowed.

“In an appropriate proceeding”- Meaning

“In an appropriate proceeding” does not refer to a petition for certiorari filed under Sec. 1 and 5
of Art. 8 of the Constitution. In fine, the phrase "in an appropriate proceeding" appearing on the
third paragraph of Section 18, Article VII refers to any action initiated by a citizen for the purpose
of questioning the sufficiency of the factual basis of the exercise of the Chief Executive's
emergency powers, as in these cases. It could be denominated as a complaint, a petition, or a
matter to be resolved by the Court.

Section 18, Article VII specifically grants authority to the Court to itself to determine the
45
sufficiency of the factual basis for the declaration of martial law and/or the suspension of the
privilege of the writ of habeas corpus, "namely (1) actual invasion or rebellion, and (2) public
safety requires the exercise of such power." Without the concurrence of the two conditions, the
President's declaration of martial law and/or suspension of the privilege of the writ of habeas
corpus must be struck down.

In determining the existence of rebellion, the President only needs to convince himself that there
is probable cause or evidence showing that more likely than not a rebellion was committed or is
being committed.

PARAMETERS FOR DETERMINING THE SUFFICIENCY OF FACTUAL BASIS:

l) actual rebellion or invasion;


2) public safety requires it; the first two requirements must concur; and
3) there is probable cause for the President to believe that there is actual rebellion or invasion.

AP: Here, for the 1st parameter, Pres. Duterte issued Proclamation 216 declaring a state of
martial law and suspending the privilege of the writ of habeas corpus in whole Mindanao because:
(1) series of violent acts committed by the Maute terrorist group such as the attack on the
military outpost in Butig, Lanao del Sur in February 2016, killing and wounding several soldiers,
and the mass jailbreak in Marawi City in August 2016, freeing their arrested comrades and other
detainees. Thus, the President deduced from the facts available to him that there was an armed
public uprising, the culpable purpose of which was to remove from the allegiance to the Philippine
Government a portion of its territory and to deprive the Chief Executive of any of his powers and
prerogatives, leading the President to believe that there was probable cause that the crime of
rebellion was and is being committed and that public safety requires the imposition of martial law
and suspension of the privilege of the writ of habeas corpus.

For the 2nd parameter, Public safety requires the declaration of martial law and the suspension o
f the privilege o f the writ o f habeas corpus in the whole o f Mindanao. In his Report, the
President noted that the acts of violence perpetrated by the ASG and the Maute Group were
directed not only against government forces or establishments but likewise against civilians and
their properties. Thus, the security of the entire Mindanao was compromised.

The overriding and paramount concern of martial law is the protection of the security of the
nation and the good and safety of the public. As Commander-in-Chief, the President receives
vital, relevant, classified, and live information which equip and assist him in making decisions.
The Court has no machinery or tool equal to that of the Commander-in-Chief to ably and properly
assess the ground conditions. The 1987 Constitution grants to the President, as Commander-in-
Chief, the discretion to determine the territorial coverage or application of martial law or
suspension of the privilege of the writ of habeas corpus.

Padilla v. Congress LB: The provision in Article VII, Section 18 of the 1987 Constitution requiring the Congress to
vote jointly in a joint session is specifically for the PURPOSE OF REVOCATION of the
President's proclamation of martial law and/or suspension of the privilege of the writ
of habeas corpus.

There is no constitutional provision governing concurrence by the Congress in the


President's proclamation of martial law and/or suspension of the privilege of the writ
of habeas corpus, and absent a specific mandate for the Congress to hold a joint
session in the event of concurrence, then whether or not to hold a joint session under
such circumstances is completely within the discretion of the Congress.

AP: Here, the Senate and House of Representatives already separately adopted resolutions
expressing support for President Duterte's Proclamation No. 216. Given the express support of
both Houses of the Congress for Proclamation No. 216, and their already evident lack of intent to
revoke the same, the provision in Article VII, Section 18 of the 1987 Constitution on revocation
did not even come into operation and, therefore, there is no obligation on the part of the
Congress to convene in joint session.

The Court highlights the particular circumstance herein that both Houses of Congress already
separately expressed support for President Duterte's Proclamation No. 216, so revocation was not
even a possibility and the provision on revocation under Article VII, Section 18 of the 1987
Constitution requiring the Congress to vote jointly in a joint session never came into operation. It
will be a completely different scenario if either of the Senate or the House of Representatives, or
if both Houses of the Congress, resolve/s to revoke the President's proclamation of martial law
and/or suspension of the privilege of the writ of habeas corpus, in which case, Article VII, Section
18 of the 1987 Constitution shall apply and the Congress must convene in joint session to vote
jointly on the revocation of the proclamation and/or suspension.

Lagman v. Pimentel LB: Congress has the power to extend and determine the period of martial law and the
suspension of the privilege of the writ of habeas corpus.

Section 18, Article VII is clear that the only limitations to the exercise of the congressional
authority to extend such proclamation or suspension are

(1) that the extension should be upon the President's initiative;


(2) that it should be grounded on the persistence of the invasion or rebellion and the

46
demands of public safety; and
(3) that it is subject to the Court's review of the sufficiency of its factual basis upon the
petition of any citizen.

Section 18, Article VII did not also fix the period of the extension of the proclamation
and suspension. However, it clearly gave the Congress the authority to decide on its
duration; thus, the provision states that that the extension shall be for a period to be
determined by the Congress." If it were the intention of the framers of the Constitution to limit
the extension to sixty (60) days, as petitioners in G.R. No. 235935 theorize, they would not have
expressly vested in the Congress the power to fix its duration.

AP: Here, Both Houses, in joint session adopted Resolution 4 extending the period of martial law
and suspension of the privilege of writ of habeas corpus in the entire Mindanao for 1 year, from
January 1 to December 31, 2018. Said Resolution 4 is Constitutional.

SECTION 19

A. EXECUTIVE CLEMENCY

Llamas v. Orbos LB: The President may grant executive clemency in administrative cases.

The Constitution does not distinguish between which cases executive clemency may be
exercised by the President, except for impeachment cases. If the President can grant
reprieves, commutations and pardons, and remit fines and forfeitures in criminal cases, with
much more reason can she grant executive clemency in administrative cases, which are clearly
less serious than criminal offenses.

No Limitation

President's executive clemency powers may not be limited in terms of coverage, except as
already provided in the Constitution, that is, "no pardon, amnesty, parole, or suspension of
sentence for violation of election laws, rules and regulations shall be granted by the President
without the favorable recommendation of the COMELEC".

If those already adjudged guilty criminally in court may be pardoned, those adjudged guilty
administratively should likewise be extended the same benefit. However, the President’s
power to grant executive clemency is limited to administrative cases in the executive
branch, and not in the judicial or legislative branches of government.

AP: Here, the President granted Governor Ocampo executive clemency from his 90-day
suspension in the administrative case filed by Petitioners for violating the Anti-Graft and Corrupt
Practices Act.

36. Monsanto v. LB: A public officer who has been granted an absolute pardon by the Chief executive
Factoran (Marcos in this case) is not entitled to reinstatement to her former position (Asst. City
Treasurer) without need of a new appointment. She must reapply and undergo the usual
procedure required for a new appointment.

AP: Here, Monsanto may apply for reappointment to the office which was forfeited by reason of
her conviction (Estafa thru Falsification of Public Documents). And in considering her qualifications
and suitability for the public post, the facts constituting her offense must be and should be
evaluated and taken into account to determine ultimately whether she can once again be
entrusted with public funds. Stated differently, the pardon granted to petitioner has resulted in
removing her disqualification from holding public employment but it cannot go beyond that. To
regain her former post as assistant city treasurer, she must reapply and undergo the usual
procedure required for a new appointment.

LB: Pardon – Meaning

Pardon is defined as “an act of grace, proceeding from the power entrusted with the
execution of the laws, which exempts the individual, on whom it is bestowed, from the
punishment the law inflicts for a crime he has committed. It is the private, though official
act of the executive magistrate, delivered to the individual for whose benefit it is intended, and
not communicated officially to the Court. x x x. A pardon is a deed, to the validity of which
delivery is essential, and delivery is not complete without acceptance.”

A pardon reaches both the punishment prescribed for the offense and the guilt of the offender;
and when the pardon is full, it releases the punishment and blots out of existence the guilt, so
that in the eye of the law the offender is as innocent as if he had never committed the offense. If
granted before conviction, it prevents any of the penalties and disabilities, consequent upon
conviction, from attaching; if granted after conviction, it removes the penalties and disabilities
and restores him to all his civil rights; it makes him, as it were, a new man, and gives him a new
credit and capacity.

While a pardon has generally been regarded as blotting out the existence of guilt so that in the

47
eye of the law the offender is as innocent as though he never committed the offense, it does not
operate for all purposes. The very essence of a pardon is forgiveness or remission of guilt. Pardon
implies guilt. It does not erase the fact of the commission of the crime and the conviction
thereof. It does not wash out the moral stain. It involves forgiveness and not
forgetfulness.

Garcia v. COA LB: Executive clemency –Effect

A person dismissed from work and acquitted from a crime based on his innocence is
entitled to the payment of back wages after having been reinstated pursuant to the
grant of executive clemency.

If the pardon is based on the innocence of the individual, it affirms this innocence and makes him
a new man and as innocent as if he had not been found guilty of the offense charged. When a
person is given pardon because he did not truly commit the offense, the pardon relieves the party
from all punitive consequences of his criminal act, thereby restoring to him his clean name, good
reputation and unstained character prior to the finding of guilt.

AP: Here, petitioner was found administratively liable for dishonesty and consequently dismissed
from the service. However, he was later acquitted by the trial court of the charge of qualified
theft based on the very same acts for which he was dismissed. The acquittal of petitioner by the
trial court was founded not on lack of proof beyond reasonable doubt but on the fact that
petitioner did not commit the offense imputed to him. Aside from finding him innocent of the
charge, the trial court commended petitioner for his concern and dedication as a public servant.
Verily, petitioner’s innocence is the primary reason behind the grant of executive clemency to
him, bolstered by the favorable recommendations for his reinstatement by the Ministry of
Transportation and Communications and the Civil Service Commission.

Petitioner’s automatic reinstatement to the government service entitles him to back wages. This
is meant to afford relief to petitioner who is innocent from the start and to make reparation for
what he has suffered as a result of his unjust dismissal from the service. To rule otherwise would
defeat the very intention of the executive clemency, i.e., to give justice to petitioner. Moreover,
the right to back wages is afforded to those who have been illegally dismissed and were thus
ordered reinstated or to those otherwise acquitted of the charges against them. There is no doubt
that petitioner’s case falls within the situations aforementioned to entitle him to back wages.

People v. Salle Jr. LB: The 1987 Constitution prohibits the grant of pardon, whether full or conditional, to
an accused during the pendency of his appeal from the judgment of conviction by the
trial court.

Any application for a pardon should not be acted upon, or the process toward its grant should not
begin, unless the appeal is withdrawn. Hence, before an appellant may be validly granted pardon,
he must first ask for the withdrawal of his appeal, i.e., the appealed conviction must first be
brought to finality.

Accordingly, the agencies or instrumentalities of the Government concerned must require proof
from the accused that he has not appealed from his conviction or that he has withdrawn his
appeal. Such proof may be in the form of a certification issued by the trial court or the appellate
court, as the case may be.

The acceptance of the pardon shall not operate as an abandonment or waiver of the appeal, and
the release of an accused by virtue of a pardon, commutation of sentence, or parole before the
withdrawal of an appeal shall render those responsible therefor administratively liable.
Accordingly, those in custody of the accused must not solely rely on the pardon as a basis for the
release of the accused from confinement.

AP: Here, considering that appellant Ricky Mengote has not filed a motion to withdraw his appeal
up to this date, the conditional pardon extended to him should not have been enforced.
Nonetheless, since he stands on the same footing as the accused-appellants in the Hinlo case, he
may be freed from the full force, impact, and effect of the rule herein pronounced subject to the
condition set forth below. This rule shall fully bind pardons extended after 31 January 1995 during
the pendency of the grantee’s appeal.

Drilon v. CA LB: The commutation of sentence given by the President is not appealable.

The "pardoning power" of the President (that is, to grant reprieves, commutations, and pardons,
remit fines and forfeitures) is final and unappealable so is commutation of sentence, in which the
Chief Executive reduces a sentence. It extinguishes criminal liability partially, and has the
effect of changing the penalty to a lesser one.

AP: Here, Ganzon served six years in the stockades of the military — no doubt as a result of his
conviction — but was released in 1978 and put under so-called house arrest (although then
President Marcos never apparently carried this out seriously as Ganzon was free apparently, to
move in and out of his residence). When President Marcos ordered Ganzon's release after six
years of imprisonment, then President Marcos, unavoidably commuted Ganzon's imprisonment to
six years (give or take a few days)." Ganzon's sentence having been commuted, he has therefore
served his sentence and he can no longer be reinvestigated, or be made to complete the service
of his sentence.

48
Vidal v. Comelec LB/AP: An absolute pardon given by the President has the effect of restoring all the
civil and political right of a public official, which naturally includes the right to seek
public elective office.

The 1987 Constitution, specifically Section 19 of Article VII and Section 5 of Article IX-C, provides
that the President of the Philippines possesses the power to grant pardons, along with other acts
of executive clemency;

Instances in which the President may not extend pardon:

The only instances in which the President may not extend pardon remain to be in:
(1) impeachment cases;
(2) cases that have not yet resulted in a final conviction; and
(3) cases involving violations of election laws, rules and regulations in which there was no
favorable recommendation coming from the COMELEC.

From both law and jurisprudence, the right to seek public elective office is unequivocally
considered as a political right.

Hence, the Court reiterates its earlier statement that the pardon granted to former President
Estrada admits no other interpretation other than to mean that, upon acceptance of the
pardon granted to him, he regained his FULL civil and political rights — including the
right to seek elective office. The statement “he is hereby restored to his civil and political
rights,” to the mind of the Court, is crystal clear — the pardon granted to former President
Estrada was absolute, meaning, it was not only unconditional, it was unrestricted in scope,
complete and plenary in character, as the term “political rights” adverted to has a settled
meaning in law and jurisprudence. Thus, Estrada is not disqualified to run for public office as
Mayor of Manila.

San Diego v. People LB: The Court is not allowed to impose a penalty which prohibits a pardon.

The exercise of the pardoning power is discretionary in the President and may not be controlled
by the legislature or reversed by the court, save only when it contravenes the limitations set forth
by the Constitution.

AP: Here, the proper penalty imposable for San Diego, the accountant of the Cooperative is, the
penalty of reclusion perpetua, and not reclusion perpetua for forty (40) years without pardon
because that would be a limitation on the part of the power of the Chief Executive.

SECTION 21

A. International Agreements

Commissioner v. John LB: International agreements entered into by the President do not require the
Gotamco and Sons concurrence of the Senate.

While treaties are required to be ratified by the Senate under the Constitution, less formal types
of international agreements may be entered into by the Chief Executive and become binding
without the concurrence of the legislative body.

AP: Here, the Host Agreement, which specifically exempts the WHO from indirect taxes, comes
within the latter category; it is a valid and binding international agreement even without the
concurrence of the Philippine Senate. The privileges and immunities granted to the WHO under
the Host Agreement have been recognized by the Court as legally binding on Philippine
authorities.

Bayan v. Exec Sec LB: Section 21, Article VII vs. Section 25, Article XVIII

Section 21, Article VII deals with treaties or international agreements in general, in which case,
the concurrence of at least 2/3 of all the Members of the Senate is required to make the subject
treaty, or international agreement, valid and binding on the part of the Philippines. This provision
lays down the general rule on treaties or international agreements and applies to any form of
treaty with a wide variety of subject matter, such as, but not limited to, extradition or tax treaties
or those economic in nature.

In contrast, Section 25, Article XVIII is a special provision that applies to treaties which
involve the presence of foreign military bases, troops or facilities in the Philippines. Under this
provision, the concurrence of the Senate is only one of the requisites to render compliance with
the constitutional requirements and to consider the agreement binding on the Philippines. Section
25, Article XVIII further requires that “foreign military bases, troops, or facilities” may be allowed
in the Philippines only by virtue of a treaty duly concurred in by the Senate, ratified by a majority
of the votes cast in a national referendum held for that purpose if so required by Congress, and
recognized as such by the other contracting state.

These constitutional provisions both embody phrases in the negative and thus, are deemed

49
prohibitory in mandate and character. In particular, Section 21 opens with the clause “No treaty,”
and Section 25 contains the phrase “shall not be allowed.” Additionally, in both instances, the
concurrence of the Senate is indispensable to render the treaty or international agreement valid
and effective.

As to the matter of voting, Section 21, Article VII particularly requires that a treaty or
international agreement, to be valid and effective, must be concurred in by at least two-thirds of
all the members of the Senate. On the other hand, Section 25, Article XVIII simply provides that
the treaty be “duly concurred in by the Senate”

AP: Undoubtedly, Section 25, Article XVIII, which specifically deals with treaties involving foreign
military bases, troops, or facilities, should apply in the instant case. To a certain extent and in a
limited sense, however, the provisions of Section 21, Article VII will find applicability with regard
to the issue and for the sole purpose of determining the number of votes required to obtain the
valid concurrence of the Senate, as will be further discussed hereunder.

Applying the foregoing constitutional provisions, a two-thirds vote of all the members of the
Senate is clearly required so that the concurrence contemplated by law may be validly obtained
and deemed present. While it is true that Section 25, Article XVIII requires, among other things,
that the treaty—the VFA, in the instant case—be “duly concurred in by the Senate,” it is very true
however that said provision must be related and viewed in light of the clear mandate embodied in
Section 21, Article VII, which in more specific terms, requires that the concurrence of a treaty, or
international agreement, be made by a two-thirds vote of all the members of the Senate . Indeed,
Section 25, Article XVIII must not be treated in isolation to Section 21, Article VII.

Note: Treaty vs. Executive Agreement

This Court is of the firm view that the phrase “recognized as a treaty” means that the other
contracting party accepts or acknowledges the agreement as a treaty. To require the other
contracting state, the United States of America in this case, to submit the VFA to the United
States Senate for concurrence pursuant to its Constitution, is to accord strict meaning to the
phrase.

Moreover, it is inconsequential whether the United States treats the VFA only as an executive
agreement because, under international law, an executive agreement is as binding as a
treaty. To be sure, as long as the VFA possesses the elements of an agreement under
international law, the said agreement is to be taken equally as a treaty.

Abaya v. Ebdane LB: An exchange of notes is considered a form of an executive agreement, which
becomes binding through executive action without the need of a vote by the Senate or
Congress.

Agreements concluded by the President which fall short of treaties are commonly referred to as
executive agreements and are no less common in our scheme of government than are the more
formal instruments—treaties and conventions. They sometimes take the form of exchange of
notes and at other times that of more formal documents denominated “agreements” or
“protocol.”

AP: Here, the Loan Agreement taken in conjunction with the exchange of notes between Japan
and Philippines is an executive agreement.

Saguisag v. Ochoa LB: EDCA (Enhanced Defense Cooperation Agreement) is an executive agreement which
does not need concurrence of the Senate.

As the sole organ of our foreign relations and the constitutionally assigned chief architect of our
foreign policy, the President is vested with the exclusive power to conduct and manage the
country’s interface with other states and governments. Being the principal representative of the
Philippines, the Chief Executive speaks and listens for the nation; initiates, maintains, and
develops diplomatic relations with other states and governments; negotiates and enters into
international agreements; promotes trade, investments, tourism and other economic relations;
and settles international disputes with other states. This constitutional mandate emanates from
the inherent power of the President to enter into agreements with other states, including the
prerogative to conclude binding executive agreements that do not require further Senate
concurrence. The existence of this presidential power is so well-entrenched that Section 5(2)(a),
Article VIII of the Constitution, even provides for a check on its exercise.

AP: Here, the admission and presence of U.S. military and civilian personnel in Philippine territory
are already allowed under the VFA, the treaty supposedly being implemented by EDCA. What
EDCA has effectively done, in fact, is merely provide the mechanism to identify the locations in
which U.S. personnel may perform allowed activities pursuant to the VFA. As the implementing
agreement, it regulates and limits the presence of U.S. personnel in the country.

Article VIII The Judicial Department

SECTION 1

Santiago v. Bautista LB: (1) Judicial power is defined as:

50
1. Authority to determine the rights of persons or property.
(grade 6 pupil, was 2. Authority vested in some court, officer or persons to hear and determine when the
adjudged 3rd honor. 2 rights of persons or property or the propriety of doing an act is the subject
days before graduation, matter of adjudication.
his parents sought the 3. The power exercised by courts in hearing and determining cases before them.
invalidation of the 4. The construction of laws and the adjudication of legal rights.
ranking of the honor
students through (2) Judicial function is an act performed by virtue of judicial powers.
Certiorari against the  The exercise of judicial function is the doing of something in the nature of the action of
committee on rating the court.
honors. Respondent filed  It may be said that the exercise of judicial function is to determine what the law is ,
a MTD claiming that and what the legal rights of parties are, with respect to a matter in controversy.
Committee is not a
tribunal, nor board, (3) Test to determine whether a tribunal or board exercises judicial functions: (S-P-J)
exercising judicial 1. There must be specific controversy involving rights of persons brought before a
functions, under RULE tribunal for hearing and determination.
65, certiorari is a remedy 2. That the tribunal must have the power and authority to pronounce judgment
against judicial function.) and render a decision.
3. The tribunal must pertain to that branch of the sovereign which belongs to the
judiciary (or at least the not the legislative nor the executive)

AP: The Committee for Rating Honor Students are neither judicial nor quasi-judicial
bodies in the performance of its assigned task.

It is necessary that there be a LAW that gives rise to some specific rights of persons or property
under which adverse claims to such rights are made, and the controversy ensuring there from is
brought in turn, to the tribunal or board clothed with power and authority to determine.

The courts cannot exercise judicial power when there is no applicable law.

In this case, an award of honors to a student by a board of teachers may not be reversed
by a court where the awards are governed by no applicable law.

Disposition: Dismissed

Echegaray v. Secretary LB: The rule on finality of judgment cannot divest the Supreme Court of its jurisdiction
(rape of 10 yr old to execute and enforce the same judgment—the finality of a judgment does not mean that
daughter, sentenced with the Court has lost all its powers over the case.
death penalty)
- By the finality of the judgment, what the court loses is its jurisdiction to amend, modify
or alter the same. Even after the judgment has become final the court retains its
jurisdiction to execute and enforce it.

- There is a difference between the jurisdiction of the court to execute its judgment and
its jurisdiction to amend, modify or alter the same.

j. to execute its judgment j. to amend, modify or alter the


same
continues even after the judgment has terminates when the judgment
become final for the purpose of becomes final. For after the judgment
enforcement of judgment has become final facts and
circumstances may transpire which
can render the execution unjust or
impossible.

- The particulars of the execution itself, which are certainly not always included in the
judgment and writ of execution are absolutely under the control of the judicial authority,
while the executive has no power over the person of the convict except to provide for
carrying out of the penalty and to pardon.

AP: In this case, the Supreme Court exercised its judicial power when it granted a TRO, which
delayed the execution of the death penalty upon petitioner in order for the Court to determine
whether Congress was set to repeal the death penalty or the petitioner would be granted
executive clemency.

LB: (2) The power to control the execution of its decision is an essential aspect of
jurisdiction. It cannot be the subject of substantial subtraction for our Constitution
vests the entirety of judicial power in one Supreme Court and in such lower courts as
may be established by law.

- the most important part of a litigation, whether civil or criminal, is the process of
execution of decisions where supervening events may change the circumstance of the
parties and compel courts to intervene and adjust the rights of the litigants to prevent
unfairness.
o It is because of these unforseen, supervening contingencies that courts have
been conceded the inherent and necessary power of control of its processes and
orders to make them conformable to law and justice.
o For this purpose, Section 6 of Rule 135 provides that “when by law jurisdiction
is conferred on a court or judicial officer, all auxiliary writs, processes and other
means necessary to carry it into effect may be employed by such court or
51
officer and if the procedure to be followed in the exercise of such jurisdiction is
not specifically pointed out by law or by these rules, any suitable process or
mode of proceeding may be adopted which appears conformable to the spirit of
said law or rules.”

AP: What the Court restrained temporarily is the execution of its own Decision to give it
reasonable time to check its fairness in light of supervening events in Congress as alleged by
petitioner. The Court, contrary to popular misimpression, did not restrain the effectivity of a law
enacted by Congress.

Tua v. Mangrobang LB: The primary judge of the necessity, adequacy, wisdom, reasonableness and
expediency of any law is primarily the function of the legislature.
(Respondent filed a The act of Congress entrusting us with the issuance of protection orders is in pursuance of our
Petition for the issuance authority to settle justiciable controversies or disputes involving rights that are enforceable and
of a protection order, demandable before the courts of justice or the redress of wrongs for violations of such rights.
pursuant to RA 9262 or
the Anti-Violence Against It is settled doctrine that there is grave abuse of discretion when there is a capricious and
Women and their whimsical exercise of judgment as is equivalent to lack of jurisdiction, such as where the power is
Children Act of 2004, exercised in an arbitrary or despotic manner by reason of passion or personal hostility, and it
against her husband, must be so patent and gross so as to amount to an evasion of positive duty or to a virtual refusal
petitioner. to perform the duty enjoined or to act at all in contemplation of law.
The RTC issued ex parte
a Temporary Protection AP: Here, the CA did not err when it found no grave abuse of discretion committed by the RTC in
Order (TPO). Petitioner the issuance of the TPO because the alleged acts of petitioner would fall under the acts of VAWC
assailed the as enumerated in Sec. 5 of RA 9262.
constitutionality of RA
9262 and sought to lift
the TPO.
Without awaiting the
resolution of the RTC on
the foregoing, Petitioner
filed a petition for
certiorari with the CA
assailing the TPO. )

Agcaoili v. Farinas LB: The petitions for certiorari and prohibition are appropriate remedies to raise
(Alleged misappropriation constitutional issues and to review and/or prohibit or nullify the acts of legislative and
on the excise tax on a executive officials.
locally made cigarette-
allegedly used to buy The above pronouncement is but an application of the Court's judicial power which Section 1,
vehicles. A legisla inquiry Article VIII of the Constitution defines as the duty of the courts of justice (1) to settle actual
was conducted, LGU controversies involving rights which are legally demandable and enforceable, and (2) to
officials were held in determine whether or not there has been a grave abuse of discretion amounting to lack or excess
contempt, detained. of jurisdiction on the part of any branch or instrumentality of the Government. Such innovation
Latter filed a petition for under the 1987 Constitution later on became known as the Court's "traditional jurisdiction" and
habeas corpus before the "expanded jurisdiction," respectively.
CA. CA judges were
issued by the Senate
Comm. a show cause "Expanded Jurisdiction"
order. Pet. for habeas
corpus, then referred to The exercise by the Court of its "expanded jurisdiction" is not limited to the determination of
SC) grave abuse of discretion to quasi-judicial or judicial acts, but extends to any act involving the
exercise of discretion on the part of the government. Indeed, the power of the Court to enjoin a
Issue: w/n SC has legislative act is beyond cavil.
jurisdiction over the pet.
for habeas corpus? No. AP: Here, while there is no question that a writ of prohibition lies against legislative functions, the
Court finds no justification for the issuance thereof in the instant case because a perusal of the
Note: prior decision of SC minutes of legislative hearings so far conducted reveals that the same revolved around the use of
LGU officials were the Province of Ilocos Norte's shares from the excise tax on locally manufactured virginia-type
released by Senate cigarettes through cash advances which co-petitioner Marcos herself admits to be the "usual
rendering the pet. for practice" and was actually allowed by the Commission on Audit (COA). In fact, the cause of
habeas corpus, moot. But petitioners' detention was not the perceived or gathered illegal use of such shares but the rather
the SC ruled on the unusual inability of petitioners to recall the transactions despite the same having involved
issue, nevertheless. considerable sums of money.

SECTION 2

Malaga v. Penachos, Jr LB: A law is passed (PD 1818) prohibiting courts from issuing injunctions in cases involving
infrastructure projects of the government. Valid?
(ISCOF published an
invitation to bid for the Such prohibition can only refer to administrative acts in controversies involving facts or
construction of a micro the existence of discretion in technical cases. Outside of this dimension and on issues
laboratory building at involving questions of law, the courts cannot be prevented from exercising their power.
ISCOF. Petitioners
submitted their bids. All AP: Here, although Iloilo State College of Fisheries (ISCOF) is covered by PD 1818 since it is a
three were denied to chartered institution, it does not automatically folly that ISCOF is covered by the prohibition in the
participate in the bid, not said decree.
being able to make the
cutoff time. Hence, all
three filed for injunction It is apparent that the present controversy did not arise from the discretionary acts of the
and TRO for the awarding administrative body nor does it involve merely technical matters. What is involved here is
of the project. RTC noncompliance with the procedural rules on bidding which required strict observance. P.D. 1818
was not intended to shield from judicial scrutiny irregularities committed by administrative
52
denied the injunction agencies such as the anomalies above described. Hence, the challenged restraining order was not
complaint on the ground improperly issued by the respondent judge and the writ of preliminary injunction should not have
that P.D. 1818, been denied.
prohibited the courts
from having jurisdiction
over infrastructure
projects, and that the
building being
constructed by ISCOF is
considered as an
infrastructure project of
the government falling
within the coverage of
P.D 1818.)

SECTION 3

A. Fiscal autonomy

Bengzon v. Drilon LB: Fiscal autonomy – Meaning

(constitutionality of the Fiscal autonomy means freedom from outside control. The Judiciary, the Constitutional
veto by the President of Commissions, and the Ombudsman must have the independence and flexibility needed in
certain provisions in the the discharge of their constitutional duties.
GAA for the Fiscal Year
1992 relating to the The imposition of restrictions and constraints on the manner the independent
payment of the adjusted constitutional offices allocate and utilize the funds appropriated for their operations is
pensions of retired anathema to fiscal autonomy and violative not only of the express mandate of the Constitution
Justices of the SC and but especially as regards the Supreme Court, of the independence and separation of powers upon
the CA) which the entire fabric of our constitutional system is based.

AP: Here, the President Aquino’s veto of these specific provisions in the General Appropriations
Act is tantamount to dictating to the Judiciary how its funds should be utilized, which is clearly
repugnant to fiscal autonomy. The freedom of the Chief Justice to make adjustments in the
utilization of the funds appropriated for the expenditures of the judiciary, including the use of any
savings from any particular item to cover deficits or shortages in other items of the judiciary is
withheld. Pursuant to the Constitutional mandate, the Judiciary must enjoy freedom in the
disposition of the funds allocated to it in the appropriations law. It knows its priorities just as it is
aware of the fiscal restraints. The Chief Justice must be given a free hand on how to augment
appropriations where augmentation is needed.

Section 4

Firestone Ceramics v. Issue: Consolidated cases involving vast tract of land with an area of 99 hectares presumptively
CA belonging to the PH govt has been adjudicated to private individuals by a court. It is alleged that
such court has no jurisdiction. Thus, Petitioners filed a motion to transfer the cases to the en
banc. The 3rd division denied their motion. As such, they submitted to the SC their Motion to
Refer to the court en banc these consolidated cases. Can SC en banc accept?

LB/AP: SC’s Residual Power

Yes. The SC, in accepting cases for the banc to pass upon is a legitimate and a valid
exercise of its RESIDUAL POWER in contemplation of Par. 9 of the Resolution en banc of
Nov. 18 1993 which states: “all other cases as the court en banc by a majority of its actual
membership may deem of sufficient importance to merit its attention”. Since in this case, 9-5
voted to accept because it was deemed of sufficient importance to merit its attention, it is valid.

Decisions or resolutions of a division of the court are decisions or resolutions of the SC itself. The
SC sitting en banc is not an appellate court of its Divisions, and as such, not an inferior body. The
only constraint is that any doctrine or principle laid down by the Court, either rendered en banc or
in division, may be overturned or reversed only by the Court sitting en banc.

Fortich v. Corona LB:


(1) “cases” vs. “matters”
(re: reclassification of - cases are “decided– while matters, which include motions, are “resolved.–
bukidnon lands from agri - Otherwise put, the word “decided– must refer to “cases–; while the word “resolved–
to agri-industrial, must refer to “matters,– applying the rule of reddendo singula singulis.
granting farmers title to - This is true not only in the interpretation of the above-quoted Article VIII, Section 4(3),
the lands) but also of the other provisions of the Constitution where these words appear.

Respondents filed for (2) The second sentence of Art VIII, Sec. 4 (3) speaks only of “case– and not “matter.
another “MR and for - Art VIII, Sec. 4 (3) pertains to the disposition of cases by a division. If there is a
“Referral of the case to tie in the voting, there is no decision. The only way to dispose of the case then is to refer
this honorable Court en it to the Court en banc
banc” claiming that since - On the other hand, if a case has already been decided by the division and the
53
the earlier MR were losing party files a motion for reconsideration, the failure of the division to
resolved by vote of 2-2, resolve the motion because of a tie in the voting does not leave the case
the required number to undecided.
carry a decision of 3 was - There is still the decision which must stand in view of the failure of the members of the
not met and that the division to muster the necessary vote for its reconsideration
case should be referred - Plainly, if the voting results in a tie, the motion for reconsideration is lost. The
and decided by SC en assailed decision is not reconsidered and must therefore be deemed affirmed.
banc pursuant to Article
VIII Sec 4
AP: The rule on referring a case to the SC en banc whenever the required number of votes is not
obtained, DOES NOT APPLY in this case.
Issue: W/N it should be
properly referred and
The rule does not apply where, as in this case, the required three votes is not obtained
decided by SC en banc?
in the resolution of a motion for reconsideration.
NO.

Here, the assailed decision is not reconsidered and must therefore be deemed affirmed. Such
was the ruling of this SC in 1998 (previous case)

Note: there are exceptional cases when this Court may entertain a second motion for
reconsideration, such as where there are extraordinarily persuasive reasons.

In this case, not only did movants fail to ask for prior leave of court, but more importantly, they
have been unable to show that there are exceptional reasons for us to give due course to their
second motions for reconsideration.

Philippine Health Care LB: The Supreme Court is NOT bound by a minute resolution in another case which
Providers, Inc. v. CIR involves different parties and subject matter.

CIR assessed the When a minute resolution denies or dismisses a petition for failure to comply with formal and
petitioner (PHCP) of substantive requirements, the challenged decision, together with its findings of fact and legal
deficiency for DST of its conclusions, are deemed sustained. When a minute resolution denies or dismisses a petition for
agreements with its failure to comply with formal and substantive requirements, the challenged decision, together
members. PHCP is not with its findings of fact and legal conclusions, are deemed sustained.
subject to DST. One of its
arguments, is that the Minute Resolution vs. Decision
Court is bound by the
ruling of the CA in CIR v.
PNB that a healthcare The constitutional requirement under the first paragraph of Section 14, Article VIII of the
agreement of Philamcare Constitution that the facts and the law on which the judgment is based must be expressed
Health Systems is not an clearly and distinctly applies only to decisions, not to minute resolutions.
insurance contract for
purposes of the DST. A minute resolution is signed only by the clerk of court by authority of the justices, unlike a
decision. It does not require the certification of the Chief Justice. Moreover, unlike decisions,
minute resolutions are not published in the Philippine Reports. Finally, the proviso of Section 4(3)
of Article VIII speaks of a decision. Indeed, as a rule, this Court lays down doctrines or principles
of law which constitute binding precedent in a decision duly signed by the members of the Court
and certified by the Chief Justice.

AP: Here, since petitioner was not a party in G.R. No. 148680 and since petitioners liability for
DST on its health care agreement was not the subject matter of G.R. No. 148680, petitioner
cannot successfully invoke the minute resolution in that case (which is not even binding
precedent) in its favor. Nonetheless, in view of the reasons already discussed, this does not
detract in any way from the fact that petitioners health care agreements are not subject to DST.

Section 5

A. Judicial review: ripeness

PACU v. Secretary of LB: "The power of courts to declare a law unconstitutional arises only when the
Education interests of litigants require the use of that judicial authority for their protection
against actual interference, a hypothetical threat being insufficient." (United Public Works
(Petitioners complain that vs. Mitchell, 330 U. S. 75; 91 L. Ed. 754.)
before opening a school
the owner must secure a - a private individual must be in immediate danger of sustaining a direct injury as the
permit from the result of that action
Secretary of Education. - And it is not sufficient that he has merely a general reason to invoke the judicial power
Such requirement was to determine the validity of executive or legislative action
not originally included in - He must show that he has sustained or is interest common to all members of the public.
Act No. 2706) - Courts will not pass upon the constitutionality of a law upon the complaint of one who
fails to show that he is injured by its operation

Mere apprehension that the Secretary of Education might under the law withdraw the
permit of one of petitioners does not constitute a justiciable controversy.

AP: Here, petitioners contend that the right of a citizen to own and operate a school is
guaranteed by the Constitution, and any law requiring previous governmental approval or permit
before such person could exercise said right, amounts to censorship of previous restraint; a
practice abhorent to our system of law and government. Petitioners obviously refer to section 3 of
Act No. 2706 as amended which provides that before a private school may be opened to the
public it must first obtain a permit from the Secretary of Education.
54
Where the petitioning private schools are actually operating by virtue of permits issued to them
by the Secretary of Education under Act No. 2706, who is not shown to have threatened to
revoke their permits, there is no justiciable controversy that would authorize the courts to pass
upon the constitutionality of said Act.

As to the assailed provision on the prohibition of certain textbook, the same has no justiciable
controversy. The SC was not informed that the Board on Textbooks has prohibited this or that
text, or that the petitioners refused or intend to refuse to submit some textbooks, and are in
danger of losing substantial privileges or rights for so refusing.

B. Judicial review: locus standi

Telecommunications LB: A citizen will be allowed to raise a constitutional question only when he can show
and Broadcast that he has personally suffered some actual or threatened injury as a result of the
Attorneys v. Comelec allegedly illegal conduct of the government; the injury is fairly traceable to the challenged
action; and the injury is likely to be redressed by a favorable action.
(TELEBAP are suing as
taxpayers and citizens A corporate entity composed of lawyers in the broadcasting industry has no legal
and registered voters. standing over a suit which involves the constitutionality of a law which prohibits the
They assail the validity of sale or donation of print space or air time for political ads.
BP 881 which requires
that radio and tv AP: TELEBAP – NO STANDING AS CITIZENS. A Citizen will be allowed to raise a constitutional
companies provide free question only when he can show that he has personally suffered some actual or threatened injury
airtime to COMELEC for as a result of the allegedly illegal conduct of government, that the injury is fairly traceable to the
the use of candidates in challenged action, and that the injury is likely to be redressed by a favorable action. In this case,
the campaign and for it has not shown that they will suffer or have suffered harm as a result of the operation
other political purposes. of BP 881.
Telebap claims that the
law takes property TELEBAP – NO STANDING AS REGISTERED VOTERS. No interest as registered voters since
without due process and this case does not concern their right to suffrage. Their interest in BP 881 should be
that it violates the precisely in upholding its validity.
eminent domain clause
which provides for TELEBAP – NO STANDING AS TAXPAYERS. No interest as taxpayers since this case does not
payment of just involve the exercise by Congress of its taxing or spending power . A party suing as a
compensation. GMA taxpayer must specifically show that he has sufficient interest in preventing the illegal
Network, also filed a expenditure of money raised by taxation and that he will sustain direct injury as a result of the
similar case.) enforcement of the questioned statute.

TELEBAP – NO STANDING AS CORPORATE ENTITY. No standing to assert the rights of radio


and television companies which they represent. The mere fact that Telebap is composed of
lawyers in the broadcast industry does not entitle them to bring this suit in their name as
representatives of the affected companies. Standing jus tertii will be recognized only if it can be
shown that the party suing has some substantial relation to the third party, or that the
third party cannot assert his constitutional right.

GMA7 – HAS STANDING. Since GMA operates radio and tv broadcast stations, they will be
affected by the enforcement of BP 881. It suffered losses amounting to several millions in
providing COMELEC time in connection with the 1992 and 1995 elections. Now, its stands to
suffer even more should it be required to do so again this year (1998 elections). GMA’s
allegations that it will suffer losses again is sufficient to give it standing to question the validity of
BP 881.

Joya v. PCGG LB: The rule is settled that no question involving the constitutionality or validity of a law or
governmental act may be heard and decided by the court unless there is compliance with the
(petitioners assails the legal requisites for judicial inquiry, namely:
auction of artworks and 1. that the question must be raised by the proper party;
silverwares acquired by 2. that there must be an actual case or controversy;
the PCGG from the 3. that the question must be raised at the earliest possible opportunity; and,
Marcos’s for allegedly 4. that the decision on the constitutional or legal question must be necessary to the
part of their ill-gotten determination of the case itself.
wealth. Petitioner
contends that such were The Court will exercise its power of judicial review only if the case is brought before it
cultural treasure of the by a party who has the legal standing to raise the constitutional or legal question.
nation and thus cannot
be sold by the gov’t) “Legal standing” means
- a personal and substantial interest in the case such that the party has
sustained or will sustain direct injury as a result of the governmental act that is
Petitioners claim that as being challenged.
Filipino citizens, - The term “interest” is material interest, an interest in issue and to be affected by the
taxpayers and artists decree, as distinguished from mere interest in the question involved, or a mere
deeply concerned with incidental interest.
the preservation and - Moreover, the interest of the party plaintiff must be personal and not one based on
protection of the a desire to vindicate the constitutional right of some third and unrelated party.
country’s artistic wealth,
they have the legal Certain instances when the Court has allowed exceptions to the rule on legal standing
personality to restrain - as when a citizen brings a case for mandamus to procure the enforcement of a public
respondents Executive duty for the fulfillment of a public right recognized by the Constitution, and
Secretary and PCGG from - when a taxpayer questions the validity of a governmental act authorizing the
acting contrary to their disbursement of public funds.
public duty to conserve
55
the artistic creations as
mandated by the 1987 AP: Petitioners have no legal standing.
Constitution. - Petitioner’s themselves allege that the paintings were donated by private persons from
different parts of the world to the Metropolitan Museum of Manila Foundation, which is a
nonprofit and non-stock corporations, established to promote non-Philippine arts. The
foundation’s chairman was former First Lady Imelda R. Marcos. The ownership of
these paintings legally belongs to the foundation or corporation or the
members thereof

- the pieces of antique silverware were given to the Marcos couple as gifts from friends
and dignitaries from foreign countries on their silver wedding anniversary, an occasion
personal to them.

- The confiscation of these properties by the Aquino administration should not be


understood to mean that the ownership of these paintings has automatically passed on
to the government without complying with constitutional and statutory requirements of
due process and just compensation.

- If these properties were already acquired by the government, any constitutional or


statutory defect in their acquisition and their subsequent disposition must be raised only
by the proper parties— the true owners thereof—whose authority to recover emanates
from their proprietary rights which are protected by statutes and the Constitution.
Having failed to show that they are the legal owners of the artworks or that the
valued pieces have become publicly owned, petitioners do not possess any clear
legal right whatsoever to question their alleged unauthorized disposition.

-
LB: In Legaspi v. Civil Service Commission, SC laid down the rule that a writ of mandamus
may be issued to a citizen only when the public right to be enforced and the concomitant duty of
the state are unequivocably set forth in the Constitution.

AP: petitioners are not after the fulfillment of a positive duty required of respondent
officials under the 1987 Constitution. What they seek is the enjoining of an official act
because it is constitutionally infirmed. Moreover, petitioners’ claim for the continued enjoyment
and appreciation by the public of the artworks is at most a privilege and is unenforceable as a
constitutional right in this action for mandamus.

LB: A taxpayer’s suit can prosper only if the governmental acts being questioned
involve disbursement of public funds upon the theory that the expenditure of public funds by
an officer of the state for the purpose of administering an unconstitutional act constitutes a
misapplication of such funds, which may be enjoined at the request of a taxpayer.

AP: petitioners are not challenging any expenditure involving public funds but the
disposition of what they allege to be public properties. Petitioners admit that the paintings and
antique silverware were acquired from private sources and not with public money.

Sandoval v. PAGCOR LB: (1) A party suing as a taxpayer must specifically prove that he has sufficient interest
in preventing the illegal expenditure of money raised by taxation—
(Petitioners filed such
petitions as taxpayers In essence, taxpayers are allowed to sue where there is a claim of illegal disbursement of
and in their capacity as public funds, or that public money is being deflected to any improper purpose, or where
members of the House of petitioners seek to restrain respondent from wasting public funds through the
Representatives. On the enforcement of an invalid or unconstitutional law.
other hand, respondents,
among other allegations, (2) However, in line with the liberal policy of this Court on locus standi when a case involves an
contend that the issue of overarching significance to our society, we find and so hold that as members of
petitioners do not have the house of representatives, petitioners have legal standing to file the petitions at bar.
the locus standi to file the
petitions because the
operation of jai-alai does (3) A member of the House of Representatives has standing to maintain inviolate the
not involve the prerogatives, powers and privileges vested by the Constitution in his office.
disbursement of funds.)
AP: Here, petitioners complain that the operation of jai-alai constitutes an infringement by
I: W/N members of the PAGCOR of the legislature’s exclusive power to grant franchise. To the extent the powers
House of Representatives of Congress are impaired, so is the power of each member thereof, since his office confers
have standing to question a right to participate in the exercise of the powers of that institution.
the validity of PAGCOR’s
operation of jai-alai Thus, petitioners, as members of the House of Representatives, have the standing to question the
without congressional operation of jai-alai by PAGCOR without a congressional franchise.
franchise? YES.
Lozano v. Nograles LB: LOCUS STANDI - “transcendental importance” doctrine

(petitions seek to trigger Generally, a party will be allowed to litigate only when he can demonstrate that
a justiciable controversy (1) he has personally suffered some actual or threatened injury because of the allegedly illegal
that would warrant a conduct of the government;
definitive interpretation (2) the injury is fairly traceable to the challenged action; and
by this Court of Section (3) the injury is likely to be redressed by the remedy being sought
1, Article XVII, which
provides for the (1) Locus standi requires a personal stake in the outcome of a controversy for
56
procedure for amending significant reasons. It assures adverseness and sharpens the presentation of
or revising the issues for the illumination of the Court in resolving difficult constitutional
Constitution) questions.

(2) A taxpayer’s suit requires that the act complained of directly involves the illegal
disbursement of public funds derived from taxation.
Standing as a citizen has been upheld by this Court in cases where a petitioner
is able to craft an issue of transcendental importance or when paramount public
interest is involved.

RATIONALE OF THE LOCUS STANDI REQUIREMENT:

A lesser but not insignificant reason for screening the standing of persons who desire to litigate
constitutional issues is economic in character. Given the sparseness of our resources, the
capacity of courts to render efficient judicial service to our people is severely limited.
For courts to indiscriminately open their doors to all types of suits and suitors is for them to
unduly overburden their dockets, and ultimately render themselves ineffective dispensers of
justice. (Kilosbayan, Incorporated v. Guingona, Jr.)

Moreover, while the Court has taken an increasingly liberal approach to the rule of
locus standi, evolving from the stringent requirements of “personal injury” to the
broader “transcendental importance” doctrine, such liberality is not to be abused. It is
not an open invitation for the ignorant and the ignoble to file petitions that prove
nothing but their cerebral deficit.

AP: In the cases at bar, petitioners have not shown the elemental injury in fact that would endow
them with the standing to sue. The lack of petitioners’ personal stake in this case is no more
evident than in Lozano’s three-page petition that is devoid of any legal or jurisprudential basis.

Neither can the lack of locus standi be cured by the claim of petitioners that they are instituting
the cases at bar as taxpayers and concerned citizen `It is undisputed that there has been no
allocation or disbursement of public funds in this case as of yet.

While the Court recognizes the potential far-reaching implications of the issue at hand, the
possible consequence of House Resolution No. 1109 is yet unrealized and does not
infuse petitioners with locus standi under the “transcendental importance” doctrine.

Southern Hemisphere LB: (1) Judicial Review; Requisites.— A-L-E-L


Engagement Network, 1. there must be an actual case or controversy;
Inc., et al. vs. Anti- 2. petitioners must possess locus standi;
Terrorism Council 3. the question of constitutionality must be raised at the earliest opportunity; and
4. the issue of constitutionality must be the lis mota of the case.
(Consolidation of 6
petitions challenging the (2) Locus standi or legal standing has been defined as a personal and substantial interest in a
constitutionality of RA case such that the party has sustained or will sustain direct injury as a result of the governmental
9372 “An Act to Secure act that is being challenged.
the State and Protect our
People from Terrorism,”
aka Human Security Act
of 2007. Petitioner- (3) Locus Standi; Requisites – Di-DI
organizations assert locus 1. A party who assails the constitutionality of a statute must have a direct and personal
standi on the basis of interest
being suspected 2. it must show not only that the law or any governmental act is invalid, but also that it
“communist fronts” by sustained or is in immediate danger of sustaining some direct injury as a result
the government, whereas of its enforcement, and not merely that it suffers thereby in some indefinite way.
individual petitioners
invoke the (4) Locus Standi; Requisites;
“transcendental
importance” doctrine and
For a concerned party to be allowed to raise a constitutional question, he must show
their status as citizens
that:
and taxpayers.)
1. He has personally suffered some actual or threatened injury;
2. The injury is fairly traceable to the challenged action;
I: W/N petitioners have 3. The injury is likely to be redressed by a favorable action.
locus standi. NO.
AP: RA 9372 is a penal statute. While Chavez v. PCGG holds that transcendental public
importance dispenses with the requirement that petitioner has experienced or is in actual danger
of suffering direct and personal injury, cases involving the constitutionality of penal
legislation belong to an altogether different genus of constitutional litigation. Such
necessitates closer judicial scrutiny of locus standi.

The mere invocation of the duty to preserve the rule of law does not, however, suffice to
clothe the IBP or any of its members with standing. They failed to sufficiently demonstrate
how its mandate under the assailed statute revolts against its constitutional rights and
duties.

Former Senator Ma. Ana Consuelo Madrigal who claims to have been the subject of “political
surveillance” also lacks locus standi. The same is true for Wigberto Tañada and Osmeña III, who
cite their being a human rights advocate and an oppositor, respectively. No concrete injury has

57
been pinpointed, hence, no locus standi.

In re Save the LB: Locus standi is defined as “a right of appearance in a court of justice on a given
Supreme Court question.”
Independence and - In private suits, standing is governed by the “real parties-in-interest” rule as contained
Fiscal Autonomy in Section 2, Rule 3 of the 1997 Rules of Civil Procedure, as amended. It provides that
Movement “every action must be prosecuted or defended in the name of the real party-in-interest.”
Accordingly, the “real party-in-interest” is “the party who stands to be benefited or
This case involves the injured by the judgment in the suit or the  party entitled to the avails of the suit.”
proposed bills abolishing Succinctly put, the plaintiff’s standing is based on his own right to the relief sought.
the Judiciary - The difficulty of determining locus standi arises in public suits.
Development Fund and o Here, the plaintiff who asserts a “public right” in assailing an allegedly illegal
replacing it with the official action, does so as a representative of the general public.
“Judiciary Support Fund.” o He may be a person who is affected no differently from any other person.
Funds collected from the o He could be suing as a “stranger,” or in the category of a “citizen,” or
proposed Judiciary “taxpayer.”
Support Fund shall be o In either case, he has to adequately show that he is entitled to seek judicial
remitted to the national protection. In other words, he has to make out a sufficient interest in the
treasury and Congress vindication of the public order and the securing of relief as a “citizen”
shall determine how the or “taxpayer.”
funds will be used
The Supreme Court (SC) has occasionally relaxed the rules on standing when the issues
involved are of “transcendental importance” to the public
- this court has stated that: the rule on standing is a matter of procedure, hence, can be
relaxed for nontraditional plaintiffs like ordinary citizens, taxpayers, and legislators when
the public interest so requires, such as when the matter is of transcendental importance,
of overreaching significance to society, or of paramount public interest.

There being no doctrinal definition of transcendental importance, the following


instructive determinants formulated by former Supreme Court Justice Florentino P. Feliciano
are instructive:1
(1) the character of the funds or other assets involved in the case;
(2) the presence of a clear case of disregard of a constitutional or statutory prohibition by the
public respondent agency or instrumentality of the government; and
(3) the lack of any other party with a more direct and specific interest in raising the questions
being raised

A mere invocation of transcendental importance in the pleading is not enough for this court to set
aside procedural rules: Whether an issue is of transcendental importance is a matter
determined by this court on a case-to-case basis. An allegation of transcendental
importance must be supported by the proper allegations.

In addition to the determinants in Francisco, it must also be shown that there is a clear or
imminent threat to fundamental rights.

AP:
None of the determinants in Francisco are present in this case. The events feared by petitioner
are merely speculative and conjectural.

The events feared by petitioner are contingent on the passing of the proposed bill in Congress.
The threat of imminent injury is not yet manifest since there is no guarantee that the bill will even
be passed into law. There is no transcendental interest in this case to justify the relaxation of
technical rules.

***

As to the justiciable controversy requirement:


There can be no justiciable controversy involving the constitutionality of a proposed bill. The
Court can exercise its power of judicial review only after a law is enacted, not before.

C. Operative Fact

The “operative fact doctrine simply means that the declaration of unconstitutionality of a law, treaty, etc., is prospective. It is
only the declaration of unconstitutionality, which is the “operative fact” which would stop the people from complying with its
provisions.
- All acts done in connection with the said law before its declaration of unconstitutionality shall be considered legal,
valid and binding.

De Agbayani v. PNB LB: The actual existence of a statute, prior to such a determination (of
unconstitutionality), is an operative fact and may have consequences which cannot
I: Whether or not the justly be ignored.
period of the effectivity of - The past cannot always be erased by a new judicial declaration . The effect of the
EO 32 and the Act subsequent ruling as to invalidity may have to be considered in various aspects,—with
extending the respect to particular relations, individual and corporate, and particular conduct, private
Moratorium Law before and official.
they were declared
invalid, stayed the period AP: The now prevailing principle is that the existence of a statute or executive order prior to

1
Francisco v. Nagmamalasakit na mga Manananggol ng mga Manggagawang Pilipino, Inc.,
58
of prescription (supposed its being declared void is an operative fact to which legal consequences are attached.
effect of the declaration
of the unconstitutionality This is because of the judicial recognition that the moratorium was a valid governmental response
of a law)? YES to the plight of the debtors who suffered during the war. The court solidified its view in a series of
cases that during the eight-year period that Executive Order No. 32 and Republic Act No. 342
were in force, prescription did not run.

From July 19, 1944, when her loan matured, to July 13, 1959, when extrajudicial foreclosure
proceedings were started by appellant Bank, the time consumed was 6 days short of fifteen
years. The prescriptive period was tolled however, from March 10, 1945, the effectivity of
Executive Order No. 32, to May 18, 1953, when the decision of Rutter v. Esteban was
promulgated, covering eight years, two months and eight days. Obviously then, when resort
was had extra-judicially to the foreclosure of the mortgage obligation, there was time to spare
before prescription could be availed of as a defense.

D. Political Questions

Marcos v. Manglapus LB: Political Question Doctrine

Is the President’s - The present Constitution limits resort to the political question doctrine and broadens the
determination that the scope of judicial inquiry into areas which the Court, under previous constitutions, would
return of former have normally left to the political departments to decide.
President Marcos and his - But nonetheless there remain issues beyond the Court’s jurisdiction the determination of
family to the Philippines which is exclusively for the President, for Congress or for the people themselves through
is a clear and present a plebiscite or referendum.
danger to national
security, public safety, or
In the exercise of the power of judicial review, the function of the court is merely to check,
public health a political
not to supplant the Executive.
question?No.
- When political questions are involved, the Constitution limits the determination
to whether or not there has been a grave abuse of discretion amounting to lack
or excess of jurisdiction on the part of the official whose action is being
questioned.
- If grave abuse is not established, the Court will not substitute its judgment for that of
the official concerned and decide a matter which by its nature or by law is for the latter
alone to decide.

AP: There is nothing in the case that precludes the Court’s determination on the political question
doctrine.

Accordingly, the question for the Court to determine is whether or not there exist factual bases
for the President to conclude that it was in the national interest to bar the return of the Marcoses
to the Philippines

Daza v. Singson LB: The term political question” connotes, in legal parlance, what it means in ordinary
parlance, namely, a question of policy.
(Daza challenged his - In other words, x x x it refers “to those questions which, under the Constitution,
removal from the are to be decided by the people in their sovereign capacity, or in regard to
Commission on which full discretionary authority has been delegated to the Legislature or
Appointments based) executive branch of the Government.” (Tanada v. Cuenco) It is concerned with
issues dependent upon the wisdom, not legality, of a particular measure.

AP: Contrary to the respondent’s assertion, the Court has the competence to act on the matter at
bar. Our finding is that what is before us is not a discretionary act of the House of
Representatives that may not be reviewed by us because it is political in nature.
- What is involved here is the legality, not the wisdom, of the act of that chamber
in removing the petitioner from the Commission on Appointments. That is not a
political question.

And, even assuming that the issue presented before us was political in nature, SC would still not
be precluded from resolving it under the expanded jurisdiction conferred upon us that
now covers, in proper cases, even the political question.

E. Review of capital offenses

People v. Mateo LB: (1) While the Fundamental Law requires a mandatory review by the Supreme Court
of cases where the penalty imposed is reclusion perpetua, life imprisonment, or death,
(10 counts of rape) nowhere, however, has it proscribed an intermediate review.
- If only to ensure utmost circumspection before the penalty of death, reclusion perpetua
or life imprisonment is imposed, the Court now deems it wise and compelling to provide
in these cases a review by the Court of Appeals before the case is elevated to the
Supreme Court

- Where life and liberty are at stake, all possible avenues to determine his guilt or
innocence must be accorded an accused, and no care in the evaluation of the facts can
ever be overdone.
- A prior determination by the Court of Appeals on, particularly, the factual
issues, would minimize the possibility of an error of judgment.
- If the Court of Appeals should affirm the penalty of death, reclusion perpetua or life
imprisonment, it could then render judgment imposing the corresponding penalty as the
59
circumstances so warrant, refrain from entering judgment and elevate the entire records
of the case to the Supreme Court for its final disposition.

(2) Allowing an intermediate review by the Court of Appeals before the case is elevated to the
Supreme Court on automatic review is a procedural matter within the rule-making prerogative of
the Supreme Court than the law-making power of Congress

AP: the instant case is REMANDED, and all pertinent records thereof ordered to be FORWARDED,
to the Court of Appeals for appropriate action and disposition

F. Rule-making power

In re Cunanan LB:(1) The Constitution has not conferred on Congress and this Tribunal equal
responsibilities governing the admission to the practice of law.
Congress passed RA 972 - The primary power and responsibility which the Constitution recognizes, continue to
or the Bar Flunkers Act reside in this court.
of 1953. In accordance - Congress may repeal, alter and supplement the rules promulgated by this court,
with the said law, the SC - but the authority and responsibility over the admission, suspension, disbarment
then passed and and reinstatement of attorneys-at-law and their supervision remain vested in
admitted to the bar those the Supreme Court
candidates who had
obtained an average of (2) Being coordinate and independent branches the power to promulgate and enforce rules
72% by raising it to 75%. for the admission to the practice of law and the concurrent power to repeal, alter and
After its approval, many supplement them may and should be exercised with the respect that each owes to the
unsuccessful candidates other, giving careful consideration to the responsibility which the nature of each department
filed petition for requires.
admission to the bar
invoking RA 972.
- the harmonious delimitation being found in that the legislature may and should examine
if the existing rules on the admission to the Bar respond to the demands which public
WON: RA 972 interest requires of a Bar endowed with high virtues, culture, training and responsibility
constitutional? No.
o The legislature may, by means of repeal, amendment or supplemental rules, fill
up any deficiency that it may find,
o and the judicial power, which has the inherent responsibility for a good and
efficient administration of justice and the supervision of the practice of the legal
profession, should consider these reforms as the minimum standards for the
elevation of the profession, and see to it that with these reforms the lofty
objective that is desired in the exercise of its traditional duty of admitting,
suspending, disbarring and reinstating attorneys-atlaw is realized

- They are powers which, exercised within their proper constitutional limits, are not
repugnant, but rather complementary to each other in attaining the
establishment of a Bar that would respond to the increasing and exacting
necessities of the administration of justice.

AP: When Congress enacted RA 972, it in effect repeals, alters and supplements the
rules on admission to the bar. This contravenes the Constitution as only the SC can decide
who may be admitted to the bar and may continue the practice of law according to its existing
rules.

Javellana v. LB: As a matter of policy, this Court accords great respect to the decisions and/or actions of
Department of Interior administrative authorities not only because of the doctrine of separation of powers but also for
their presumed knowledgeability and expertise in the enforcement of laws and regulations
(private respondent entrusted to their jurisdiction.
allege that petitioner
engages in practice in AP: Petitioner’s contention that Section 90 of the Local Government Code of 1991 and DLG
law while being elected Memorandum Circular No. 90-81 violate Article VIII, Section 5 of the Constitution is completely
as city councilor, in off tangent.
violation of DILG Memo)
- Neither the statute nor the circular trenches upon the Supreme Court’s power
and authority to prescribe rules on the practice of law. The Local Government
Issue: W/N DLG Memo Code and DLG Memorandum Circular No. 90-81 simply prescribe rules of conduct for
circulars are issued in public officials to avoid conflicts of interest between the discharge of their public duties
violation of the and the private practice of their profession, in those instances where the law allows it.
Constitution for
encroachment on SC’s Section 90 of the Local Government Code does not discriminate against lawyers and doctors. It
exclusive authority to applies to all provincial and municipal officials in the professions or engaged in any occupation.
regulate the practice of Section 90 explicitly provides that sanggunian members “may practice their professions, engage
law in any occupation, or teach in schools except during session hours.” If there are some prohibitions
that apply particularly to lawyers, it is because of all the professions, the practice of law is more
likely than others to relate to, or affect, the area of public service.

Fabian v. Desierto LB: (1) In determining whether a rule prescribed by the Supreme Court, for the practice and
procedure of the lower courts, abridges, enlarges, or modifies any substantive right, the test is
whether the rule really regulates procedure, that is, the judicial process for enforcing
rights and duties recognized by substantive law and for justly administering remedy
and redress for a disregard or infraction of them.

60
• If the rule takes away a vested right, it is not procedural.
• If the rule creates a right such as the right to appeal, it may be classified as a
substantive matter;
• But if it operates as a means of implementing an existing right then the rule
deals merely with procedure.

(2)A transfer by the Supreme Court, in the exercise of its rule-making power, of
pending cases involving a review of decisions of the Office of the Ombudsman in
administrative disciplinary actions to the Court of Appeals which shall now be vested
with exclusive appellate jurisdiction thereover, relates to procedure only.
- This is so because it is not the right to appeal of an aggrieved party which is affected
by the law. That right has been preserved.

- Only the procedure by which the appeal is to be made or decided has been
changed. The rationale for this is that no litigant has a vested right in a particular
remedy, which may be changed by substitution without impairing vested rights, hence
he can have none in rules of procedure which relate to the remedy

AP: It cannot be said that the transfer of appellate jurisdiction to the Court of Appeals in
this case is an act of creating a new right of appeal because such power of the Supreme
Court to transfer appeals to subordinate appellate courts is purely a procedural and not
a substantive power.
- Neither can we consider such transfer as impairing a vested right because the parties
have still a remedy and still a competent tribunal to administer that remedy.
-
Baguio Market Vendors LB: (1) The 1987 Constitution textually altered the power-sharing scheme under the previous
Cooperative vs. Cortes charters by deleting in Section 5(5) of Article VIII Congress’ subsidiary and corrective power. This
glaring and fundamental omission led the Court to observe in Echegaray v. Secretary of Justice,
I: whether petitioner’s 301 SCRA 96 (1999) that this Court’s power to promulgate judicial rules “is no longer
application for shared by this Court with Congress.”
extrajudicial foreclosure
is exempt from legal fees (2)The payment of legal fees is a vital component of the rules promulgated by this Court
under Article 62(6) of RA concerning pleading, practice and procedure, it cannot be validly annulled, changed or
6938? Or modified by Congress —as one of the safeguards of the Supreme Court’s institutional
Whether Congress may independence, the power to promulgate rules of pleading, practice and procedure is now the
grant exemptions from Court’s exclusive domain.
legal fees? No.
AP: First, a petition for extrajudicial foreclosure of mortgage under Act 3135 is outside the ambit
of the Cooperative Code because the legal exemption granted therein is limited to two types of
actions: 1) actions brought under the Cooperative Code; and 2) actions brought by the
Cooperative Development Authority to enforce the payment of obligations contracted in favor of
cooperatives; and

Second, the 1987 Constitution textually altered the power-sharing scheme under the previous
charters by deleting in Section 5(5) of Article VIII Congress’ subsidiary and corrective power.

Hence, Petitioner cannot rely on Article 62(6) of RA 6938 for its exemption to legal fees.

Estipona v. Lobrigo LB: It is within the Supreme Court’s power to make exceptions to the rules of court.
Under proper conditions, it may permit the full and exhaustive ventilation of the
parties’ arguments and positions despite the supposed technical infirmities of a petition
or its alleged procedural flaws.

The power to promulgate rules of pleading, practice and procedure is now the Supreme Court’s
(SC’s) exclusive domain and no longer shared with the Executive and Legislative departments

The separation of powers among the three (3) coequal branches of our government has erected
an impregnable wall that keeps the power to promulgate rules of pleading, practice and procedure
within the sole province of the Supreme Court.
- The other branches trespass upon this prerogative if they enact laws or issue
orders that effectively repeal, alter or modify any of the procedural rules
promulgated by the Court.

The Supreme Court’s (SC’s) sole prerogative to issue, amend, or repeal procedural rules
is limited to the preservation of substantive rights, i.e., the former should not diminish,
increase or modify the latter.
- “Substantive law is that part of the law which creates, defines and regulates rights, or
which regulates the right and duties which give rise to a cause of action; that part of the
law which courts are established to administer; as opposed to adjective or remedial law,
which prescribes the method of enforcing rights or obtain redress for their invasions.”

AP: Section 6, Rule 120 of the Rules, which provides that an accused who failed to appear at the
promulgation of the judgment of conviction shall lose the remedies available against the
judgment, does not take away substantive rights but merely provides the manner through which
an existing right may be implemented.

It is the failure of the accused to appear without justifiable cause on the scheduled date of
promulgation of the judgment of conviction that forfeits their right to avail themselves of the
remedies against the judgment. It is not correct to say that Section 6, Rule 120, of the Rules of
Court diminishes or modifies the substantive rights of petitioners. It only works in pursuance of

61
the power of the Supreme Court to “provide a simplified and inexpensive procedure for the
speedy disposition of cases.” This provision protects the courts from delay in the speedy
disposition of criminal cases — delay arising from the simple expediency of nonappearance of the
accused on the scheduled promulgation of the judgment of conviction.

Section 6

A. Supervision of courts

People v. Gacott, Jr LB: The Court is fully aware that not every error or mistake of a judge in the
performance of his duties is subject to censure. But where, as in the present case, the
(Judge dismissed the error could have been entirely avoided were it not for public respondent’s irresponsibility in
case on the ground that the performance of his duties, it is but proper that respondent judge be reprimanded and his
RTC has no jurisdiction order of dismissal set aside for grave ignorance of the law. For, respondent judge’s error is not a
over the case and it is simple error in judgment but one amounting to gross ignorance of the law which could
the Anti Dummy Board easily undermine the public’s perception of the court’s competence.
who has j. over the case,
without knowing/ignorant AP: Respondent judge is REPRIMANDED AND FINED in the amount of P10,000.00 for gross
of the fact that the latter ignorance of the law with a stern warning that a repetition of the same or a similar offense
was already abolished by shall merit serious consequences
law).

Judge Caoibis v. LB: Under Section 6, Article VIII of the Constitution, it is the Supreme Court which is vested with
Ombudsman exclusive administrative supervision over all courts and its personnel.
 Prescinding from this premise, the Ombudsman cannot determine for itself and by
RTC Judge Alumbres filed itself whether a criminal complaint against a judge, or court employee, involves
before the Ombudsman a an administrative matter.
criminal complaint for
physical injuries,  The Ombudsman is duty bound to have all cases against judges and court
malicious mischief and personnel filed before it, referred to the Supreme Court for determination as to
assault upon a person in whether and administrative aspect is involved therein.
authority against fellow
Judge Caoibis. -This  This rule should hold true regardless of whether an administrative case based on
started when Alumbre the act subject of the complaint before the Ombudsman is already pending with
requested Caoibis to the Court.
return the executive
table he borrowed. [LOL]
 For, aside from the fact that the Ombudsman would not know of this matter unless he is
informed of it, he should give due respect for and recognition of the administrative
authority of the Court, because in determining whether an administrative matter is
involved, the Court passes upon not only administrative liabilities but also other
WON the Ombudsman administrative concerns
have the jurisdiction over
the administrative case By virtue of its constitutional power of administrative supervision over all courts and
between the two judges? court personnel, from the Presiding Justice of the Court of Appeals down to the lowest
No. municipal trial court clerk, it is only the Supreme Court that can oversee the judges’
and court personnel’s compliance with all laws, and take the proper administrative
action against them if they commit any violation thereof.
- No other branch of government may intrude into this power, without running afoul of the
doctrine of separation of powers.

AP: The Ombudsman cannot dictate to, and bind the Court, to its findings that a case before it
does or does not have administrative implications. To do so is to deprive the Court of the exercise
of its administrative prerogatives and to arrogate unto itself a power not constitutionally
sanctioned. This is a dangerous policy which impinges, as it does, on judicial independence.

Section 8

A. Judicial and Bar Council

Chavez v. Judicial and LB: The Framers of our Constitution intended to create a Judicial and Bar Council (JBC) as
Bar Council an innovative solution in response to the public clamor in favor of eliminating politics in
the appointment of members of the Judiciary
In 1994, the composition - To ensure judicial independence, they adopted a holistic approach and hoped that, in
of the JBC was creating a JBC, the private sector and the three branches of government would have an
substantially altered. active role and equal voice in the selection of the members of the Judiciary.
Instead of having only - to allow the Legislature to have more quantitative influence in the JBC by having more
seven (7) members, an than one voice speak, whether with one full vote or one-half (1/2) a vote each, would, as
eighth (8th) member was one former congressman and member of the JBC put it, “negate the principle of equality
added to the JBC as two among the three branches of government which is enshrined in the Constitution.”
(2) representatives from
Congress began sitting in “A perusal of the records of the Constitutional Commission reveals that the composition of the
the JBC—one from the JBC reflects the Commission’s desire “to have in the Council a representation for the major
House of Representatives elements of the community.” xxx The ex-officio members of the Council consist of representatives

62
and one from the Senate, from the three main branches of government while the regular members are composed of various
with each having one-half stakeholders in the judiciary. The unmistakeable tenor of Article VIII, Section 8(1) was to
(1/2) of a vote. Then, treat each ex-officio member as representing one co-equal branch of government. xxx
curiously, the JBC En Thus, the JBC was designed to have seven voting members with the three ex-officio
Banc, in separate members having equal say in the choice of judicial nominees.
meetings held in 2000
and 2001, decided to AP: the use of the singular letter “a” preceding “representative of Congress” is
allow the representatives unequivocal and leaves no room for any other construction. It is indicative of what the members
from the Senate and the of the Constitutional Commission had in mind, that is, Congress may designate only one (1)
House of Representatives representative to the JBC. Had it been the intention that more than one (1) representative from
one full vote each the legislature would sit in the JBC, the Framers could have, in no uncertain terms, so provided.

Issue: Does the first the word “Congress” used in Article VIII, Section 8(1) of the Constitution is used in its generic
paragraph of Section 8, sense. No particular allusion whatsoever is made on whether the Senate or the House of
Article VIII of the 1987 Representatives is being referred to, but that, in either case, only a singular representative may
Constitution allow more be allowed to sit in the JBC.
than one (1) member of
Congress to sit in the
JBC? Is the practice of the seven-member composition of the JBC serves a practical purpose, that is, to provide
having two (2) a solution should there be a stalemate in voting.
representatives from - This underlying reason leads the Court to conclude that a single vote may not be divided
each house of Congress into half (1/2), between two representatives of Congress, or among any of the sitting
with one (1) vote each members of the JBC for that matter.
sanctioned by the - This unsanctioned practice can possibly cause disorder and eventually muddle the JBC’s
Constitution? voting process, especially in the event a tie is reached.

Conclusion: Court’s finding that the current composition of the JBC is unconstitutional,
it bears mentioning that as a general rule, an unconstitutional act is not a law; it confers no
rights; it imposes no duties; it affords no protection; it creates no office; it is inoperative as if it
has not been passed at all. This rule, however, is not absolute. In the interest of fair play under
the doctrine of operative facts actions, previous to the declaration of unconstitutionality are
legally recognized.

Villanueva v. Judicial LB: The JBC is mandated to recommend appointees to the judiciary and only those
and Bar Council nominated by the JBC in a list officially transmitted to the President may be appointed
by the latter as justice or judge in the judiciary.
Judge Ferdinand - Thus, the JBC is burdened with a great responsibility that is imbued with public interest
Villanueva, one-year as it determines the men and women who will sit on the judicial bench. While the 1987
after being appointed as Constitution has provided the qualifications of members of the judiciary, this
MCTC Presiding Judge of does not preclude the JBC from having its own set of rules and procedures and
Compostela, applied for providing policies to effectively ensure its mandate.
vacant positions as
Presiding Judge for 3 - The functions of searching, screening, and selecting are necessary and incidental to the
RTCs. JBC did not include JBC’s principal function of choosing and recommending nominees for vacancies in the
him in the list of judiciary for appointment by the President. However, the Constitution did not lay
applicants having failed down in precise terms the process that the JBC shall follow in determining
to meet the 5 years of applicants’ qualifications.
service requirement. He
then assailed the policy
- In carrying out its main function, the JBC has the authority to set the
of the Judicial and Bar
standards/criteria in choosing its nominees for every vacancy in the judiciary,
Council (JBC), requiring
subject only to the minimum qualifications required by the Constitution and law
five years of service as
for every position. The search for these long held qualities necessarily requires
judges of first-level
a degree of flexibility in order to determine who is most fit among the
courts before they can
applicants. Thus, the JBC has sufficient but not unbridled license to act in
qualify as applicant to
performing its duties.
second-level courts, on
the ground that it is
unconstitutional - JBC’s ultimate goal is to recommend nominees and not simply to fill up judicial vacancies
in order to promote an effective and efficient administration of justice. Given this
pragmatic situation, the JBC had to establish a set of uniform criteria in order to
I: Whether the policy of
ascertain whether an applicant meets the minimum constitutional qualifications and
JBC requiring five years
possesses the qualities expected of him and his office
of service as judges of
first-level courts before
they can qualify as AP: Thus, the adoption of the five-year requirement policy applied by JBC to the
applicant to second-level petitioner’s case is necessary and incidental to the function conferred by the
courts is constitutional? Constitution to the JBC.
YES
Umali v. Judicial and LB: To add another member in the Judicial and Bar Council (JBC) or to increase the
Bar Council representative of Congress to the JBC, the remedy is not judicial but constitutional
amendment.
Petitioner assails the - To add another member in the Judicial and Bar Council (JBC) or to increase the
practice of six-month representative of Congress to the JBC, the remedy is not judicial but constitutional
rotational representation amendment
of Congress in the - And though it is unnecessary for the JBC composition to be an odd number as no tie-
Judicial and Bar Council breaker is needed in the preparation of a shortlist since judicial nominees are not
(JBC) for it unfairly decided by a “yes” or “no” vote, still, JBC’s membership cannot be increased from seven
deprives both Houses of to eight for it will be a clear violation of the aforesaid constitutional provision.
Congress of their full
participation in the said AP: Stare decisis applies. SC did not overthrow Chavez for it is in accord with the constitutional
body. The mandate of giving Congress “a representative” in the JBC. In the same manner, the adoption of
aforementioned practice the rotational scheme will not in any way deprive Congress of its full participation in the JBC for
was adopted by the JBC such an arrangement is also in line with that constitutional mandate.
in light of the ruling in

63
Chavez v. Judicial and
Bar Council.

Section 9

A. Appointment of Justices

Dulay v. Judicial and LB: A plain reading of the constitutional provisions on the Judicial Department in Article VIII of
Bar Council the 1987 Constitution clearly shows that the phrase "Members of the Supreme Court" and
the words "Members" and "Member" are repeatedly used to refer to the Justices of the
With the removal and Supreme Court without distinction whether he be the Chief Justice or any of the
impeachment of Hon. Associate Justices or all fifteen Justices
Renato Corona, the
position of Chief Justice AP: in Section 9 on the appointment of Justices and Judges, the phrase "Members of the
was left vacant. Famela Supreme Court" clearly refers to the fifteen Justices of the Court - one Chief Justice and
R. Dulay raises this fourteen (14) Associate Justices - who are within the appointing power of the
petition against the President. There can be no doubt that the Chief Justice and Associate Justices required
Judicial and Bar Council to compose the Supreme Court are the regular members of the Court
(JBC) and Executive
Secretary Paquito N. ***
Ochoa, claiming: That Other issue:
the President cannot
validly appoint the
Chief Justice of the SC, Dulay argues that the JBC cannot perform its task without an incumbent Chief Justice. To follow
because the Constitution this logic would lead to an eventuality where a vacancy in the Judiciary will not be filled if a
only empowers him to vacancy occurs in the JBC. We can likewise infer from this argument that if the Office of the Chief
appoint members or Justice is vacated, the same will not be filled because there will be no "incumbent Chief Justice" to
Justices, but not the act as Chairman of the JBC.
Chief Justice; and That
the JBC cannot be The principal function of the JBC is to recommend appointees to the Judiciary. For every vacancy,
validly headed by a the JBC submits to the President a list of at least three nominees and the President may not
retired Associate appoint anybody who is not in the list. Any vacancy in the Supreme Court is required by the
Justice of the SC, Constitution to be filled within 90 days from the occurrence thereof. This 90-day period is
because the Constitution mandatory. It cannot, therefore, be compromised only because the constitutionally-named
specifically provides that Chairman could not sit in the JBC.
it be headed by the
incumbent Chief Justice
Although it would be preferable if the membership of the JBC is complete, the JBC can still
and no other.
operate to perform its mandated task of submitting the list of nominees to the
President even if the constitutionally-named ex-officio Chairman does not sit in the JBC.

Aguinaldo v. Aquino LB: As long as in the end, the President appoints someone nominated by the Judicial
and Bar Council (JBC), the appointment is valid, and he, not the JBC, determines the
JBC asserts that in seniority of appointees to a collegiate court.
submitting six short lists - the power to recommend of the JBC cannot be used to restrict or limit the President’s
for six vacancies, it was power to appoint as the latter’s prerogative to choose someone whom he/she considers
only acting in accordance worth appointing to the vacancy in the Judiciary is still paramount.
with the clear and
unambiguous mandate of Part of the President’s power to appoint members of a collegiate court, such as the
Article VIII, Section 93 of Sandiganbayan, is the power to determine the seniority or order of preference of such newly
the 1987 Constitution for appointed members by controlling the date and order of issuance of said members’ appointment
the JBC to submit a list or commission papers. By already designating the numerical order of the vacancies, the JBC
for every vacancy. would be establishing the seniority or order of preference of the new Sandiganbayan Associate
Considering its Justices even before their appointment by the President and, thus, unduly arrogating unto itself a
independence as a vital part of the President’s power of appointment.
constitutional body, the
JBC has the discretion AP: On the faces of said short lists, it could only mean that President Aquino was to make the
and wisdom to perform appointments in the order of seniority predetermined by the JBC, and that nominees who applied
its mandate in any for any of the vacant positions, requiring the same qualifications, were deemed to be qualified to
manner as long as it is be considered for appointment only to the one vacant position to which his/her cluster was
consistent with the specifically assigned.
Constitution

By arbitrarily clustering the nominees for appointment to the six simultaneous vacancies for
Sandiganbayan Associate Justice into separate short lists, the JBC influenced the appointment
process and encroached on the President’s power to appoint members of the Judiciary and
determine seniority in the said court, beyond its mandate under the 1987 Constitution.
- The President’s option for every vacancy was limited to the five to seven nominees in
each cluster. Once the President had appointed a nominee from one cluster, then he was
proscribed from considering the other nominees in the same cluster for the other
vacancies. All the nominees applied for and were found to be qualified for appointment
to any of the vacant Associate Justice positions in the Sandiganbayan, but the JBC failed
to explain why one nominee should be considered for appointment to the position
assigned to one specific cluster only
- Correspondingly, the nominees’ chance for appointment was restricted to the
consideration of the one cluster in which they were included, even though they applied
and were found to be qualified for all the vacancies.
- Moreover, by designating the numerical order of the vacancies, the JBC established the
seniority or order of preference of the new Sandiganbayan Associate Justices.

The clustering by the JBC of nominees for simultaneous or closely successive vacancies in
collegiate courts can actually be a device to favor or prejudice a particular nominee. A favored

64
nominee can be included in a cluster with no other strong contender to ensure his/her
appointment; or conversely, a nominee can be placed in a cluster with many strong contenders to
minimize his/her chances of appointment.

Conclusion: the clustering of nominees by the JBC for the simultaneous vacancies that
occurred by the creation of six new positions of Associate Justice of the Sandiganbayan
is unconstitutional and President Aquino is not bound by such clustering in making his
appointments to the vacant Sandiganbayan Associate Justice posts.

Section 10

A. Salary of Justices and Judges

Nitafan v. LB: Salaries of Justices and Judges subject to income taxation


Commissioner on
Internal Revenue The clear intent of the Constitutional Commission was to delete the proposed express grant of
exemption from payment of income tax to members of the Judiciary, so as to "give substance
Petitioners in this case to equality among the three branches of Government".
are the duly appointed
and qualified Judges - In the course of the deliberations, it was further expressly made clear, specially with
seeking to prohibit and regard to Commissioner Joaquin F. Bernas' accepted amendment to the amendment of
enjoin respondent (CIR) Commissioner Rigos, that the salaries of members of the Judiciary would be
from making any subject to the general income tax applied to all taxpayers.
deduction of withholding - The Court hereby reiterates that the salaries of Justices and Judges are properly
taxes from their salaries.
subject to a general income tax law applicable to all income earners and that
Claiming that “any tax
the payment of such income tax by Justices and Judges does not fall within the
withheld from their
constitutional protection against decrease of their salaries during their
emoluments or
continuance in office.
compensations as
- It is plain that the Constitution authorizes Congress to pass a law fixing another rate of
judicial officers
compensation of Justices and Judges but such rate must be higher than that which they
constitutes a decrease
are receiving at the time of enactment, or if lower, it would be applicable only to those
or diminution of their
appointed after its approval. It would be a strained construction to read into the
salaries, violates Sec
provision an exemption from taxation in the light of the discussion in the Constitutional
10 of Article VII
Commission.

AP: SC accord due respect to the intent of the people, through the discussions and deliberations
of their representatives, in the spirit that all citizens should bear their aliquot part of the cost of
maintaining the government and should share the burden of general income taxation equitably.

Section 11

A. Security of tenure

Vargas v. Rilloraza LB: For repugnancy to result it is not necessary that there should be an actual removal
of the disqualified Justice from his office. What matters here is not only that the Justice
Petitioner assails the affected continue to be a member of the court and to enjoy the emoluments as well as to exercise
validity of Sec. 14 of the the other powers and fulfill the other duties of his office, but that he be left unhampered to
People's Court Act, exercise all the powers and fulfill all the responsibilities of said office in all cases
Commonwealth Act 682, properly coming before his court under the constitution, again without prejudice to
which provided that the proper cases of disqualification under Rule 126. Any statute enacted by the legislature
President could designate which would impede him in this regard simply cannot become a law.
Judges of First Instance,
Judges-at-large of First Deprivation of a designated judge of his or their judicial power is equivalent to the
Instance or Cadastral deprivation of the judicial power of the court itself.
Judges to sit as
substitute Justices of the AP: were it not for the challenged section 14 of People’s Court Act (the provides for an additional
Supreme Court in treason grounds for disqualification for SC Justices and the Power of the president to appoint on account
cases without them of such DQ) there would have been an uninterrupted continuity in the tenure of the displaced
necessarily having to Justice and in his exercise of the powers and fulfillment of the duties appertaining to his office,
possess the required saving only proper cases or disqualification under Rule 126.
constitutional
qualifications of a regular
Supreme Court Justice. Pursuant to Article VIII, sections 4, 5, 6, and 9 of the Constitution the jurisdiction of the Supreme
Court may only be exercised by the Chief Justice and Associate Justices appointed by the
President with the consent of the Commission on Appointments, sitting in banc or in division, and
in cases like those involving treason they must sit in banc. If, according to section 4 of said Article
VIII, "the Supreme Court shall be composed" of the Chief Justice and Associate Justices therein
referred to, its jurisdiction can only be exercised by it as thus composed. To disqualify any of
these constitutional component members of the Court—particularly, as in the instant
case, a majority of them—in a treason case, is nothing short of pro tanto depriving the
Court itself of its jurisdiction as established by the fundamental law

De La Llana v. Alba LB: It is well-known rule also that valid abolition of offices is neither removal nor
separation of the incumbents. x x x And, of course, if the abolition is void, the incumbent is
BP 129, entitled “An Act deemed never to have ceased to hold office.
Reorganizing the - As well-settled as the rule that the abolition of an office does not amount to an
65
Judiciary, Appropriating illegal removal of its incumbent is the principle that, in order to be valid, the
Funds Therefor and for abolition must be made in good faith.”
Other Purposes”, was
passed. Gualberto De la Removal is to be distinguished from termination by virtue of the abolition of the office.
Llana, a judge in - There can be no tenure to a non-existent office. After the abolition, there is in
Olongapo, was assailing law no occupant.
its validity because he - In case of removal, there is an office with an occupant who would thereby lose
would be one of the
his position.
judges that would be
- As to its effect, no distinction exists between removal and the abolition of the office.
removed due to the
Realistically, it is devoid of significance. He ceases to be a member of the judiciary
reorganization. Judge
De la Llana asserts
that BP 129 AP: The question of any impairment of security of tenure does not arise. Nonetheless, for
contravened the the incumbents of inferior courts abolished, the effect is one of separation.
constitutional
provision which
provides the security
of tenure of judges of
the courts. He also
averred that only the
Supreme Court can
remove judges NOT
the Congress.

I: WON a judge like


Judge De La Llana can be
validly removed by the
legislature by such
statute (BP 129)? YES

Section 14

A. "Decisions" and "petitions," "minute resolutions."

Cobarrubias vs. People LB: The general rule is that where there is a conflict between the fallo, or the dispositive
part, and the body of the decision or order, the fallo prevails on the theory that the fallo is
Petitioner’s main the final order and becomes the subject of execution, while the body of the decision merely
contention is that there is contains the reasons or conclusions of the court ordering nothing,
a clerical error in the fallo - but where one can clearly and unquestionably conclude from the body of the
or the dispositive portion decision that there was a mistake in the dispositive portion, the body of the
of Judge Alumbres’ Order decision will prevail.
dated 20 March 2001,
which should have AP: In this case, considering the clear finding of the trial court that the prosecution failed to
dismissed Criminal Case prove beyond reasonable doubt the guilt of petitioner in the charges for Homicide and Frustrated
No. 945038 instead of Homicide, while the two other charges for Illegal Possession of Firearms and Violation of the
Criminal Case No. 94- Omnibus Election Code require further evidence, it is only just and proper to correct the
5037, considering that in dispositive portion to reflect the exact findings and conclusions of the trial court. Thus, in
the body of the order, accordance with the findings of the trial court, Criminal Case No. 94-5036 (Frustrated Homicide)
the trial court ruled that and Criminal Case No. 94-5038 (Homicide) should be dismissed, while Criminal Case No. 94-5037
the prosecution failed to (Illegal Possession of Firearms under Presidential Decree No. 1866) and Criminal Case No. 24-392
prove beyond reasonable (Violation of Section 261(Q) of the Omnibus Election Code in relation to Section 32 of Republic
doubt the guilt of Act No. 7166) should be set for further trial.
petitioner in the charges
for Frustrated Homicide
(Criminal Case No. 94-
5036) and Homicide
(Criminal Case No. 94-
5038).

Tayamura v. IAC LB: The Court reminds all lower courts, lawyers, and litigants that it disposes of the bulk of
its cases by minute resolutions and decrees them as final and executory,
Borromeo charges the - as where a case is patently without merits
division’s clerks of court. - where the issues raised are factual in nature,
Chief of the Judicial - where the decision appealed from is supported by: substantial evidence and, is in
Records Office of this accord with the facts of the case and the applicable laws,
Court with usurpation of - where it is clear from the records that the petition is filed merely to forestall the
judicial functions, for early execution of judgment and for non-compliance with the rules.
allegedly "maliciously and The resolution denying due course or dismissing the petition always gives the legal basis.
deviously issuing biased,
fake, baseless and For a prompt dispatch of actions of the Court, minute resolutions are promulgated by
unconstitutional the Court through the Clerk of Court, who takes charge of sending copies thereof to the
'Resolution' and 'Entry of parties concerned by quoting verbatim the resolution issued on a particular case.
Judgment' in G.R. No. - It is the Clerk of Court's duty to inform the parties of the action taken on their cases by
82273. quoting the resolution adopted by the court. The Clerk of Court never participates in the
deliberations of case. All decisions and resolutions are actions of the Court. The Clerk of
I: Whether the minute Court merely transmits the Court's action.
resolutions issued by the
Court did not comply with Minute resolutions need not be signed by the members of the Court who took part in
the constitutional the deliberations of a case nor do they require the certification of the Chief Justice.
requirements? Compliant. - For to require members of the court to sign all resolutions issued would not only unduly
66
delay the issuance of its resolutions but a great amount of their time would be spent on
functions more properly performed by the Clerk of court and which time could be more
profitably used in the analysis of cases and the formulation of decisions and orders of
important nature and character.

AP: The September 13, 1989 resolution of the Supreme Court through its Third Division which
disposed of Borromeo's petition is a four-page resolution which more than adequately complies
with the constitutional requirements governing resolutions refusing to give due course to petitions
for review. The petition and its incidents were discussed and deliberated upon by the Justices of
the Third Division.

Francisco v. Permskul LB: The memorandum decision authorized under Sec. 40, BP 129 should actually embody the
findings of fact and conclusions of law of the lower court in an annex attached to and made an
The question squarely indispensable part of the decision.
presented to the Court is - The memorandum decision, to be valid, cannot incorporate the findings of fact and the
the validity of the conclusions of law of the lower court only by remote reference, which is to say that the
memorandum decision challenged decision is not easily and immediately available to the person reading the
authorized under Section memorandum decision.
40 of B.P. Blg. 129 in the - For the incorporation by reference to be allowed, it must provide for direct access to the
light of Article VIII, facts and the law being adopted, which must be contained in a statement attached to the
Section 14 of the said decision.
Constitution.
Memorandum decision should be sparingly used lest it become an addictive excuse for
judicial sloth
- It is an additional condition for its validity that this kind of decision may be resorted to
only in cases where the facts are in the main accepted by both parties or easily
determinable by the judge and there are no doctrinal complications involved that will
require an extended discussion of the laws involved.
- The memorandum decision may be employed in simple litigations only, such as ordinarily
collection cases, where the appeal is obviously groundless and deserves no more than
the time needed to dismiss it.
-
Odchigue-Bondoc vs. LB: Section 14, Article VIII of the Constitution does not extend to resolutions issued by
Tan Tiong Bio  the Department of Justice (DOJ) Secretary.
- even the action of the Secretary of Justice in reviewing a prosecutor’s order or
Respondent filed a resolution via appeal or petition for review cannot be considered a quasi-judicial
complaint for estafa proceeding since the “DOJ is not a quasi-judicial body.”
against Fil-Estate officials
including its Corporate AP: Sec. 14, Art. VIII of the Constitution does not extend to decisions or rulings of
Secretary, herein executive departments such as the DOJ; and that Section 12(c) of the NPS Rule on Appeal
respondent. Petitioner allows the DOJ to dismiss a petition for review motu proprio
denies the allegations.
The DOJ, by resolution Other issues:
signed by the Chief State A preliminary investigation is not a quasi-judicial proceeding since “the prosecutor in a
Prosecutor for the preliminary investigation does not determine the guilt or innocence of the accused.”
Secretary of Justice,
motu proprio dismissed
the petition on finding
that there was no
showing of any reversible
error.

The CA set aside the DOJ


Secretary’s resolution
holding that it committed
grave abuse of discretion
in issuing its Resolution
in violation of Art VIII
sec. 14.

Section 15

Court Administrator v. LB: Failure to resolve cases submitted for decision within the period fixed by law constitutes a
Quianola serious violation of the constitutional right of the parties to a speedy disposition of their cases,
and this transgression is compounded when a judge continues to collect his salaries upon
OCA conducted a judicial certification that all cases and motions pending before him had been determined and decided
and physical inventory of within the reglementary period.
cases pending before the
MTC of San Pedro, The failure of a judge to decide cases promptly and expeditiously within the constitutionally
Laguna, in view of the prescribed 90-day period constitutes gross inefficiency, which consequently warrants
then impending administrative sanctions.
compulsory retirement of
its presiding judge, AP: In this case, respondent judge was remiss in the performance of his duties. As borne out by
Leonardo Quiñanola. The the findings of the OCA, he failed to decide twelve (12) cases within the prescribed period and to
OCA's Judicial Audit resolve matters pending in forty-one (41) others.
Team 1 reported that,
there are 1,268 pending
cases before the sala of That he was burdened with a heavy case load and suffered “intracerebral hematoma (L) [in the]
Judge Quiñanola basal ganglia” serves only to mitigate the penalty, not to exonerate him. To be completely
faultless, he should have written this Court to explain his predicament and to ask for extensions
of time

67
Re: Problem of Delays LB: Article VIII, Section 15(1) and (2) of the 1987 Constitution does not apply to the
in Cases before the Sandiganbayan for the said court is not a regular court but a special one.
Sandiganbayan - Special courts are judicial tribunals exercising limited jurisdiction over
particular or specialized categories of actions.
Administrative Circular - The above provision does not apply to the Sandiganbayan. The provision refers to
No. 10-94 requires all regular courts of lower collegiate level that in the present hierarchy applies
Courts to submit to the only to the Court of Appeals.
Supreme Court a bi- - The Sandiganbayan is a special court of the same level as the Court of Appeals and
annual report indicating possessing all the inherent powers of a court of justice, with functions of a trial court.
the title, date of filing, - The Sandiganbayan was originally empowered to promulgate its own rules of procedure.
date of pre-trial in and However, on March 30, 1995, Congress repealed the Sandiganbayan’s power to
arraignment, the date of promulgate its own rules of procedure and instead prescribed that the Rules of Court
initial trial, the date of promulgated by the Supreme Court shall apply to all cases and proceedings
last hearing and the date filed with the Sandiganbayan.
that the case is - Special courts are judicial tribunals exercising limited jurisdiction over particular or
submitted for decision, specialized categories of actions. They are the Court of Tax Appeals, the Sandiganbayan,
and to post, in a and the Shari’a Courts.
conspicuous place within
its premises, a monthly AP: Since sec. 15 does not apply with SB:
list of cases submitted for
decision.
Under Article VIII, Section 5 (5) of the Constitution “Rules of procedure of special courts and
quasi-judicial bodies shall remain effective unless disapproved by the Supreme Court.” The law
I: What is the creating the Sandiganbayan, P.D. No. 1606 is clear on this issue: judgment shall be rendered
reglementary period within three (3) months from the date the case was submitted for decision
within which the
Sandiganbayan must
As to the applicability of Administrative Circular No. 10-9:
decide/resolve cases
falling within its
jurisdiction? Is Supreme Yes it is applicable with SB.
Court Administrative Given the rationale behind the Administrative Circular, SC hold that it is applicable to the
Circular No. 1094 Sandiganbayan with respect to cases within its original and appellate jurisdiction.
applicable to the - Decision making is the primordial and most important duty of the member of the bench.
Sandiganbayan? Yes Hence, judges are enjoined to decide cases with dispatch. Their failure to do so
constitutes gross inefficiency that warrants disciplinary sanction, including fine,
suspension and even dismissal. The rule particularly applies to justices of the
Sandiganbayan.
- Delays in the disposition of cases erode the faith and confidence of our people in the
judiciary, lower its standards, and bring it into disrepute. Delays cannot be sanctioned or
tolerated especially in the anti-graft court, the showcase of the nation’s determination to
succeed in its war against graft.
-
Conclusion: Due to a finding of lack of bad faith on the part of respondent justice, we issued only
a warning. However, the dispositive portion of the decision cautioned respondent justice that “a
repetition of the same or similar act in the future shall be dealt with more severely.

Article IX – Constitutional Commissions

Art. IX-A

Sec. 2 Prohibition to Hold Other Office

Funa v. CSC LB: 1) To safeguard the independence of these Commissions, the 1987 Constitution,
among others,31 imposes under Section 2, Article IX-A of the Constitution certain
(President Arroyo inhibitions and dis qualifications upon the Chairmen and members to strengthen their
appointed Duque as integrity, to wit:
CSC. PGMA then issued (a) Holding any other office or employment during their tenure;
EO 864 designating (b) Engaging in the practice of any profession;
Duque as an ex officio (c) Engaging in the active management or control of any business which in any way may be
member of the Board affected by the functions of his office; and
of Directors/ Trustees (d) Being financially interested, directly or indirectly, in any contract with, or in any
of the GSIS, franchise or privilege granted by the Government, any of its subdivisions, agencies or
PhilHealth, ECC, and instrumentalities, including government-owned or -con -trolled corporations or their
HDMF. This is in subsidiaries.
pursuance of the
2) GR: All appointive officials in the civil service are allowed to hold other office or
Administrative Code.
employment in the government during their tenure when such is allowed by law or by the
Funa filed a petition
primary functions of their positions.
challenging the
constitutionality of
Duque’s designation, EXC: The President, Vice-President, Members of the Cabinet, their deputies and assistants. May
claiming that violates hold another office or employment in the government only when authorized by the Constitution.
Sec. 1 and 2 or Art. IX-
A of the Constitution. AP: Being an appointive public official who does not occupy a Cabinet position (i.e., President, the
That the designation Vice-President, Members of the Cabinet, their deputies and assistants), Duque was thus
violates the covered by the general rule.
independence of the
CSC and violates the
LB: 3) Ex officio means “from office; by virtue of office.” It refers to an “authority derived from
prohibition imposed
official character merely, not expressly conferred upon the individual character, but rather
68
upon members of annexed to the official position.” “act done in an official character, or as a consequence of office,
constitutional and without any other appointment or authority other than that conferred by the office."
commissions from REASON: These services are already paid for and covered by the compensation attached to his
holding any other principal office.
office or employment.) AP: The SC also notes that Duque’s designation as member of the governing Boards of the GSIS,
I: Does the PHILHEALTH, ECC and HDMF entitles him to receive per diem, a form of additional compensation
designation of Duque that is disallowed by the concept of an ex officio position by virtue of its clear contravention of the
as member of the proscription set by Section 2, Article IX-A of the 1987 Constitution.
Board of Directors or
Trustees of the GSIS, LB: 4) The CSC Chairman cannot be a member of a government entity that is under the control
PHILHEALTH, ECC and of the President without impairing the independence vested in the CSC by the 1987
HDMF, in an ex officio Constitution.
capacity, impair the
independence of the
CSC and violate the AP: Duque’s designation as member of the governing Boards of the GSIS, PHILHEALTH, ECC and
constitutional HDMF impairs the independence of the CSC. Undoubtedly, the GSIS, PHILHEALTH, ECC and HDMF
prohibition against the and the members of their respective governing Boards are under the control of the President.
holding of dual or As such, the CSC Chairman cannot be a member of a government entity that is under the control
multiple offices for the of the President without impairing the independence vested in the CSC by the 1987 Constitution.
Members of the
Constitutional
Commissions? YES. EO
864 is unconstitutional.

Sec. 5 Fiscal Autonomy

CSC v. Department of LB: “Automatic release” of approved annual appropriations to the Civil Service Commission, a
Budget constitutional commission which is vested with fiscal autonomy, should be construed to mean
that no condition to fund releases to it may be imposed.
(CSC via the present
petition for mandamus  This conclusion is consistent with the une 3, 1993 Resolution of this Court which
seeks to compel the effectively prohibited the enforcement of a “no report, no release” policy against the
Department of Budget Judiciary which has also been granted fiscal autonomy by the Constitution.
and Management to
release the balance of
its budget for fiscal Even assuming that there is a shortfall in revenues, that does not justify non-compliance
year 2002. At the with the mandate of Article IX (A), Section 5 of the Constitution.
same time, it seeks a
determination by this To hold that the Civil Service Commission may be subjected to withholding or reduction
Court of the extent of of funds in the event of a revenue shortfall would, to that extent, place it and the other
the constitutional entities vested with fiscal autonomy on equal footing with all others which are not
concept of fiscal granted the same autonomy, thereby reducing to naught the distinction established by the
autonomy.) Constitution; 
CSC’s contention:
Balance was
intentionally withheld
 The agencies which the Constitution has vested with fiscal autonomy should be given
by respondent on the priority in the release of their approved appropriations over all other agencies not
basis of its “no report, similarly vested when there is a revenue shortfall.
no release” policy
whereby allocations The plain implication of the omission in Article IX (A), Section 5 of the Constitution of the
for agencies are provision proscribing a reduction of appropriations below that for the previous year is that
withheld pending their Congress is not prohibited from reducing the appropriations of Constitutional
submission of the Commissions below the amount appropriated for them for the previous year.
documents mentioned
in National Budget AP: Here, the “no report, no release” policy may not be validly enforced against offices
Circular No. 478 vested with fiscal autonomy is not disputed. Indeed, such policy cannot be enforced against
I: W/N DBM’s act of offices possessing fiscal autonomy without violating Article IX (A), Section 5 of the Constitution
withholding the subject
funds from CSC due to
revenue shortfall is Respecting respondent’s justification for the withholding of funds from petitioner as due to a
UNCONSTITUTIONAL? shortfall in revenues, the same does not lie. In the first place, the alleged shortfall is totally
YES unsubstantiated. In the second place, even assuming that there was indeed such a shortfall, that
does not justify non-compliance with the mandate of above-quoted Article IX (A), Section 5 of the
Constitution.

 If respondent’s theory were adopted, then the constitutional mandate to automatically


and regularly release approved appropriations would be suspended every year, or even
every month that there is a shortfall in revenues, thereby emasculating to a significant
degree, if not rendering insignificant altogether, such mandate.

While the retention or reduction of appropriations for an office is generally allowed when there is
an unmanageable budget deficit, the Year 2002 GAA, in conformity with the Constitution,
excepted from such rule the appropriations for entities vested with fiscal autonomy. 

 Thus, even assuming that there was a revenue shortfall as respondent claimed, it could
not withhold full release of petitioner’s funds without violating not only the Constitution
but also Section 64 of the General Provisions of the Year 2002 GAA.

Disposition: Petition granted. DBM’s act of withholding the subject funds from CSC due to
revenue shortfall is UNCONSTITUTIONAL.

NOTE: 
The phrase “subject to availability of funds” in paragraph 4 of the guiding principles on the
69
Constitutional Mandate on the Judiciary’s Fiscal Autonomy as contained in the Court’s Resolution
in A.M. No. 92-9-029-SC —contemplates a situation where total revenue collections are so
low that they are not sufficient to cover the total appropriations for all entities vested
with fiscal autonomy; Considering that the budget for agencies enjoying fiscal autonomy is only
a small portion of the total national budget, only in the most extreme circumstances will the
total revenue collections fall short of the requirement of such agencies

Sec. 6 Rules of Procedure

Aruelo v. CA Petitioner filed the election protest with the RTC, whose proceedings are governed by the Revised
Rules of Court.
(Aruelo claims that in
election contests, the LB: 1) The COMELEC Rules of Procedure is not applicable to proceedings before the regular
COMELEC Rules of courts. As expressly mandated by the COMELEC Rules of Procedure, the filing of motions to
Procedure gives the dismiss and bill of particulars, shall apply only to proceedings brought before the
respondent therein COMELEC.
only five days from  It must be noted that nowhere in Part VI of the COMELEC Rules of Procedure is it
receipt of summons provided that motions to dismiss and bill of particulars are not allowed in election
within which to file his protests or quo warranto cases pending before the regular courts.
answer to the petition
(Par and that this five- 2) Constitutionally speaking, the COMELEC cannot adopt a rule prohibiting the filing of
day period had lapsed certain pleadings in the regular courts. The power to promulgate rules concerning
when Gatchalian filed pleadings, practice and procedure in all COURTS is vested on the Supreme Court.
his answer.
AP: Private respondent received a copy of the order of the RTC denying his motion for a bill of
According to him, the particulars on August 6, 1992.
filing of motions to  Under Revised Rules of Court, a party has at least five days to file his answer after
dismiss and motions receipt of the order denying his motion for a bill of particulars.
for bill of particulars is
prohibited by the  Private respondent, therefore, had until August 11, 1992 within which to file his
COMELEC Rules of answer. The Answer with Counter-Protest and Counterclaim filed by him on August
Procedure; hence, the 11, 1992 was filed timely.
filing of said pleadings
did not suspend the
 The instant case is different from a pre-proclamation controversy which the law
running of the five-day
expressly mandates to be resolved in a summary proceeding.
period, or give
Gatchalian a new five-
day period to file his  Pre-proclamation controversies should be summarily decided, consistent with the
answer.) legislators’ desire that the canvass of the votes and the proclamation of the winning
I: W/N RTC committed candidate be done with dispatch and without unnecessary delay.
GADALEJ when it
allowed respondent  Questions as those involving the appreciation of the votes and the conduct of the
Gatchalian to file his balloting, which require more deliberate and necessarily longer consideration, are left for
pleading beyond the examination in the corresponding election protest.
five-day period
prescribed in Section
1, Rule 13, Part III of
the COMELEC Rules of
Procedure. NO.

Macalintal v. Comelec LB: Congress may not assume power to review rules promulgated by Comelec.

Macalintal seeking a The ambit of legislative power under Article VI of the Constitution is circumscribed by other
declaration that certain constitutional provisions. One such provision is Section 1 of Article IX-A of the 1987
provisions of Republic Act Constitution ordaining that constitutional commissions such as the COMELEC shall be
No. 9189 (The Overseas “independent.”
Absentee Voting Act of  the Court has held that “[w]hatever may be the nature of the functions of the
2003) Commission on Elections, the fact is that the framers of the Constitution wanted it to be
independent from the other departments of the Government
Macalintal’s contention: 
Once a law is enacted and approved, the legislative function is deemed accomplished and
The creation of the complete. The legislative function may spring back to Congress relative to the same law only if
Joint Congressional that body deems it proper to review, amend and revise the law, but certainly not to
Oversight Committee approve, review, revise and amend the IRR of the COMELEC.
with the power to
review, revise, amend
and approve the IRR AP: It is not correct to hold that because of its recognized extensive legislative power to enact
promulgated by the election laws, Congress may intrude into the independence of the COMELEC by exercising
COMELEC intrudes into supervisory powers over its rule-making authority. Congress trampled upon the constitutional
the independence of mandate of independence of the COMELEC. 
the COMELEC which, as
a constitutional body.

I: May Congress
assume power to
review rules
promulgated by the
Comelec? NO.

70
Sec. 7 How Commissions Decide

Estrella v. Comelec LB:


1.) Nowhere in the COMELEC Rules does it allow a Commissioner to voluntarily inhibit
(A status quo ante with reservation.
order was issued by
the COMELEC and from 2.) The COMELEC En Banc shall decide a case or matter brought before it by a majority vote of
the 7 members only 5 “ALL ITS MEMBERS,” and not majority of the members who deliberated and voted thereon—had
participated. From the the framers intended that it should be the majority of the members who participated or
5 participants 1 deliberated, it would have clearly phrased it that way
dissented. 3.) D: The provision of the Constitution is clear that it should be the majority vote of all its
members and not only those who participated and took part in the deliberations.
According to the  Under the rules of statutory construction, it is to be assumed that the words in which
Constitution 4 will constitutional provisions are couched express the objective sought to be attained.
constitute the  Since the above-quoted constitutional provision states “all of its members,” without any
majority. However, qualification, it should be interpreted as such.
Lantion, one of the 4
AP: COMELEC= 7 Incumbent Members.
members who agreed
In this case only five of the seven incumbent members participated (Abalos; Tangcangco;
to the order was
Javier; Borra; and Lantion). Out of 5 who participated, FOUR (4) voted for issuance
disallowed by the SC
(Abalos; Tangcangco; Javier; and Lantion) and 1 dissented (Borra)
to participate because
 For COMELEC En Banc to arrive at a decision; Section 5. Quorum; Votes Required. – (a)
he had previously
When sitting en banc, four (4) Members of the Commission shall constitute a quorum for
inhibited himself on
the purpose of transacting business. The concurrence of a majority of the Members of
the case. The effect is
the Commission shall be necessary for the pronouncement of a decision, resolution,
that there are
order or ruling.
effectively only 4
participants and from Since Commissioner Lantion could not participate and vote in the issuance of the questioned
these there is 1 order, thus leaving three (3) members concurring therewith, the necessary votes of four (4)
dissent.) or majority of the members of the COMELEC was not attained.
The order thus failed to comply with the number of votes necessary for the
I: W/N the vote of 3 pronouncement of a decision or order, as required under Rule 3, Section 5(a) of the
who agreed to the COMELEC Rules of Procedure stated above.
order can be
considered a majority
vote. NO.

MAJORITY OF 7 IS 4 NOT
3
Mateo v. CA LB:   Under the present rule, final resolutions of the Civil Service Commission shall be
appealable to the Court of Appeals
Whether the RTC has  Revised Circular No. 1-91 as amended by Revised Administrative Circular No. 1-95 which
jurisdiction over the took effect on June 1, 1995, final resolutions of the Civil Service Commission shall be
special civil case appealable to the Court of Appeals.
involving dismissal of
an employee of quasi-
public corporation? NO.  In any event, whether under the old rule or the present rule, Regional Trial Courts have
no jurisdiction to entertain cases involving dismissal of officers and employees covered
by the Civil Service Law.

AP: MOWAD is a quasi-public corporation, which fall under the jurisdiction of CSC. CSC rules
intially provides
 that the party aggrieved by a decision, ruling, order, or action of an agency of the
government involving termination of services may appeal to the Commission within
fifteen (15) days.
 Thereafter, private respondent could go on certiorari to the Supreme Court under Rule
65 of the Rules of Court if he still feels aggrieved by the ruling of the Civil Service
Commission as held in Mancita v. Barcinas case. 
HOWEVER, Mancita, however, no longer governs. Private respondent may now appeal CSC’s
final resolutionl before CA, not RTC.

Ambil v. Comelec LB: 1.) The Supreme Court has no power to review via certiorari an interlocutory order or even a
final resolution of a Division of the Commission on Elections.
(W/N interlocutory
orders by the 2.) A decision, order or resolution of a division of the Comelec must be reviewed by the
COMELEC in division Comelec en banc via a motion for reconsideration before the final en banc decision may
are appealable be brought to the Supreme Court on certiorari. The pre-requisite filing of a motion for
through certiorari to reconsideration is mandatory.
the Supreme Court?) 3.) D: The exceptions to the rule in certiorari cases, dispensing with a motion for reconsideration
NO. prior to the filing of the petition, do not apply to election cases where a motion for reconsideration
is mandatory by Constitutional fiat to elevate the case to the Comelec en banc.

AP: In this case, petitioner filed a premature petition for certiorari with the SC,
challenging an interlocutory order issued by the COMELEC First Division, without
waiting for a promulgation of the case or filing a motion for reconsideration should he
have been aggrieved by the same. The petitioner failed to exhaust all available
administrative remedies before the COMELEC. Thus, the petition was dismissed.
ISSUE about the Resolution of Commissioner Guaini
W/N the Resolution has any value. NONE.

LB: Before that resolution or decision is signed and promulgated, there is no valid resolution or
decision to speak of.
 A final decision or resolution becomes binding only after it is promulgated and not
71
before.
 Accordingly, one who is no longer a members of the Commission at the time the final
decision or resolution is promulgated cannot validly take part in that resolution or
decision. Much less could he be the ponente of the resolution or decision.

The SC declared the resolution signed by Commissioner Guani as void for various reasons.
(a) First, one who is no longer a member of the Commission at the time the final decision or
resolution is promulgated cannot validly take part in that resolution or decision.
(b) Second, the Clerk of the 1st Division denied the release or promulgation of the
resolution.
(c) Third, the 1st Division even later said that the parties should ignore the resolution since
it was not yet promulgated.
(d) Lastly, Commissioner Guiani could not have affixed her signature on the resolution, since
on the same date an order was issued where she said that she still wanted to see both
positions before making her decision.

Art. IX-B

A. Civil Service Commission (“CSC”)

Scope

CSC v. Alfonso LB: As the central personnel agency of the government, the CSC has jurisdiction to
supervise the performance of and discipline, if need be, all government employees,
Alfonso, Director of the including those employed in government-owned or controlled corporations with original
HRM Department of PUP charters such as PUP.
was charged with grave  The CSC has appellate jurisdiction over disciplinary cases decided by government
misconduct, conduct departments, agencies and instrumentalities. 
prejudicial to the best
interest of the Service,
and violation of the Civil  However, a complaint may be filed directly with the CSC, and the Commission has the
Service Law (worked authority to hear and decide the case, although it may opt to deputize a department or
24hrs straight daily in 3 an agency to conduct the investigation.
consecutive weeks with
OT pay) While  there are certain special laws that allow the creation of disciplinary committees and
governing bodies in different branches the government to hear and decide administrative
Alfonso’s contention:  complaints against their respective officers and employees, it cannot be interpreted as to divest
CSC of its inherent power to supervise and discipline government employees, including those in
the academe.
the CSC had no
jurisdiction to hear and
decide the administrative  To hold otherwise would not only negate the very purpose for which the CSC was
case because it is the PUP established, i.e. to instill professionalism, integrity, and accountability in our civil service,
Board of Regents that has but would also impliedly amend the Constitution itself.
the exclusive authority to
appoint and remove PUP AP: Accordingly, all PUP officers and employees, whether they be classified as teachers or
employee
professors pursuant to certain provisions of law, are deemed, first and foremost, civil servants
accountable to the people and answerable to the CSC in cases of complaints lodged by a citizen
I: Whether CSC has against them as public servants.
jurisdiction to hear the
case against respondent The SC also ruled that since the complaints were filed directly with the CSC and the CSC had
Alfonso? YES opted to assume jurisdiction over the complaint, the CSC’s exercise of jurisdiction shall be to the
exclusion of other tribunals exercising concurrent jurisdiction.

PNOC-EDC v. Leogardo LB: Test in determining whether a government-owned or controlled corporation is


I: W/N the PNOC-EDC’s subject to the Civil Service Law
employees are governed
by the Labor Code. YES. The test in determining whether a government-owned or controlled corporation is subject to the
Civil Service Law is the manner of its creation such that government corporations created
by special charter are subject to its provisions while those incorporated under the
general Corporation Law are not within its coverage.

AP: PNOC-EDC, having been incorporated under the general Corporation Law, is a
government-owned or controlled corporation whose employees are subject to the provisions of
the Labor Code. This is apparently the intendment in the NASECO case notwithstanding the fact
that the NASECO therein was a subsidiary of the PNB, a government-owned corporation.

Phil. Fisheries v. NLRC LB: There being no employer-employee relationship between a GOCC and the security
Private Respondent filed guards, the jurisdiction of the Civil Service Commission may not be invoked
a case before LA for
unpaid readjusted wages AP: The petitioner is a GOCC with a special charter. This places it under the scope of the civil
of security guards of service.
Petitioners. LA dismissed
the case saying that since
the petitioner is a GOCC  However, the guards are not employees of the petitioner. The contract of services
then it is under the scope explicitly states that the security guards are not considered employees of the petitioner.
and jurisdiction of the
CSC.  The contract entered into by the petitioner which is merely job contracting makes the
petitioner an indirect employer. 
72
I: W/N CSC has
jurisdiction over security
guards of Petitioner? No.
 Notwithstanding that the petitioner is a government agency, its liabilities, which are
joint and solidary with that of the contractor, are provided in Articles 106, 107 and 109
of the Labor Code. This places the petitioner’s liabilities under the scope of the
NLRC.

The petitioner can not escape liability since the law provides the joint and solidary liability of the
principal and the contractor for the protection of the laborers. 

Disposition: petitioner and the private respondent jointly and severally liable to the security
guards for the unpaid wage differentials under Wage Order No. 6. 

Buenaflor v. Ramirez LB: 1.) Disciplinary cases and cases involving personnel actions affecting employees in the Civil
(The petitioner Service, like appointment or separation from the service, are within the exclusive jurisdiction of
terminated Ramirez as of the CSC.
the same date as
Chairman Eugenio’s AP: It cannot be disputed that Ramirez’s complaint was thereby challenging the validity of his
resignation on the ground termination from the service. Such challenge was outside of the RTC’s sphere of authority.
that his tenure had Instead, it was the CSC that was vested by law with jurisdiction to do so.
expired by virtue of the
position being personal
and confidential, and, LB: 2.) The Constitution vests in the CSC the jurisdiction over all employees of the
coterminous with that of Government, including all its branches, subdivisions, instrumentalities, and agencies, as well as
the appointing authority. government-owned or -controlled corporations with original charters.
Ramirez sued in the RTC
to declare his dismissal AP: Here, Ramirez was one such employee. The agency in which he had been appointed by
null and void. Buenaflor, Chairman Domingo was the PAGC, an office established by PGMA through Executive Order No.
represented by the OSG, 1225 as an agency under the Office of the President. His complaint thus came under the
contended that Ramirez jurisdiction of the CSC.
had failed to exhaust
administrative remedies
CLARIFICATION TO THE ABOVE RULING:
and should have instead
It is clarified that the CSC has jurisdiction over a case involving a civil servant if it can be
filed an administrative
regarded as equivalent to a labor dispute resoluble under the Labor Code; conversely, the
complaint in the CSC. )
regular court has jurisdiction if the case can be decided under the general laws , such as
I: W/N the RTC has no
when the case is for the recovery of private debts, or for the recovery of damages due to
jurisdiction since the case
slanderous remarks of the employer, or for malicious prosecution of the employees.
is under the sole
The mere fact that the parties are members of the Civil Service should not remove the
jurisdiction of the CSC.
controversy from the general jurisdiction of the courts of justice and place them under the special
jurisdiction of the CSC.

Career Executive LB: As the central personnel agency of the government, the CSC has broad authority to
Service Board v. CSC pass upon all civil service matters.

PAO assailed petitioner’s Article IX-B of the 1987 Constitution entrusts to the CSC the administration of the civil service,
rqmt on third level which is comprised of “all branches, subdivisions, instrumentalities, and agencies of the
eligibility, as the position Government, including government- owned or -controlled corporations with original charters.”
in question are
permanent employees as
In accordance with the Consti Comm’s deliberations, the mandate of the CSC should therefore
provided by law. PAO
be read as the comprehensive authority to perform all functions necessary to ensure
appealed to CSC and
the efficient administration of the entire civil service, including the CES.
ruled in favor of PAO and
reversed CESB’s reso.
The Administrative Code of 1987 further reinforces this view. Book V, Title I, Subtitle A,
Issue: Whether CSC has Chapter 3, Section 12 thereof enumerates the specific powers and functions of the CSC while
jurisdiction to review the recognizing its comprehensive authority over all civil service matters.
Resolution of the CESB,
given the latter’s SECTION 12. Powers and Functions.—The Commission shall have the following powers and
autonomy as an attached functions:
agency? Yes (1) Administer and enforce the constitutional and statutory provisions on the merit system for all
levels and ranks in the Civil Service;
(2) Prescribe, amend and enforce rules and regulations for carrying into effect the provisions of
the Civil Service Law and other pertinent laws;
(3) Promulgate policies, standards and guidelines for the Civil Service and adopt plans and
programs to promote economical, efficient and effective personnel administration in the
government;
(4) Formulate policies and regulations for the administration, maintenance and implementation of
position classification and compensation and set standards for the establishment, allocation and
reallocation of pay scales, classes and positions;
(5) Render opinion and rulings on all personnel and other Civil Service matters which shall be
binding on all heads of departments, offices and agencies and which may be brought to the
Supreme Court on certiorari;
xxxx
(11) Hear and decide administrative cases instituted by or brought before it directly or on appeal,
including contested appointments, and review decisions and actions of its offices and of the
agencies attached to it. Officials and employees who fail to comply with such decisions, orders, or
rulings shall be liable for contempt of the Commission. Its decisions, orders, or rulings shall be
final and executory. Such decisions, orders, or rulings may be brought to the Supreme Court on
certiorari by the aggrieved party within thirty (30) days from receipt of a copy thereof;
(14) Take appropriate action on all appointments and other personnel matters in the Civil Service
including extension of Service beyond retirement age;
xxxx
(19) Perform all functions properly belonging to a central personnel agency and such other
functions as may be provided by law.
73
AP: The CSC acted within its jurisdiction when it resolved the PAO’s ap- peal and
reversed CESB Resolution No. 918.

Other Issue: As to harmonizing the function of CESB and CSC

D: the specific powers of the CESB over members of the CES must be interpreted in a manner
that takes into account the comprehensive mandate of the CSC under the Constitution and other
statutes.

AP: The present case involves the classification of positions belonging to the CES and the
qualifications for these posts. These are matters clearly within the scope of the powers granted to
the CESB under the Administrative Code and the Integrated Reorganization Plan. However, this
fact alone does not push the matter beyond the reach of the CSC.

D: The CSC, as the central personnel agency of the government, is given the comprehensive
mandate to administer the civil service. It has also been expressly granted the power to
promulgate policies, standards, and guidelines for the civil service; and to render opinions and
rulings on all personnel and other civil service matters.

The CSC has been granted the authority to review the decisions of agencies attached to it under
Administrative Code

AP: Here, the question of whether the subject PAO positions belong to the CES is clearly a civil
service matter falling within the comprehensive jurisdiction of the CSC.

Further, considering the repercussions of the issue concerning the appointments of those
occupying the posts in question, the jurisdiction of the CSC over personnel actions is implicated.

Since the CESB is an attached agency of the CSC, the former’s decisions are expressly
subject to the CSC’s review on appeal.

Disposition: The CSC correctly ruled that third level eligibility is not re- quired for the
subject positions.
 The CESB effectively amended the law when it required the occupants of the subject PAO
positions to obtain third-level eligibility.

Appointments

Santiago v. CSC LB: 1) No mandatory nor peremptory requirement in the (Civil Service Law) that
(Customs Commissioner persons next-in-rank are entitled to preference in appointment.
Tanada extended to  What it does provide is that they would be among the first to be considered for the
Santiago a permanent vacancy, if qualified, and if the vacancy is not filled by promotion, the same shall be
promotional appointment filled by transfer or other modes of appointment.
from Customs Collector I
to Customs Collector III.  One who is next-in-rank is entitled to preferential consideration for promotion to the
Jose, a Customs Collector higher vacancy but it does not necessarily follow that he and no one else can be
II, protested, claiming appointed.
that he was the next-in-
rank to the position of
Collector of Customs III.  The rule neither grants a vested right to the holder nor imposes a ministerial
The Civil Service duty on the appointing authority to promote such person to the next higher
Commission revoked position.
Santiago’s promotion.
Hence, the present 2.) Ocampo vs. Subido- The power to appoint is a matter of discretion. The appointing
petition.) power has a wide latitude of choice as to who is best qualified for the position
I: W/N the next-in-rank  To apply the next-in-rank rule peremptorily would impose a rigid formula on the
rule applies in making appointing power contrary to the policy of the law that among those qualified and
appointments? NO. eligible, the appointing authority is granted discretion and prerogative of choice
of the one he deems fit for appointment.

3.) While it is true that the CSC is empowered to approve all appointments to positions in the civil
service and disapprove those where the appointees do not possess the appropriate eligibility or
required qualification, all the CSC is actually allowed to do is check whether or not the appointee
possesses the appropriate civil service eligibility or the required qualifications.
 If he does, his appointment is approved; if not, it is disapproved. No other criterion is
permitted by law to be employed by the Commission when it acts on, or as the decree
says, "approves" or "disapproves" an appointment made by the proper authorities.

 It has no authority to revoke the said appointment simply because it believed that the
jose was better qualified for that would have constituted an encroachment on the
discretion vested solely (in the appointing authority.

AP: Here, the Court fail to see any reason to disturb SANTIAGO’s promotional appointment. The
minimum qualifications and the standard of merit and fitness have been adequately satisfied as
found by the appointing authority. The latter has not been convincingly shown to have committed
any grave abuse of discretion.

74
Aquino v. CSC LB: It is well-settled that once an appointment is issued and the moment the appointee
Victor Aquino was assumes a position in the civil service under a completed appointment, he acquires a
designated as Property legal, not merely equitable right (to the position), which is protected not only by
Inspector and In-Charge statute, but also by the Constitution, and cannot be taken away from him either by
of the Supply Office revocation of the appointment, or by removal, except for cause, and with previous
performing the duties and notice and hearing.
responsibilities of the
Supply Officer.  Complete appointment: the acts of the head of a department or office making the appointment
and the Commissioner of Civil Service acting together, though not concurrently, but
2 years later, the Division consecutively, are necessary to make an appointment complete. (Favis v. Rupisan)
Superintendent of City
Schools, Tagle, issued a When the appointing power has once acted and the appointee has accepted the office and done
promotional appointment what is required of him upon its acceptance, his title to the office becomes complete, and he can
to Leonarda D. de la Paz then be removed only in the regular way. The appointing power can not effect his removal
as Supply Officer.  indirectly by rescinding or revoking his appointment after it is complete.
At the time of her  
appointment, she was The moment the discretionary power of appointment has been exercised and the
then holding the position appointee assumed the duties and functions of the position, the said appointment
of Clerk II, Division of cannot be revoked by the appointing authority on the ground merely that the protestant
City Schools.  is more qualified than the first appointee.
 Otherwise, the security of tenure guaranteed by Article IX- B, Section 2 par. (3) of the
Aquino filed a protest 1987 Constitution would be rendered meaningless if the appointing authority is allowed
with the DECS Secretary to flip-flop in exercising its discretionary power of appointment.
questioning the
qualification and
competence of de la Paz Definition of the concept of “for cause”
for the position of Supply  It means for reasons which the law and sound public policy recognized as sufficient
Officer I. warrant for removal, that is, legal cause, and not merely causes which the appointing
 DECS Secretary power in the exercise of discretion may deem sufficient.
revoked the
appointment of  It is implied that officers may not be removed at the mere will of those vested with the
de la Paz and power of removal, or without any cause. Moreover, the cause must relate to and affect
appointed the administration of the office, and must be restricted to something of a substantial
Aquino in the nature directly affecting the rights and interests of the public.”
same position.
 CSC revoked An appointment to an office which is not vacant is null and void ab initio.
Aquino’s
appointment
 There can be no appointment to a non-vacant position. The incumbent must first be
and restored legally removed or his appointment validly terminated 
de la Paz’s.
AP: In the instant case,  the CSC revoked the appointment of the successful protestant,
I: W/N the appointment petitioner herein, principally because the right to security of tenure of the prior appointee,
of Aquino is valid? No. private respondent, to the contested position had already attached. 
 It must be noted that CSC did not direct the appointment of a substitute of its choice. It
merely restored the appointment of private respondent who was first appointed to the
contested position.

The conclusion of respondent Commission in the questioned decision that private respondent is
more qualified than petitioner merely supports the validity of the restoration of private
respondent to her previously approved appointment.

The ground relied upon by petitioner in his protest that he is more qualified than private
respondent in terms of education, experience and training does not fall within the meaning of “for
cause” contemplated by Article IX-B, Section 2 par. (3) of the 1987 Constitution which would
warrant the revocation, if not removal, of the appointment of private respondent.
 Therefore, the protest of petitioner did not adversely affect the approval of the
appointment of private respondent.  

Appointment of Aquino to a non-vacant position is null and void.


Even on the assumption that the revocation of private respondent’s appointment was validly
exercised by DECS Secretary Quisumbing, still the appointment extended to petitioner was
tainted with irregularity as it was issued before the finality of the decision on the
protest in violation of CSC Resolution No. 83- 343 which prohibits the issuance of an
appointment to protestant (petitioner) if the protest case is not yet finally resolved, since there is
no vacancy in the position pending resolution of the protest case.

CSC v. Salas LB: 1.) Two recognized instances when a position may be considered primarily
confidential
(Salas was appointed by
the PAGCOR Chairman as Prior to the passage of the Civil Service Act of 1959, there were two recognized instances
Internal Security Staff when a position may be considered primarily confidential:
(ISS) member and (a) Firstly, when the President, upon recommendation of the Commissioner of Civil Service,
assigned to the casino. has declared the position to be primarily confidential; and,
His employment was
terminated by the (b) Secondly, in the absence of such declaration, when by the nature of the functions of
PAGCOR Board for loss of the office there exists “close intimacy” between the appointee and appointing
confidence, after a covert power which insures freedom of intercourse without embarrassment or freedom from
investigation yielded an misgivings of betrayals of personal trust or confidential matters of state.
alleged involvement of
Salas in proxy betting. ) 2.) The primary purpose of the framers of the 1987 Constitution in providing for the
75
I: W/N Salas, a member declaration of a position as policy-determining, primarily confidential or highly
of the PACGOR’s Internal technical is to exempt these categories from competitive examination as a means for
Security Staff is a determining merit and fitness .
confidential employee.  It must be stressed further that these positions are covered by security of tenure.
NO

Security of Tenure

Hernandez v. Villegas LB: Officers occupying primarily confidential positions may not be removed or
suspended without cause.
Atty. Villegas was  Even officers and employees of the civil service occupying primarily confidential positions
appointed Director for are subject to the constitutional safeguard against removal or suspension except for
Security of the Bureau of cause.
Customs. He was sent to
the US for further  How appointments to primarily confidential positions terminated.
studies. When he went
 Officials and employees holding primary confidential positions continue only for so
back his previous position
long as confidence in them endures. 
was assigned temporarily
to Keefe. He was the  The termination of their official relation can be justified on the ground of loss of
temporarilly assigned as confidence because in that case their cessation from office involves no removal but
Arrastre Supervisor. Their merely the expiration of the term of office.
assignment were then
made permanent.  The statement in De los Santos v. Mallare, to the effect that appointment to any of the three
classes of positions is terminable at the will of the appointing power, must be deemed a mere
The Directorship is a obiter. 
confidential position.  It has been correctly criticized as misleading. For if these three special positions do not
Arrastre Supervisor is a really belong to the Civil Service, the Constitution would not have specifically named
classified position. them as an exception to the general rule that all appointments must be made on the
basis of merit and fitness to be determined by competitive examinations.
Petitioner’s (sec. Of DOF)
contention:  AP: Here, as long as confidence in them endures—and it has been shown that it has been lost in
he Directorship is a this case—the incumbent is entitled to continue in office. Therefore Villegas’ removal from the
confidential position.  office of Director for Security is without cause and is therefore illegal.
Hence, terminable at the
will of the appointing
power. 

I:  W/N it is valid to


transfer Atty. Villegas
from Director of Security
to Arrastre
Superintendent?  NO.

Astraquillo v. LB: The appointment of government employees as “non-career” members of the


Manglapus Foreign Service may be terminated without justifiable cause.

(CONSOLIDATED The Civil Service Law, Presidential Decree No. 807, classifies employment in Government into
PETITIONS: Petitioners "career" and "non-career service." It identifies the peculiar characteristics of each category, and
were appointed enumerates the positions falling under each class.
ambassadors to different
countries. The Secretary AP: Here, it seems plain that all 3 petitioners: Isabelo J. Astraquillo, Alunan C. Glang, and
of Foreign Affairs, by the Alejandro Melchor, Jr., pertained to the Non-Career Service. Their appointments to the Foreign
authority of the Service as ambassadors of UAE, Kuwait and Moscow by Pres. Cory were made on bases other
President, then than those of the usual test of merit and fitness utilized for the career service; their entrance was
terminated their services. not 'based on merit and fitness determined by competitive examinations, or based on highly
Petitioners complained technical qualifications. This being so, their tenure was coterminous with that of the appointing
and contended that they authority or subject to his pleasures. Here, their removal is valid since it was in fact the President,
were employed under the by virtue of the Secretary of Foreign Affairs’ recommendation, who ordered their removal.
career service and thus
granted security of
tenure.) Note: The distinction between career and non-career officers may be derived by implication from
I: the provisions of the Foreign Service Act of 1952, R.A. No. 708, as amended. Part B, Title III of
said Foreign Service Act deals with the Career Service Corps composed of Foreign Service Officers
appointed by the President upon the recommendation of the Secretary," and declares that no
person shall be eligible for appointment (in such corps) unless he has passed such competitive
examinations as the Board of Foreign Service examination may prescribe to determine his fitness
and aptitude for the work of the service and has demonstrated his loyalty to the Government of
the Republic of the Philippines and his attachment to the principles of the Constitution." Those
who thus qualify are certified by the Secretary of Foreign Affairs as eligible for appointment as
Foreign Service Officers, and it is exclusively from these officers so certified that the President
shall appoint Foreign Service Officers.

Gloria v. CA LB: The transfer of a permanent employee to another permanent position without the
Dr. Icasiano was consent of the employee violates security of tenure.
appointed Schools
Division Superintendent, As to temporary transfer: While a temporary transfer or assignment of personnel is
Division of City Schools, permissible even without the employees’ prior consent, it cannot be done when the
QC.  transfer is a preliminary step toward his removal, or is a scheme to lure him away from
his permanent position, or designed to indirectly terminate his service, or force his
Sec. Gloria recommended resignation. Such a transfer would in effect circumvent the provision which safeguards the
to Pres. Aquino that  tenure of office of those who are in the Civil Service
76
Icasiano be reassigned as
Superintendent of the AP: Here, Dr. Icasiano’s transfer from incumbent Schools Division Superintendent of Quezon City
MIST [Marikina Institute to Vocational Schools Superintendent of the Marikina Institute of Science and Technology (MIST)
of Science and was without his consent. Hence, it violates his security of tenure.
Technology], to fill up the
vacancy created by the
retirement of its Also, his reassignment to MIST appears to be indefinite and not temporary since Dr. Icasiano has
Superintendent.- been described as fit for the (reassigned) job, being an expert in the field.
Approved. 

I: Whether the
reassignment of private
respondent from School
Division Superintendent
of QC to Vocational
School Superintendent of
MIST is violative of his
security of tenure?  YES.

Dimayuga v. Benedicto LB: Security of tenure in an office is acquired only by one who has the qualifications for
that office. Hence, it is acquired with respect to rank, and not to position.

AP: Here, during Dimayuga’s appointment as Executive Director 2 of the Toll Regulatory Board,
her position was excluded from the coverage of the CES, so Dimayuga was able to occupy the
said position although she was not a Career Executive Service Officer. The subsequent inclusion
of her position under the CES, however, did not automatically qualify her for the said position as
she lacked the required eligibility. Hence, she is not entitled to security of tenure.

Abolition of Office

Mayor v. Macaraig LB: Abolition of an office is NOT THE SAME as the declaration that that office is vacant. 

RA No. 6715 Declaring While abolition by law as a result of reorganization is a recognized cause for termination of a
Vacant “all positions of government employee, it is not the same as a declaration that the office is vacant.
the Commissioners,
Executive Labor Arbiters
and Labor Arbiters of the
 While it is undoubtedly a prerogative of the legislature to abolish certain offices, it can
present National Labor not be conceded the power to simply pronounce those offices vacant and
Relations Commissions”. thereby effectively remove the occupants or holders thereof from the civil
The old positions were service. 
declared vacant because  Such an act would constitute, on its face, an infringement of the constitutional
of the “need to guarantee of security of tenure, and will have to be struck down on that account. 
professionalize the higher  It can not be justified by the professed “need to professionalize the higher levels of
levels of officialdom officialdom invested with adjudicatory powers and functions, and to upgrade their
invested with qualifications, ranks, and salaries or emoluments.”
adjudicatory powers and
functions, and upgrade
AP: The petitioners have the right to remain in office until the expiration of the terms for which
their qualifications, ranks
they have been appointed, unless sooner removed “for cause provided by law.”
and salaries or
emoluments.”
RA. 6715 did not abolish the NLRC, or change its essential character as a supervisory and
adjudicatory body.
I: Whether the provisions
of RA No. 6715 are  The provisions conferring
constitutional? Whether   a somewhat greater measure of autonomy; requiring that its membership be
or not RA 6715 has drawn from tripartite sectors (workers, employees and the public sector); 
worked such an abolition  changing the official stations of the Commission’s divisions;
of the petitioners’ offices,
 and even those prescribing higher or other qualifications for the positions of
expressly or impliedly?
Commissioner which, if at all, should operate only prospectively, not to
NO.
mention the fact that the petitioners have asserted without dispute that they
possess the new qualifications—
 none of these can be said to work so essential or radical a revision of the
nature, powers and duties of the NLRC as to justify a conclusion that the Act in
truth did not merely declare vacant but actually abolished the offices of
commissioners and created others in their place.

Right to Organize

SSS Employees v. CA LB: Government employees has the right to organize but they do not have the right to
strike.

The terms and conditions of government employment are fixed by law, government workers
cannot use the same weapons employed by workers in the private sector to secure concessions
from their employers.

AP: Here, SSS being a GOCC with an original charter having been created under RA 1161, its
employees are part of the civil service and are covered by the CSC’s memorandum prohibiting
strikes.

77
Double Compensation

Peralta v. Mathay LB: No double compensation unless authorized by law.

Respondent Auditor  This is to manifest a commitment to the fundamental principle that a public
General Mathay inquires office is a public trust.
into the meaning and
significance of the
constitutional inhibition  It is expected of a government official or employee that he keeps uppermost in mind the
against an officer or demands of public welfare. He is there to render public service. 
employee of the
government receiving
 He is of course entitled to be rewarded for the performance of the functions
additional or double
entrusted to him, but that should not be the overriding consideration. 
compensation unless
specifically authorized by
law  The intrusion of the thought of private gain should be unwelcome. The
temptation to further personal ends, public employment as a means for the acquisition of
I: W/N the cost of living wealth, is to be resisted. That at least is the ideal. 
allowance as well as
incentive and
Christmas bonuses  There is then to be an awareness on the part of an officer or employee of the
paid to petitioner, a government that he is to receive only such compensation as may be fixed by law. With
Trustee GSIS, did fall such a realization, he is expected not to avail himself of devicus or circuitous
within such a ban? YES means to increase the remuneration attached to his position. 

 It is an entirely different matter if the legislative body would itself determine for reasons
satisfactory to it that he should receive something more. If it were to be thus though,
there must be a law to that effect.

AP: A “per diem” is commonly identified with the daily allowance “for each day he (an officer or
employee) was away from his home base.” Its usual signification is thus that of a reimbursement
for expenses incurred in the performance of one’s duties. If employed in a statute, as in this case,
in the concept of remuneration, however, there must be, to justify an additional compensation, a
specific law that so provides. Otherwise, fidelity to the constitutional command is lacking.

A similar approach is called for in determining the nature of a cost of living allowance. If it
could rightfully be considered as in the nature of a reimbursement rather than additional
emoluments or perquisites, then the ruling of respondent Auditor General cannot find support in
the Constitution. 
 Here, petitioner was unable to show that the cost of living allowance received
by him was in the nature of a reimbursement. It did amount then to an
additional compensation.

It is quite obvious that by its very nature, a bonus partakes of an additional remuneration
or compensation.
 The very characterization of what was received by petitioner as bonuses being intended
by way of an incentive to spur him possibly to more diligent, efforts and to add to the
feeling of well-being traditionally associated with the Christmas season would remove
any doubt that the Auditor General had no choice except to deduct from petitioner’s
gratuity such items.

Disposition: Auditor’s General Affirmed. Allowance and Bonus are considered additional
remuneration.

Santos v. CA LB: The petitioner cannot take refuge under Section 8 (2) of Article IX-B of the Constitution,
which provides “pensions or gratuities shall not be considered as additional, double, or indirect
compensation.” This provision simply means that a retiree receiving pension or gratuity can
continue to receive such pension or gratuity even if he accepts another government position to
which another compensation is attached.

AP: Here, the retirement benefits which Santos had received or has been receiving under RA 910,
as amended, do not constitute double compensation. He could continue receiving the same even
if after his retirement he had been receiving salary from the defunct MMA as Director III thereof.
This is but just because said retirement benefits are rewards for his services as MeTC Judge,
while his salary was his compensation for his services as Director III of the MMA.

However, to credit his years of service in the Judiciary in the computation of his separation pay
under R.A. No. 7924 notwithstanding the fact that he had received or has been receiving the
retirement benefits under RA 910, would be to countenance double compensation for exactly the
same services, i.e., his services as MeTC Judge. Such would run counter to the policy of this
Court against double compensation for exactly the same services. More important, it would be in
violation of Section 8 (1) of Article IX-B of the Constitution, which proscribes additional, double,
or indirect compensation. The law creating the MMA does not specifically authorize payment of
additional compensation for years of government service outside of the MMA.

Singson v. COA LB: Payment of additional monthly Representation and Transportation Allowance
(RATA) does not constitutes double compensation proscribed by the Constitution.

The RATA is distinct from salary, as a form of compensation. Unlike salary which is paid for
services rendered, the RATA is a form of allowance intended to defray expenses deemed
unavoidable in the discharge of office. Hence, the RATA is paid only to certain officials who, by
78
the nature of their offices, incur representation and transportation expenses.

AP: Here, Singson’s receipt of RATA from PICCI, in addition to their per diem as members of PICC
BOD, and another RATA from BSP pursuant to MB 15 does not constitute double compensation.

Art. IX-C

B. Comelec

Qualifications and Prohibitions

Brilliantes v. Yorac LB: The choice of a temporary chairman of the COMELEC, is within the discretion of the
Commission itself, and such discretion cannot be exercised for it, even with its consent,
by the President of the Philippines.

AP:
- The lack of a statutory rule covering the situation at bar is no justification for the President of
the Philippines to fill the void by extending the temporary designation in favor of the
respondent.

- This is still a government of laws and not of men. The problem allegedly sought to be
corrected, if it existed at all, did not call for presidential action. The situation could have been
handled by the members of the Commission on Elections themselves without the
participation of the President, however well meaning.

- In the choice of the Acting Chairman, the members of the Commission on Elections would
most likely have been guided by the seniority rule as they themselves would have
appreciated it. In any event, that choice and the basis thereof were for them and not the
President to make.

- That guaranty on security of tenure is not available to the respondent as Acting Chairman of
the Commission on Elections by designation of the President of the Philippines.
- WHEREFORE, the designation by the President of the Philippines of respondent Haydee B.
Yorac as Acting Chairman of the Commission on Elections is declared UNCONSTITUTIONAL.

Cayetano v. Monsod LB: Practice of law


- means any activity, in or out of court,
- which requires the application of law, legal procedure, knowledge, training and
experience.
- “To engage in the practice of law is to perform those acts which are characteristics of the
profession. Generally, to practice law is to give notice or render any kind of service,
which device or service requires the use in any degree of legal knowledge or skill.”

AP: Interpreted in the light of the various definitions of the term “practice of law”, particularly
the modern concept of law practice, and taking into consideration the liberal construction
intended by the framers of the Constitution, Atty. Monsod’s past work experiences as
- A lawyer-economist,
- a lawyer-manager,
- a lawyer-entrepreneur of industry,
- a lawyer-negotiator of contracts, and
- a lawyer-legislator of both the rich and the poor

—verily more than satisfy the constitutional requirement—that he has been engaged in
the practice of law for at least ten years.

Powers and Functions

LDP v. Comelec LB: The ascertainment of the identity of a political party and its legitimate officers is a
matter that is well within the authority of the Commission on Elections.
I: Who as between the
Party Chairman and the The source of this authority is no other than the fundamental law itself, which vests upon the
Secretary General has COMELEC the power and function to enforce and administer all laws and regulations
the authority to sign relative to the conduct of an election.
certificates of candidacy - In the exercise of such power and in the discharge of such function, the Commission is
of the official candidates endowed with ample “wherewithal” and “considerable latitude in adopting means
of the party? – The party and methods that will ensure the accomplishment of the great objectives for which
chairman it was created to promote free, orderly and honest elections.”

The solomonic solution of AP:


the Comelec is to - COMELEC need only to turn to the Party Constitution. The LDP’s Constitution states that
recognize both factions: the Party Chairman is the Chief Executive Officer of the Party, whose powers and functions
79
LDP Angara wing and LDP include signing documents for and on behalf of the party. The Secretary General, on the
Aquino wing. Both can other hand, assists the Party Chairman in overseeing the day-to-day operations of the Party
field local and national and only when empowered by the Party Chairman, is he to sign documents for and on behalf
candidates and are of the Party. Thus, the Sec Gen has only a delegated power, which originally pertains to the
entitled to copies of Party Chairman.
certificates of canvass.
The even numbered - By creating two wings of LDP (Angara Wing and Aquino Wing), the COMELEC effectively
precincts will go to diffused the LDPs strength and undeniably emasculated its chance of obtaining the
Angara-wing while the Commissions nod as the dominant minority party.
odd-numbered will go to
Aquino
- As if to rationalize its folly, the COMELEC invokes the constitutional policy towards a free and
open party system. This policy, however, envisions a system that shall “evolve according to
the free choice of the people,” not one molded and whittled by the COMELEC. When the
Constitution speaks of a multi-party system, it does not contemplate the COMELEC
splitting parties into two.

Thus, Comelec Resolution is ANNULLED.

Sison v. Comelec LB: Presentation of evidence before the COMELEC is not at all indispensable in order to
satisfy the demands of due process.

- Under the amendment introduced by R.A. No. 7166, particularly Section 18 thereof, all that
is required now is that the COMELEC shall dispose of pre-proclamation
controversies “on the basis of the records and evidence elevated to it by the board
of canvassers.”

- This is but in keeping with the policy of the law that cases of this nature should be summarily
decided and the will of the electorate as reflected on the election returns be determined as
speedily as possible.

AP:
- Owing to the presumption of regularity of performance of official duty and the precept that
factual findings of the COMELEC based on its assessments and duly supported by gathered
evidence, are conclusive upon the court, that the COMELEC did arrive at its conclusion with
due regard to the available evidence before it.

- That this is so can, in fact, be gleaned from petitioners own allegation and admission
in his petition that the election returns themselves as well as the minutes of the
Canvassing Committees and the City Board of Canvassers x x x are in the
possession of the COMELEC. He even cites paragraph (g), Section 20 of the Omnibus
Election Code to validate such allegation.

Hence, it is not really correct to say that the COMELEC acted without evidentiary basis at
all or that petitioner was deprived of his right to due process.

Sambarani v. Comelec (1) LB: Section 2(1) of Article IX(C) of the Constitution gives the COMELEC the broad
power to “enforce and administer all laws and regulations relative to the conduct of
Failure of elections in 11 an election, plebiscite, initiative, referendum, and recall.”
barangays in Lanao Del
Sur, a special election - Indisputably, the text and intent of this constitutional provision is to give COMELEC
was held but it failed all the necessary and incidental powers for it to achieve its primordial objective of
again holding free, orderly, honest, peaceful and credible elections.

Comelec Reso: (2) LB: The functions of the COMELEC under the Constitution are essentially executive and
The COMELEC explained administrative in nature. It is elementary in administrative law that courts will not interfere
that it is no longer in a in matters which are addressed to the sound discretion of government agencies entrusted
position to call for with the regulation of activities coming under the special technical knowledge and training of
another special election such agencies. The authority given to COMELEC to declare a failure of elections and
since Section 6 of the to call for special elections falls under its administrative function.
Omnibus Election Code
provides that special (3) LB: Prohibition on conducting special elections after thirty days from the cessation of the
elections shall be held on cause of the failure of elections is not absolute; COMELEC possesses residual power,
a date reasonably close (directory in nature) to conduct special elections beyond the deadline prescribed by
to the date of the election law.
not held, but not later
than thirty days after
cessation of the cause of - The deadline in Section 6 cannot defeat the right of suffrage of the people as guaranteed by
such postponement. The the Constitution.
COMELEC noted that -
more than thirty days - The COMELEC has broad power or authority to fix other dates for special elections to
had elapsed since the enable the people to exercise their right of suffrage.
failed election. - The COMELEC may fix other dates for the conduct of special elections when the same cannot
be reasonably held within the period prescribed by law.
To hold another special
election in these AP: In this case, the cause of postponement after the second failure of elections was COMELECs
barangays will not only refusal to hold a special election because of (1) its erroneous interpretation of the law, and (2) its
be tedious and perceived logistical, operational and financial problems.
cumbersome, but a waste
of its precious resources.
- The operational, logistical and financial problems which COMELEC claims it will encounter
with the holding of a second special election can be solved with proper planning, coordination
80
Note: It also left to the and cooperation among its personnel and other deputized agencies of the government.
DILG the process of - A special election will require extraordinary efforts, but it is not impossible. In applying
appointing brgy election laws, it would be better to err in favor of popular sovereignty than to be right in
chairman/kagawads/Sks complex but little understood legalisms. In any event, this Court had already held that special
– Wrong. Hold-over elections under Section 6 would entail minimal costs because it covers only the precincts in
principle shall apply. the affected barangays.
Thus, COMELEC’s reasons for refusing to hold another special election are void.

Datu Abas Kida vs. LB: COMELEC has no power to call for the holding of special elections unless pursuant to
Senate a specific statutory grant.
- The constitutional power of the COMELEC, in contrast with the power of Congress to call for,
and to set the date of, elections, is limited to enforcing and administering all laws and
regulations relative to the conduct of an election.
- Statutorily, COMELEC has no power to call for the holding of special elections unless pursuant
to a specific statutory grant.

AP: In this case, the petitioners proposed that COMELEC hold a special election for applicable
positions for which would be effective during the duration of the interim period between the
originally and the new schedule of the ARMM elections.
- One of their arguments was that since COMELEC had been given the power to postpone
elections in certain circumstances, it could exercise the same to the case at bar by analogy.
- The Supreme Court disagreed, ruling that such circumstances were limited to
extralegal and unforeseen events, which does not include a law passed by
Congress.

Thus, since Congress did not fix a date for a special election, COMELEC does not have
the power to hold the same.

Pangilinan v. Comelec LB: While the COMELEC has exclusive original jurisdiction over election contests of all
elective regional, provincial and city officials, the Electoral Tribunal of the House of
Petitioner filed an Urgent Representatives is the sole judge over election contests of all its members.
Motion to Suspend the
Canvass and - The Constitution vests in the COMELEC “exclusive original jurisdiction over all contests
Proclamation of Belmonte relating to the elections, returns, and qualifications of all elective regional provincial and city
(proclaimed rep of QC) officials.” It has no jurisdiction over contests relating to the election, returns, and
on the ground that the qualifications of Members of the House of Representatives.
latter committed election
law violations (by giving - On the other hand, under Sec 17, Article VI of the 1987 Constitution, the Electoral Tribunal of
money and other
the House of Representatives is the “sole judge of air contests relating to the election,
material considerations
returns, and qualifications” of its members.
to influence, induce or
corrupt the voters) but
COMELEC ignored the - Consequently, the phrase “including pre-proclamation controversies” used in Sec 3,
said motion. – should’ve Article IX-C of the Constitution should be construed as referring only to “pre-
been filed before the proclamation controversies” in election cases that fall within the exclusive original
HRET jurisdiction of the COMELEC, i.e., election cases pertaining to the election of
regional, provincial and city officials.

The 1987 Constitution abolished the Batasang Pambansa and consequently divested the
COMELEC of its jurisdiction over the Batasan (Congress). The Congress under the 1987
Constitution made up of the Senate and the House of Representatives now have their respective
Electoral Tribunals which are the sole judge of all election contests of their respective members

AP: In this case, since, Belmonte was already a member of the Congress, petitioner should file
any other issues to the HRET.

Thus, COMELEC is correct in not taking cognizance over the case.

Roque, Jr. v. Comelec LB: The first function of the Comelec under the Constitution—and the Omnibus Election Code for
that matter—relates to the enforcement and administration of all laws and regulations relating to
Petitioner’s claim: By the conduct of elections to public office to ensure a free, orderly and honest electoral exercise.
acceding to Art. 3.3 of
the automation contract, AP: The RFP (Request for Proposal), which forms an integral part of the automation contract,
Comelec relinquished, so states the requirement of a complete solutions provider which can provide effective overall
petitioners claim, nationwide project management service under COMELEC supervision and control.
supervision and control of
the system to be used for
the automated elections. Sec. 6.7 of the contract also states: Subject to the provisions of the General Instructions to be
To a more specific point, issued by the Commission En Banc, the entire processes of voting, counting, transmission,
the loss of control, as consolidation and canvassing of votes shall be conducted by COMELECs personnel and
may be deduced from the officials, and their performance, completion and final results according to specifications
ensuing exchanges, arose and within the specified periods shall be the shared responsibility of COMELEC and the
from the fact that PROVIDER.
Comelec would not be
holding possession of The role of Smartmatic TIM Corporation is basically to supply the goods necessary for the
what in IT jargon are the automation project, such as but not limited to the PCOS machines, PCs, electronic transmission
public and private keys devices and related equipment, both hardware and software, and the technical services pertaining
pair. to their operation. As lessees of the goods and the back-up equipment, the corporation and its
operators would provide assistance with respect to the machines to be used by the Comelec
I: Whether there is an which, at the end of the day, will be conducting the election thru its personnel and whoever it
abdication by the deputizes.
COMELEC of its
Constitutional functions -
81
NO
Arroyo v. DOJ LB: Under the present law, the Comelec and other prosecuting arms of the government,
such as the Department of Justice (DOJ), now exercise concurrent jurisdiction in the
investigation and prosecution of election offenses

- While recognizing the Comelec’s exclusive power to investigate and prosecute cases under
Batas Pambansa Bilang 881 or the Omnibus Election Code, the Court pointed out that the
framers of the 1987 Constitution did not have such intention. This exclusivity is thus a
legislative enactment that can very well be amended by Section 43 of RA 9369. Therefore,
under the present law, the Comelec and other prosecuting arms of the government, such as
the DOJ, now exercise concurrent jurisdiction in the investigation and prosecution of election
offenses.

AP: Notwithstanding the grant of concurrent jurisdiction, the Comelec and the DOJ nevertheless
included a provision in the assailed Joint Order whereby the resolutions of the Joint Committee
finding probable cause for election offenses shall still be approved by the Comelec in accordance
with the Comelec Rules of Procedure.

Therefore, the creation of the Joint Committee as an abdication of the Comelec’s


independence enshrined in the 1987 Constitution.

Cagas v. Comelec LB: The Constitution recognizes that the power to fix the date of elections is legislative in nature,
which is shown by the exceptions in previously mentioned Constitutional provisions, as well as in
the election of local government officials.

The Constitution, however, grants the COMELEC the power to “[e]nforce and administer all laws
and regulations relative to the conduct of an election, plebiscite, initiative, referendum and
recall.” The COMELEC has “exclusive charge of the enforcement and administration of all laws
relative to the conduct of elections for the purpose of ensuring free, orderly and honest elections.”
The text and intent of Section 2(1) of Article IX(C) is to give COMELEC “all the
necessary and incidental powers for it to achieve the objective of holding free, orderly,
honest, peaceful and credible elections.”

LB: The Commission on Elections has residual power to conduct a plebiscite even
beyond the deadline prescribed by law.

- It is thus not novel for this Court to uphold the COMELEC’s broad power or authority to fix
other dates for a plebiscite, as in special elections, to enable the people to exercise their right
of suffrage.

LB: The legal compass from which the COMELEC should take its bearings in acting upon
election controversies is the principle that "clean elections control the appropriateness of
the remedy."

In fixing the date for special elections the COMELEC should see to it that:
1. It should not be later than thirty (30) days after the cessation of the cause of the
postponement or suspension of the election or the failure to elect; and

2. It should be, suspended or which resulted in the failure to elect. The first involves a
question of reasonably close to the date of the election not held fact.

This must be determined in the light of the peculiar circumstances of a case. Thus, the holding of
elections within the next few months from the cessation of the cause of the postponement,
suspension or failure to elect may still be considered "reasonably close to the date of the election
not held."

In the exercise of the plenitude of its powers to protect the integrity of elections, the COMELEC
should not and must not be straitjacketed by procedural rules in the exercise of its discretion to
resolve election disputes

AP: The date 28 October 2013 is reasonably close to 6 April 2013, and there is no reason why
the plebiscite should not proceed as scheduled by the COMELEC.

Indeed, Cagas’ insistence that only Congress can cure the alleged legal infirmity in the date of
holding the plebiscite for the creation of the Province of Davao Occidental fails in light of the
absence of abuse of discretion of the COMELEC. Finally, SC finds it unacceptable to utilize more of
our taxpayers’ time and money by preventing the COMELEC from holding the plebiscite as now
scheduled.

Go v. Bayron LB: When the Commission on Elections (COMELEC) receives a budgetary appropriation
for its “Current Operating Expenditures,” such appropriation includes expenditures to
I: Whether the COMELEC carry out its constitutional functions, including the conduct of recall elections.
can suspend the conduct
of holding of a recall AP: The 2014 GAA provides the line item appropriation to allow the COMELEC to perform its
election by reason of lack constitutional mandate of conducting recall elections. There is no need for supplemental
of appropriated funding? legislation to authorize the COMELEC to conduct recall elections for 2014.
NO.
Contrary to the COMELEC's assertion, the appropriations for personnel services and maintenance
and other operating expenses falling under "Conduct and supervision of elections, referenda,

82
recall votes and plebiscites" constitute a line item which can be augmented from the COMELEC's
savings to fund the conduct of recall elections in 2014. The conduct of recall elections requires
only operating expenses, not capital outlays. The COMELEC's existing personnel in Puerto
Princesa are the same personnel who will evaluate the sufficiency of the recall petitions. and
conduct the recall elections

Thus, there is no reason why the COMELEC is unable to perform its constitutional
mandate to “enforce and administer all laws and regulations relative to the conduct of
x x x recall.”

Should the funds appropriated in the 2014 GAA be deemed insufficient, then the COMELEC
Chairman may exercise his authority to augment such line item appropriation from the
COMELEC’s existing savings, as this augmentation is expressly authorized in the 2014 GAA.

Jurisdiction

Sarmiento v. Comelec LB: Decision of COMELEC en banc void where case, inclusive of pre-proclamation
controversy, not first resolved by a Division

- Election cases include pre-proclamation controversies, and all such cases must first
be heard and decided by a Division of the Commission. The Commission, sitting en
banc, does not have the authority to hear and decide the same at the first instance.

- In the COMELEC RULES OF PROCEDURE, pre-proclamation cases are classified as Special


Cases and, in compliance with the above provision of the Constitution, the two (2) Divisions
of the Commission are vested with the authority to hear and decide these Special Cases. Rule
27 thereof governs Special Cases; specifically, Section 9 of the said Rule provides that
appeals from rulings of the Board of Canvassers are cognizable by any of the Divisions to
which they are assigned and not by the Commission en banc.

AP: The COMELEC en banc acted without jurisdiction, or with grave abuse of discretion, when it
resolved the appeals of petitioners in the abovementioned Special Cases without first referring
them to any of its Divisions. Said resolutions are, therefore, null and void and must be set aside.
Consequently, the appeals are deemed pending before the Commission for proper referral to a
Division.

LB: Section 16 of R.A. No. 7166 provides that all pre- proclamation cases pending before it
shall be deemed terminated at the beginning of the term of the office involved and the
rulings of the boards of canvassers concerned shall be deemed affirmed, without prejudice to the
filing of a regular election protest by the aggrieved party. However, proceedings may continue
when on the basis of the evidence thus far presented, the Commission determines that the
petition appears meritorious and accordingly issues an order for the proceeding to continue or
when an appropriate order has been issued by the Supreme Court in a petition for certiorari.”

AP: The terms of the offices involved in the Special Cases subject of these petitions commenced
at noon of 30 June 1992. These cases have thus been rendered moot and such a
resolution would only be an exercise in futility.

Flores v. Comelec (1) LB: Decision of the municipal or metropolitan court in barangay election appealable
to Regional Trial Court, unconstitutional.
- Section 9 of Rep. Act No. 6679, insofar as it provides that the decision of the municipal or
metropolitan court in a barangay election case should be appealed to the regional trial court,
must be declared unconstitutional.

AP: Petitioner had a right to rely on the presumed validity of the law.

- the Court does not disregard the fact that the petitioner was only acting in accordance with
the said law when he appealed the decision of the Municipal Circuit Trial Court of Tayum to
the Regional Trial Court of Abra
- That is what the statute specifically directed in its Section 9 which, at the time the appeal
was made, was considered constitutional. The petitioner had a right to rely on its presumed
validity as everyone apparently did.

Therefore, SC considered his appeal to the Commission on Elections as having been


made directly from the Municipal Circuit Trial Court of Tayum, Abra, disregarding the
detour to the Regional Trial Court.

(2) LB: Decisions of the COMELEC on election contests involving municipal and
barangay offices shall be final and not appealable applies to questions of fact.

- the provision of Article IX-C, Section 2(2) of the Constitution that “decisions, final orders, or
rulings of the Commission on election contests involving elective municipal and barangay
offices shall be final, executory, and not appealable” applies only to questions of fact and
not of law.
- That provision was not intended to divest the Supreme Court of its authority to resolve
questions of law as inherent in the judicial power conferred upon it by the Constitution. We
eschew a literal reading of that provision that would contradict such authority.

People v. Hon. Delgado LB: Decisions of COMELEC in election contests or administrative questions brought

83
before it are subject to judicial review only by the Supreme Court.
The respondents contend
that since the cases were - As provided in Section 7, Article IX of the Constitution, unless otherwise provided by law, any
filed in court by the decision, order or ruling of the COMELEC may be brought to the Supreme Court on certiorari by
COMELEC as a public the aggrieved party within thirty days from receipt of a copy thereof.
prosecutor, and not in the
exercise of its power to - “what is contemplated by the term ‘final orders, rulings and decisions’ of the COMELEC
decide election contests, reviewable on certiorari by the Supreme Court as provided by law are those rendered in actions
the trial court has or proceedings before the COMELEC and taken cognizance of by said body in the exercise of its
authority to order a adjudicatory or quasi-judicial powers.” (Filipinas Engineering and Machine Shop vs. Ferrer)
reinvestigation.

- Thus, the decisions of the COMELEC on election contests or administrative questions brought
before it are subject to judicial review only by this Court.

LB: Powers of COMELEC; Aside from its adjudicatory or quasi-judicial power, the
COMELEC is also vested the power of a public prosecutor with exclusive authority to
conduct preliminary investigation and prosecution of election offenses.

The Regional Trial Court has the authority to review actions of the COMELEC in the
investigation and prosecution of election offenses filed in said court.

AP: Thus, when the COMELEC, through its duly authorized law officer, conducts the preliminary
investigation of an election offense and upon a prima facie finding of a probable cause, files the
information in the proper court, said court thereby acquires jurisdiction over the case.
Consequently, all the subsequent disposition of said case must be subject to the approval of the
court.
o The records of the preliminary investigation required to be produced by the court
must be submitted by the COMELEC.
o The trial court may rely on the resolution of the COMELEC to file the information, by
the same token that it may rely on the certification made by the prosecutor who
conducted the preliminary investigation, in the issuance of the warrant of arrest.
o Nevertheless the court may require that the record of the preliminary investigation
be submitted to it to satisfy itself that there is probable cause which will warrant the
issuance of a warrant of arrest.
o The refusal of the COMELEC or its agents to comply with the order of the trial court
requiring them to conduct a reinvestigation in this case and to submit to the court
the record of the preliminary investigation on the ground that only this Court may
review its actions is certainly untenable.

Note: The petition is brought in the name of the People of the Philippines. Only the Solicitor
General can represent the People of the Philippines in this proceeding. In the least, the consent of
the Office of the Solicitor General should have been secured by the COMELEC before the filing of
this petition. On this account alone, the petition should be dismissed.

People v. Judge Inting LB: Comelec is empowered to conduct preliminary investigations in cases involving
election offenses for the purpose of helping the Judge determine probable cause and
I: Does a preliminary for filing an information in Court.
investigation conducted - This power is exclusive with COMELEC.
by a Provincial Election
Supervisor involving The Provincial Fiscal as such assumes no role in the prosecution of election cases
election offenses have to - If the Fiscal or Prosecutor files an information charging an election offense or prosecutes a
be coursed through the violation of election law, it is because he has been deputized by the COMELEC. He does not
Provincial Prosecutor, do so under the sole authority of his office.
before the Regional Trial
Court may take
AP: It is apparent that the respondent trial court misconstrued the constitutional provision when
cognizance of the
it quashed the information filed by the Provincial Election Supervisor. What the respondent trial
investigation and
court should have done was to enforce its September 30, 1988 order, to wit: It should have said
determine whether or not
probable cause exists, issued a warrant of arrest and placed bail at Php 5,000. The order to get
probable cause exists?
the approval of the Provincial Fiscal is not only superfluous but unwarranted.
NO.

Note: Difference of Preliminary Investigation and Preliminary Examination


Preliminary investigation should be distinguished as to whether it is an investigation for the
determination of a sufficient ground for the filing of the information or it is an investigation for the
determination of a probable cause for the issuance of a warrant of arrest. The first kind of
preliminary investigation is executive in nature. It is part of the prosecution’s job. The
second kind of preliminary investigation which is more properly called preliminary
examination is judicial in nature and is lodged with the judge.

Jaramilla v. Comelec (1) LB: Election cases including preproclamation controversies should first be heard
and decided by a division of the COMELEC, and then by the commission en banc
if a motion for reconsideration of the division is filed.

(2) LB: When the case demands only the exercise by the COMELEC of its
administrative functions, such as the correction of a manifest mistake in the addition
of votes or an erroneous tabulation in the statement of votes, the COMELEC en banc
can directly act on it in the exercise of its constitutional function to decide
questions affecting elections.

(3) COMELEC has authority to suspend the reglementary periods provided by the
rules, or the requirement of certification of non-forum shopping in the interest
84
of justice and speedy resolution of the cases before it

(4) COMELEC is not constrained to dismiss a case before it by reason of non-


payment of filing fees; COMELEC has discretion whether to entertain the petition
or not in case of nonpayment of legal fees

AP: The Petition for Correction of Manifest Errors in the case at bar alleges an erroneous
copying of figures from the election return to the Statement of Votes by Precinct.
- Such an error in the tabulation of the results, which merely requires a clerical
correction without the necessity of opening ballot boxes or examining ballots,
demands only the exercise of the administrative power of the COMELEC.

Hence, the Commission en banc properly assumed original jurisdiction over the aforesaid petition.

Buac v. Comelec LB: A case assailing the regularity of the conduct of a plebiscite does not fit the kind of
I: Whether or not a case calling for the exercise of judicial power—it does not involve the violation of any
COMELEC has jurisdiction legally demandable right and its enforcement.
over petitions contesting
the results of plebiscites? AP: The case at bar involves the determination of whether the electorate of Taguig voted in favor
YES of, or against the conversion of the municipality of Taguig into a highly urbanized city in the
plebiscite conducted for the purpose. Respondents submit that the regular courts of justice, more
specifically, the Regional Trial Court, has the jurisdiction to adjudicate any controversy concerning
the conduct of said plebiscite.

- The invocation of judicial power to settle disputes involving the conduct of a plebiscite is
misplaced.
- Section 1, Article VIII of the Constitution defines judicial power as including “the duty of the
courts of justice to settle actual controversies involving rights which are legally demandable
and enforceable and to determine whether or not there has been a grave abuse of discretion
amounting to lack or excess of jurisdiction on the part of any branch or instrumentality of the
Government.”
- According to Mr. Justice Isagani Cruz, “the first part of the authority represents the
traditional concept of judicial power involving the settlement of conflicting rights as conferred
by law.”

- The case at bar assailing the regularity of the conduct of the Taguig plebiscite does not fit
the kind of a case calling for the exercise of judicial power. It does not involve the
violation of any legally demandable right and its enforcement.

- There is no plaintiff or defendant in the case at bar for it merely involves the
ascertainment of the vote of the electorate of Taguig whether they approve or
disapprove the conversion of their municipality to a highly urbanized city.
- There is no invocation of a private right conferred by law that has been violated and
which can be vindicated alone in our courts of justice in an adversarial proceeding.
- Rather, the issue in the case at bar is the determination of the sovereign decision of
the electorate of Taguig. The purpose of this determination is more to protect the
sovereignty of the people and less to vindicate the private interest of any individual. Such a
determination does not contemplate the clash of private rights of individuals and
hence cannot come under the traditional jurisdiction of courts.

LB: The intent of the Constitution and election laws to subject only contests relating to the
elections, returns and qualifications of elected officials—from the barangay to the
President of the Philippines—to the exercise of judicial or quasi-judicial powers of
courts or administrative tribunals; The enforcement and administration of a law relative
to a plebiscite falls under the jurisdiction of the COMELEC under Section 2(1), Article IX (C)
of the Constitution which gives it the power to enforce and administer all laws and regulations
relative to the conduct of a plebiscite.

LB: Another reason why the jurisdiction of the COMELEC to resolve disputes involving plebiscite
results should be upheld is that such a case involves the appreciation of ballots which is
best left to the COMELEC
- As an independent constitutional body exclusively charged with the power of enforcement
and administration of all laws and regulations relative to the conduct of an election,
plebiscite, initiative, referendum and recall, the COMELEC has the indisputable expertise
in the field of election and related laws. Consequently, we should be extra cautious
in delimiting the parameters of the COMELEC’s broad powers.

- COMELEC should be given enough latitude in the exercise of its expertise, for to
straightjacket its discretion in the enforcement and administration of laws relating to the
conduct of election, plebiscite or referendum may render it impotent

AP: In the case at bar, the conduct of the Taguig plebiscite is the core of the controversy. This is
a matter that involves the enforcement and administration of a law relative to a
plebiscite. It falls under the jurisdiction of the COMELEC under Section 2(1), Article IX (C)
of the Constitution which gives it the power “to enforce and administer all laws and regulations
relative to the conduct of a x x x plebiscite x x x.”

Note: if jurisdiction is given to the regular courts, the result will not enhance the orderly
administration of justice. Any regional trial court from every nook and corner of the country will
have jurisdiction over a petition questioning the results of a nationwide plebiscite . Bearing in mind
that the jurisdiction of these courts is limited only within their respective judicial regions, the
85
difficulties that will attend their exercise of jurisdiction would be many if not unmanageable.

People v. Basilla LB: Prosecution officers designated by the COMELEC become deputies and agents of
COMELEC;
The contention of private
respondents that the - There is nothing in Section 2 (4) of Article IX-C of the Constitution which requires such a
deputation by the pinched and niggardly interpretation of the authority of the Comelec to appoint as its
Comelec of the deputies, officials or employees of other agencies and instrumentalities of the government.
prosecuting arms of the - The prompt investigation and prosecution and disposition of election offenses constitute an
Government would be indispensable part of the task of securing free, orderly, honest, peaceful and credible
warranted only before the elections.
elections and only to - The investigation and prosecution of election offenses are, in an important sense, more
ensure tree, honest, important than the maintenance of physical order in election precincts.
orderly, peaceful and - Without the assistance of provincial and city fiscals and their assistants and staff members,
credible elections, that is, and of the state prosecutors of the Department of Justice, the prompt and fair investigation
to perform the peace- and prosecution of election offenses committed before or in the course of nationwide
keeping functions of elections would simply not be possible, unless, perhaps, the Comelec had a bureaucracy
policemen, lack many times larger than what it actually has.
substance
LB: Authority of COMELEC over its deputies through administrative sanctions

Moreover, the prosecution officers designated by the Comelec become deputies or agents
of the Comelec and pro tanto subject to the authority, control and supervision of the
Comelec in respect of the particular functions covered by such deputation.
- The acts of such deputies within the lawful scope of their delegated authority are, in legal
contemplation, the acts of the Comelec itself.
- The only limitation the Constitution itself places upon the Comelec’s authority over
its deputies relates to the enforcement of such authority through administrative
sanctions. Such sanctions—e.g., suspension or removal—may be recommended by the
Comelec to the President (Sec. 2 [8], Article IX-C, 1987 Constitution) rather than directly
imposed by the Comelec, evidently, to pre-empt and avoid potential difficulties with the
executive department of the Government where the prosecution and other officers deputized
are ordinarily located.

AP: Respondent Judge erred in dismissing the criminal informations filed by


prosecution officers deputized by COMELEC.

Tan v. Comelec LB: Upon the COMELEC’s recommendation, the Secretary of the Department of Justice
shall take appropriate action, whether to suspend or remove from office the officer or
Petitioner Tan is the employee who may, after due process, be found guilty of violation of election laws or
incumbent city prosecutor failure to comply with instructions, orders, decisions or rulings of the COMELEC.
of Davao City, and was - The COMELEC’s mandate includes its authority to exercise direct and immediate supervision
designated by the and control over national and local officials or employees, including members of any national
petitioner to be the Vice- or local law enforcement agency and instrumentality of the government, required by law to
Chairman of the City perform duties relative to the conduct of elections.
Board of Canvassers of
Davao City. Based on the - In order to help ensure that such duly deputized officials and employees of government carry
canvassed votes, Garcia out their respective assigned tasks, the law has also provided that upon the COMELEC’s
was proclaimed the recommendation, the corresponding proper authority (the Secretary of the Department of
winning candidate for the Justice in the case at bar) shall take appropriate action, either to suspend or remove from
Congressman of District 2 office the officer or employee who may, after due process, be found guilty of violation of
of Davao City. election laws or failure to comply with instructions, orders, decisions or rulings of the
Respondent Alterado, COMELEC.
also a candidate, filed
numerous cases
LB: Prior to making recommendation, it is the COMELEC, being in the best position to
questioning the validity of
assess how its deputized officials and employees perform or have performed in their
the proclamation, alleging
duties,that should conduct the administrative inquiry.
unlawful, erroneous, and
irregular canvassing.
- To say that the COMELEC is without jurisdiction to look into charges of election offenses
committed by officials and employees of government outside the regular employ of the
Petitioner alleged lack of
COMELEC would be to unduly deny to it the proper and sound exercise of such
jurisdiction of the
recommendatory power and, perhaps more than that, even a possible denial of due process
COMELEC, he being
to the official or employee concerned.
under the executive
department.
LB: The COMELEC merely may issue a recommendation for disciplinary action but that it
is the executive department to which the charged official or employee belongs which has the
ultimate authority to impose the disciplinary penalty.

AP: It should be stressed that the administrative case against petitioner, taken cognizance of by,
and still pending with, the COMELEC, is in relation to the performance of his duties as an election
canvasser and not as a city prosecutor.

Petitioner’s assertion that private respondent Alterado has resorted to forum-shopping is


unacceptable. The investigation then being conducted by the Ombudsman on the criminal case for
falsification and violation of the AntiGraft and Corrupt Practices Act, on the one hand, and the
inquiry into the admin-istrative charges by the COMELEC, on the other hand, are entirely
independent proceedings. Neither would the results in one conclude the other. Thus, an
absolution from a criminal charge is not a bar to an administrative prosecution.

Panlilio v. Comelec LB: The remedy to assail an interlocutory order of the Commission on Elections
(COMELEC) in Division is neither to file a motion for reconsideration for certification to the
86
COMELEC En Banc nor to elevate the issue to the Court via a petition for certiorari under Rule 65
of the Rules of Civil Procedure; It is the same COMELEC’s Division which issued the
interlocutory order who should resolve the motion for reconsideration of the order.

LB: The Commission on Elections (COMELEC) En Banc shall decide motions for
reconsideration only of “decisions” of a Division, meaning those acts having a final
character.

AP: Here, the assailed Second Division order did not completely dispose of the case, as there was
something more to be done, which was to decide the election protest. Being interlocutory, the
assailed Second Division orders may not be resolved by the COMELEC En Banc.

Other doctrines:

(1) When there is an allegation in an election protest that would require the perusal,
examination or counting of ballots as evidence, it is the ministerial duty of the trial court
to order the opening of the ballot boxes and the examination and counting of ballots
deposited therein

(2) Filing of a protest before the Board of Election Inspectors is not a condition sine
qua non before the Commission on Elections (COMELEC) acquires jurisdiction
over the present election protest.

o Jurisdiction is conferred only by law and cannot be acquired through, or waived by,
any act or omission of the parties.

Bedol v. Comelec LB: The Commission on Elections possesses the power to conduct investigations as an
adjunct to its constitutional duty to enforce and administer all election laws, by virtue of
the explicit provisions of paragraph 6, Section 2, Article IX of the 1987 Constitution, which reads:
Article IX-C, Section 2. xxx (6) xxx; investigate and, where appropriate, prosecute cases of
violations of election laws, including acts or omissions constituting election frauds, offenses, and
malpractices.

Quasi-judicial or administrative adjudicatory power on the other hand is the power of the
administrative agency to adjudicate the rights of persons before it.
- It is the power to hear and determine questions of fact to which the legislative policy is
to apply and to decide in accordance with the standards laid down by the law itself in
enforcing and administering the same law.
- In carrying out their quasi-judicial functions the administrative officers or bodies are required
to investigate facts or ascertain the existence of facts, hold hearings, weigh evidence, and
draw conclusions from them as basis for their official action and exercise of discretion in a
judicial nature. Since rights of specific persons are affected, it is elementary that in the
proper exercise of quasi-judicial power due process must be observed in the conduct of the
proceedings.

AP: Task Force Maguindanao, was exercising its quasi-judicial power in pursuit of the
truth behind the allegations of massive fraud during the elections in Maguindanao.

—The Creation of Task Force Maguindanao was impelled by the allegations of fraud and
irregularities attending the conduct of elections in the province of Maguindanao and the non-
transmittal of the canvassing documents for all municipalities of said province. Task Force
Maguindanao’s factfinding investigation—to probe into the veracity of the alleged fraud
that marred the elections in said province; and consequently, to determine whether
the certificates of canvass were genuine or spurious, and whether an election
offense had possibly been committed—could by no means be classified as a purely
ministerial or administrative function.

- To achieve its objective, the Task Force conducted hearings and required the attendance of
the parties concerned and their counsels to give them the opportunity to argue and support
their respective positions.

Contempt Power

LB: The effectiveness of the quasi-judicial power vested by law on a government institution
hinges on its authority to compel attendance of the parties and/or their witnesses at
the hearings or proceedings;

To withhold from the Commission on Elections the power to punish individuals who
refuse to appear during a fact-finding investigation, despite a previous notice and order
to attend, would render nugatory the COMELEC’s investigative power, which is an
essential incident to its constitutional mandate to secure the conduct of honest and
credible elections

The language of the Omnibus Election Code and the Commission on Elections Rules of Procedure
is broad enough to allow the initiation of indirect contempt proceedings by the Commission on
Elections motu proprio.

AP: In this case, the purpose of the investigation was however derailed when petitioner
obstinately refused to appear during said hearings and to answer questions regarding the various
election documents which, he claimed, were stolen while they were in his possession and custody.
87
Undoubtedly, the COMELEC could punish petitioner for such contumacious refusal to
attend the Task Force hearings.

The fact that the indirect contempt charges against petitioner were initiated motu proprio by the
COMELEC did not by itself prove that it had already prejudged the case against him. As borne out
by the records, the COMELEC gave petitioner several opportunities to explain his side and to
present evidence to defend himself. All of petitioner’s belatedly filed pleadings were admitted and
taken into consideration before the COMELEC issued the assailed Resolution finding petitioner
guilty of indirect contempt

LB: The powers of the board of canvassers are not purely ministerial—the board exercises quasi-
judicial functions, such as the function and duty to determine whether the papers transmitted to
them are genuine election returns signed by the proper officers.

The board exercises quasi-judicial functions, such as the function and duty to determine whether
the papers transmitted to them are genuine election returns signed by the proper officers. When
the results of the elections in the province of Maguindanao were being canvassed, counsels for
various candidates posited numerous questions on the certificates of canvass brought before the
COMELEC. The COMELEC asked petitioner to appear before it in order to shed light on the issue of
whether the election documents coming from Maguindanao were spurious or not. When petitioner
unjustifiably refused to appear, COMELEC undeniably acted within the bounds of its jurisdiction
when it issued the assailed resolutions.

Ibrahim vs. Comelec LB: Section 7, Article IX of the 1987 Constitution in part substantially provides that any
decision, order or ruling of any of the Constitutional Commissions may be brought for
review to the Supreme Court on certiorari within 30 days from receipt of a copy thereof.

- The orders, ruling and decisions rendered or issued by the COMELEC en banc must be final
and made in the exercise of its adjudicatory or quasi-judicial power. Further, Section 1, Rule
64 of the Rules of Court states that it shall govern the review of final judgments and orders
or resolutions of the COMELEC and the Commission on Audit.

AP: In the case at bar, the now assailed Resolutions were issued with finality by the COMELEC en
banc. Under the Constitution and the Rules of Court, the said resolutions can be reviewed by way
of filing before SC a petition for certiorari.

Besides, the issues raised do not at all relate to alleged irregularities in the preparation,
transmission, receipt, custody and appreciation of the election returns or to the composition and
proceedings of the board of canvassers. What the instant Petition challenges is the
authority of the MBOC to suspend Ibrahim’s proclamation and of the COMELEC en banc
to issue the assailed resolutions. The crux of the instant Petition does not qualify as one
which can be raised as a pre-proclamation controversy.

LB: It is the COMELEC sitting in division and not the COMELEC en banc which has
jurisdiction over petitions to cancel a certificate of candidacy (Garvida v. Sales, Jr.,)

- In relation thereto, Rule 23 of the COMELEC Rules of Procedure provides that a petition to
deny due course to or cancel a certificate of candidacy for an elective office may be filed with
the Law Department of the COMELEC on the ground that the candidate has made a false
material representation in his certificate. The petition may be heard and evidence received by
any official designated by the COMELEC after which the case shall be decided by the
COMELEC itself.
- Under the same Rules of Procedure, jurisdiction over a petition to cancel a certificate of
candidacy lies with the COMELEC sitting in Division, not en banc. Cases before a Division may
only be entertained by the COMELEC en banc when the required number of votes to reach a
decision, resolution, order or ruling is not obtained in the Division. Moreover, only motions to
reconsider decisions, resolutions, orders or rulings of the COMELEC in Division are resolved
by the COMELEC en banc.
-
AP: In the case at bar, the COMELEC en banc, through the herein assailed resolutions,
ordered Ibrahim’s disqualification even when no complaint or petition was filed against
him yet.

If filed before the conduct of the elections, a petition to deny due course or cancel a certificate of
candidacy under Section 78 of the OEC is the appropriate petition which should have been
instituted against Ibrahim considering that his allegedly being an unregistered voter of his
municipality disqualified him from running as Vice-Mayor.

His supposed misrepresentation as an eligible candidate was an act falling within the purview of
Section 78 of the OEC. Moreover, even if we were to assume that a proper petition had
been filed, the COMELEC en banc still acted with grave abuse of discretion when it took
cognizance of a matter, which by both constitutional prescription and jurisprudential
declaration, instead aptly pertains to one of its divisions.

The MBOC has no authority to suspend Ibrahim’s proclamation especially since the herein
assailed resolutions, upon which the suspension was anchored, were issued by the COMELEC en
banc outside the ambit of its jurisdiction.

LB: The simple purpose and duty of the canvassing board is to ascertain and declare the
apparent result of the voting while all other questions are to be tried before the court
or other tribunal for contesting elections or in quo warranto proceedings.
88
AP: the MBOC motu proprio suspended Ibrahim’s proclamation when the issue of the latter’s
eligibility is a matter which the board has no authority to resolve.

Such suspension can only be ordered upon the motion of a complainant or intervenor relative to a
case for disqualification, or a petition to deny due course or cancel a certificate of candidacy
pending before the COMELEC, and only when the evidence of the winning candidate’s guilt is
strong. Besides, the COMELEC en banc itself could not have properly ordered Ibrahim’s
disqualification because in taking cognizance of the matter, it had already exceeded its
jurisdiction.

Jalosjos vs. Comelec LB: Section 3, Article IX-C of the 1987 Constitution requiring a motion for reconsideration before
the COMELEC En Banc may take action is confined only to cases where the COMELEC exercises its
quasi-judicial power. It finds no application, however, in matters concerning the COMELEC’s
exercise of administrative functions.

(Villarosa v. COMELEC) – Distinction: Admin vs. QJ powers


- [T]he term ‘administrative’ connotes, or pertains, to ‘administration, especially management,
as by managing or conducting, directing or superintending, the execution, application, or
conduct of persons or things. It does not entail an opportunity to be heard, the production
and weighing of evidence, and a decision or resolution thereon.

- While a ‘quasijudicial function’ is a term which applies to the action, discretion, etc., of public
administrative officers or bodies, who are required to investigate facts, or ascertain the
existence of facts, hold hearings, and draw conclusions from them, as a basis for their official
action and to exercise discretion of a judicial nature.

LB: The COMELEC’s denial of due course to and/or cancellation of a CoC in view of a
candidate’s disqualification to run for elective office based on a final conviction is
subsumed under its mandate to enforce and administer all laws relating to the conduct
of elections. Accordingly, in such a situation, it is the COMELEC’s duty to cancel motu
proprio the candidate’s CoC, notwithstanding the absence of any petition initiating a
quasi-judicial proceeding for the resolution of the same.

AP: As petitioner’s disqualification to run for public office had already been settled in a previous
case and now stands beyond dispute, it is incumbent upon the COMELEC En Banc to cancel his
Certificate of Candidacy as a matter of course, else it be remiss in fulfilling its duty to enforce and
administer all laws and regulations relative to the conduct of an election.

Equally compelling is the fact that the denial of petitioner’s Petition for Inclusion as a registered
voter in Zamboanga City had already attained finality by virtue of the RTC’s Order. In this accord,
petitioner’s non-compliance with the voter registration requirement under Section 39(a) of the
LGC is already beyond question and likewise provides a sufficient ground for the cancellation of
his CoC altogether.

Art. IX-D

C. Commission on Audit ("COA")

Appointment and Term

Funa v. COA LB: Reappointment” refers to a movement to one and the same office. Necessarily, a
movement to a different position within the commission (from Commissioner to Chairman) would
Whether Villar’s constitute an appointment, or a second appointment, to be precise, but not reappointment.
appointment as COA
Chairman, while sitting in LB: Where the Constitution or, for that matter, a statute, has fixed the term of office of a public
that body and after official, the appointing authority is without authority to specify in the appointment a term shorter
having served for four (4) or longer than what the law provides—if the vacancy calls for a full seven-year appointment, the
years of his seven (7) President is without discretion to extend a promotional appointment for more or for less than
year term as COA seven (7) years.
commissioner, is valid in
light of the term The President has two and only two options on term appointments.
limitations imposed - Either he extends an appointment for a full 7-year term when the vacancy results from the
under, and the expiration of term,
circumscribing concepts - or for a shorter period corresponding to the unexpired term of the predecessor when the
tucked in, Sec. 1 (2), Art.
vacancy occurs by reason of death, physical disability, resignation or impeachment.
IX(D) of the Constitution
- If the vacancy calls for a full seven-year appointment, the Chief Executive is barred
– NO, unconsti
from extending a promotional appointment for less than seven years. Else, the
President can trifle with terms of office fixed by the Constitution.

LB: A commissioner who resigns after serving in the Commission for less than seven
years is eligible for an appointment to the position of Chairman for the unexpired
portion of the term of the departing chairman.

- Such appointment is not covered by the ban on reappointment, provided that the aggregate

89
period of the length of service as commissioner and the unexpired period of the term of the
predecessor will not exceed seven (7) years and provided further that the vacancy in the
position of Chairman resulted from death, resignation, disability or removal by impeachment.
The Court clarifies that “reappointment” found in Sec. 1(2), Art. IX(D) means a movement to
one and the same office (Commissioner to Commissioner or Chairman to Chairman). On the
other hand, an appointment involving a movement to a different position or office
(Commissioner to Chairman) would constitute a new appointment and, hence, not, in the
strict legal sense, a reappointment barred under the Constitution.

LB: Reappointment is prohibited for the obvious intent of the framers is to prevent the
president from “dominating” the Commission by allowing him to appoint an additional
or two more commissioners.

- On the other hand, the provision, on its face, does not prohibit a promotional
appointment from commissioner to chairman as long as the commissioner has not
served the full term of seven years, further qualified by the third sentence of Sec. 1(2),
Article IX (D) that “the appointment to any vacancy shall be only for the unexpired portion of
the term of the predecessor.” In addition, such promotional appointment to the
position of Chairman must conform to the rotational plan or the staggering of terms
in the commission membership such that the aggregate of the service of the
Commissioner in said position and the term to which he will be appointed to the
position of Chairman must not exceed seven years so as not to disrupt the
rotational system in the commission prescribed by Sec. 1(2), Art. IX(D).

AP: The appointment of then Commissioner Reynaldo A. Villar to the position of Chairman of the
Commission on Audit to replace Guillermo N. Carague, whose term of office as such chairman
has expired, is hereby declared UNCONSTITUTIONAL for violation of Sec. 1(2), Art. IX(D) of
the Constitution.

The appointment of members of any of the three constitutional commissions, after the expiration
of the uneven terms of office of the first set of commissioners, shall always be for a fixed term of
seven (7) years; an appointment for a lesser period is void and unconstitutional.

The appointing authority cannot validly shorten the full term of seven (7) years in case of the
expiration of the term as this will result in the distortion of the rotational system prescribed by
the Constitution.

In conclusion, there is nothing in Sec. 1(2), Article IX(D) that explicitly precludes a promotional
appointment from Commissioner to Chairman, provided it is made under the aforestated
circumstances or conditions.

Powers and Functions

Dingcong v. Guingona, LB: The Commission on Audit (COA) vested with the power and authority, and the duty, to
Jr. examine, audit and settle all accounts pertaining to x x x the expenditures or uses of
funds x x x owned x x x by, or pertaining to, the Government or any of its subdivisions,
agencies, or instrumentalities (Article IX [D], Section 2 [1], 1987 Constitution).

AP: Here, the COA did not usurp the BOT management authority when it disallowed the contract
for the hiring of a carpenter and electrician on paykyao basis.

The disallowance made by COA is neither illegal nor a usurpation of a management function. The
authority of the petitioner, as agency head, to enter into a contract is not being curtailed. What
COA maintains is that the "pakyao" contract has proved disadvantageous to the
government.

NOTE: But, since here its actions are reviewable by the SC and the SC deems that the contract
was not excessive then the SC set aside the decision of the COA.

Danville Maritime, Inc. LB: The Constitution has ordained that the COA shall have exclusive authority to define the scope
v. COA of its audit and examination, establish the techniques and methods required therefore, and
promulgate accounting and auditing rules and regulations, including those for the prevention and
disallowance of irregular, unnecessary, excessive, extravagant, or unconscionable expenditures,
or use of government funds and properties.

AP: In the case at bar, there is no showing that the COA committed grave abuse of
discretion. COA has clearly shown its position to the PNOC in its questioned letter-directive
advising the latter of its misgivings as to why the award was given to the lone bidder in
spite of regulations previously made known to PNOC and to top it all, why the PNOC
perfunctorily rejected a much higher bid which appears to be more beneficial to the
corporation. Rather than condemn the COA as petitioner proposes, the COA should be
comme nded for its zeal and care in insuring that the disposition of the subject vessel would
be in a manner most advantageous to the government. A rebidding removes any suspicion
that may arise out of the sale of the vessel to petitioner under present circumstances.

Ramos v. Aquino LB: The Auditor General is vested with the power to examine, audit and settle all accounts
pertaining to the revenues and receipts from whatever source, and to audit, in accordance with
law and administrative regulations, all expenditures of funds or property pertaining to or held in
trust by the government as well as the provinces or municipalities thereof.8 That is one thing.
90
The ascertainment of whether a crime committed and by whom is definitely another (which the
fiscal did is another). Hence, The Auditor general’s functions are distinct from that of the fiscal
that determines criminal responsibility.

Nowhere does it appear that such a statutory grant of authority of the Auditor General to open
revised accounts carries with it the power to determine who may be constituted in the event that
in the preparation thereof a crime has been committed. The conclusive effect of the finality of his
decision on the execution of branch of the government thus relates solely to the administrative
aspect of the matter. His powers do not include participation in the investigation of charges
(otherwise it stretches his Constitutional power to “unwarranted limits”).

AP: Hence, there is no encroachment on the constitutional prerogatives of the Auditor General if
provincial fiscal conducts a preliminary investigation for cases of malversation through falsification
of public, official and commercial documents.

Mamaril v. Domingo LB: The responsibility for state audit is vested by the Constitution on the Commission on Audit.
State audit is not limited to the auditing of the accountable officers and the settlement of
accounts but includes accounting functions and the adoption in the audited agencies of internal
control, to see to it, among other matters, that the correct fees and penalties due the
government are collected.

The verification of the correctness of the evaluation and computation of the fees and penalties
collectible under the Land Transportation Law (R.A. No. 4136) are parts of the functions of the
COA, which examines and audits revenue accounts.

AP: When any person is indebted to any government agency, the COA may direct the proper
officer to withhold the payment of any money due such person or his estate to be applied in
satisfaction of the indebtedness. Thus, COA has the power over petitioner, who was not an
accountable officer.

Osmeña v. COA LB: It is true that COA is guided by certain principles and state policies to assure that
government funds shall be managed, expended, utilized in accordance with law and
WON COA commited regulations and safeguarded against loss or wastage, but the compromise of a civil suit in
grave abuse of discretion this case wherein the City is involved as a party, is a perfectly legitimate
in disallowing the transaction, not only recognized but even encouraged by law.
payment of P30,000.00
for the compromise AP: That the City of Cebu complied with the relevant formalities contemplated by law can hardly
agreement between the be doubted. The compromise agreement was submitted to its legislative council, the Sangguniang
parties herein involved - Panlungsod, which approved it conformably with its established rules and procedure, particularly
Yes the stipulation for the payment of P30,000.00 to the de la Cerna family. Neither may it be
disputed that since, as a municipal corporation, Cebu City has the power to sue and be sued, it
has the authority to settle or compromise suits, as well as the obligation to pay just and valid
claims against it.

Respondent refused to take account of the foregoing legal principles in relation to the antecedents
of the provision in the supplemental budget of the City for payment of P30,000.00. It failed to
realize that payment thereof was part of the consideration, not merely for the settlement of a
claim, but for the settlement of an actual controversy, and constituted one of the “reciprocal
concessions” which the law considers “the very heart and life of every compromise.”

By making reciprocal concessions, the parties in the civil case before the trial court put an end to
the action in a manner acceptable to all of them. It is noteworthy that the compromise in
question was approved by, and embodied in the judgment of, the Court, which pronounced it “to
be in conformity with law, morals and public policy” and enjoined the parties “to comply strictly
with the terms and conditions thereof.”

This judicial compromise is conclusive and binding on all the parties, including the City of Cebu. It
is enforceable by execution, as above stressed. There was no reason whatever to object to it,
much less disallow any disbursement therein stipulated. It should have been approved as a
matter of course.

Bustamante v. COA LB: The Commission on Audit’s power and duty to examine, audit and settle accounts owned or
held in trust by government-owned and controlled corporations, such as the NPC is granted by
the Constitution. Hence, it cannot be trumped by a resolution of a GOCC, such as the NPC.

If the Commission's power and duty to examine, audit and settle accounts pertaining to this
particular expenditures or use of funds and property, owned or held in trust by this government-
owned and controlled corporation (NPC) is not sustained, this Constitutional Body, which has been
tasked to be vigilant and conscientious in safeguarding the proper use of the government's, and
ultimately, the people's property, will be rendered inutile.

AP: Thus, here, the NPC’s resolution that authorizes the monthly reimbursement of
representation and transportation allowance cannot beat the COA’s power. The auditor was
correct in disallowing the transportation allowance of P pursuant to COA rules.

Bustamante’s contention that the Commission, in the exercise of its power granted by the
Constitution, usurped the statutory functions of the NPC Board of Directors, cannot be sustained
for its leads to the absurd conclusion that a mere Board of Directors of a government-owned and
controlled corporation, by issuing a resolution, can put to naught a constitutional provision which
91
has been ratified by the majority of the Filipino people.

Orocio v. COA LB: Pursuant to the COA’s power of determining whether an expenditure of a Government
agency or instrumentality such as the NPC is irregular , the COA should not be bound by the
opinion of the legal counsel of said agency or instrumentality which may have been the basis
for the questioned disbursement. Otherwise, its auditing function would be a meaningless
and futile exercise.

AP: However, since in this case the General Counsel of the COA issued a memo/order to make
Petitioner personally liable for the hospital bills without seeking the guidance of the COA as a
collegial body, said memo/order should be set aside. COA’s General Counsel cannot act for the
Commission for he is not even a Commissioner thereof. He can only offer legal advice or render
an opinion in order to aid the COA in the resolution of a case or a legal question.

DBP v. COA LB: The mere fact that private auditors may audit government agencies does not divest the COA
of its power to examine and audit the same government agencies. Also, under existing laws, the
The DBP reiterates that COA does not have the sole and exclusive power to examine and audit government banks. The
the income of the Fund Central Bank has concurrent jurisdiction to examine and audit, or cause the examination and
should be treated and audit, of government banks.
recorded as separate
from the income of DBP AP: The Agreement indisputably transferred legal title over the income and properties of the Fund
itself, and charges that to the Funds trustees. Thus, COAs directive to record the income of the Fund in DBPs books of
COA committed grave account as the miscellaneous income of DBP constitutes grave abuse of discretion. The income of
abuse of discretion. YES the Fund does not form part of the revenues or profits of DBP, and DBP may not use such income
for its own benefit. The principal and income of the Fund together constitute the res or subject
matter of the trust. The Agreement established the Fund precisely so that it would eventually be
sufficient to pay for the retirement benefits of DBP employees under RA 1616 without additional
outlay from DBP. COA itself acknowledged the authority of DBP to set up the Fund. However,
COAs subsequent directive would divest the Fund of income, and defeat the purpose for the
Funds creation.

The funds being under trusteeship, is not part of DBP funds. Therefore, COA does not have the
power to require that DBP use the funds for its own benefit. its not public funds.

Boy Scouts of the LB: The Boy Scouts of the Philippines (BSP) is a public corporation and its funds are subject to
Philippines vs. the Commission on Audit’s (COA’s) audit jurisdiction
Commission on Audit
There are three classes of juridical persons under Article 44 of the Civil Code and the BSP, as
presently constituted under Republic Act No. 7278,falls under the second classification. Article 44
reads:

Art. 44. The following are juridical persons:


(1) The State and its political subdivisions;
(2) Other corporations, institutions and entities for public interest or purpose created by
law; their personality begins as soon as they have been constituted according to
law;
(3) Corporations, partnerships and associations for private interest or purpose to which
the law grants a juridical personality, separate and distinct from that of each
shareholder, partner or member.

The Boy Scouts of the Philippines (BSP) which was created by a special law to serve a public
purpose in pursuit of a constitutional mandate, comes within the class of “public corporations”
defined by paragraph 2, Article 44 of the Civil Code.

AP: Evidently, the BSP, which was created by a special law to serve a public purpose in pursuit of
a constitutional mandate, comes within the class of "public corporations" defined by paragraph 2,
Article 44 of the Civil Code and governed by the law which creates it, pursuant to Article 45 of the
same Code, hence within the jurisdiction of COA.

Dela Llana vs. COA LB: The conduct of a pre-audit is not a mandatory duty that this Court may compel the COA to
perform. This discretion on its part is in line with the constitutional pronouncement that the COA
Is the pre-audit a has the exclusive authority to define the scope of its audit and examination. When the language
constitutional mandate of the law is clear and explicit, there is no room for interpretation, only application. Neither can
on the COA? NO. the scope of the provision be unduly enlarged by this Court

AP: There is nothing in Sec. 2 of Article IX-D that requires the COA to conduct a pre-audit of all
government transactions and for all government agencies. The only clear reference to a pre-audit
requirement is found in Sec. 2, par. 1, which provides that a post-audit is mandated for certain
government or private entities with state subsidy or equity and only when the internal control
system of an audited entity is inadequate. In such a situation, the COA may adopt measures,
including a temporary or special pre-audit, to correct the deficiencies. Hence, it is not mandatory.

Province of Aklan v. LB: COA and not the RTC which has primary jurisdiction to pass upon money claims against
Jody King Construction respondent local government unit.
& Devlopment Corp
The doctrine of primary jurisdiction holds that if a case is such that its determination requires the
expertise, specialized training and knowledge of the proper administrative bodies, relief must first
be obtained in an administrative proceeding before a remedy is supplied by the courts even if the
matter may well be within their proper jurisdiction. . It applies where a claim is originally
cognizable in the courts, and comes into play whenever enforcement of the claim requires the

92
resolution of issues which, under a regulatory scheme, have been placed within the special
competence of an administrative agency. In such a case, the court in which the claim is sought to
be enforced may suspend the judicial process pending referral of such issues to the administrative
body for its view or, if the parties would not be unfairly disadvantaged, dismiss the case without
prejudice.

The objective of the doctrine of primary jurisdiction is to guide the court in determining
whether it should refrain from exercising its jurisdiction until after an administrative
agency has determined some question or some aspect of some question arising in the
proceeding before the court.

AP: COA has primary jurisdiction over private respondent’s money claims Petitioner is not
estopped from raising the issue of jurisdiction.

Respondent seeks to enforce a claim for sums of money allegedly owed by petitioner, a local
government unit. Under Commonwealth Act No. 327, as amended by Section 26 of Presidential
Decree No. 1445, it is the COA which has primary jurisdiction over money claims against
government agencies and instrumentalities.

Respondent’s collection suit being directed against a local government unit, such money claim
should have been first brought to the COA. Hence, the RTC should have suspended the
proceedings and refer the filing of the claim before the COA. Moreover, petitioner is not estopped
from raising the issue of jurisdiction even after the denial of its notice of appeal and before the
CA.

Funa v. Manila LB: COA has the authority to audit non-governmental entities receiving equity or subsidy with
Economic Cultural respect to those “funds xxx coming from or through the government.
Office
AP: In this case, MECO was declared a sui generis entity, which, although governed by the
Whether or not MECO is Corporation Code, was entrusted by the government to collect verification fees and consular fees
subject to the audit as its agent in Taiwan. Thus, with respect to those fees, MECO is subject to the auditing authority
jurisdiction of COA? It of COA.
depends.
The MECO is not a GOCC or government instrumentality. The fact of the incorporation of the
MECO under the Corporation Code is key. However, the MECO was “ entrusted” by the
government with the “delicate and precarious” responsibility of pursuing “ unofficial” relations with
the people of a foreign land whose government the Philippines is bound not to recognize. Thus, it
is a sui generis private entity especially entrusted by the government with the facilitation of
unofficial relations with the people in Taiwan without jeopardizing the country’s faithful
commitment to the One China policy of the PROC. However, despite its non-governmental
character, the MECO handles government funds in the form of the “verification fees” it
collects on behalf of the DOLE and the “consular fees” it collects under Section 2(6) of EO
No. 15, s. 2001. Hence, under existing laws, the accounts of the MECO pertaining to its collection
of such “verification fees” and “consular fees” should be audited by the COA.

Fernando v. COA LB: The determination of COA's jurisdiction over a specific entity does not merely require an
examination of the nature of the entity. Should the entity be found to be non-governmental,
further determination must be had as to the source of its funds or the nature of the account
sought to be audited by the COA. The totality of an entity’s relations with the State must be
considered. If the corporation is created by the State as the latter's own agency or
instrumentality to help it in carrying out its governmental functions, then that corporation is
considered public; otherwise, it is private. The mere public purpose of an entity's existence does
not, per se, make it a public corporation. This class of corporations may be considered quasi-
public corporations, which are private corporations that render public service, supply public wants
or pursue other eleemosynary objectives. While purposely organized for the gain or benefit of its
members, they are required by law to discharge functions for the public benefit. Regardless of
the nature of the corporation, the determining factor of COA's audit jurisdiction is government
ownership or control of the corporation.

AP:
- the Executive Committee is subject to COA jurisdiction, considering its administrative
relationship to the Metro Manila Development Authority, a government agency tasked to
perform administrative, coordinating and policy-setting functions for the local government
units in the Metropolitan Manila area.

- The records simply establish that the Executive Committee is an office under the MMDA, a
public agency, subject to the audit jurisdiction of the COA. Moreover, the funds become
public once it is received on behalf of the Govt. for the purposes for which the MMFF was
established.

NOT A GOCC
 Considering the establishment and mechanism of the Executive Committee of the MMFF, it is
at once apparent that it is not a government-owned and controlled corporation. In this case,
there is nothing in the records which establishes that the Executive Committee of the MMFF is
organized as a stock or non-stock corporation. It does not have capital which is to be divided
into shares of stock, nor stockholders and voting shares, as to qualify as a stock corporation. 
 We cannot also deem it a non-stock corporation. Though undoubtedly organized for cultural
purposes, the Executive Commitee of the MMFF is ostensibly just a group of representatives
of various stakeholders m the Philippine movie industry, it has no other members.

BUT STILL UNDER COA JURISDICTION

93
 BUT, Such finding notwithstanding, We find that the Executive Committee is subject to COA
jurisdiction, considering its administrative relationship to the Metro Manila Development
Authority, a government agency tasked to perform administrative, coordinating and policy-
setting functions for the local government units in the Metropolitan Manila area. 
 The public nature of MMDA is apparent in its charter, Republic Act (R.A.) No. 7924. It is an
agency created for the purpose of laying down policies and coordinating with the various
national government agencies, people's organizations, non-governmental organizations and
the private sector for the efficient and expeditious delivery of basic services in the vast
metropolitan area. All its functions are administrative in nature
 The Executive Committee, having been created to assist the MMDA in the conduct of the
annual Manila Film Festival, cannot be treated separately from the legal existence and nature
of the agency it is tasked to give assistance to. It is likewise apparent that the observance of
the annual film festival, entails activities which impacts some, if not all local government
units of the Metropolitan Manila. 
 The link between MMDA and the Executive Committee is likewise evident from the
establishment of a Secretariat within the MMDA, which will assist the committee in the
discharge of its function. 
 In addition, this Court notes that the multi-sectoral membership of the executive committee
mirrors the network MMDA is authorized to establish under its Charter
 This Court cannot accord merit to petitioner's arguments which seek to treat separately the
Executive Committee from the MMDA. Certainly, that would amount to creating another
entity without basis in law and in fact. The records simply establish that the Executive
Committee is an office under the MMDA, a public agency, subject to the audit jurisdiction of
the COA.
 The conclusion reached by the Court in Funa cannot be applied in the case at bar. Compared
to MECO, which is an incorporated body, the Executive Committee is merely an office under
MMDA, created pursuant to a Presidential Proclamation passed in 1975, when the legislative
power was exercised by the President. It cannot likewise be denied that some of the original
members of the Executive Committee, as well as current ones, are public officials. 
The membership of representatives from private companies, such as those coming from the
cinema owners, merely incidental to the operations and the activities held during the duration
of the annual film festival.
 Two Sources of Funds of ExecComm:
o The donations from the local government units comprising the Metropolitan Manila
covering the period of holding the MMFF from December 25 to January 3; and 
o The non-tax revenues that come in the form of donations from private entities.
 As a committee under MMDA, a public office, this Court finds that both sources of funds can
properly be subject of COA's audit jurisdiction. 
 That the Executive Committee of the MMFF administers funds from the government is
apparent in the following portion of Proclamation No. 1459: 
o The Executive Committee is authorized to engage in fund raising campaign among
all sectors of society including the local governments concerned which may donate
their amusement tax shares to the MOWELFUND during the period of the celebration
to make it a success.
 Verily, if non-governmental entities maybe audited by the COA as long as its funds are partly
coming from the government, with more reason should this principle apply to the Executive
Committee.
 As to the committee's funds coming from non-tax revenues, the fact that such funds come
from purported private sources, do not convert the same to private funds. Such funds must
be viewed with the public purpose for which it was solicited, which is the management of the
MMFF.
 Furthermore, despite the private source of funds, ownership over the same was already
transmitted to the government by way of donation. As donee, the government had become
the owner of the funds, with full ownership rights and control over the use and disposition of
the same, subject only to applicable laws and COA rules and regulations. Thus, upon
donation to the government, the funds became public in character.
 Applying the principles enunciated in the aforesaid cases, and considering the purpose for
which COA was created, this Court finds that any such funds, though coming from private
sources, become public upon receipt by the Executive Committee, for use in the purpose for
which it was created. 

ARTICLE III BILL OF RIGHTS

Basic Principles of Police Power, Eminent Domain and Taxation

City of Manila v. Judge D:


Laguio
(1)POLICE POWER OF LOCAL GOVERNMENTS IS SUBJECT TO LIMITATIONS

The police power of the City Council, however broad and far-reaching, is subordinate to
the constitutional limitations thereon; and is subject to the limitation that its exercise
must be reasonable and for the public good.

The police power granted to local government units must always be exercised with
utmost observance of the rights of the people to due process and equal protection of
the law;
- Due process requires the intrinsic validity of the law in interfering with the rights of the person
to his life, liberty and property

(2) REASONABLE RELATION: PURPOSE AND MEANS EMPLOYED

A reasonable relation must exist between the purposes of the police measure and the
94
means employed for its accomplishment, for even under the guise of protecting the public
interest, personal rights and those pertaining to private property will not be permitted to be
arbitrarily invaded.

Requisites:

1. It must appear that the interests of the public generally, as distinguished from those of a
particular class, require an interference with private rights, [LAWFUL SUBJECT]

2. Means adopted must be reasonably necessary for the accomplishment of the purpose
and not unduly oppressive upon individuals. [LAWFUL MEANS]
- It must be evident that no other alternative for the accomplishment of the purpose
less intrusive of private rights can work.

Lacking a concurrence of these two requisites, the police measure shall be struck down as an
arbitrary intrusion into private rights—a violation of the due process clause.

(3) TAKING OF A PROPERTY WITHOUT JUST COMPENSATION

An ordinance which permanently restricts the use of property that it can not be used for
any reasonable purpose goes beyond regulation and must be recognized as a taking of the
property without just compensation.

AP:

LGU’s Ordinance No. 7783 is NOT a valid exercise of police power.

In passing the Ordinance No. 7783, Manila LGU’s purpose was


- the promotion and protection of the social and moral values of the community
- by addressing and arresting the social ills purportedly spawned by the establishments in the
Ermita-Malate area which are allegedly operated under the deceptive veneer of legitimate,
licensed and tax-paying nightclubs, bars, karaoke bars, girlie houses, cocktail lounges, hotels
and motels.

While it pursued a valid purpose, the means planned to be employed was not
reasonable and unduly oppressive.

- The closing down and transfer of businesses or their conversion into businesses “allowed” under
the Ordinance have no reasonable relation to the accomplishment of its purposes.
- The prohibition of the enumerated establishments will not per se protect and promote
the social and moral welfare of the community; it will not in itself eradicate the
alluded social ills of prostitution, adultery, fornication nor will it arrest the spread of
sexual disease in Manila.

WHAT SHOULD HAVE BEEN DONE


- If the City of Manila so desires to put an end to prostitution, fornication and other social ills, it
can instead impose reasonable regulations such as daily inspections of the establishments for
any violation of the conditions of their licenses or permits; it may exercise its authority to
suspend or revoke their licenses for these violations; and it may even impose increased license
fees. In other words, there are other means to reasonably accomplish the desired end.

***
Other Doctrines:

Tests of a valid ordinance:


1. Must not contravene the Constitution or any statute;
• Test of constitutionality and
• Test of consistency with the prevailing laws.
2. Must not be unfair or oppressive;
3. Must not be partial or discriminatory;
4. Must not prohibit but may regulate trade;
5. Must be general and consistent with public policy; and
6. Must not be unreasonable.

The Ordinance infringes the Due Process Clause.


• Violates the constitution.
• It was not a valid exercise of police power.

Equal Protection Clause requires that all persons or things similarly situated should be treated
alike, both as to rights conferred and responsibilities imposed.
- Similar subjects, in other words, should not be treated differently, so as to give undue favor to
some and unjustly discriminate against others.
Requirements:
1. It must be based on substantial distinctions.
2. It must be germane to the purposes of the law.
3. It must not be limited to existing conditions only.
4. It must apply equally to all members of the class.

AP: Here the ordinance discriminates against motels and women. Both men and women can
engage in prostitution.

Mun. of Paranaque v. D:
VM Realty
(1)POWER OF EMINENT DOMAIN
Municipality of Paranaque
95
filed a complaint for A Local Government Unit (LGU) may exercise the power to expropriate private property
expropriation against VM - only when authorized by Congress and
Realty Corporation over 2 - subject to the latter’s control and restraints, imposed through the law conferring the
parcels of land for the power or in other legislations.
purpose of alleviating
conditions of the Note: The power of eminent domain is lodged in the legislative branch of government, which may
underprivileged by delegate the exercise thereof to LGUs, other public entities and public utilities.
providing homes for the
homeless through a
socialized housing (2) ESSENTIAL REQUISITES BEFORE AN LGU CAN EXERCISE THE POWER OF EMINENT
project. DOMAIN.

The following essential requisites must concur before an LGU can exercise the power of eminent
domain [Local Government Code]:
1. An ordinance is enacted by the local legislative council authorizing the local chief
executive, in behalf of the LGU, to exercise the power of eminent domain or pursue
expropriation proceedings over a particular private property.
2. The power of eminent domain is exercised for public use, purpose or welfare , or for
the benefit of the poor and the landless.
3. There is payment of just compensation , as required under Section 9, Article III of the
Constitution, and other pertinent laws.
4. A valid and definite offer has been previously made to the owner of the property
sought to be expropriated, but said offer was not accepted.
AP:

In the case at bar, the local chief executive sought to exercise the power of eminent domain
pursuant to a resolution of the municipal council. Thus, there was no compliance with the
first requisite that the mayor be authorized through an ordinance.

LGU of Parañaque failed to comply with first requisite of exercise of eminent domain, because it
is based on mere resolution, not ordinance. LGC requires an ordinance for valid
expropriation.

(3) AN ORDINANCE IS DIFFERENT FROM A RESOLUTION

ORDINANCE
- A law
- Possesses a general and permanent character.
- Third reading is necessary for its enactment.

RESOLUTION
- Merely a declaration of the sentiment or opinion of the lawmaking body on a specific matter.
- Temporary in nature.
- Does not requires a third reading, unless decided otherwise by a majority of the Sangguinan
members.

Roxas v. Court of D:
Appeals
THE IMPLEMENTATION OF THE CARL IS AN EXERCISE OF THE STATE’S POLICE POWER
AND THE POWER OF EMINENT DOMAIN

- To the extent that the CARL prescribes retention limits to the landowners, there is an exercise
of police power for the regulation of private property,
- but where, to carry out such regulation, the owners are deprived of lands they own in excess of
the maximum area allowed, there is also a taking under the power of eminent domain

NOTE: The taking contemplated is not a mere limitation of the use of the land. What is required
is the surrender of the title to and physical possession of the said excess and all beneficial rights
accruing to the owner in favor of the farmer beneficiary.

THE EXERCISE OF THE POWER OF EMINENT DOMAIN REQUIRES THAT DUE PROCESS BE
OBSERVED IN THE TAKING OF PRIVATE PROPERTY.

- The procedure in the sending of notices in the implementation of the CAR Program is
important to comply with the requisites of due process especially when the owner is a juridical
entity.

2-NOTICE RULE:

For a valid implementation of the CARP in addition to proper identification of subject lands, two
notices are required:
1. the Notice of Coverage and letter of invitation to a preliminary conference sent to the
landowner, the representatives of the BARC, LBP, farmer beneficiaries and other interested
parties pursuant to DAR A.O. No. 12, Series of 1989; and
2. the Notice of Acquisition sent to the landowner under Section 16 of the CARL.

AP: Proceedings were nullified for failure of DAR to observe due process but remanded
to DAR for proper acquisition proceedings and determination of petitioner's application for
conversion.

In this case the notices were sent to Pimentel (hacienda administrator).


· Neither the president, manager, secretary, cashier or director of petitioner corporation
(He was only the administrator of the said Haciendas).
96
· Even assuming that Pimentel was Roxas’ agent, and the notices and letters of invitation
were validly served on Roxas through him, there is no showing that Pimentel himself was
duly authorized to attend the conference meeting with the MARO, BARC and LBP
representatives and farmer beneficiaries for purposes of compulsory acquisition of
petitioner’s landholdings.
· Roxas even claims that it had no knowledge of the letter-invitation, hence, could not
have given Pimentel the authorit

LTO v. City of Butuan D: THE POWER OF LGU TO REGULATE THE OPERATION OF TRICYCLES AND TO GRANT
FRANCHISES FOR THE OPERATION THEREOF IS STILL SUBJECT TO THE GUIDELINES
Sangguniang PRESCRIBED BY THE DEPARTMENT OF TRANSPORTATION AND COMMUNICATIONS.
Panglungsod of Butuan
passed an ordinance “To regulate” means to fix, establish, or control; to adjust by rule, method, or established
regulating the operation mode; to direct by rule or restriction; or to subject to governing principles or laws.
of tricycles and providing
for the mechanism for the A franchise is defined to be a special privilege to do certain things conferred by government on
issuance of franchise, an individual or corporation, and which does not belong to citizens generally of common right.
registration and permit.
“to register” means to record formally and exactly, to enroll, or to enter precisely in a list or the
like, and a “driver’s license” is the certificate or license issued by the government which
LTO claims that it has the authorizes a person to operate a motor vehicle.
authority to register all
motor vehicles. POLICE POWER AND TAXATION ARE SEPARATE AND DISTINCT POWERS

To settle this issue, the All these inherent powers are for a public purpose and legislative in nature but the similarities
City of Butuan, just about end there.
represented by its Mayor, - The basic aim of police power is public good and welfare.
filed a petition seeking - Taxation, in its case, focuses on the power of government to raise revenue in order to
the declaration of the support its existence and carry out its legitimate objectives.
validity of the ordinance, - Although correlative to each other in many respects, the grant of one does not necessarily carry
which the LTO opposed. with it the grant of the other. The two powers are, by tradition and jurisprudence, separate and
distinct powers, varying in their respective concepts, character, scopes and limitations.
I: Whether the
registration of tricycles AP: LGU has no power to give registration and license to tricyles and tricycle drivers.
was given to LGU's,
hence the ordinance is a Based on the-"Guidelines to Implement the Devolution of LTFRBs Franchising Authority over
valid exercise of police Tricycles-For-Hire to Local Government units pursuant to the Local Government Code"- the
power. newly delegated powers to LGU's pertain to the franchising and regulatory powers
exercised by the LTFRB and not to the functions of the LTO relative to the registration of motor
vehicles and issuance of licenses for the driving thereof. Corollarily, the exercised of a police
power must be through a valid delegation. In this case t he police power of registering
tricycles was not delegated to the LGU’s, but remained in the LTO.

Under LGC, certain functions of the DOTC were transferred to the LGUs, such as the power to
regulate the operation of tricycle and to grant franchise for the operation thereof.

LTO:
- Registration
- Record or enroll or enter in a list and issue a driver’s license for driving thereof. Hence,
licensing.

LGU
- Regulate (Fix, establish or control)
- Grant franchise (A special privilege to do certain things conferred by the government)

LGUs are in good position to curb the alarming increase of accidents in national highways
involving tricycles because of their proximity to the situation that can enable them to address that
serious concern better than the national government
White Light D: Police power, while incapable of an exact definition, has been purposely veiled in general
Corporation vs. City of terms to underscore its comprehensiveness to meet all exigencies and provide enough
Manila room for an efficient and flexible response as the conditions warrant

The City of Manila issued Police power is based upon the concept of necessity of the State and its corresponding
an ordinance prohibiting right to protect itself and its people.
Short-Time Admission, - Police power has been used as justification for numerous and varied actions by the State. These
Short-Time Admission range from the regulation of dance halls, movie theaters, gas stations and cockpits. The
Rates, and Wash-Up Rate awesome scope of police power is best demonstrated by the fact that in its hundred or so years
Schemes in Hotels, of presence in our nation’s legal system, its use has rarely been denied.
Motels, Inns, Lodging
Houses, Pension Houses, Police power is rebuked by the Bill of Rights, particularly the right to due process,
and Similar which has 2 aspects – procedural and substantive
Establishments in the
City of Manila to minimize Procedural - procedures the government follows before it deprives a person of life, liberty or
if not eliminate the use of property - notice and hearing
the covered
establishments for illicit Substantive due process - There are other legitimate activities which the Ordinance would
sex, prostitution, drug impair – families during brownout, transit passengers who wish to wash up between trips.
use, etc.
It must appear that the interests of the public generally, as distinguished from a particular
The ordinance sanctions class, require an interference with private rights and the means must be reasonably
any person or corporation necessary for the accomplishment of the purpose and not unduly oppressive of private
who will allow the rights, that no other alternative for its accomplishment that is less intrusive can work,
admission and charging and a reasonable relation must exist between the purposes of the measure and the
97
of room rates for less means employed
than 12 hours or the
renting of rooms more AP: Ordinance is not valid.
than twice a day.
While the intent of the ordinance was to curtail illicit sexual behavior, it nevertheless
also curtailed legitimate sexual behaviors among consenting married and consenting
adults. The ordinance is an intrusion to the private right of the clients.
it cannot also be ignored that there are other legitimate activities which the Ordinance would
impair
- Families during brownout
- Transit passengers who wish to wash up between trips.
Southern Drug D:
Corporation vs. DSWD (1) It is in the exercise of its police power that the Congress enacted Republic Act (RA)
Nos. 9257 and 9442, the laws mandating a twenty percent (20%) discount on
I: (1) Whether the State, purchases of medicines made by senior citizens and persons with disabilities (PWDs). It
in promoting the health is also in further exercise of this power that the legislature opted that the said discount be
and welfare of a special claimed as tax deduction, rather than tax credit, by covered establishments.
group of citizens, can
impose upon private In Gerochi v. Department of Energy: [P]olice power
establishments the - is the power of the state to promote public welfare by restraining and regulating the use of
burden of partly liberty and property.
subsidizing a government - It is the most pervasive, the least limitable, and the most demanding of the three fundamental
program? YES powers of the State.
- The justification is found in the Latin maxim salus populi est suprema lex (the welfare of the
(2) The petitioner, claims people is the supreme law) and sic utere tuo ut alienum non laedas (so use your property as
that the change in the tax not to injure the property of others).
treatment of the discount - As an inherent attribute of sovereignty which virtually extends to all public needs,
is illegal as it constitutes police power grants a wide panoply of instruments through which the State, as parens
taking without just patriae, gives effect to a host of its regulatory powers. We have held that the power
compensation: WRONG to “regulate” means the power to protect, foster, promote, preserve, and control, with
(SEE D (4)) due regard for the interests, first and foremost, of the public, then of the utility and of
its patrons.
(3) Is is it violative of
equal protection clause? (2) In the exercise of police power, “property rights of private individuals are subjected
NO to restraints and burdens in order to secure the general comfort, health, and prosperity
of the State.”
- It is a measure, which by sheer necessity, the State exercises, even to the point of interfering
with personal liberties or property rights in order to advance common good.

To warrant such interference, two requisites must concur: [LAWFUL SUBJECT AND LAWFUL
METHOD]
(a) the interests of the public generally, as distinguished from those of a particular class,
require the interference of the State; and
(b) the means employed are reasonably necessary to the attainment of the object sought to
be accomplished and not unduly oppressive upon individuals.

(3) Republic of the Philippines v. Vda. de Castellvi: 5 circumstances must be present in order
to qualify “taking” as an exercise of eminent domain:

First, the expropriator must enter a private property.


Second, the entrance into private property must be for more than a momentary period.
Third, the entry into the property should be under warrant or color of legal authority.
Fourth, the property must be devoted to a public use or otherwise informally appropriated or
injuriously affected.
Fifth, the utilization of the property for public use must be in such a way as to oust the owner
and deprive him of all beneficial enjoyment of the property.

(4) Regulatory laws are within the category of police power measures from which
affected persons or entities cannot claim exclusion or compensation.

Unlike in the exercise of the power of eminent domain, just compensation is not
required in wielding police power. This is precisely because there is no taking involved,
but only an imposition of burden.

AP:

THE LAW WAS PASSED IN THE EXERCISE OF POLICE POWER.

The Senior Citizens Act was enacted primarily to maximize the contribution of senior citizens to
nation-building, and to grant benefits and privileges to them for their improvement and well-
being as the State considers them an integral part of our society.
The law is a legitimate exercise of police power which, similar to the power of eminent
domain, has general welfare for its object.

The duty to care for the elderly and the disabled lies not only upon the State, but also
on the community and even private entities.

As to the State, the duty emanates from its role as parens patriae which holds it under obligation
to provide protection and look after the welfare of its people especially those who cannot tend to
themselves. Parens patriae means parent of his or her country, and refers to the State in
its role as “sovereign,” or the State in its capacity as a provider of protection to those
unable to care for themselves. In fulfilling this duty, the State may resort to the
exercise of its inherent powers: police power, eminent domain and power of taxation.

98
NOT BASED ON EMINENT DOMAIN, AS THERE IS NO TAKING

1. The first requirement speaks of entry into a private property which clearly does not obtain in
this case. There is no private property that is invaded or appropriated by the State. As
it is, the petitioner precipitately deemed future profits as private property and then proceeded
to argue that the State took it away without full compensation. This seemed preposterous
considering that the subject of what the petitioner supposed as taking was not even earned
profits but merely an expectation of profits, which may not even occur. For obvious
reasons, there cannot be taking of a contingency or of a mere possibility because it
lacks physical existence that is necessary before there could be any taking. Further, it
is impossible to quantify the compensation for the loss of supposed profits before it is
earned.

2. The supposed taking also lacked the characteristics of permanence and consistency. The
presence of these characteristics is significant because they can establish that the effect of the
questioned provisions is the same on all establishments and those losses are indeed its
unavoidable consequence. But apparently these indications are wanting in this case. The
reason is that the impact on the establishments varies depending on their response
to the changes brought about by the subject provisions. To be clear, establishments are
not prevented from adjusting their prices to accommodate the effects of the granting of the
discount and retain their profitability while being fully compliant to the laws. It follows that
losses are not inevitable because establishments are free to take business measures
to accommodate the contingency. Lacking in permanence and consistency, there can be no
taking in the constitutional sense. There cannot be taking in one establishment and none in
another, such that the former can claim compensation but the other may not. Simply told,
there is no taking to justify compensation; there is only poor business decision to
blame.

3. There is also no ousting of the owner or deprivation of ownership. Establishments are


neither divested of ownership of any of their properties nor is anything forcibly taken from
them. They remain the owner of their goods and their profit or loss still depends on the
performance of their sales.

Apart from the foregoing, covered establishments are also provided with a mechanism to recoup
the amount of discounts they grant the senior citizens and PWDs. Basically, whatever amount was
given as discount, covered establishments may claim an equal amount as an expense or tax
deduction

Zaval v. Duterte D: Police power, amongst the three fundamental and inherent powers of the state, is the most
pervasive and comprehensive
Assailed law: closure of - It has been defined as the 'state authority to enact legislation that may interfere with personal
Boracay. liberty or property in order to promote general welfare.
- As defined, it consists of
Petitioners are residents (1) imposition or restraint upon liberty or property,
of Boracay who assails (2) in order to foster the common good.
that the President’s
statute closing Boracay - It is not capable of exact definition but has been purposely, veiled in general terms to
affected their livelihood underscore its all-comprehensive embrace.
and violated the right to - The police power "finds no specific Constitutional grant for the plain reason that it does not owe
travel of tourists. its origin to the Charter since "it is inborn in the very fact of statehood and sovereignty.”
- It is said to be the "inherent and plenary power of the State which enables it to
I: does Proclamation No. prohibit all things hurtful to the comfort, safety, and welfare of the society.”
475 constitute an - Thus, police power constitutes an implied limitation on the Bill of Rights. After all, "the
impairment on the right Bill of Rights itself does not purport to be an absolute guaranty of individual rights and liberties.
to travel? NO. 'Even liberty itself, the greatest of all rights, is not unrestricted license to act according to one's
will.' It is subject to the far more overriding demands and requirements of the greater number."
Was the temporary
closure of Boracay as a NOT ABSOLUTE
tourist destination for six "It has to be exercised within bounds – lawful ends through lawful means, i.e.,
months reasonably 1. that the interests of the public generally, as distinguished from that of a particular class,
necessary under the require its exercise, and
circumstances? YES 2. that the means employed are reasonably necessary for the accomplishment of the
purpose while not being unduly oppressive upon individuals."

AP: The assailed governmental measure is within the scope of police power .

As to lawful subject:
The statutes from which the said measure draws authority and the constitutional provisions which
serve as its framework are primarily concerned with the environment and health, safety, and
well-being of the people, the promotion and securing of which are clearly legitimate objectives of
governmental efforts and regulations. The motivating factor in the issuance of Proclamation
No. 475 is without a doubt the interest of the public in general.

As to lawful means:

One of the root causes of the problems that beset Boracay was tourist influx.
- Tourist arrivals in the island were clearly far more than Boracay could handle. As early as 2007,
the DENR had already determined this as the major cause of the catastrophic depletion of the
island's biodiversity.
- Also part of the equation is the lack of commitment to effectively enforce pertinent
environmental laws.
Unfortunately, direct action on these matters has been so elusive that the situation reached a
critical level. Hence, by then, only bold and sweeping steps were required by the situation.

99
The closure of Boracay, albeit temporarily, gave the island its much needed breather, and
likewise afforded the government the necessary leeway in its rehabilitation program.
- The rehabilitation works in the first place were not simple, superficial or mere cosmetic but
rather quite complicated, major, and permanent in character as they were intended to serve as
long-term solutions to the problem.
- Also, time is of the essence. Every precious moment lost is to the detriment of Boracay's
environment and of the health and well-being of the people thereat.
- operations of establishments in Boracay had to be halted in the course thereof since majority, if
not all of them, need to comply with environmental and regulatory requirements in order to
align themselves with the government's goal to restore Boracay into normalcy and develop its
sustainability.
- It could not be said that Boracay, at the time of the issuance of the questioned proclamation,
was in such a physical state that would meet its purpose of being a tourist destination. For one,
its beach waters could not be said to be totally safe for swimming.

To the mind of the Court, this period constitutes a reasonable time frame, if not to
complete, but to at least put in place the necessary rehabilitation works to be done in
the island. Indeed, the temporary closure of Boracay, although unprecedented and radical as it
may seem, was reasonably necessary and not unduly oppressive under the circumstances.

Section 1

A. Life, Liberty or Property

American Inter D: A privilege turned into a property right shall not be removed arbitrarily and without
Fashion Corporation v. due process.
Office of the President Mabuhay Textile Mills Corporation v. Ongpin:
While it is true that such allocations as alleged by the Board are mere privileges which it can
Glorious Sun was found revoke and cancel as it may deem fit, these privileges have been accorded to petitioner for
guilty of misdeclaration of so long that they have become impressed with property rights.
imported raw materials
resulting in dollar salting AP: In this case, the private respondent's export quota allocation which initially was a
abroad. privilege evolved into some form of property right which should not be removed from it
arbitrarily and without due process only to hurriedly confer it on another.

The Garments and Textile Glorious Sun was denied due process when GETB failed to disclose evidence it used in
Export Board (GETB) rendering a resolution against it. Contrary to petitioner’s posture, the record clearly manifests
cancelled its export that in cancelling the export quotas granted to it. Before its cancellation in 1984, it had been
quotas. enjoying export quotas since 1977. Thus, its allocation which initially was a privilege evolved
The export quota was into some form of property right which should not be removed from it arbitrarily and without due
then awarded to 2 newly- process only to hurriedly confer it on another.
formed corporations (De
Soleil and American
Inter-Fashion)

The Office of the Pres. set


aside the GETB decision
on the ground that
Glorious Sun was not
accorded due process
during trial.

Chavez v. Romulo D: A license authorizing a person to enjoy a certain privilege is neither a property nor a
property right.
Tan vs. The Director of Forestry: “a license is merely a permit or privilege to do what
A ban on the carrying
of firearms outside of otherwise would be unlawful, and is not a contract between the authority granting it and the
person to whom it is granted; neither is it property or a property right, nor does it create a
residence was imposed
in order to deter the vested right.”
- The issuance of the license it is a mere privilege subject to revocation and restriction as the law
rising crime rates.
may provide.

Chavez questions the


ban as a violation of his AP: A Permit to Carry Firearms Outside Residence (PTCFOR) does not constitute a
right to property. property right protected under our Constitution
- The PNP Chief is granted broad discretion in the issuance of PTCFOR as provided in the IRR of
• That the revocation of PD No. 1866.
his permit to carry - PTCFOR, just like ordinary licenses in other regulated fields, may be revoked any time—it does
firearms is not confer an absolute right, but only a personal privilege to be exercised under existing
unconstitutional; and restrictions, and such as may thereafter be reasonably imposed.
• That right to carry
firearms is a vested Even if it’s a property right, it’s still under the police power of the Government
property right. At any rate, assuming that petitioner’s PTCFOR constitutes a property right protected by the
Constitution, the same cannot be considered as absolute as to be placed beyond the reach of the
State’s police power. All property in the state is held subject to its general regulations, necessary
to the common good and general welfare.

Executive Secretary v. D: A profession, trade or calling is a property right within the meaning of our
Court of Appeals constitutional guarantees;

100
- One cannot be deprived of the right to work and the right to make a living because these rights
Assailed law: RA8042 are property rights, the arbitrary and unwarranted deprivation of which normally constitutes an
(Migrant Worker’s Act) actionable wrong.

HOWEVER, IT IS NOT ABSOLUTE


private respondents - The State, in the exercise of its police power, may, through legislative action legislative
assailed the action, define the mode and manner in which every one may so use his own property so as not
constitutionality of the to pose injury to himself or others.
Migrant Workers and
- To pretend that licensing or accreditation requirements violates the due process clause is to
Overseas Filipino Act,
ignore the settled practice, under the mantle of the police power, of regulating entry to the
which, among others,
practice of various trades or professions.
mandated that only
- In addition, the equal protection clause of the Constitution does not forbid classification
skilled Filipino workers be
for so long as:
deployed as overseas
1. such classification is based on real and substantial differences having a reasonable
workers, except those
relation to the subject of the particular legislation;
deployed by local service
2. is germane to the purpose of the law,
contractors and manning
3. concerns all members of the class, and
agents.
4. applies equally to present and future conditions,
the classification does not violate the equal protection guarantee.
Whether or not the State
may allow only skilled AP: In this case, Migrant Worker’s Act does not deprive private respondent of their right to
workers to be deployed property.
as overseas workers? Yes To pretend that licensing or accreditation requirements violates the due process clause is to
ignore the settled practice, under the mantle of the police power, of regulating entry to the
practice of various trades or professions.
- Professionals leaving for abroad are required to pass rigid written and practical exams before
they are deemed fit to practice their trade.
- Seamen are required to take tests determining their seamanship.
- Locally, the Professional Regulation Commission has begun to require previously licensed
doctors and other professionals to furnish documentary proof that they had either re-trained or
had undertaken continuing education courses as a requirement for renewal of their licenses.

It is not claimed that these requirements pose an unwarranted deprivation of a property right
under the due process clause. So long as professionals and other workers meet reasonable
regulatory standards no such deprivation exists.

As to violation non-impairment clause

No violation of non-impairment clause. Equally important, into every contract is read provisions of
existing law, and always, a reservation of the police power for so long as the agreement deals
with a subject impressed with the public welfare.

As to violation of equal protection clause

Singling out of entertainers and performing artists under the assailed department orders DOES
NOT constitute class legislation. All the requisites of a vaild classification were met.

Duncan Association v. D: It is a settled principle that the commands of the equal protection clause are addressed
Glaxo only to the state or those acting under color of its authority.

GR: Corollarily, it has been held in a long array of U.S. Supreme Court decisions that the equal
protection clause erects no shield against merely private conduct, however,
discriminatory or wrongful.
EXC: The only exception occurs when the state in any of its manifestations or actions has
been found to have become entwined or involved in the wrongful private conduct.

AP: Glaxo’s policy prohibiting an employee from having a relationship with an employee of a
competitior company is a valid exercise of management prerogative.

The challenged company policy does not violate the equal protection clause of the
Constitution as petitioners erroneously suggest.

- Glaxo has a right to guard its trade secrets, manufacturing formulas, marketing strategies, and
other confidential programs and information from competitors.
- The prohibition against pesonal or marital relationships with employees of competitor
companies upon Glaxo's employees is reasonable under the circumstances because
relationships of that nature might compromise the interests of the company
- That Glaxo possesses the right to protect its economic interest cannot be denied as the
Constitution likewise protects the right of enterprises.

Remman Enterprises v. D:
Professional
Regulatory Board (1)No right is absolute, and the proper regulation of a profession, calling, business or
trade has always been upheld as a legitimate subject of a valid exercise of the police
Assailed law: RA 9646 power of the State particularly when their conduct affects the
(Real Estate Service Act - execution of legitimate governmental functions,
of the Philippines) - aims - the preservation of the State,
to professionalize the real - public health and
estate service sector - welfare and public morals.
under a regulatory
scheme of licensing, (2) Equal Protection Clause - Requisites
registration and If classification is germane to the purpose of the law, concerns all members of the class, and
supervision of real estate applies equally to present and future conditions, the classification does not violate the equal
service practitioners protection guarantee.
101
Petitioner’s Claim: They AP: RA 9646 a valid exercise of the State’s police power.
stress that real estate
developers are now Here, the legislature recognized the importance of professionalizing the ranks of real estate
burdened by law to practitioners by increasing their competence and raising ethical standards as real property
employ licensed real transactions
estate brokers to sell, - It "susceptible to manipulation and corruption, especially if they are in the hands of unqualified
market and dispose of persons working under an ineffective regulatory system.”
their properties.
No Violation of Equal Protection Clause - there is substantial distinction between real estate
developers and ordinary property owners.

- Real estate developers at present constitute a sector that hires or employs the largest number
of brokers, salespersons, appraisers and consultants due to the sheer number of products (lots,
houses and condominium units) they advertise and sell nationwide.
- To protect the interest of home and lot buyers from fraudulent acts and manipulations
perpetrated by these unscrupulous subdivision and condominium sellers and operators, P.D. No.
957 was issued to strictly regulate housing and real estate development projects
- Hence, in approving R.A. No. 9646, the legislature rightfully recognized the necessity of
imposing the new licensure requirements to all real estate service practitioners, including and
more importantly, those real estate service practitioners working for real estate developers.

The foregoing shows that substantial distinctions do exist between ordinary property
owners exempted under Section 28(a) and real estate developers like petitioners, and
the classification enshrined in R.A. No. 9646 is reasonable and relevant to its legitimate
purpose. The Court thus rules that R.A. No. 9646 is valid and constitutional.

B. Procedural due process

Banco Español Filipino D:


v. Palanca (1) REQUISITES OF DUE PROCESS:
As applied to judicial proceedings, due process of law implies that there must be:
Engracio Palanca was 1. a court or tribunal clothed with power to hear and determine the matter before it,
indebted to El Banco, 2. that jurisdiction shall have been lawfully acquired,
which was secured by his 3. that the defendant shall have an opportunity to be heard, and
parcel of land, worth 4. that judgment shall be rendered upon lawful hearing.
more than his debt. He
failed to pay his debt but (2) CONSTRUCTIVE OR SUBSTITUTED SERVICE
he left for China and died - Though commonly called constructive, or substituted service, such notification does not
there. Since Engracio is a constitute a service of process in any true sense.
non resident, El Banco - It is merely a means provided by law whereby the owner may be admonished that his property
has to notify Engracio is the subject of judicial proceedings and that it is incumbent upon him to take such steps as he
about their intent to sue sees fit to protect it.
him by means of - It will be observed that this mode of notification does not involve any absolute assurance
publication using a that the absent owner shall thereby receive actual notice.
newspaper. The lower - The provision of our law relative to the mailing of notice does not absolutely require the mailing
court further ordered the of notice unconditionally and in every event, but only in the case where the defendant's
clerk of court to furnish residence is known.
Engracio a copy to be
sent to Amoy, China
AP: It is evident that actual notice to the defendant in cases of this kind is not, under the
law, to be considered absolutely necessary.
El Banco to foreclose
Engracio’s property. 7
The Code of Civil Procedure, indeed, expressly declares that there is a presumption that things
years thereafter, Vicente
have happened according to the ordinary habits of life
surfaced on behalf of
Engracio as his
In support of this presumption, as applied to the present case,
administrator to petition
- it is permissible to consider the probability that the defendant may have received
for the annulment of the
actual notice of these proceedings from the unofficial notice addressed to him in
ruling, averring that there
Manila which was mailed by an employee of the bank's attorneys.
had been no due process
as Engracio never - in view of the well-known skill of postal officials and employees in making proper delivery
received the summons. of letters defectively addressed, we think the presumption is clear and strong that this
notice reached the defendant, there being no proof that it was ever returned by the
postal officials as undelivered.
- And if it was delivered in Manila, instead of being forwarded to Amoy, China, there is a
probability that the recipient was a person sufficiently interested in his affairs to send
it or communicate its contents to him.

THUS,
(1) that the failure of the clerk to send the notice to the defendant by mail did not
destroy the jurisdiction of the court and
(2) that such irregularity did not infringe the requirement of due process of law.

Bautista v. Court of D: Where a party was afforded an opportunity to participate in the proceedings but
Appeals failed to do so, he cannot complain of deprivation of due process.

Petitioner filed a Due process is satisfied as long as the party is accorded an opportunity to be heard.
complaint for quieting of - If it is not availed of, it is deemed waived or forfeited without violating the constitutional
title against Manila guarantee.
Papermills, International, - Moreover, the grant of a motion for continuance or postponement is not a matter of right. It is
Inc. addressed to the sound discretion of the court.

102
After several delays That the absence of a party during trial constitutes a waiver of his right to present
spanning more than 2 evidence and cross-examine the opponent’s witnesses is firmly supported by
years, the case was jurisprudence
finally set for trial.
Bautista, et al. filed an AP: In this case, petitioners were not denied of due process as they were accorded the
Urgent Motion for opportunity to be heard when the case was set for trial. Their failure to appear on the day of the
Postponement to cancel trial constitutes a waiver of his right to present evidence and cross-examine the opponent’s
the hearing on the witnesses.
ground that his lawyer
was in the US attending
an important matter.

Trial court denied the


postponement and
considered Bautista, et
al. to have waived the
presentation of their
evidence..
Rural Bank of Buhi v. D: In cases of closure and liquidation of banks, appointment of a receiver may be made
Court of Appeals by the Monetary Board without notice and hearing but its action is subject to judicial
inquiry to insure the protection of the banking institution.
- Petitioner bank became
insolvent Rationale: Due process does not necessarily require a prior hearing; a hearing or an
- The bank was thereafter
opportunity to be heard may be subsequent to the closure. One can just imagine the dire
placed under consequences of a prior hearing: bank runs would be the order of the day, resulting in panic and
receivership. hysteria. In the process, fortunes may be wiped out, and disillusionment will run the gamut of the
- The manager of the
entire banking community.
bank filed a petition
contending that due AP: In this case, there is no violation of the due process when the Monetary Board issued a
process was not resolution placing Buhi under receivership, but such act may be subject to judicial inquiry
observed by the thereafter, this is for the protection of the banking community and the interest of the public.
Monetary Board before Although in this case, it was remanded in the trial court, due to failure of the RTC to decide on
it was placed under the merits.
receivership.
Note: It has long been established and recognized that the closure and liquidation of a bank may
be considered as an exercise of police power.

Pollution Adjudication D: When an ex parte order is allowed, such is not violative of the requirement of
Board v. Court of procedural due process, especially when the right to be heard is allowed even after the
Appeals issuance of an order. What matters is that the individual or entity should be heard and
allowed to contest the correctness of the order.
Based on a series of
inspection conducted by Ex parte cease and desist orders are permitted by law and regulations in situations
the NPC and DENR, the like that here presented precisely because stopping the continuous discharge of pollutive and
Pollution Adjudication untreated effluents into the rivers and other inland waters of the Philippines cannot be made
Board issued an exparte to wait until protracted litigation over the ultimate correctness or propriety of such orders has
order directing Solar to run its full course, including multiple and sequential appeals such as those which Solar has
immediately cease and taken, which of course may take several years.
desist from utilizing its
wastewater pollution The relevant pollution control statute and implementing regulations were enacted and
source installations which promulgated in the exercise of that pervasive, sovereign power to protect the safety, health,
were discharging and general welfare and comfort of the public, as well as the protection of plant and animal life,
untreated wastewater commonly designated as the police power.
directly into a canal
leading to the adjacent It is a constitutional commonplace that the ordinary requirements of procedural due process
Tullahan-Tinejeros River. yield to the necessities of protecting vital public interests like those here involved,
through the exercise of police power.

Solar went to court on


certiorari contending that AP: SC order and writ of execution are valid without prejudice on the part of Solar to
the exparte order was contest the correctness of the basis of such order
violative of the
requirements of due Instead of filing with the RTC for an injunction, Solar should have asked the PAB to conduct a
process. public hearing subsequent to the PAB order to determine its correctness. However, the order here
sought to be annulled was based on several inspections by the PAB, and Solar as the new owner
sent a notice to the PAB regarding its acquisition of the plant. Solar knows that its Wastewater
Treatment Plant was non-operational. Thus, it violated the law by still discharging untreated
wastewater to the that fails to meet the standards set by the law to the Tullahan-Tinejeros River.

Lao Gi v. Court of D: Although a deportation proceeding does not partake of the nature of a criminal
Appeals action, the constitutional right of a person to due process shall not be denied; The rules
on criminal procedure in the Rules of Court are applicable to deportation proceedings.
- Manuel Chia was
charged with Hence,
falsification for alleging 1. the charge against an alien must specify the acts or omissions complained of which
in a deed of sale of real must be stated in ordinary and concise language to enable a person of common
property that he was a understanding to know on what ground he is intended to be deported and enable the CID to
Filipuno Citizen. pronounce a proper judgment.
- He was acquitted.
2. Before any charge should be filed in the CID a preliminary investigation must be conducted
- Later on, Petitioners
to determine if there is a sufficient cause to charge the respondent for deportation
were charged for 3. The issuance of warrants of arrest, arrests without warrant and service of warrants should
deportation before the be in accordance with the Rules of Court
Commission of 4. There shall be no private prosecutor assisting the special prosec of CID
Immigration and 5. a private party to have no right to intervene (his recourse is to ordinary courts)
Deportation (CID).
103
- An order of arrest was
issued in case they fail Note: The power to deport an alien is an act of the State. It is an act by or under the authority of
to register as aliens. the sovereign power. It is a police measure against undesirable aliens whose presence in the
- They contend that this country is found to be injurious to the public good and domestic tranquility of the people.
is premature as there
was no competent AP: CID is directed to continue hearing the deportation case.
determination yet that Where the very citizenship of the petitioners is in issue there should be a previous determination
their citizenship was by the CID that they are aliens before the petitioners may be directed and required to register as
procured by fraud. aliens.
People v. de la Piedra D: Due process requires that the terms of a penal statute must be sufficiently explicit to
inform those who are subject to it what conduct on their part will render them liable to
Through an entrapment its penalties.
planned by POEA, Carol
Figueroa aka Carol Llena A criminal statute that “fails to give a person of ordinary intelligence fair notice that his
and Carol dela Piedra was contemplated conduct is forbidden by the statute,” or is so indefinite that “it encourages
caught conducting illegal arbitrary and erratic arrests and convictions,” is void for vagueness. The constitutional vice in a
recruitment, to which vague or indefinite statute is the injustice to the accused in placing him on trial for an offense, the
upon charging in the trial nature of which he is given no fair warning.
court, she was convicted.
Apart from denial and Test of vagueness
alibi, she asked the court As a rule, a statute or act may be said to be vague when it lacks comprehensible standards
to consider that that that men “of common intelligence must necessarily guess at its meaning and differ as
Article 13 (b) of the Labor to its application.”
Code defining
"recruitment and Unconstitutional
placement" is void for It is repugnant to the Constitution in two respects:
vagueness and, thus, (1) it violates due process for failure to accord persons, especially the parties targeted by
violates the due process it, fair notice of the conduct to avoid; and
clause. (2) it leaves law enforcers unbridled discretion in carrying out its provisions and become an
arbitrary flexing of the Government muscle.

The act must be utterly vague on its face, that is to say, it cannot be clarified by either a
saving clause or by construction.

AP: The assailed provision not void for vagueness.


The definition of “recruitment and placement” were given clear definitions under Under Section
38-39 of RA 8042.
Section 13 (b), therefore, is not a perfectly vague act whose obscurity is evident on its face. If at
all, the proviso therein is merely couched in imprecise language that was salvaged by proper
construction. It is not void for vagueness.

Estrada v. D: A statute is not rendered uncertain and void merely because general terms are used
Sandiganbayan therein, or because of the employment of terms without defining them; much less do
we have to define every word we use.
Petitioner’s contention:
the Plunder Law is void A law is not vague as long as the law affords some comprehensible guide or rule that would
for vagueness, as it inform those who are subject to it what conduct would render them liable to its penalties, its
denies him the right to be validity will be sustained.
informed of the nature
and cause of the “void-for-vagueness” doctrine
accusation against him.
The “void-for-vagueness” doctrine states that a statute establishing a criminal offense must
define the offense with sufficient definiteness that persons of ordinary intelligence can understand
what conduct is prohibited by the statute. It can only be invoked against that specie of legislation
that is utterly vague on its face, i.e., that which cannot be clarified either by a saving clause or by
construction.

When not applicable:


The doctrine does not apply as against
- legislations that are merely couched in imprecise language but which nonetheless specify a
standard though defectively phrased; or
- to those that are apparently ambiguous yet fairly applicable to certain types of activities.
The first may be “saved” by proper construction, while no challenge may be mounted as
against the second whenever directed against such activities.
- With more reason, the doctrine cannot be invoked where the assailed statute is clear and free
from ambiguity, as in this case.

AP: In this case the definition of the assailed term of "combination" and "series" are of
general term which can be understood in its ordinary meaning in connection to the offenses
provided in the plunder law.

Further the law does not circumvent the proof “beyond reasonable doubt” by use of the term
pattern of evidence, it only means that prosecution needs to prove beyond reasonable doubt is
only a number of acts sufficient to form a combination or series which would constitute a pattern
and involving an amount of at least P50,000,000.00.

Hence the plunder law is constitutional, because the due process of understanding what the crime
is clear in the law and the proof of beyond reasonable is the quantum of evidence needed to
prove the crime.

Sec. 2 Right to be Secure against Unreasonable Search and Seizures


104
A. Checkpoint

Valmonte v. De Villa Checkpoints are not illegal per se. The Court did not legalize all checkpoints at all times
and under all circumstances

A.) Under exceptional circumstances: (GP)


1. where the survival of organized government is on the balance; or
2. where the lives and safety of the people are in grave peril, checkpoints may be allowed and
installed by the government.

 Implicit in this proposition is, that when the situation clears and such grave perils are
removed, checkpoints will have absolutely no reason to remain.

B.) As long as the vehicle is neither searched nor its occupants subjected to a body
search and the inspection of the vehicle is limited to a visual search, said routine checks cannot
be regarded as violative of an individual’s right against unreasonable search.

C.) What constitutes a reasonable or unreasonable search and seizure in any particular case is
purely a judicial question, determinable from consideration of the circumstances involved.

B. Requisites of a Valid Warrant

Alvarez v. CFI A.) Before the issuance of a search warrant there must be:
1. Probable Cause
2. Warrant must be based upon an Application supported by oath of the applicant and the
witness he may produce.

 Oath includes any form of attestation which signifies that he is bound by it.
 Oath must refer to the truth of the facts within the personal knowledge of the applicant
or his witnesses.

B.) The True Test of sufficiency of an affidavit to warrant issuance of a search warrant is whether
it has been drawn in such a manner that perjury could be charged and affiant be held liable.

People v. Veloso GR: Warrants must have a NAME AND DESCRIPTION of the accused.
 Warrants for apprehension of unidentified persons are void.

EXC: if it contains the best descriptio personae possible that will enable the officer to identify
the accused.
 Must be sufficient to indicate clearly the proper person.
 Should state: (APORO)
1. Personal Appearance;
2. Peculiarities
3. Occupation;
4. Place of Residence;
5. Other circumstances by which he may be identified.

Burgos v. Chief of Staff A.) In the determination of whether a Search Warrant describes the premises to be
searched with sufficient particularity, it has been held that the executing officer’s prior
knowledge as to the place intended in the warrant is relevant.

B.) Rules of Court does not require that the property to be seized should be owned by
the person against whom the search warrant is directed.
 Ownership is of no consequence, and it is sufficient that the person against whom the
warrant is directed has control over the property to be seized.
C.) “General Warrants” are proscribed and unconstitutional.
 Application and supporting affidavits must contain specification, stating with particularity
the alleged subversive materials.
 Mere generalization will not suffice.
 Affidavits merely stated the basis is the “evidence gather by their unit” is
considered general warrant.

People v. CA LB: What is material in determining the validity of a search is the place stated in the
warrant itself, not what the applicant had in their thoughts, or had represented in the proofs
they submitted to the court issuing the warrant.

Ratio: Constitution removed from the officers the discretion as to search process to avoid abuses.

AP: Here, the police despite having personal knowledge of the store and apartments, failed to
make the Judge to understand the need to pinpoint Apartment No. 1 in the warrant.

Even after having received the warrant, they did not ask the judge to correct said description.

Corro v. Lising A.) A search warrant:


1. May only be issued upon probable cause, supported by fact and circumstances;
105
2. Particularly describes the place to be searched and thing to be seized.

B.) PURPOSE: The evident purpose of the requirement is to leave the officers of the law with
no discretion regarding what articles they should seize, to the end that unreasonable
searches and seizures may be made and abuses may no be committed.
 It is also aimed at preventing violation of security of persons and property, and unlawful
invasion of the sanctity of home, and giving remedy against such usurpation.

Soliven v. Makasiar A. It is sufficient that the judge “personally determine” the existence of probable cause.

B. It is NOT necessary that he should personally examine the complainant and his
witnesses.

Lim v. Judge Felix A.) If the judge relied solely on the certification of the Prosecutor (since all the records of
the preliminary investigation were still in Masbate), then he cannot be said to have
PERSONALLY DETERMINED the existence of probable cause, and, therefore, the warrant of
arrest issued by the judge is null and void.
 There was no basis for him to make his own personal determination of the existence of a
probable cause for the issuance of a warrant of arrest.

The Judge before issuing his own Warrant of Arrest should have gone over the records of the
preliminary examination conducted.

B.) The issuance of warrant of arrest is not a ministerial function of the judge.
 While he could rely on the findings of the fiscal, he is not bound thereby.

Laud v. People A.) The SC upheld the validity of the search warrant issued by Judge Peralta, Vice Executive
Judge of RTC-Manila, upon application of the PNP.
 Search Warrant for the three caves in Laud Compound in Davao City where the remains
of the victims of the Davao Death Squad.

B.) The ruling was based on Sec. 12, Chapter V, of A.M no. 03-8-02-SC, which authorized
the RTCs of Manila and Quezon City to issue search warrants in special criminal cases
when the following requirements are complied with: (HPE)
1. Case involves HEINOUR CRIMES;
2. Application is made by PNP;
3. Executive Judge, or when he is on official leave or not physically present, the Vice
Executive Judge, issues warrant if justified.

C.) Such warrant may be served in places outside the territorial jurisdiction of the RTCs of Manila
and Quezon City.

People v. Castillo A.) A motion to quash a search warrant may be based on grounds extrinsic of the search
warrant, such as
1. the place searched or the property seized are not those specified or described in the
search warrant; and
2. (2) there is no probable cause for the issuance of the search warrant.

B.) A search warrant may be issued by any court pursuant to Section 2, Rule 126 of the
Rules of Court and the resultant case may be filed in another court that has jurisdiction
over the offense committed.
 What controls here is that a search warrant is merely a process, generally issued by
a court in the exercise of its ancillary jurisdiction, and not a criminal action to be
entertained by a court pursuant to its original jurisdiction.

C.) In certain cases when no criminal action has yet been filed, any court may issue a search
warrant even though it has no jurisdiction over the offense allegedly committed , provided that all
the requirements for the issuance of such warrant are present.

C. Warrantless Searches and Seizures

Nolasco v. Pano Arrested for rebellion outside of her dwelling place, but authorities searched the house without a
search warrant, claiming the search was a valid exercise of warrantless search.

The better and established rule is a strict application of the exception provided in Rule 126, sec.
12 and that is to absolutely limit a warrantless search of a person who is lawfully
arrested to his or her person at the time of, and incident to his or her arrest and to
“dangerous weapons or anything which may be used as proof of the commission of the
offense.”
 Such warrantless search obviously cannot be made in a place other than the
place of arrest

The seizure of documents and personal effects with such a void warrant could not be justified “as
an incident of an arrest” outside petitioner’s dwelling and the Constitution bars their
admissibility as evidence and ordains their return to petitioners.

People v. CFI A.) Searches and seizures without a warrant may be made upon probable cause that an
automobile or other vehicle contains that which by law is subject to seizure.
106
 Persons exercising authority under the customs law may effect a search and seizure
without a warrant made upon probable cause.

B.) The requirement that a warrant must be issued judicially borders on the impossible
considering that the time, place, things, persons to be searched must be described particularly to
the satisfaction of the judge.
 In the case of smuggling with the use of a moving vehicle, such particularity of
description cannot be accomplished since contraband may be easily transported.

People v. Lo Ho Wing A warrantless search of a moving vehicle is justified on the ground that "it is not practicable
to secure a warrant because the vehicle can be quickly moved out of the locality or jurisdiction in
which the warrant must be sought."

People v. Evaristo LB: Objects falling in the PLAIN VIEW OF AN OFFICER who has the right to be in the position to
have that view, are subject to seizure and may be introduced in evidence. (Exception)

AP: The records in this case show that Sgt. Romerosa was granted permission by the appellant
Evaristo to enter his house. The officer’s purpose was to apprehend Rosillo whom he saw had
sought refuge therein.
 Therefore, it is clear that the search for firearms was not Romerosa’s purpose in entering
the house, thereby rendering his discovery of the subject firearms as inadvertent and
even accidental.

Malacat v. CA LB: A “stop-and-frisk” serves a two-fold interest:

1. the general interest of effective crime prevention and detection, which underlies the
recognition that a police officer may, under appropriate circumstances and in an appropriate
manner, approach a person for purposes of investigating possible criminal behavior even
without probable cause; and

2. the more pressing interest of safety and self-preservation which permit the police
officer to take steps to assure himself that the person with whom he deals is not
armed with a deadly weapon that could unexpectedly and fatally be used against the police
officer.

AP: Here, there are at least three (3) reasons why the “stop-and-frisk” was invalid:

First, we harbor grave doubts as to Yu’s claim that petitioner was a member of the group
which attempted to bomb Plaza Miranda two days earlier. This claim is neither supported by
any police report or record nor corroborated by any other police officer.

Second, there was nothing in petitioner’s behavior or conduct which could have
reasonably elicited even mere suspicion other than that his eyes were “moving very fast”—
an observation which leaves us incredulous since Yu and his teammates were nowhere near
petitioner and it was already 6:30 p.m., thus presumably dusk.
 Petitioner and his companions were merely standing at the corner and were not creating
any commotion or trouble, as Yu explicitly declared on cross-examination.

Third, there was at all no ground, probable or otherwise, to believe that petitioner was
armed with a deadly weapon.
 None was visible to Yu, for as he admitted, the alleged grenade was “discovered” “inside
the front waistline” of petitioner, and from all indications as to the distance between Yu
and petitioner, any telltale bulge, assuming that petitioner was indeed hiding a grenade,
could not have been visible to Yu.

People v. De Garcia LB: Warrantless arrests are valid:


1. When, in his presence, the person to be arrested has committed, is actually committing,
or is attempting to commit an offense;
2. When an offense has in fact just been committed, and he has personal knowledge of
facts indicating that the person to be arrested has committed it; and
3. When the person to be arrested is a prisoner who has escaped.

AP: At the height of the coup d’etat staged in December 1989, A warrantless search may be
conducted when the urgency and exigency of the moment calls for it and there exists a
reasonable ground to believe that a crime was committed.

People v. Johnson A.) The SC upheld the validity of searches conducted on passengers attempting to board an
aircraft whose carry-on baggage, as well as checked- in luggage, are subjected to x-ray scans,
and passengers themselves are made to pass through metal detectors.

B.) These searches are REASONABLE given the:


1. Minimal Intrusiveness,
2. the Gravity of the Safety interest, and the
3. Reduced Privacy Expectations associated with airline travel.

C.) Travelers are often notified through airport public address systems, signs and notices in their
airline tickets that they are subject to search and if any prohibited materials or substances are
found such would be subject to seizure.
 These announcements place passengers on notice that ordinary constitutional
protections against warrantless searches and seizures do not apply to routine airport
107
procedure.

AP: 3 plastic packs of methamphetamine from her girdle caught right before she boarded her
flight back to US. A valid warrantless arrest for being caught in flagrante delicto.

David v. Arroyo Rules of Court requires Search warrant:

1. Be issued upon PROBABLE CAUSE in CONNECTION WITH ONE SPECIFIC OFFENSE to


be personally determined by the judge.

2. Search of house, room, or any other premise be made:


 In PRESENCE OF the lawful occupant or any member of his family or in the absence of
the latter, in the presence of 2 witnesses of sufficient age and discretion residing in the
same locality.

3. Direct that it be served in DAYTIME.


 Unless the property is on the person or in the place ordered to be searched, in which
case it may by day or night.

People v. Nuevas A.) Search of Evidence in “PLAINT VIEW”; Elements


1. Prior valid intrusion based on the valid warrantless arrest in which the police are legally
present in the pursuit of official duties.
2. Evidence was discovered by the police who have the right to be there.
3. Evidence must be immediately apparent;
4. “Plain view” justified mere seizure of evidence without further search.

B.) if the package is such that an experienced observer could infer from its appearance
that it contains prohibited articles, then the article is deemed in plain view.

C.) An object is in plain view if it is plainly exposed to sight.


 Where the object seized was inside a closed package, the object itself is not in
plain view and therefore cannot be seized without a warrant.

 However, if the package proclaims its contents, whether by its distinctive configuration,
its transparency, or if its contents are obvious to an observer, then the contents are in
plain view and may be seized.

 In other words, if the package is such that an experienced observer could infer from its
appearance that it contains the prohibited article, then the article is deemed in plain
view. It must be immediately apparent to the police that the items that they observe
may be evidence of a crime, contraband or otherwise subject to seizure.

AP: In this case, the Supreme Court held that the marijuana leaves were not in the plain view of
the police officers, as they were wrapped in newspaper inside the plastic bag.

In this case, the Supreme Court held that the search was not done incidentally to a lawful arrest,
because the police officers did not have personal knowledge that defendants had committed, were
actually committing, or were attempting to commit an offense.

Oncoma v. People The Jay Walker, after being told to walk through allowed areas, was searched and found to
have .03 grams of shabu.

A.) The indispensability of the intent to arrest an accused in a warrantless search


incident to a lawful arrest was emphasized in Luz vs. People of the Philippines.

 The Court held that the shabu confiscated from the accused in that case was
inadmissible as evidence when the police officer who flagged him for traffic
violation had no intent to arrest him.

 According to the Court, due to the lack of intent to arrest, the subsequent search
was unlawful.

 This is notwithstanding the fact that the accused, being caught in flagrante delicto for
violating an ordinance, could have been therefore lawfully stopped or arrested by
the apprehending officers.

B.) The waiver of an illegal warrantless arrest does not also mean a waiver of the
inadmissibility of evidence seized during an illegal warrantless arrest.

Pollo v. David This case involves a search of office computer assigned to a government employee who was
charged administratively and eventually dismissed from the service. The employee’s personal files
stored in the computer were used by the government employer as evidence of misconduct.

A.) The right to privacy has been accorded recognition in this jurisdiction as a facet of the right
protected by the guarantee against unreasonable search and seizure under Section 2, Article III
of the 1987 Constitution.

AP: The CSC in this case had implemented a policy that put its employees on notice that
108
they have no expectation of privacy in anything they create, store, send or receive on
the office computers, and that the CSC may monitor the use of the computer resources
using both automated and human means. This implies that on-the-spot inspections may be
done to ensure that the computer resources were used only for such legitimate business
purposes.

B.) A search by a government employer of an employee’s office is justified at inception


when there are reasonable grounds for suspecting that it will turn up evidence that the
employee is guilty of work-related misconduct.

 Government employee had no legitimate expectation of privacy as to the use


and contents of his office computer, and therefore evidence found during
warrantless search of the computer was admissible in prosecution for child
pornography.

 Also, the computer from which the personal files were retrieved is a
government issued computer hence government property, the use of which the CSC
has absolute right to regulate and monitor.

E. Warrantless Arrests

In re: Umil v. Ramos Par. 1, Rule 113, Sec. 5

A.) The SC held that rebellion is a continuing offense. Accordingly, a rebel may be arrested at any
time, with or without warrant, as he is deemed to be in the act of committed the offense at any
time of day or night.

MAIN: The grounds of suspicion are reasonable when, in the absence of actual belief of the
arresting officers, the suspicion that the person to be arrested is probably guilty of committing
the offense, is based on actual facts, i.e., supported by circumstances sufficiently strong in
themselves to create the probable cause of guilt of the person to be arrested.

MR: A reasonable suspicion therefore must be founded on probable cause, coupled with good
faith on the part of the peace officers making the arrest.

B.) Writ of habeas corpus is no longer available after an information is filed against the person
detained and a warrant of arrest or and order of commitment, is issued by the court where said
information has been filed.

People v. Aminudin A.) Warrantless arrest allowed under Rule 113 of the Rules of Court are not justified unless the
accused was caught in flagrante or a crime was about to be committed or had just been
committed. The evidence of probable caused should be determined by a judge and not by law-
enforcement agents.

B.) It is not disputed, and in fact it is admitted by the PC officers who testified for the
prosecution, that they had no warrant when they arrested Aminnudin and seized the bag
he was carrying.
 Their only justification was the tip they had earlier received from a reliable and regular
informer who reported to them that Aminnudin was arriving in Iloilo by boat with
marijuana.

 When he was arrested, he was merely descending the gangplank and there was no
indication that called of his arrest.

C.) The present case presented no urgency.


 From the conflicting declarations of the PC witnesses, it is clear that they had at least
two days within which they could have obtained a warrant to arrest and search
Aminnudin who was coming to Iloilo on the M/V Wilcon 9.

 His name was known. The vehicle was identified. The date of its arrival was
certain. And from the information they had received, they could have persuaded
a judge that there was probable cause, indeed, to justify the issuance of a
warrant. Yet they did nothing.

 No effort was made to comply with the law. The Bill of Rights was ignored altogether
because the PC lieutenant who was the head of the arresting team, had determined on
his own authority that a "search warrant was not necessary."

Harvey v. Defensor A.) The SC upheld the validity of the arrest of pedophiles on order of Immigration Commissioner
Santiago Santiago because there was probable cause, occasioned by months of surveillance made by CID
agents on the suspected pedophiles.

B.) According to the Court, the requirement that probable cause is to be determined only by a
judge does not extend to deportation cases which are not criminal but purely administrative
in nature.

C.) That petitioners were not "caught in the act" does not make their arrest illegal. Petitioners
109
were found with young boys in their respective rooms, the ones with John Sherman being naked.
Under those circumstances the CID agents had reasonable grounds to believe that petitioners had
committed "pedophilia" defined as "psychosexual perversion involving children".

People v. Mengote A. The requirements under Rule 113 to justify a warrantless arrest have not been
established in the case at bar. At the time of the arrest in question, the accused-appellant
was merely “looking from side to side” and “holding his abdomen,” according to the arresting
officers themselves. There was apparently no offense that had just been committed or
was being actually committed or at least being attempted by Mengote in their
presence.

B. In this case, the defendant was arrested after the police received a tip that there were
“suspicious-looking” people in the vicinity who were about to commit a robbery.

C. The defendant was arrested at 11:30 in the morning and in a crowded street shortly after
alighting from a passenger jeep with his companion. He was not skulking in the shadows but
walking in the clear light of day. There was nothing clandestine about his being on that
street at that busy hour in the blaze of the noonday sun.

Posadas v. A.) Personal knowledge on the part of the arresting officers means knowledge of facts in arrests
Ombudsman without a warrant under Section 5 (b) of Rule 113 must be based upon “probable cause” which
means an “actual belief or reasonable grounds of suspicion.”

B.) When the arrest does not fall under Rule 113, the NBI agents cannot effect the arrest
themselves, because only the courts could decide the question of probable cause.

AP: Here, the arresting officers did not witness the crime being committed. Neither are the
students fugitives from justice. Par.b does not also apply since the respondents were not
committing a crime or anything that would create suspicion. The officers do not have personal
knowledge of facts.

People v. Laguio A.) For a warrantless arrest of an accused caught in flagrante delicto under par. A of sec. 5 of
Rule 113 to be valid, two requisites must concur:

1. Person to be arrested must EXECUTE AN OVERT ACT indicating that he has just committed,
is actually committing, or is attempting to commit a crime;

2. OVERT ACT IS DONE IN THE PRESENCE or within the view of the arresting officer.

B.) It is settled that “reliable information” alone, absent any overt act indicative of a
felonious enterprise in the presence and within the view of the arresting officers, is not
sufficient to constitute probable cause that would justify an in flagrante delicto arrest.

Antiquera v. People LB: Section 5(a), Rule 113 of the Rules of Criminal Procedure provides that a "peace officer or a
private person may, without a warrant, arrest a person when, in his presence, the person to be
arrested has committed, is actually committing, or is attempting to commit an offense."
 This is an arrest in flagrante delicto .
 The overt act constituting the crime is done in the presence or within the view of the
arresting officer.

AP: The officers in this case were conducting a police visibility patrol when they saw 2
unidentified men rush out of a house and immediately board a jeep.

Suspecting that a crime had been committed, the police officers approached the house from
where they came and were faced with a partially opened door.

During cross-examination, they admitted that they pushed the door in order to get a
better view of the inside. They saw the accused holding a bong and a lighter and the other
accused holding aluminium foil and an improvised burner. The police entered the house,
introduced themselves and arrested the accused.

No crime was plainly exposed to the view of the arresting officer because the officers
admitted that they pushed the door in order to look inside the house.

Sec. 3 Right to Privacy

A. Exclusionary Rule

People v. Marti A.) The constitutional proscription against unlawful searches and seizures applies as restraint
directed only against the government. Thus it could only be invoked against the State.
 The protection against unreasonable searches and seizures cannot be extended to acts
committed by private individuals.

B.) Unlawful searches and seizures renders the collected articles inadmissible as evidence.

110
Sesbreno v. CA The protection against unreasonable searches and seizures cannot be extended to acts committed
by private individuals so as to bring it within the ambit of alleged unlawful intrusion by the
government.

B. Waiver of Rights
Sps. Veroy v. Layague A.) While the offense of illegal possession of fire arms is malum prohibitum, it does not follow
that the subject firearm is illegal per se.
 Thus, inasmuch as the consent to the search was, in scope, limited to search for
NPA rebels, the confiscation of the firearms was held invalid.

B.) The articles seized, having been confiscated illegally, are protected by the exclusionary
principle and cannot be used as evidence against the petitioners in the criminal action against
them for illegal possession of firearms.

C.) Where the permission to enter the residence is given to search for rebels, it is illegal to search
the rooms therein and seize firearms without a search warrant.

C. Anti-Wire Tapping Act

Ramirez v. CA The SC held that R.A. 4200 clearly and unequivocally makes it illegal for any person, not
authorized by ALL the parties to any private communication, to secretly record such
communications by means of a tape recorder.

 The law does not make any distinction.


 It applies against one of the parties taping the private conversation.

Navarro v. CA The law prohibits overhearing, intercepting, or recording of private communications.

Where the exchange between two persons is not private, its tape recording is not prohibited.

Salcedo-Ortanez v. CA Absent a clear showing that both parties to the telephone conversations allowed the recording of
the same, the inadmissibility of the subject tapes is mandatory under Rep. Act No. 4200.

 Section 4. Any communication or spoken word, or the existence, contents, substance,


purport, or meaning of the same or any part thereof, or any information therein
contained, obtained or secured by any person in violation of the preceding sections of
this Act shall not be admissible in evidence in any judicial, quasi-judicial, legislative or
administrative hearing or investigation.”

D. Privacy of Bank Accounts

Marquez v. Desierto Before an in camera inspection may be allowed:


1. There must be a PENDING CASE;
2. ACCOUNT MUST BE CLEARLY IDENTIFIED, the inspection limited to the subject matter
of the pending case before the court of competent jurisdiction.
3. Bank personnel and the account holder MUST BE NOTIFIED to be PRESENT during the
inspection, andl
4. Inspection may cover only ACCOUNT IDENTIFIED IN THE PENDING CASE.

AP: In the case at bar, there is yet no pending litigation before any court of competent authority.
What is existing is an investigation by the Office of the Ombudsman. In short, what the Office of
the Ombudsman would wish to do is to fish for additional evidence to formally charge Amado
Lagdameo, et al., with the Sandiganbayan. Clearly, there was no pending case in court which
would warrant the opening of the bank account for inspection.

Phil v. Eugenio Section 11 of the AMLA does not generally authorize the issuance ex parte of the bank
inquiry order would be that such orders cannot be issued unless notice is given to the owners of
the account, allowing them the opportunity to contest the issuance of the order.

The source of the right to privacy governing bank accounts in the Philippines is statutory. (Bank
Secrecy Act)

 The framers of the 1987 Constitution recognized that bank accounts are not covered by
either the right to information under Section 7, Article III or under the requirement of
full public disclosure under Section 28, Article II.

 A bank inquiry order is not a search warrant or warrant of arrest as it contemplates a


direct object but not the seizure of persons or property.

E. Privacy of Communication

Roxas v. Zuzuaregui Court En Banc ordered Atty. Roxas to explain in writing why he should not be held in contempt of
court and subjected to disciplinary action when he, in a letter addressed to Associate Justice
Nazario with copies thereof furnished the Chief Justice and all the other Associate Justices, called
111
the Supreme Court a “dispenser of injustice.”

To prevent liability from attaching on account of his letter, he invokes his rights to free
speech and privacy of communication.

 Atty. Roxas likewise cannot hide under the mantle of the right to privacy.

 It must be disclosed that prior to his letter addressed to Justice Nazario, Atty. Roxas first
wrote then Chief Justice Panganiban asking for an investigation as to how the assailed
decision was rendered and to sanction the perpetrators.

 The accusations contained therein are similar to those in his letter to Justice Nazario.

 The fact that his letters were merely addressed to the Justices of this Court and were not
disseminated to the media is of no moment.

DOCTRINE: Letters addressed to individual Justices, in connection with the


performance of their judicial functions, become part of the judicial record and are a
matter of concern for the entire court.

 As can be gathered from the records, the letter to then Chief Justice Panganiban was
merely noted and no show-cause order was issued in the hope that Atty. Roxas would
stop his assault on the Court.

 However, since Atty. Roxas persisted in attacking the Court via his second letter, it
behooved the Court to order him to explain why he should not be held in contempt of
court and subjected to disciplinary action.

F. Writ of Habeas Data

Gamboa v. Chan A.) The writ of habeas data is an independent and summary remedy designed to protect the
image, privacy, honor, information, and freedom of information of an individual , and to
provide a forum to enforce one’s right to the truth and to informational privacy.

B.) It seeks to protect a person’s right to control information regarding oneself,


particularly in instances in which such information is being collected through unlawful
means in order to achieve unlawful ends.

C.) It must be emphasized that in order for the privilege of the writ to be granted, there
must exist a nexus between the right to privacy on the one hand, and the right to life,
liberty or security on the other.

AP: In this case, respondents admitted the existence of the Report, but emphasized its
confidential nature. That it was leaked to third parties and the media was regrettable, even
warranting reproach. But it must be stressed that Gamboa failed to establish that
respondents were responsible for this unintended disclosure. In any event, there are
other reliefs available to her to address the purported damage to her reputation,
making a resort to the extraordinary remedy of the writ of habeas data unnecessary
and improper.

It is clear from the foregoing discussion that the state interest of dismantling PAGs far outweighs
the alleged intrusion on the private life of Gamboa, especially when the collection and forwarding
by the PNP of information against her was pursuant to a lawful mandate. Therefore, the
privilege of the writ of habeas data must be denied.

Vivares v. St. Theresa’s A.) The writ of Habeas date is an independent and summary remedy available to any person
College whose right to privacy in life, liberty or security is violated or threatened by an unlawful act or
omission of a:
1. Public official or employee; or
2. Private individual; or
3. Entity

 Engaged in the gathering, collecting or storing of data or information regarding the


person, family, home and correspondence of the aggrieved party.

B.) Such remedy is not limited to cases involving extra-judicial killings or enforced
disappearances.

C.) To be entitled to the writ, it must be proved:


1. Existence of a right to informational privacy;
2. A showing, by substantial evidence, of an actual or threatened violation of the right
to privacy in life, libery or security of the victim.

D. Three standards of the right to privacy:


1. Locational or situational privacy;
2. Information privacy;
112
3. Decisional privacy.

E.) There can only be a violation of privacy if there is a reasonable expectation of


privacy.

AP: children involve did not enjoy a reasonable expectation of privacy over digital photos posted
on their FB since they had not utilized such privacy tools.

Lee v. Illagan A.) Habeas Data Rule was conceived as a response to address the extraordinary rise in the
number of killings and enforced disappearances.
 Judicial remedy enforcing the right to privacy.

B.) As the rules and jurisprudence evoke, alleging and eventually proving the nexus
between one’s privacy right to the cogent rights to life, liberty or security are crucial in
Habeas Data Cases, so much so that a failure on either account certainly renders a habeas data
petition dismissible.

Cam v. Almario LB: SEC. 3. Where to File. The petition may be filed with the Regional Trial Court where
petitioner or respondent resides, or that which has jurisdiction over the place where the data
or information are gathered, collected or stored, at the option of the petitioner.

The petition may also be filed with the Supreme Court or the Court of Appeals or the
Sandiganbayan when the action concerns public data files of government offices.
(Emphasis supplied) ·

AP: The subject Petition was filed directly with this Court. However, there is no allegation to
the effect that the subject Petition involves or concerns public data files of government
offices. Rather, the ·subject Petition principally involves alleged menacing surveillance activities
through unidentified and unregistered vehicles.

 Further, no special or compelling reason has been proffered by petitioner which would
otherwise justify the relaxation of the rule on venue in this case.

LB: Section 6 requires specific and verified allegations in support of petitioner's cause of action. It
further requires that petitioner "allege the courses of action he or she has undertaken to protect
the right to privacy or the right to life, liberty or security of the petitioner."

AP: It is reasonable to presume that the Rule requires the petitioner to allege the courses of
action he or she has undertaken in order to determine if there are sufficiently important or
compelling reasons to issue the writ considering that it is an extraordinary remedy.

LB: Writ of Habeas Data is a remedy enforceable against "a public official or employee, or
a private individual or entity engaged in the gathering, collecting or storing of data or
information regarding the person, family, home and correspondence of the aggrieved
party."

AP: As correctly pointed out by respondents, their functions as public officials do not extend
to gathering, collecting or storing of data or information regarding the person, family,
home and correspondence of certain individuals. As such, they are not the proper parties to
this case.

LB: Section 6(d) of the Rule on the Writ of Habeas Data requires the Petition to allege:
1. location of the files,
2. registers or database,
3. the government office, and
4. the person in charge, in possession or in control of the data or information, if known.

AP: In the instant case, petitioner utterly failed to comply with this requirement. Nowhere was it
mentioned in the Petition that herein respondents are the persons in charge, in possession, or in
control of the data or information.

Sec. 4 Freedom of Expression

What is the clear and If the words are used in such circumstances and are of such a nature as to create a clear and
present danger rule? present danger that they will bring about the substantive evils that the lawmaker has a right to
prevent – Eastern Broadcasting v. Dans, Jr. (DYRE radio station)
Whether the EVAT law No. Since the law granted the press a privilege, the law could take back the privilege anytime
discriminates against without offense to the Constitution. The reason is simple: by granting exemptions, the State does
the press because it not forever waive the exercise of its sovereign prerogative. – Tolentino v. Secretary of Finance
removed the (EVAT law)
exemption from VAT
still granted to others?
What is prior restraint? Prior restraint refers to official governmental restrictions on the press or other forms of
expression in advance of actual publication or dissemination. – Chavez v. Gonzales (Garci
Tapes)
Distinguish between
content-neutral and
CN CB
contest-based
113
restraint Definitio CN CB restraint or
n regulation is censorship, i.e.
merely , the
concerned restriction is
with the based on the
incidents of subject matter
the speech, of the
or one that utterance or
merely speech. The
controls the cast of the
time, place restriction
or manner, determines the
and under test by which
well-defined the challenged
standards; act is assayed
or with.

Test Substantial Strict scrutiny;


government clear and
al interest present danger

- Chavez v. Gonzales

A. Prior Restraint

Eastern Broadcasting A.) The Closure of DYRE in Cebu was a violation of Freedom of Expression and therefore
v. Dans unconstitutional.

 All forms of media, whether print or broadcast, are entitled to the broad protection of the
freedom of speech and expression clause.

B.) Freedom of television and radio broadcasting is somewhat lesser in scope than the freedom
accorded to newspaper and print media.

 SC explained that more than other forms of communications, radio broadcasting have the
most limited protection from free expression clause because:

1. broadcast media is uniquely pervasive, with airwaves confronting citizens both in public
and in the privacy of their homes;

2. broadcasting is accessible to children unlike other medium or media.

B.) The test for limitations on freedom of expression continues to be the CLEAR AND PRESENT
DANGER RULE.

 The clear and present danger test, however, does not lend itself to a simplistic and all-
embracing interpretation applicable to all utterances in all forums.

 The clear and present danger test, therefore, must take the particular circumstances of
broadcast media into account.

C.) The supervision of radio stations—whether by government or through self-regulation by the


industry itself calls for thoughtful, intelligent and sophisticated handling. The government has a
right to be protected against broadcasts which incite the listeners to violently overthrow it.

D.) Radio and television may not be used to organize a rebellion or to signal the start of
widespread uprising.

At the same time, the people have a right to be informed.

Radio and television would have little reason for existence if broadcasts are limited to bland,
obsequious, or pleasantly entertaining utterances.

Since they are the most convenient and popular means of disseminating varying views on public
issues, they also deserve special protection.
Tolentino v. Sec. of The withdrawal of the “exempt-status” of the press from VAT DOES NOT constitute
Finance discrimination against the press and a form of “prior restraint.”

Since the law granted the press a privilege, the law could take back the privilege anytime without
offense to the Constitution. The reason is simple: by granting exemptions, the State does not
forever waive the exercise of its sovereign prerogative.

Chavez v. Gonzalez A.) Acts of the Secretary of Justice and the National Telecommunications Commission in warning
television stations against playing the “Garci Tapes” under pain of revocation of their licenses
were content-based restrictions and as subjected to the “clear and present danger” test, it is
INVALID.

114
B.) It is not enough to determine whether the challenged act constitutes some form of restraint
on freedom of speech.

A distinction has to be made whether the restraint is


 content-neutral regulation, i.e., merely concerned with the incidents of the speech, or one
that merely controls the time, place or manner, and under well-defined standards; o
o only a substantial governmental interest is required for its validity.
o not designed to suppress any particular message
 content-based restraint or censorship, i.e., the restriction is based on the subject matter
of the utterance or speech. The cast of the restriction determines the test by which the
challenged act is assayed with.
o governmental action that restricts freedom of speech or of the press based
on content is given the strictest scrutiny in light of its inherent and invasive
impact.

o Only when the challenged act has overcome the clear and present danger rule will it
pass constitutional muster, with the government having the burden of overcoming
the presumed unconstitutionality.

o Unless the government can overthrow this presumption, the content-based


restraint will be struck down.

AP: They focused only on one subject – a specific content – the alleged tape conversations
between the president and comelec official.
 They did not merely provide regulations as to time, place or manner of the dissemination
of speech or expression.

 Respondent evidence falls short of satisfying the clear and present danger test.

Tordesillas v. Puno I: W/N the Advisory issued by the respondents is not content-neutral and thus constitute prior
restraint, censorship, and are content-restrictive, which resulted to a "chilling effect" in violation
of the freedom of the press. NO

LB: Jurisprudence has recognized four aspects of freedom of the press, to wit:
(1) freedom from prior restraint;
(2) freedom from punishment subsequent to publication;
(3) freedom of access to information; and
1. (4) freedom of circulation.

LB: Prior restraint refers to official governmental restrictions on the press or other forms of
expression in advance of actual publication or dissemination.

 Prior restraint is understood to be any form of governmental restriction on, or


interference to any form of expression in advance of actual expression, or exercise of the
right.

 There is prior restraint when the government totally prohibits and/or in some way,
restricts the expression of one's view or the manner of expressing oneself.

AP: A plain reading of the questioned advisory clearly shows that no media network or personnel
is prohibited nor restricted from reporting or writing on any subject matter or from being present
and covering newsworthy event.

B. Subsequent Punishment

People v. Perez The Supreme Court ruled Criticism, no matter how severe, on the Executive, the
Legislature, and the Judiciary, is within the range of liberty of speech, UNLESS the
intention and effect be seditious.
(Exception)

Gonzalez v. Comelec The case at bar raised the validity of the prohibition in R.A. No. 4880 of the too early nomination
of candidates and limitation found on the period of election campaign or partisan political
activity alleged by petitioners to offend against the rights of free speech, free press,
freedom of assembly and freedom of association, which include:
1. "Making speeches, announcements or commentaries or holding interviews for or
against the election of any party or candidate for public office:
2. Publishing or distributing campaign literature or materials;
3. Directly or indirectly soliciting votes and/or under-taking any campaign or propaganda for
or against any candidate or party.

A.) GR: previous restraint on the communication of views or subsequent liability


whether in libel suits, prosecution for sedition, or action for damages, or contempt
proceedings are prohibited

115
EXC: unless there is a clear and present danger of substantive evil that Congress has a right to
prevent. There are other social values that press for recognition.

 AP: In this case, the Supreme Court held that the substantive evil sought to be avoided
is the debasement of the electoral process.

 Freedom of expression is not an absolute.

B.) The Supreme Court applied the clear and present danger test.
 The term clear seems to point to a causal connection with the danger of the substantive
evil arising from the utterance questioned.
 Present refers to the time element.
 It used to be identified with imminent and immediate danger.
 The danger must not only be probable but very likely inevitable.

C. Freedom of Expression and the Electoral Process

Sanidad v. Comelec A.) Comelec Resolution No. 2167, which provides: During the plebiscite campaign period, on the
day before and on plebiscite day, no mass media columnist, commentator, announcer or
personality shall use his column or radio or television time to campaign for or against the
plebiscite issues, is INVALID.

Adiong v. Comelec A.) The prohibition on posting of decals and stickers on “mobile” places whether public or private
except in the authorized areas designated by the COMELEC becomes censorship is Unonstitutional
for infringement of freedom of expression.

 The prohibited acts were found to present no substantial danger to government interest
(it did not satisfy the clear and present danger test).

B.) The restriction was so broad as to include even the privately owned vehicles, equivalent to
deprivation of property without due process of law.

C.) the constitutional objective of giving the rich and poor candidates equal opportunity to inform
the electorate is not violated by the posting of decals and stickers on card and other vehicles.

ABS-CBN v. Comelec The SC declared that there is no law prohibiting the holding and reporting of exit polls.
 Exit poll – survey conducted for the purpose of determining the probable result of an
election.

The freedom of speech and of the press should all the more be upheld when what is sought to be
curtailed is the dissemination of information meant to add meaning to the right of suffrage.

SWS v. Comelec A.) Fair Elections Act provision that surveys affecting national candidates shall not be published
fifteen (15) days before an election and surveys affecting local candidates shall not be published
seven (7) days before an election is INVALID.

B.) (O’brien test) Government regulation is sufficiently justified:


1. if it is within the constitutional Power of the Government;
2. if it furthers an important or substantial governmental interest;
3. if the governmental interest is unrelated to the suppression of free expression; and
4. if the incidental restriction on alleged First Amendment freedoms [of speech,
expression and press] is no greater than is essential to the furtherance of that interest.

C.) Under this test, even if a law furthers an important or substantial governmental interest, it
should be invalidated if such governmental interest is “not unrelated to the suppression of free
expression.” Moreover, even if the purpose is unrelated to the suppression of free speech, the law
should nevertheless be invalidated if the restriction on freedom of expression is greater than is
necessary to achieve the governmental purpose in question.

AP: the subject law is invalid because


1. it imposes a prior restraint on the freedom of expression,
2. it is a direct and total suppression of a category of expression even though such suppression
is only for a limited period, and
3. the governmental interest sought to be promoted can be achieved by means other than the
suppression of freedom of expression.

David v. Arroyo A.) the SC held that:


1. Warrantless arrest of David and Llamas;
2. Dispersal of the rallies and warrantless arrest of the KMU and NAFLU member;
3. Imposition of standards on media or any prior restraint on the press
4. Warrantless search and seizure of the Daily Tribune offices

ARE NOT AUTHORIZED BY LAW AND THE CONSTITUTION.


B.) Assembly – is the right on the part of the citizens to meet peaceably for consultation in
respect to public affairs.
116
AP: here, David et.al. were arrested while they were exercising their right to peaceful assembly.
Not committing any crime, neither was there a showing a clear and present danger hat warranted
the limitation of that right.

Diocese of Bacolod v. A.) COMELEC does not have the authority to regulate the enjoyment of the preferred
Comelec right to freedom of expression exercised by a non-candidate in this case.

The tarpaulin was not paid for or posted “in return for consideration” by any candidate, political
party, or party-list group.

B.) Regulation of election paraphernalia will still be constitutionally valid if it reaches


into speech of persons who are not candidates or who do not speak as members of a
political party if they are not candidates, only if what is regulated is declarative speech
that, taken as a whole, has for its principal object the endorsement of a candidate only.

The regulation
(a) should be provided by law,
(b) reasonable,
(c) narrowly tailored to meet the objective of enhancing the opportunity of all candidates to
be heard and considering the primacy of the guarantee of free expression, and
(d) demonstrably the least restrictive means to achieve that object.

The regulation must only be with respect to the time, place, and manner of the rendition of the
message.

In no situation may the speech be prohibited or censored on the basis of its content. For this
purpose, it will not matter whether the speech is made with or on private property.

GMA v. Comelec LB: Aggregate-based airtime limits goes against the constitutional guaranty of freedom of
expression, of speech and of the press.

AP: The SC held that when the Comelec drastically reduced the airtime within which
national candidates and political parties in the 2013 elections may air political
advertisements on television and radio to an aggregate of 120 minutes and 180 minutes,
respectively, the Comelec unduly restricted and constrained the ability of the candidates
and political parties to reach out and communicate with the people.

 The guaranty to speak is useless without the ability to communicate and


disseminate what is said. The prohibition will only allow each candidate 81.81
seconds per day TV exposure, if the will consider the three mainstream TV channels, will
only have allocation of 27.27 seconds or airtime per network.

Reason – levelling the playing field – does not constitute a compelling state interest which would
justify such a substantial restriction on the freedom of candidates and political parties to
communicate their ideas, philosophies, platforms and programs of government.

D. Freedom of Expression and the Courts

In re: Emil Jurado A.) The law does not protect a journalist who deliberately prints lies or distorts the truth; or that
a newsman may escable liability who publishes derogatory or defamatory allegations against a
person or entity, but recognizes no obligation to establish the factual basis of such imputations
and refuses to submit proof thereof.

B.) Publication that tends to impede, embarrass or obstruct the court and constitutes a clear and
present danger to the administration of justice is not protected by the guarantee of press freedom
and is punishable by contempt.

C.) The Court said that Jurado is being called to account as a journalist who has misused and
abused press freedom to put the judiciary in clear and present danger of disrespute and of public
odium and opprobrium, to the detriment of the administration of justice.

People v. Godoy Reading of the subject article in its entirety will show that the same does not constitute
contempt, but at most, merely constitutes fair criticism.

Snide remarks or sarcastic innuendoes do not necessarily assume that level of


contumely which is actionable under Rule 71 of the Rules of Court.

Re: Radio/TV Coverage A.) The SC denied the request to televise and broadcast live trial of President Estrada before the
of Estrada Trial Sandiganbayan.
 A public trial is not synonymous with publicized trial; it only implies that the
court doors must be open to those who wish to come, sit in the available seats,
conduct themselves with decorum and observe the trial process.

B.) The SC said when constitutional guarantees of freedom of the press and right to public
information and the rights of the accused, jurisprudence tells that the right of the accused must
117
be preferred to win.

 With the possibility of losing not only liberty but also the very life of an accused, it
behoves all to make certain that an accused receives a verdict solely on the basis of a
just judgment, unbiased and unswayed by any kind of pressure.

C.) MR-RESOLUTION: Court ordered the Audio-Visual recording for documentary purposes, not
for live or real time broadcast.

E. Libel

Borjal v. CA A.) Fair commentaries on matters of public interest are privileged and constitute a valid defense
in an action for libel or slander. The doctrine of fair comment means that while in general every
discreditable imputation publicly made is deemed false, because every man is presumed innocent
until his guilt is judicially proved, and every false imputation is deemed malicious, nevertheless,
when the discreditable imputation is directed against a public person in his public capacity, it is
not necessarily actionable.

B.) In order to maintain a libel suit, it is essential that the victim be identifiable although it
is not necessary that he be named. It is also not sufficient that the offended party
recognized himself as the person attacked or defamed, but it must be shown that at
least a third person could identify him as the object of the libelous publication.
 Regrettably, these requisites have not been complied with in the case at bar.

Villanueva v. PDI A.) LIBEL is “a public and malicious imputation of a crime, or of a vice or defect, real or
imaginary, or any act, omission, condition, status, or circumstance tending to cause the
dishonor, discredit, or contempt of a natural person or juridical person, or to blacken
the memory of one who is dead.” Any of these imputations is defamatory and under the
general rule stated in Article 354 of the Revised Penal Code, every defamatory
imputation is PRESUMED TO BE MALICIOUS.

B.) The presumption of malice, however, does not exist in the following instances:
1.  A private communication made by any person to another in the performance of any legal,
moral, or social duty; and
2.  A fair and true report, made in good faith, without any comments or remarks, of any
judicial, legislative, or other official proceedings which are not of confidential nature, or of any
statement, report, or speech delivered in said proceedings, or of any other act performed by
public officers in the exercise of their functions.”

AP: In the instant case, there is no denying that the questioned articles dealt with MATTERS OF
PUBLIC INTEREST. These are matters about which the public has the right to be
informed, taking into account the very public character of the election itself. For this
reason, they attracted media mileage and drew public attention not only to the election itself but
to the candidates. As one of the candidates, petitioner consequently assumed the status of a
public figure. But even assuming a person would not qualify as a public figure, it would not
necessarily follow that he could not validly be the subject of a public comment (e.g. he is involved
in a public issue).

Disini v. Sec of Justice A.) The SC invalidates SEC. 4,c,3 of R.A. 10175 (Cybercrime Law) which penalizes the
transmission of spam emails.
 Ruling that section is unconstitutional, the Court said that:
1. No basis for holding that unsolicited emails reduce the efficiency of computers;
2. People have already been receiving junk mails before the arrival of computers;
3. To prohibit such would deny a person the right to read his emails, and the state cannot
rob him of his right without violating his freedom of expression.

B.) Sec. 4,c,4 is VALID which penalizes online libel with respect to the original author of the
post, it declared as VOID the same provision as applied to other who simply receive the post
and react to it.

C.) INVALIDATED Sec. 5 – punishes aiding and abetting the commission of cyber
crimes.
 The court said that the provision cannot stand scrutiny. Its vagueness raises
apprehensions on the part of internet users because of its obvious chilling effect on
freedom of expression, especially since the crime of aiding and abetting ensnares all
actors in the cyberspace.
 But it is not constitutionally infirm with respect to:
1. Illegal Access;
2. Illegal Interference;
3. Data Interference;
4. Misuse of Device;
5. Cyber-squatting;
6. Forgery;
7. Fraud;
8. Theft
 None of these offenses borders on the exercise of freedom of expressions. The actors
can be identified.

D.) Sec. 12 is VOID which grants law enforcement agencies the real time collection of traffic
data since it is too sweeping and lacks restraint.
 This power is limitless, enabling agencies in fishing expeditions threatening the
118
right of privacy of individuals.

Belen v. People (1) W/N petitioner is guilty of libel despite the absence of the element of publication. YES. Guilty

(2) W/N CA erred in ruling that the privileged communication rule is inapplicable

LB (1) Publication in libel means making the defamatory matter, after it has been written,
known to someone other than the person to whom it has been written.
 In the same vein, a defamatory letter contained in a closed envelope addressed to
another constitutes sufficient publication if the offender parted with its possession in
such a way that it can be read by person other than the offended party.

AP: In claiming that he did not intend to expose the Omnibus Motion to third persons, but only
complied with the law on how service and filing of pleadings should be done, petitioner conceded
that the defamatory statements in it were made known to someone other than the person to
whom it has been written.

LB (2) A communication is absolutely privileged when it is not actionable, even if the


author has acted in bad faith.

This class includes allegations or statements made by parties or their counsel in pleadings or
motions or during the hearing of judicial and administrative proceedings, as well as answers given
by the witness in reply to questions propounded to them in the course of said proceedings.
 The reason for the rule that pleadings in judicial proceedings are considered privileged is
not only because said pleadings have become part of public record open to the public to
scrutinize, but also to the undeniable fact said pleadings are presumed to contain
allegations and assertions lawful and legal in nature, appropriate to the disposition of
issues ventilated before the courts for proper administration of justice and, therefore, of
general public concern.
 Moreover, pleadings are presumed to contain allegations substantially true because they
can be supported by evidence in good faith, the contents of which would be under
scrutiny of courts and, therefore, subject to be purged of all improprieties and illegal
statements contained therein

LB (3) Relevancy Test


 The absolute privilege remains regardless of the defamatory tenor and the
presence of malice, if the same are relevant, pertinent or material to the cause
in and or subject of the inquiry.
 Sarcastic, pungent and harsh allegations in a pleading although tending to detract from
the dignity that should characterize proceedings in courts of justice, are absolutely
privileged, if relevant to the issues.
 If the pleader goes beyond the requirements of the statute, and alleges an irrelevant
matter which is libelous, he loses his privilege.
 The reason for this is that without the requirement of relevancy, pleadings could be
easily diverted from their original aim to succinctly inform the court of the issues in
litigation and pervaded into a vehicle for airing charges motivated by a personal rancor

AP: The statements in petitioner’s Omnibus Motion filed before the OCP of San Pablo City as a
remedy for the dismissal of his estafa complaint during preliminary investigation, fall short of the
test of relevancy.

An examination of the motion shows that the following defamatory words and phrases used, even
if liberally construed, are hardly material or pertinent to his cause, which is to seek a
reconsideration of the dismissal of his estafa complaint.

F. Obscenity and Indecency

Pita v. CA The determination of what is obscene is a judicial function. The public respondents should have
first obtained a judicial determination that the materials in question were pornographic and
obtained a search warrant in order to search and seize the materials in question.

Undoubtedly, "immoral" lore or literature comes within the ambit of free expression, although not
its protection. In free expression cases, this Court has consistently been on the side of the
exercise of the right, barring a "clear and present danger" that would warrant State interference
and action. But the burden to show this lies with the authorities.

"There must be objective and convincing, not subjective or conjectural, proof of the existence of
such clear and present danger."

The burden is on the State to demonstrate the existence of a danger, a danger that must not only
be: (1) clear but also, (2) present, to justify State action to stop the speech

AP: The Court was not convinced that the private respondents have shown the required proof to
justify a ban and to warrant confiscation of the literature for which mandatory injunction had
been sought below. First of all, they were not possessed of a lawful court order: (1) finding the
119
said materials to be pornography, and (2) authorizing them to carry out a search and seizure, by
way of a search warrant.

Section 8.

A. Government employees

SSS Employees v. Issue: Whether employees of SSS has the right to strike? NO.
Court of Appeals
LB: In recognizing the right of government employees to organize, the constitutional
GOV’T EMPLOYEES commissioners intended to limit the right to the formation of unions or associations only,
HAVE NO RIGHT TO without including the right to strike.
STRIKE
AP: SSS is one such GOCC with an original charter, its employees are part of the civil service and
are covered by the CSC's memorandum prohibiting strikes. This being the case, the strike staged
by the employees of the SSS was illegal.

LB: Therefore, government employees may, through their unions or associations, either
- petition the Congress for the betterment of the terms and conditions of employment which
are within the ambit of legislation or
- negotiate with the appropriate government agencies for the improvement of those which
are not fixed by law.

If there be any unresolved grievances,


- the dispute may be referred to the Public Sector Labor - Management Council for appropriate
action.
- But employees in the civil service may not resort to strikes, walk-outs and other
temporary work stoppages, like workers in the private sector, to pressure the Government
to accede to their demands.
United Pepsi-Cola Issue: Whether the law which prohibits managerial employees from forming, joining or assisting
Supervisory Union v. labor unions is unconstitutional? NO.
Laguesma
LB: The right guaranteed in Art. 3, Sec. 8 is subject to the condition that its exercise should
PROHIBITION ON be for purposes not contrary to law.
MANAGERS TO JOIN
OR FORM UNION - AP: In the case of Art. 245, there is a rational basis for prohibiting managerial employees
VALID from forming or joining labor organizations.

- The rationale for this inhibition has been stated to be, because if these managerial employees
would belong to or be affiliated with a Union, the latter might not be assured of their
loyalty to the Union in view of evident conflict of interests. The Union can also become
company-dominated with the presence of managerial employees in Union membership.

Section 9.

Elements of “taking

Republic v. Vda. De Issue: Whether the taking of the 3 properties in Pampanga under expropriation has taken place
Castellvi - 58 SCRA when Republic commenced to occupy the property as lessee thereof? NO.
336
LB: A number of circumstances must be present in the "taking" of property for purposes of
5 ELEMENTS OF eminent domain.
“TAKING” OF
PROPERTY FOR 1. The expropriator must enter a private property.
PURPOSES OF E.D. 2.The entrance into private property must be for more than a momentary period. The word
"momentary" when applied to possession or occupancy of (real) property should be construed
IF WITH CONTRACT OF to mean "a limited period" — not indefinite or permanent.
LEASE - NOT 3.The entry into the property should be under warrant or color of legal authority.
TANTAMOUNT TO
TAKING BEC. 4. The property must be devoted to a public use or otherwise informally appropriated or
TEMPORARY injuriously affected.
5. The utilization of the property for public use must be in such a way as to oust the owner and
deprive him of all beneficial enjoyment of the property.

AP: In this case, 2 essential elements were not present (2nd and 5th elements) namely that
the entrance and occupation by the condemner bus be for a permanent period and that in
devoting the property to public use the owner was ousted from the property and deprived of its
beneficial use.

1. By virtue of the lease agreement, the Republic, through the AFP, took possession of the
property of Castellvi

2. The entry on the property, under the lease, is temporary, and considered transitory.
The fact that the Republic, through the AFP, constructed some installations of a permanent

120
nature does not alter the fact that the entry into the land was transitory, or intended to last a
year, although renewable from year to year by consent of 'The owner of the land.’ By express
provision of the lease agreement, the Republic, as lessee, undertook to return the
premises in substantially the same condition as at the time the property was first
occupied by the AFP.

3. The entry was with warrant because the Republic entered the Castellvi property as lessee.

4. It may be conceded that the circumstance of the property being devoted to public use is
present because the property was used by the air force of the AFP.

5. Here, the entry of the Republic into the property and its utilization of the same for public use
did not oust Castellvi and deprive her of all beneficial enjoyment of the property.
Castellvi remained as owner, and was continuously recognized as owner by the
Republic, as shown by the renewal of the lease contract from year to year, and by the
provision in the lease contract whereby the Republic undertook to return the property to
Castellvi when the lease was terminated. Neither was Castellvi deprived of all the
beneficial enjoyment of the property, because the Republic was bound to pay, and had been
paying, Castellvi the agreed monthly rentals until the time when it filed the complaint for
eminent domain on June 26, 1959.

It is clear, therefore, that the "taking" of Catellvi's property for purposes of eminent domain
cannot be considered to have taken place in 1947 when the Republic commenced to occupy the
property as lessee thereof.

Mere notice of intention to expropriate cannot bind the landowner. All actions of expropriation
must be commenced in court and just compensation is to be determined as of the date of the
filing of the complaint.

Garcia v. Court of Issue: Whether there was a taking of petitioner’s Block 19 property by the government for the
Appeals - 102 SCRA purposes of eminent domain? NO.
597
LB: Where the entry was gained through permission, the government did not have the
IF ENTRY WITH intention to acquire ownership either by voluntary purchase or by the exercise of eminent
PERMISSION, THEN NO domain.
INTENTION TO OWN,
THUS THERE IS NO AP: As the private respondent's entry was gained through permission, it did not have the
TAKING OF THE intention to acquire ownership either by voluntary purchase or by the exercise of eminent
PROPERTY domain.

- And the fact remains that the private respondent never completed the negotiation as to
compensation.
- Not only this, private respondent went on to construct another line - the 69 KV Mexico-Tarlac
without defendants' permission nor a court authorization.
All these prove the private respondent's intention not to expropriate Block 19, as it did not seek
so in the action it instituted on 1969. Neither did it have the intention to do so in 1953. It is clear,
therefore, that the private respondent not only did not take possession with intent to expropriate
Block 19, but that it did not institute expropriation proceedings over the same.

LB: Where there is no taking of property for purposes of eminent domain nor
condemnation proceedings instituted, the basis for determination of just compensation
is the time when the trial court made its order of expropriation; Factors to be considered
in estimating market value of property for purposes of compensation, case at bar.

AP: Consequently, since the areas covered by Block 19 were never entered into or possessed for
purposes of eminent domain, nor did they become the subject of an action for eminent domain,
neither the date of entry nor the filing of the action by private respondent for expropriation of a
"right-of-way" easement on December 8, 1969 could be reckoned with as the basis for the
determination of just compensation.

It is significant that the expropriation of Block 19 came about only when the trial court declared
that inasmuch as the private respondent cannot acquire easement of right-of- way over Block 19,
much less own it through prescription, the only way for the private respondent to justify its
continued occupation of Block 19 is to expropriate the same.

City Government v. Issue: Whether the taking of respondent’s property through Sec. 9 of an Ordinance which set
Judge Ericta - 122 aside an area of 6% of Memorial Park for charity burial, is a valid and reasonable exercise
SCRA 759 of police power? NO.

QC ORDINANCE LB: An ordinance of Quezon City requiring memorial park operators to set aside at least
REQUIRING six percent (6%) of their cemetery for charity burial of deceased persons is not a valid
MEMORIAL PARKS TO exercise of police power, and one that constitute taking of property without just
ALLOT 6% OF THEIR compensation.
CEMETERY FOR
CHARITY BURIALS IS AP: There is no reasonable relation between the setting aside of at least six (6) percent
TAKING OF PROPERTY of the total area of all private cemeteries for charity burial grounds of deceased
WITHOUT JUST paupers and the promotion of health, morals, good order, safety, or the general welfare
COMPENSATION of the people. The ordinance is actually a taking without compensation of a certain area from a
private cemetery to benefit paupers who are charges of the municipal corporation. Instead of
building or maintaining a public cemetery for this purpose, the city passes the burden to private

121
cemeteries.

LB: Police power is usually exercised in the form of mere regulation or restriction in the use of
liberty or property for the promotion of the general welfare. It does not involve the taking or
confiscation of property with the exception of a few cases where there is a necessity to confiscate
private property in order to destroy it for the purpose of protecting the peace and order and of
promoting the general welfare.

AP: The ordinance is not a police regulation but outright confiscation, it deprives a person of his
private property without due process of law and compensation. No reasonable relation as to
promotion of health, morals, good order, safety, or the general welfare of the people

Other LB:

The expropriation without compensation of a portion of private cemeteries is not covered by Sec.
12(t) of RA 537, the Revised Charter of Quezon City which empowers the city council to prohibit
the burial of the dead within the center of population of the city and to provide for their burial in a
proper place subject to the provisions of general law regulating burial grounds and cemeteries.
When the LGC, BP 337 provides in Sec. 177 (q) that a Sangguniang panlungsod may "provide for
the burial of the dead in such place and in such manner as prescribed by law or ordinance" it
simply authorizes the city to provide its own city owned land or to buy or expropriate
private properties to construct public cemeteries. This has been the law and practise in
the past. It continues to the present.

Expropriation, however, requires payment of just compensation. The questioned ordinance is


different from laws and regulations requiring owners of subdivisions to set aside certain areas for
streets, parks, playgrounds, and other public facilities from the land they sell to buyers of
subdivision lots. The necessities of public safety, health, and convenience are very clear from said
requirements which are intended to insure the development of communities with salubrious and
wholesome environments. The beneficiaries of the regulation, in turn, are made to pay by the
subdivision developer when individual lots are sold to home-owners.

MERALCO v. Pineda - Issue: Whether the respondent court can dispense with the assistance of a Board of
206 SCRA 196 Commissioners in an expropriation proceeding and determine for itself the just compensation?
NO.
TRIAL BEFORE THE
COMMISSIONERS IS LB: There are 2 stages in every action of expropriation.
INDISPENSABLE
The first is concerned with the determination of the authority of the plaintiff to exercise
the power of eminent domain and the propriety of its exercise in the context of the facts
involved in the suit.

The second phase of the eminent domain action is concerned with the determination by the
Court of "the just compensation for the property sought to be taken."
- This is done by the Court with the assistance of not more than 3 commissioners .
- The order fixing the just compensation on the basis of the evidence before, and findings of, the
commissioners would be final, too. It would finally dispose of the second stage of the suit, and
leave nothing more to be done by the Court regarding the issue.
- Obviously, one or another of the parties may believe the order to be erroneous in its
appreciation of the evidence or findings of fact or otherwise. Obviously, too, such a dissatisfied
party may seek reversal of the order by taking an appeal therefrom.

TRIAL BEFORE THE COMMISSIONERS IS INDISPENSABLE

In an expropriation case where the principal issue is the determination of just compensation, a
trial before the Commissioners is indispensable to allow the parties to present evidence
on the issue of just compensation.

- the appointment of at least 3 competent persons as commissioners to ascertain just


compensation for the property sought to be taken is a mandatory requirement in expropriation
cases.

- While it is true that the findings of commissioners may be disregarded and the court may
substitute its own estimate of the value, the latter may only do so for valid reasons, i.e.,
where the Commissioners have applied illegal principles to the evidence submitted to them or
where they have disregarded a clear preponderance of evidence, or where the amount allowed
is either grossly inadequate or excessive.

- Thus, trial with the aid of the commissioners is a substantial right that may not be
done away with capriciously or for no reason at all. Moreover, in such instances, where
the report of the commissioners may be disregarded, the trial court may make its own estimate
of value from competent evidence that may be gathered from the record.

AP: The respondent judge's act of determining and ordering the payment of just compensation
without the assistance of a Board of Commissioners is a flagrant violation of petitioner's
constitutional right to due process and is a gross violation of the mandated rule
established by the Revised Rules of Court.

NPC v. Jocson - 206 Issue: Whether the respondent judge order fixing the provisional values of the properties for the
SCRA 196 purpose of issuing a writ of possession on the basis of the market value and the daily opportunity
profit petitioner may derive is valid? NO.

122
PD 42- IN AN ACTION LB: There are 2 stages in every action of expropriation.
OF EXPROPRIATION,
NO HEARING IS The first is concerned with the determination of the authority of the plaintiff to exercise the
REQUIRED, ONLY power of eminent domain and the propriety of its exercise in the context of the facts involved in
NOTICE TO THE the suit.
OWNER SOUGHT TO BE
CONDEMNED
The second phase of the eminent domain action is concerned with the determination by the
Court of the "just compensation for the property sought to be taken." This is done by the Court
with the assistance of not more than 3 commissioners.

However, upon the filing of the complaint or at any time thereafter, the petitioner has the right
to take or enter upon the possession of the property involved upon compliance with PD
42 which requires the petitioner, after due notice to the defendant, to deposit with the
PNB in its main office or any of its branches or agencies, "an amount equivalent to the
assessed value of the property for purposes of taxation." This assessed value is that
indicated in the tax declaration.

It will be noted that under the aforequoted section, the court has the discretion to determine the
provisional value which must be deposited by the plaintiff to enable it "to take or enter upon the
possession of the property." Notice to the parties is not indispensable. PD 42, however,
effectively removes the discretion of the court in determining the provisional value.
What is to be deposited is an amount equivalent to the assessed value for taxation
purpose. No hearing is required for that purpose. All that is needed is notice to the
owner of the property sought to be condemned.

AP: Clearly, therefore, respondent Judge either deliberately disregarded PD 42 or was totally
unaware of its existence and the cases applying the same.

B. Public use

NPC v. Jocson - 206 Issue: Whether the respondent judge order fixing the provisional values of the properties for the
SCRA 196 purpose of issuing a writ of possession on the basis of the market value and the daily opportunity
profit petitioner may derive is valid? NO.
PD 42- IN AN ACTION
OF EXPROPRIATION, LB: There are 2 stages in every action of expropriation.
NO HEARING IS
REQUIRED, ONLY The first is concerned with the determination of the authority of the plaintiff to exercise the
NOTICE TO THE power of eminent domain and the propriety of its exercise in the context of the facts involved in
OWNER SOUGHT TO BE the suit.
CONDEMNED
The second phase of the eminent domain action is concerned with the determination by the
Court of the "just compensation for the property sought to be taken." This is done by the Court
with the assistance of not more than 3 commissioners.

However, upon the filing of the complaint or at any time thereafter, the petitioner has the right
to take or enter upon the possession of the property involved upon compliance with PD
42 which requires the petitioner, after due notice to the defendant, to deposit with the
PNB in its main office or any of its branches or agencies, "an amount equivalent to the
assessed value of the property for purposes of taxation." This assessed value is that
indicated in the tax declaration.

It will be noted that under the aforequoted section, the court has the discretion to determine the
provisional value which must be deposited by the plaintiff to enable it "to take or enter upon the
possession of the property." Notice to the parties is not indispensable. PD 42, however,
effectively removes the discretion of the court in determining the provisional value.
What is to be deposited is an amount equivalent to the assessed value for taxation
purpose. No hearing is required for that purpose. All that is needed is notice to the
owner of the property sought to be condemned.

AP: Clearly, therefore, respondent Judge either deliberately disregarded PD 42 or was totally
unaware of its existence and the cases applying the same.

Sumulong v. Guerrero Issue: Whether the “socialized housing” under PD 1224 for the purpose of expropriation may be
- 154 SCRA 461 considered as public use? YES.

SOCIALIZED HOUSING LB: The taking to be valid must be for public use. Whatever may be beneficially
IS CONSIDERED AS employed for the general welfare satisfies the requirement of public use.
‘PUBLIC USE’
Socialized housing is defined as the construction of dwelling units for the middle and lower class
members of our society, including the construction of the supporting infrastructure and other
facilities. Housing is a basic human need. Shortage in housing is a matter of state concern since it
directly and significantly affects public health, safety, the environment and in sum, the general
welfare. The public character of housing measures does not change because units in housing
projects cannot be occupied by all but only by those who satisfy prescribed qualifications.

AP: In the light of the foregoing, this Court is satisfied that "socialized housing" fans within
the confines of "public use".

Here, the use to which it is proposed to put the subject parcels of land meets the requisites of
"public use". The lands in question are being expropriated by the NHA for the expansion of
123
Bagong Nayon Housing Project to provide housing facilities to low- salaried government
employees.

C. Just compensation

EPZA v. Dulay - 149 Issue: Whether PD No. 1533 which vested on the assessors and property owners themselves the
SCRA 305 power and duty to fix the amount of just compensation is valid? NO.

DETERMINATION OF LB: The determination of "just compensation" in eminent domain cases is a judicial
JUST COMPEN. IS A function. The executive department or the legislature may make the initial determinations but
JUDICIAL FUNCTION when a party claims a violation of the guarantee in the Bill of Rights that private property may
not be taken for public use without just compensation, no statute, decree, or executive order can
mandate that its own determination shall prevail over the court's findings. Much less can the
courts be precluded from looking into the "justness" of the decreed compensation.

AP: The provisions of the Decrees on just compensation is unconstitutional and void because
it constitutes impermissible encroachment on judicial prerogatives. It tends to render this
Court inutile in a matter which under the Constitution is reserved to it for final determination.

Thus, although in an expropriation proceeding the court technically would still have the power to
determine the just compensation for the property, following the applicable decrees, its task would
be relegated to simply stating the lower value of the property as declared either by the owner or
the assessor. As a necessary consequence, it would be useless for the court to appoint
commissioners under Rule 67 of the Rules of Court.

Moreover, the need to satisfy the due process clause in the taking of private property is
seemingly fulfilled since it cannot be said that a judicial proceeding was not had before the actual
taking. However, the strict application of the decrees during the proceedings would be nothing
short of a mere formality or charade as the court has only to choose between the valuation of the
owner and that of the assessor, and its choice is always limited to the lower of the two. The court
cannot exercise its discretion or independence in determining what is just or fair.

Municipality of Daet v. Issue: Whether the decision of the Court fixing the market value of the property subject of the
Court of Appeals - 93 expropriation at the time of rendition of judgment is valid? YES.
SCRA 503
LB: The value of just compensation should be fixed as of the time of the taking of the
VALUE OF J.C. IS possession of the property.
FIXED AT THE TIME OF
THE TAKING OF PROP AP: We hold that the decision of the CA fixing the market value of the property to be that
obtaining, at least, as of the date of the rendition of the judgment on December 2, 1969 as
prayed by private respondent, which the Court fixed at P200.00 per square meter is in conformity
with doctrinal rulings that the value should be fixed as of the time of the taking of the possession
of the property because firstly, at the time judgment was rendered on December 2, 1969 by the
trial court disregarding the valuation made by the commissioners and using the appraisal of
Engineer Aurelio B. Aquino in 1969 as the basis in determining the value of the land in 1962,
petitioner had not actually taken possession of the property sought to be expropriated and
secondly, we find the valuation determined by the CA to be just, fair and reasonable.

National Power Issue: When is just compensation determined? 1961, which is the time of taking.
Corporation v. Court of
Appeals - 129 SCRA LB: In determining the amount of compensation, the value at the time of taking or filing
665 of complaint, whichever is earlier.

AMOUNT OF J.C. = Thus, when plaintiff takes possession before the institution of the condemnation
VALUE AT THE TIME OF proceedings, the value should be fixed as of the time of the taking of said possession,
TAKING OR FILING, not of filing of the complaint, and that the latter should be the basis for the determination of
WHICHEVER IS the value, when the taking of the property involved coincides with or is subsequent to, the
EARLIER commencement of the proceedings.

AP: The time of taking here is 1961 when it constructed the access road on the property which
was then still cogonal. Even if complaint was only filed in 1963, the time of taking is 1961, which
is earlier, is thus controlling. It is the time of taking and not as potential building site that is the
determining factor, the subdivision was not in existence when NPC entered the land.

Republic of the Issue: Whether the filing of an ejectment suit against the Government for its failure to acquire
Philippines vs. Primo ownership of a privately owned property that it had long used as a school site and to pay just
Mendoza - G.R. No. compensation for it is valid? NO.
185091, 8 August
2010 LB: Where the owner agrees voluntarily to the taking of his property by the government
for public use, he thereby waives his right to the institution of a formal expropriation
IF W/O EXPRO. proceeding covering such property.
PROCEEDING AND
OWNER FILED AN Further, the failure for a long time of the owner to question the lack of expropriation
ACTION FOR RECOVERY proceedings covering a property that the government had taken constitutes a waiver of his
BEFORE TH right to gain back possession.
COMMENCEMENT OF THE
EXPRO. PROCEEDING, AP: The evidence on record shows that the Mendozas intended to cede the property to the
THE VALUE OF THE City Government of Lipa permanently. In fact, they allowed the city to declare the property in
PROP. AT THE TIME OF its name for tax purposes.
THE TAKING IS

124
CONTROLLING The Mendozas remedy is an action for the payment of just compensation, not ejectment.

LB: As to the time when just compensation should be fixed, it is settled that where
property was taken without the benefit of expropriation proceedings and its owner filed an action
for recovery of possession before the commencement of expropriation proceedings, it is the
value of the property at the time of taking that is controlling.

Republic of the Issue: Whether just compensation must be paid to respondents-intervenors for the subdivision
Philippines vs. Spouses road lots owned by them? YES.
Llamas - 25 January
2017 LB: A "positive act" must first be made by the owner-developer before the city or
municipality can acquire dominion over the subdivision roads. As there is no such thing as
A POSITIVE ACT MUST an automatic cession to government of subdivision road lots, an actual transfer must first be
FIRST BE MADE BY THE effected by the subdivision owner. Subdivision streets belonged to the owner until donated to the
OWNER BEFORE A LGU government or until expropriated upon payment of just compensation. Stated otherwise, "the
CAN ACQUIRE local government should first acquire them by donation, purchase, or expropriation, if
DOMINION OVER THE they are to be utilized as a public road.
SUBDIVISION ROADS,
OTHERWISE J.C. MUS An owner may not be forced to donate his or her property even if it has been delineated as road
BE PAID TO THE lots because that would partake of an illegal taking. He or she may even choose to retain said
OWNER properties.

AP: The DPWH makes no claim here that the road lots covered by TCT No. 179165 have actually
been donated to the government or that their transfer has otherwise been consummated by
respondents. It only theorizes that they have been automatically transferred. Respondents have
not made any positive act enabling the City Government of Parañaque to acquire
dominion over the disputed road lots. Therefore, they retain their private character
(albeit all parties acknowledge them to be subject to an easement of right of way). Accordingly,
just compensation must be paid to respondents as the government takes the road lots
in the course of a road widening project.

Land Bank vs. Rural Issue: Whether the CA erred in upholding the RTC’s valuation fixing the just compensation for
Bank of Hermosa - G.R. the subject land at P30.00 per sq.m? YES.
No. 181953, 25 July
2017 LB: Settled is the rule that when the agrarian reform process is still incomplete, such as in this
case where the just compensation due the landowner has yet to be settled, just compensation
IF AGRARIAN REFORM should be determined and the process be concluded under RA 6657, as amended.
PROCESS IS PENDING,
RA 6657 SHALL "For purposes of determining just compensation, the fair market value of an
GOVERN - (VALUE AT expropriated property is determined by its character and its price at the time of
THE TIME OF TAKING taking," or the time when the landowner was deprived of the use and benefit of his
OR AT THE TIME WHEN property, such as when title is transferred in the name of the Republic of the Philippines, or
THE OWNER WAS Certificates of Land Ownership Award (CLOAs) are issued in favor of the farmer-beneficiaries.
DEPRIVED OF THE USE
AND BENEFIT OF HIS In addition, the factors enumerated under Sec. 17 of RA 6657, as amended, i.e.,
PROP.) (a) the acquisition cost of the land,
(b) the current value of like properties,
(c) the nature and actual use of the property, and the income therefrom,
(d) the owner's sworn valuation,
(e) the tax declarations,
(f) the assessment made by government assessors,
(g) the social and economic benefits contributed by the farmers and the farmworkers, and by the
government to the property, and
(h) the non-payment of taxes or loans secured from any government financing institution on the
said land, if any, must be equally considered.

AP: Here, the CA merely upheld the just compensation fixed by the RTC which considered only
the nature of the land's use, and its assessed value based on the tax declarations, without a
showing, however, that the other factors under Sec. 17 of RA 6657, as amended, were taken into
account or otherwise found to be inapplicable, and completely disregarded the pertinent DAR
formula contrary to what the law requires. On this score alone, the CA clearly erred in
sustaining the RTC's valuation as having been made in accordance with Sec. 17 of RA 6657, as
amended.

Nonetheless, the Court cannot likewise adopt the LBP's computation. It bears to reiterate that just
compensation must be valued at the time of taking, such as when title is transferred in the name
of the Republic, or CLOAs are issued in favor of the farmer-beneficiaries. Accordingly, the just
compensation for the subject land should have been computed based on the values
prevalent for like agricultural lands in accordance with the pertinent DAR regulations
effective during such time of taking. However, while the subject land was placed under
CARP coverage in 1991, records do not bear out the date when title was issued in the name of
the Republic or CLOAs were issued in favor of the farmer-beneficiaries.

To this end, the RTC is hereby directed to observe the following guidelines in the remand of the
case:
1. Just compensation must be valued at the time of taking, or the time when the owner
was deprived of the use and benefit of his property, such as when title is transferred in the name
of the Republic or CLOAs were issued in favor of the farmer-beneficiaries. Hence, the evidence to
be presented by the parties before the RTC for the valuation of the subject land must be based on
the values prevalent on such time of taking for like agricultural lands.

125
2. Courts should consider the factors in Sec. 17 of RA 6657, as amended, prior to its
amendment by RA 9700, as translated into the applicable DAR formula. However, if the
RTC finds that a strict application of the relevant DAR formulas is not warranted, it may depart
therefrom upon a reasoned explanation.

3. Interest may be awarded as may be warranted by the circumstances of the case and
based on prevailing jurisprudence. In previous cases, the Court has allowed the grant of legal
interest in expropriation cases where there is delay in the payment since the just compensation
due to the landowners was deemed to be an effective forbearance on the part of the State. Thus,
legal interest on the unpaid balance shall be pegged at the rate of 12% per annum from the date
of taking, as shall be determined by the RTC, until June 30, 2013 only. Thereafter, or beginning
July 1, 2013, until fully paid, the just compensation due the landowners shall earn interest at the
new legal rate of 6% per annum in line with the amendment introduced by BSP-Monetery Board
Circular No. 799, Series of 2013.

D. Judicial review

De Knecht v. Bautista - Issue: Whether the respondent court committed grave abuse of discretion in allowing the
100 SCRA 660M Republic to take immediate possession of the expropriated properties of the petitioners? YES.
(EXTENSION OF EDSA TO
FERNANDO REIN-DEL LB: There is no question that the government has the right to take private property for public use
PAN STREETS) upon payment of just compensation. However, the government may not capriciously or
arbitrarily choose what private property should be taken. It is a judicial question
JUDICIAL QUESTION = whether in the exercise of such competence, the party adversely affected is the victim
W/N IN THE EXERCISE of partiality and prejudice, as in this case.
OF CHOOSING WHAT
PROPERTY SHOULD BE AP: Here, it is a fact that the DPWH originally establish the extension of EDSA along Cuneta
TAKEN THE GOV’T Avenue. It is to be presumed that the DPWH made studies before deciding on Cuneta Avenue. It
ACTED CAPRICIOUSLY is indeed odd why suddenly the proposed extension of EDSA to Roxas Blvd was changed to go
OR ARBITRARILY through Fernando Rein-Del Pan Streets which the Solicitor General concedes the Del Pan —
Fernando Rein Streets line follows northward and inward direction. While admit that both lines,
Cuneta Avenue and Del Pan — Fernando Rein Streets lines, meet satisfactorily planning and
design criteria and therefore are both acceptable, the Solicitor General justifies the change to Del
Pan — Fernando Rein Streets on the ground that the government wanted to the social impact
factor or problem involved.

It is doubtful whether the extension of EDSA along Cuneta Avenue can be objected to on the
ground of social impact. The improvements and buildings along Cuneta Avenue to be
affected by the extension are mostly motels. Even granting, arguendo, that more people be
affected, the Human Settlements Commission has suggested coordinative efforts of said
Commission with the NHA and other government agencies in the relocation and
resettlement of those adversely affected.

It is clear that the choice of Fernando Rein — Del Pan Streets as the line through which
the EDSA should be extended to Roxas Blvs is arbitrary and should not receive judicial
approval. The respondent judge committed a grave abuse of discretion in allowing the Republic
of the Philippines to take immediate possession of the properties sought to be expropriated.

Republic v. De Knecht - Issue: Whether the passage of BP 340 which provides for the expropriation of the Fernando
182 SCRA 141 Rein-Del Pan Streets superseded the final and executory decision of the Supreme Court in De
Knecht v. Bautista? YES.
SUPERSEDED THE
PREVIOUS CASE LB: Expropriation of lands by the government may be undertaken not only by voluntary
BASED ON negotiation with the land owners, but also by taking appropriate court action or by
SUPERVENING EVENTS legislation. B.P. 340 superseded the final and executory decision of the Supreme Court
THAT OCCURED AFTER in De Knecht vs. Bautista.
THE PREVIOUS
DECISION AP: When the BP 340 was passed expropriating the very properties subject of the present
proceedings, and for the same purpose, it appears that it was based on supervening events
that occurred after the decision of this Court was rendered in De Knecht in 1980
justifying the expropriation through the Fernando Rein-Del Pan Streets. The Court finds
justification in proceeding with the said expropriation proceedings through the Fernando Rein-Del
Pan streets from ESDA to Roxas Boulevard due to the aforestated supervening events after the
rendition of the decision of this Court in De Knecht. The basis of the previous decision seems
to have disappeared justifying their expropriation as all residents have been relocated
and duly compensated.

BP 340 therefore effectively superseded the aforesaid final and executory decision of this Court.
And the trial court committed no grave abuse of discretion in dismissing the case pending before
it on the ground of the enactment of BP 340.

SECTION 10. No law impairing the obligation of contracts shall be passed. Impairment -
anything that diminishes the efficacy of the contract.

Limitations on non-impairment clause

1. Police power

2. Eminent Domain

126
3. Taxation

Note: Franchises, privileges, licenses, etc., do not come within the context of Sec. 10.

Section 10

Ortigas & Co. v. Court Issue: Whether MMC Ordinance No. 81-01 which classified the lot as commercial are violates the
of Appeals - 346 SCRA non-impairment clause of the Constitution? NO.
748
LB: Generally, laws have prospective application but this admits of certain exceptions. One of
NON- IMPAIRMENT OF which is police power. Police power legislation is applicable not only to future
CONTRACTS MUST contracts, but equally to those already in existence.
YIELD TO STATE’S
EXERCISE OF POLICE Non-impairment of contracts or vested rights clauses will have to yield to the superior
POWER and legitimate exercise by the State of police power to promote the health, morals, peace,
education, good order, safety, and general welfare of the people. Moreover, statutes in exercise
of valid police power must be read into every contract. Noteworthy, in Sangalang vs. IAC, we
already upheld MMC Ordinance No. 81-01 as a legitimate police power measure.

While our legal system upholds the sanctity of contract so that a contract is deemed law between
the contracting parties, nonetheless, stipulations in a contract cannot contravene law, morals,
good customs, public order, or public policy. Otherwise such stipulations would be deemed null
and void.

AP: The condition that the property shall be only for residential purpose was superset by the
ordinance.

The contractual stipulations annotated on the Torrens Title, on which Ortigas relies,
must yield to the ordinance. When that stretch of Ortigas Avenue from Roosevelt Street to
Madison Street was reclassified as a commercial zone by the Metropolitan Manila Commission in
March 1981, the restrictions in the contract of sale between Ortigas and Hermoso, limiting all
construction on the disputed lot to single-family residential buildings, were deemed
extinguished by the retroactive operation of the zoning ordinance and could no longer
be enforced.

CA correctly found that the trial court committed GADALEJ in refusing to treat Ordinance No. 81-
01 as applicable to Civil Case No. 64931. In resolving matters in litigation, judges are not only
duty-bound to ascertain the facts and the applicable laws, they are also bound by their oath of
office to apply the applicable law.

Pryce Corporation v. Issue: Whether Court’s approval of rehabilitation plan impaired the contracts between Pryce
China Banking Corporation and China Banking Corporation? NO.
Corporation - G.R. No.
172302 , February 18, LB: Sec. 10, Art. of the Constitution mandates that no law impairing the obligations of contract
2014 shall be passed.

NON-IMPAIRMENT Property rights and contractual rights are not absolute. The constitutional guaranty of non–
CLAUSE MAY NOT BE impairment of obligations is limited by the exercise of the police power of the State for the
INVOKED IN common good of the general public.
CORPORATE
REHABILITATION AP: This case does not involve a law or an executive issuance declaring the modification
CASE (BEC. VALID of the contract between Pryce Corporation and its creditors. Thus, the non–impairment
EXERCISE OF POLICE clause may not be invoked. Furthermore, even assuming that the same may be invoked, the
POWER - FOR THE non–impairment clause must yield to the police power of the State for the common good of the
COMMON GOOD OF general public
THE GENERAL PUBLIC)
Successful rehabilitation of a distressed corporation will benefit its debtors, creditors,
employees, and the economy in general. The court may approve a rehabilitation plan even
over the opposition of creditors holding a majority of the total liabilities of the debtor if, in its
judgment, the rehabilitation of the debtor is feasible and the opposition of the creditors is
manifestly unreasonable. The rehabilitation plan, once approved, is binding upon the debtor and
all persons who may be affected by it, including the creditors, whether or not such persons have
participated in the proceedings or have opposed the plan or whether or not their claims have
been scheduled.

Corporate rehabilitation is one of many statutorily provided remedies for businesses that
experience a downturn. Rather than leave the various creditors unprotected, legislation
now provides for an orderly procedure of equitably and fairly addressing their concerns.
Necessarily, a business in the red and about to incur tremendous losses may not be able to pay
all its creditors. Rather than leave it to the strongest or most resourceful amongst all of them, the
state steps in to equitably distribute the corporation’s limited resources.

Section 12

Magtoto v. Manguera - Issue: Whether Miranda rights may be applied retrospectively? NO.
63 SCRA 4
LB: Sec. 20, Art. 4 of the 1973 Constitution (now Sec. 12, Art. 3) granted, for the first time, to a
MIRANDA RIGHTS person under investigation for the commission of an offense, the right to counsel and to be
informed of such right. And the last sentence thereof which, in effect, means that any
127
CANNOT BE APPLIED confession obtained in violation of this right shall be inadmissible in evidence, can and
RETROSPECTIVELY should be given effect only when the right already existed and had been violated.

In Miranda vs. Arizona, it held that when an individual is taken into custody or otherwise deprived
of his freedom by the authorities in any significant way and is subjected to questioning, the
privilege against self-incrimination is jeopardized.
- Procedural safeguards must be employed to protect the privilege and unless other fully
effective means are adopted to notify the person of his right of silence and to assure that the
exercise of the right will be scrupulously honored, the following measures are required.
1. He must be warned prior to any questioning that he has the right to remain silent, that
anything he says can be used against him in a court of law,
2. that he has the right to the presence of an attorney, and
3. that if he cannot afford an attorney one will be appointed for him prior to any
questioning if he so desires.
- Opportunity to exercise these rights must be afforded to him throughout the interrogation.
- After such warning have been given, and such opportunity afforded him, the individual may
knowingly and intelligently waive these rights and agree to answer questions or make
statement. But unless and until such warning and waiver are demonstrated by the prosecution
at trial, no evidence obtained as a result of interrogation can be used against him.

AP: Here, because the confessions of the accused in G.R. Nos. L-37201-02, 37424 and 38929
were taken before the effectivity of the New Constitution (Jan. 17, 1973) in accordance
with the rules then in force, no right had been violated as to render them inadmissible in
evidence although they were not informed of "their right to remain silent and to
counsel," "and to be informed of such right," because no such right existed at the time.

When the rights become available

People v. Taylaran - Issue: Whether an accused who voluntarily surrendered to a police and admits the crime
108 SCRA 373 committed may invoked the constitutional safeguards to be informed of his rights to silence and
to counsel? NO.
MIRANDA RIGHTS NOT
APPLICABLE IF THE LB: The applicability of the provision does not seem to contemplate cases where no written
ACCUSED confession was sought to be presented in evidence as a result of formal custodial
VOLUNTARILY investigation.
SURRENDERED AND
ADMITS THE CRIME HE AP: Here, what was testified to is only what appellant told the police why he is surrendering to
COMMITTED them (admission of his declaration to Pat. Basilad that he killed his mother because of her alleged
vow to kill him by witchcraft). It is but natural for one who surrenders to the police to give
reason or explanation for his act of surrendering. It can hardly be said that under such
circumstance, the surrendered is already "under investigation within the meaning of
the constitutional provision. As the Solicitor General correctly observes on the circumstances
of this case: "If however, he voluntarily admits the killing and it was precisely because
he surrendered to admit the killing, the constitutional safeguards to be informed of his
rights to silence and to counsel may not be invoked.”

Galman v. Pamaran - Issue: Whether the testimonies given by the private respondents who did not invoke their right
138 SCRA 295 against self-incrimination before the Agrava Board is admissible in evidence?

Ruling: NO.

LB: The fact that the framers of our Constitution did not choose to use the term "custodial" by
having it inserted between the words "under" and investigation", as in fact the sentence opens
with the phrase "any person " goes to prove that they did not adopt in toto the entire fabric of the
Miranda doctrine. Neither are we impressed by

petitioners' contention that the use of the word "confession" in the last sentence of said Section
20, Article 4 connotes the idea that it applies only to police investigation, for although the word
"confession" is used, the protection covers not only "confessions" but also "admissions" made in
violation of this section. They are inadmissible against the source of the confession or admission
and against third person.

AP: Here, the records show that Generals Ver and Olivas were among the last witnesses called
by the Agrava Board. The subject matter dealt with and the line of questioning as shown by the
transcript of their testimonies before the Agrava Board, indubitably evinced purposes other than
merely eliciting and determining the so-called surrounding facts and circumstances of the
assassination. In the light of the examination reflected by the record, it is not far-fetched to
conclude that they were called to the stand to determine their probable involvement in the crime
being investigated. Yet they have not been informed or at the very least even warned while so
testifying, even at that particular stage of their testimonies, of their right to remain silent and
that any statement given by them may be used against them.

This exclusionary rule applies not only to confessions but also to admissions, whether made by a
witness in any proceeding or by an accused in a criminal proceeding or any person under
investigation for the commission of an offense.

People v. Judge Ayson Issue: Whether the right to counsel and right against self-incrimination is available during
administrative investigations?

128
Ruling: NO.

LB: Miranda rights exist only in “custodial interrogations,” or "in-custody interrogation of accused
persons." By custodial interrogation is meant "questioning initiated by law enforcement officers
after a person has been taken into custody or otherwise deprived of his freedom of action in any
significant way."

AP: Here, Felipe Ramos was not in any sense under custodial interrogation, as the term should be
properly understood, prior to and during the administrative inquiry into the discovered
irregularities in ticket sales in which he appeared to have had a hand. The constitutional rights of
a person under custodial interrogation under Sec. 20, Art. 4 of 1973 Constitution did not
therefore come into play, were of no relevance to the inquiry. It is also clear, too, that Ramos had
voluntarily answered questions posed to him on the first day of the administrative investigation,
Feb. 9, 1986 and agreed that the proceedings should be recorded, the record having thereafter
been marked during the trial of the criminal action subsequently filed against him as Exhibit A,
just as it is obvious that the note that he sent to his superiors on Feb. 8,1986, the day before the
investigation, offering to compromise his liability in the alleged irregularities, was a free and even
spontaneous act on his part. They may not be excluded on the ground that the so-called "Miranda
rights" had not been accorded to Ramos.

Police line -ups: paraffin test: signature

Gamboa v. Judge Cruz Issue: Whether the petitioner is entitled to the right to counsel during the conduct of police line-
up?

Ruling: NO. The police line-up was not part of the custodial inquest. Hence, petitioner was not
yet entitled, at such stage, to counsel.

LB: The right to counsel attaches upon the start of an investigation, i.e. when the investigating
officer starts to ask questions to elicit information and/or confessions or admissions from the
respondent/accused. At such point or stage, the person being interrogated must be assisted by
counsel to avoid the pernicious practice of extorting false or coerced admissions or confessions
from the lips of the person undergoing interrogation, for the commission of an offense.

AP: Here, when petitioner was identified by the complainant at the police line-up, he had not
been held yet to answer for a criminal offense. When the process had not yet shifted from the
investigatory to the accusatory as when police investigation does not elicit a confession the
accused may not yet avail of the services of his lawyer. Since petitioner in the course of his
identification in the police line-up had not yet been held to answer for a criminal offense, he was,
therefore, not deprived of his right to be assisted by counsel because the accusatory process had
not yet set in. The police could not have violated petitioner's right to counsel and due process as
the confrontation between the State and him had not begun. In fact, when he was identified in
the police line-up by complainant he did not give any statement to the police. He was, therefore,
not interrogated at all as he was not facing a criminal charge. Far from what he professes, the
police did not, at that stage, exact a confession to be used against him. For it was not he but the
complainant who was being investigated at that time. He "was ordered to sit down in front of the
complainant while the latter was being investigated". Petitioner's right to counsel had not
accrued.

People v. Dimaano Issue: Whether the identification of the appellants without the assistance of their counsel at the
police headquarters by the private complainants is violative of the accused constitutional right to
counsel?

LB: NO. The right to counsel attaches upon the start of an investigation, i.e., when the
investigating officer starts to ask questions to elicit information and/or confessions or admissions
from the accused. At such point or state, the person being interrogated must be assisted by
counsel to avoid the pernicious practice of extorting false or coerced admissions or confessions
from the lips of the person undergoing interrogation for the commission of an offense.

AP: Here, when the appellants were identified by the complainants at the police line-up, the
former had not yet been held to answer for the criminal offense for which they have been charged
and convicted. The police could not have, therefore, violated their right to counsel as the
confrontation between the State and them had not yet begun. As held in the case of Gamboa v.
Cruz, police line up is not part of the custodial inquest, hence, the appellants were not yet
entitled, at such stage, to counsel.

People of the Issue: Whether the out-of-court identification by Nancy and Ryan to the accused is reliable and
Philippines vs. Musa admissible as evidence?

Ruling: YES. Out-of-court identification is conducted by the police in various ways. It is done thru
show-ups where the suspect alone is brought face to face with the witness for identification. It is
done thru mug shots where photographs are shown to the witness to identify the suspect. It is
also done thru line-ups where a witness identifies the suspect from a group of persons lined up
for the purpose.

In resolving the admissibility of and relying on out-of-court identification of suspects, courts have
adopted the totality of circumstances test where they consider the following factors, viz:
1. the witness' opportunity to view the criminal at the time of the crime;
129
2. the witness' degree of attention at that time;
3. the accuracy of any prior description given by the witness;
4. the level of certainty demonstrated by the witness at the identification;
5. the length of time between the crime and the identification; and,
6. the suggestiveness of the identification procedure.

The totality test has been formulated precisely to assure fairness as well as compliance with
constitutional due process requirements in out-of-court identification.

AP: Applying this test, we find Nancy's out-of-court identification to be reliable and, hence,
admissible.

First, Nancy testified that she was seated on the first seat of the jeepney's left rear side. From
this vantage point, she had a good view of the faces of the four persons clinging to the jeepney
as well as the two who were seated inside.

Second, no competing event took place to draw her attention from the hold-up. Nothing in the
records shows the presence of any distraction that could have disrupted her attention at the time
of the robbery or that could have prevented her from having a clear view of the faces and
appearances of the robbers.

Third, the identification took place within five days after the robbery; she sufficiently explained
why it took her five days to go to the police station.

Fourth, she described the suspects to a police inspector prior to identifying them in the police
station on June 16, 2001.

Finally, nothing persuasive supports the appellants' contention that their identification at the
police station was the result of an unduly suggestive procedure. When Nancy went to the Marikina
Police Station, the police merely informed her of the date when the appellants were arrested.
Afterwards, she went to the cell where the appellants were detained; she identified them as the
persons who were her co-passengers and who participated in the robbery. The records are silent
on whether other inmates were detained together with the appellants. Nonetheless, there was no
evidence that the police either prodded Nancy to point to the appellants as the robbers, or
suggested to her that the appellants were the suspects in the June 11, 2001 robbery. That she
readily recognized them was not surprising as they were her fellow passengers before the hold-up
took place.

The records are bereft of any evidence showing that Nancy's photographic identification was
attended by an impermissible suggestion that singled out the appellants and Barredo as the
robbers. More importantly, if there was one person among the perpetrators who would have
caught her attention, it would have been Barredo because he was the one who pointed a gun at
her and at Harold, who took their bag and watch, and who shot Harold. Thus, we uphold the
integrity and reliability of Nancy's in-court identification of the appellants.

D. Right to counsel

People v. De Jesus Issue: Whether admissions obtained during custodial interrogations in the absence of counsel are
admissible in evidence when the same are later reduced into writing and signed in the presence of
counsel?

Ruling: NO. Here, the investigating officer Pfc. Rogelio Lorbes admitted that the two accused,
Tupaz and de Jesus were turned over to him for investigation on Sep. 9, 1987 by the CID,
interviewed them and solicited from them facts and information surrounding the robbery hold-up
with homicide without the assistance of a lawyer. The facts and information were later reduced to
writing on Sep. 10, 1987 in the presence of a CLAO lawyer, a certain Atty. Saldivar. Considering
such circumstances, there was an apparent violation of the accused right to counsel. The right to
counsel attaches upon the start of an investigation, i.e., when the investigating officer starts to
ask questions to elicit information or confession or admission from the accused.

Appellant in this case was already arrested as one of the principal suspects in the killing of Garcia
when he was turned over to Pfc. Lorbes for investigation. According to the testimony of Cpl.
Limpoco, he, together with his team, was ordered to make a follow-up investigation of a stabbing
incident that happened at the New Pasig Public Market on or about September 8, 1987 and that
their target persons were one Eddie and another Tupaz. Consequently, the "interview" conducted
by Pfc. Lorbes cannot be considered merely as a general inquiry but rather a custodial
investigation.

Custodial investigation is the stage where the police investigation is no longer a general inquiry
into an unsolved crime but has began to focus on a particular suspect who had been taken into
custody by the police who carry out a process of interrogation that lends itself to elicit
incriminating statements. It is when questions are initiated by law enforcement officers after a
person has been taken into custody or otherwise deprived of his freedom of action in any
significant way.

People v. Lucero - 249 Issue: Whether the conviction of the accused based on the extrajudicial confessions made by the
SCRA 425 accused without the presence of counsel of his own choice or engaged by any person of his behalf
or appointed by the lower court is valid?

130
Ruling: NO. When the Constitution requires the right to counsel, it did not mean any kind of
counsel but effective and vigilant counsel. The circumstances in the case at bench clearly
demonstrate that appellant received no effective counseling from Atty. Peralta. Here, at the
crucial point when the interrogation was just starting, Atty. Peralta left appellant to attend the
wake of a friend. At that critical stage, appellant gave his uncounselled extra-judicial a
confession. Surely, such a confession where appellant was unprotected from mischief cannot
convict.

Lenido Lumanog, et al. Issue: Whether the accused right to counsel preferably of his choice was not complied with when
vs. People of the the CPDC investigators provided him an IBP counsel?
Philippines-, 7
September 2010 Ruling: NO. Since Joel was provided with a lawyer secured by CPDC investigators from the IBP-
Quezon City chapter, it cannot be said that his right to a counsel preferably of his own choice was
not complied with, particularly as he never objected to Atty. Sansano when the latter was
presented to him to be his counsel for the taking down of his statement.

The phrase preferably of his own choice does not convey the message that the choice of a lawyer
by a person under investigation is exclusive as to preclude other equally competent and
independent attorneys from handling the defense; otherwise the tempo of custodial investigation
would be solely in the hands of the accused who can impede, nay, obstruct the progress of the
interrogation by simply selecting a lawyer who, for one reason or another, is not available to
protect his interest.

Thus, while the choice of a lawyer in cases where the person under custodial interrogation cannot
afford the services of counsel or where the preferred lawyer is not available is naturally lodged in
the police investigators, the suspect has the final choice, as he may reject the counsel chosen for
him and ask for another one. A lawyer provided by the investigators is deemed engaged by the
accused when he does not raise any objection against the counsel’s appointment during the
course of the investigation, and the accused thereafter subscribes to the veracity of the statement
before the swearing officer.

An effective and vigilant counsel necessarily and logically requires that the lawyer be present and
able to advise and assist his client from the time the confessant answers the first question asked
by the investigating officer until the signing of the extrajudicial confession. Moreover, the lawyer
should ascertain that the confession is made voluntarily and that the person under investigation
fully understands the nature and the consequence of his extrajudicial confession in relation to his
constitutional rights. A contrary rule would undoubtedly be antagonistic to the constitutional
rights to remain silent, to counsel and to be presumed innocent.

Atty. Sansano, who supposedly interviewed Joel and assisted the latter while responding to
questions propounded by SPO2 Garcia, Jr., did not testify on whether he had properly discharged
his duties to said client. While SPO2 Garcia, Jr. testified that Atty. Sansano had asked Joel if he
understood his answers to the questions of the investigating officer and sometimes stopped Joel
from answering certain questions, SPO2 Garcia, Jr. did not say if Atty. Sansano, in the first place,
verified from them the date and time of Joels arrest and the circumstances thereof, or any
previous information elicited from him by the investigators at the station, and if said counsel
inspected Joel’s body for any sign or mark of physical torture.

The right to counsel has been written into our Constitution in order to prevent the use of duress
and other undue influence in extracting confessions from a suspect in a crime. The lawyer’s role
cannot be reduced to being that of a mere witness to the signing of a pre-prepared confession,
even if it indicated compliance with the constitutional rights of the accused. The accused is
entitled to effective, vigilant and independent counsel. Where the prosecution failed to discharge
the States burden of proving with clear and convincing evidence that the accused had enjoyed
effective and vigilant counsel before he extrajudicially admitted his guilt, the extrajudicial
confession cannot be given any probative value.

After a thorough and careful review, we hold that there exists sufficient evidence on record to
sustain appellants’ conviction even without the extrajudicial confession of appellant Joel de Jesus.

E. Right to be informed

People v. Pinlac - 165 Issue: Whether the prosecution was able to establish that the accused Pinlac was effectively
SCRA 675 informed of his right to remain silent and of counsel that will justify his conviction?

Ruling: NO. When the Constitution requires a person under investigation "to be informed" of his
right to remain silent and to counsel, it must be presumed to contemplate the transmission of a
meaningful information rather than just the ceremonial and perfunctory recitation of an abstract
constitutional principle. As a rule, therefore, it would not be sufficient for a police officer just to
repeat to the person under investigation the provisions of the Constitution. The right of a person
under interrogation "to be informed" implies a correlative obligation on the part of the police
investigator to explain, and contemplates an effective communication that results in
understanding what is conveyed. Short of this, there is a denial of the right, as it cannot truly be
said that the person has been "informed" of his rights.

The Fiscal has the duty to adduce evidence that there was compliance with the duties of an
interrogating officer. As it is the obligation of the investigating officer to inform a person under
investigation of his right to remain silent and to counsel, so it is the duty of the prosecution to
affirmatively establish compliance by the investigating officer with his said obligation. Absent such
affirmative showing, the admission or confession made by a person under investigation cannot be
131
admitted in evidence.

Thus, in People vs. Ramos, the Court ruled that the verbal admission of the accused during
custodial investigation was inadmissible, although he had been apprised of his constitutional
rights to silence and to counsel, for the reason that the prosecution failed to show that those
rights were explained to him, such that it could not be said that "the apprisal was sufficiently
manifested and intelligently understood" by the accused.

AP: Here, the evidence for the prosecution failed to prove compliance with these constitutional
rights. Furthermore, the accused was not assisted by counsel and his alleged waiver was made
without the assistance of counsel. The record of the case is also replete with evidence which was
not satisfactorily rebutted by the prosecution, that the accused was maltreated and tortured for
seven (7) solid hours before he signed the prepared extra-judicial confession.

Section 13.

A. Right to bail or recognizance

People v. Nitcha Issue: Whether the accused charged with the crime of murder which is punishable by reclusion
perpetua, is entitled to bail as a matter of right?

Ruling: NO. If an accused who is charged with a crime punishable by reclusion perpetua is
convicted by the trial court and sentenced to suffer such a penalty, bail is neither a matter of
right on the part of the accused nor of discretion on the part of the court. In such a situation, the
court would not have only determined that the evidence of guilt is strong — which would have
been sufficient to deny bail even before conviction — it would have likewise ruled that the
accused's guilt has been proven beyond reasonable doubt. Bail must not then be granted to the
accused during the pendency of his appeal from the judgment of conviction.

Thus, an accused who is charged with a capital offense or an offense punishable by reclusion
perpetua, shall no longer be entitled to bail as a matter of right even if he appeals the case to this
Court since his conviction clearly imports that the evidence of his guilt of the offense charged is
strong.

Government of Hong Issue: Whether constitutional provision on bail applies to extradition proceedings?
Kong Special
Administrative Region, Ruling: YES. If bail can be granted in deportation cases, we see no justification why it should not
etc. Vs. Felixberto T. also be allowed in extradition cases. Likewise, considering that the UDHR applies to deportation
Olalia, Jr cases, there is no reason why it cannot be invoked in extradition cases. After all, both are
administrative proceedings where the innocence or guilt of the person detained is not in issue.

Clearly, the right of a prospective extraditee to apply for bail in this jurisdiction must be viewed in
the light of the various treaty obligations of the Philippines concerning respect for the promotion
and protection of human rights. Under these treaties, the presumption lies in favor of human
liberty. Thus, the Philippines should see to it that the right to liberty of every individual is not
impaired.

Here, private respondent was arrested on Sep. 23, 1999, and remained incarcerated until Dec.
20, 2001, when the trial court ordered his admission to bail. In other words, he had been
detained for over 2 years without having been convicted of any crime. By any standard, such an
extended period of detention is a serious deprivation of his fundamental right to liberty. In fact, it
was this prolonged deprivation of liberty which prompted the extradition court to grant him bail.
While our extradition law does not provide for the grant of bail to an extraditee, however, there is
no provision prohibiting him or her from filing a motion for bail, a right to due process under the
Constitution.

An extradition proceeding being sui generis, the standard of proof required in granting or denying
bail can neither be the proof beyond reasonable doubt in criminal cases nor the standard of proof
of preponderance of evidence in civil cases. While administrative in character, the standard of
substantial evidence used in administrative cases cannot likewise apply given the object of
extradition law which is to prevent the prospective extraditee from fleeing our jurisdiction. The
potential extraditee must prove by "clear and convincing evidence" that he is not a flight risk and
will abide with all the orders and processes of the extradition court.

Revilla v. Issue: Whether the Sandiganbayan erred in denying bail to Cambe and Napoles, who are
Sandiganbayan charged with the crime of plunder, after finding strong evidence of their guilt?

Ruling: NO. The grant or denial of bail in an offense punishable by reclusion perpetua, such as
plunder, hinges on the issue of whether or not the evidence of guilt of the accused is strong. This
requires the conduct of bail hearings where the prosecution has the burden of showing that the
evidence of guilt is strong, subject to the right of the defense to cross-examine witnesses and
introduce evidence in its own rebuttal. The court is to conduct only a summary hearing, or such
brief and speedy method of receiving and considering the evidence of guilt as is practicable and
consistent with the purpose of the hearing which is merely to determine the weight of evidence
for purposes of bail.

Here, in finding strong evidence of guilt against Cambe, the Sandiganbayan considered the PDAF
documents and the whistleblowers' testimonies in finding that Cambe received, for Revilla, the
total amount of P103,000,000.00, in return for Revilla's endorsement of the NGOs of Napoles as
132
the recipients of Revilla's PDAF. It gave weight to Luy's summary of rebates and disbursement
ledgers containing Cambe's receipt of money, which Luy obtained from his hard drive. The
Sandiganbayan likewise admitted Narciso as expert witness, who attested to the integrity of Luy's
hard drive and the files in it.

In finding strong evidence of guilt against Napoles, the Sandiganbayan considered the AMLC
Report, as attested by witness Santos, stating that Napoles controlled the NGOs, which were the
recipients of Revilla's PDAF. The Sandiganbayan found that the circumstances stated in the AMLC
Report, particularly that the bank accounts of these NGOs were opened by the named presidents
using JLN Corp. IDs, these accounts are temporary repository of funds, and the withdrawal from
these accounts had to be confirmed first with Napoles, are consistent with the whistleblowers'
testimonies that they were named presidents of Napoles' NGOs and they withdrew large amounts
of cash from the NGOs' bank accounts upon instruction of Napoles. The Sandiganbayan also took
note of the COA report, as confirmed by the testimony of Garcia, that Revilla's PDAF projects
failed to comply with the law, Napoles' NGOs were fake, no projects were implemented and the
suppliers selected to supply the NGOs were questionable.

Accordingly, there is no basis for the allegation of Cambe that the Sandiganbayan Resolutions
were based on mere presumptions and inferences. On the other hand, the Sandiganbayan
considered the entire record of evidence in finding strong evidence of guilt.

For purposes of bail, we held in People v. Cabral that: "[b]y judicial discretion, the law mandates
the determination of whether proof is evident or the presumption of guilt is strong. 'Proof evident'
or 'Evident proof' in this connection has been held to mean clear, strong evidence which leads a
well-guarded dispassionate judgment to the conclusion that the offense has been committed as
charged, that accused is the guilty agent, and that he will probably be punished capitally if the
law is administered. 'Presumption great' exists when the circumstances testified to are such that
the inference of guilt naturally to be drawn therefrom is strong, clear, and convincing to an
unbiased judgment and excludes all reasonable probability of any other conclusion.” The weight
of evidence necessary for bail purposes is not proof beyond reasonable doubt, but strong
evidence of guilt, or "proof evident," or "presumption great." A finding of "proof evident" or
"presumption great" is not inconsistent with the determination of strong evidence of guilt,
contrary to Cambe's argument.

Accordingly, an examination of the entire record —totality of evidence —is necessary to determine
whether there is strong evidence of guilt, for purposes of granting or denying bail to the accused.

B. Waiver of the right

People vs. Judge Issues: 1. Whether the right to bail may, under certain circumstances, be denied to a person
Donato who is charged with an otherwise bailable offense?
2. Whether the right to bail may be waived?

Ruling:

LB: 1. NO. Before conviction, bail is either a matter of right or of discretion. It is a matter of right
when the offense charged is punishable by any penalty lower than reclusion perpetua. To that
extent the right is absolute. Upon the other hand, if the offense charged is punishable by
reclusion perpetua bail becomes a matter of discretion. It shall be denied if the evidence of guilt is
strong. The court's discretion is limited to determining whether or not evidence of guilt is strong.

Here, at the time the original and the amended Informations for rebellion and the application for
bail were filed before the court below the penalty imposable for the offense for which the private
respondent was charged was reclusion perpetua to death. During the pendency of the application
for bail, EO. 187 was issued by the President, by virtue of which the penalty for rebellion as
originally provided for in Art. 135 of the RPC was restored. The restored law was the governing
law at the time the respondent court resolved the petition for bail.

We agree with the respondent court that bail cannot be denied to the private respondent for he is
charged with the crime of rebellion as defined in Article 134 of the Revised Penal Code to which is
attached the penalty of prision mayor and a fine not exceeding P20,000.00. It is, therefore, a
bailable offense under Section 13 of Article III of the 1987 Constitution.

2. YES. Rights guaranteed to one accused of a crime falls naturally into 2 classes: (a) those in
which the state, as well as the accused, is interested; and (b) those which are personal to the
accused, which are in the nature of personal privileges. Those of the first class cannot be waived;
those of the second may be.

The right to bail is another of the constitutional rights which can be waived. It is a right which is
personal to the accused and whose waiver would not be contrary to law, public order, public
policy, morals, or good customs, or prejudicial to a third person with a right recognized by law.

i. Excessive Bail

De La Camara vs. Issue: Whether the bail is excessive?


Enage
Ruling: YES. The guidelines in the fixing of bail:

133
1.Ability of the accused to give bail;
2.Nature of the offense;
3.Penalty for the offense charged;
4.Character and reputation of the accused;
5.Health of the accused;
6.Character and strength of the evidence;
7.Probability of the accused appearing in trial;
8.Forfeiture of other bonds;
9.Whether the accused was a fugitive from justice when arrested;
10.If the accused is under bond for appearance at trial in other cases.

While the court is granted discretion to rule on question of bail, the court will supervise where
conditions imposed upon seeking bail would amount to a refusal thereof and render nugatory this
constitutional right to bail.

Here, there being only two offenses charged, the amount required as bail could not possibly
exceed P50,000.00 for the information for murder and P25,000.00 for the other information for
frustrated murder. Nor should it be ignored in this case that the DOJ did recommend the total
sum of P40,000.00 for the two offenses.

However, the Judge fixed the amount of bail at the excessive amount of P840K for murder and
P355K for frustrated murder. Thus, the bail is excessive.

Section 14

A. Military tribunals

Olaguer v. Military - Issue: Whether military tribunal has jurisdiction to try civilians while the civil courts are open and
150 SCRA 144 functioning during the period of martial law?

Ruling: NO. A military commission or tribunal cannot try and exercise jurisdiction, even during
the period of martial law, over civilians for offenses allegedly committed by them as long as the
civil courts are open and functioning, and that any judgment rendered by such body relating to a
civilian is null and void for lack of jurisdiction. A different ruling would result to the violation of the
right of due process.

Due process of law demands that in all criminal prosecutions, the accused shall be entitled to a
trial. The trial contemplated by the due process clause of the Constitution is a trial by judicial
process, not by executive or military process. Military commissions or tribunals, by whatever
name they are called, are not courts within the Philippine judicial system.

Here, Olaguer and other petitioners who were all civilians were tried by the military commission
for the crime of subversion while the civil courts were still operational.

B. Due process

Scoty's Department Issue: Whether the Court of Industrial Relations has jurisdiction to impose the penalties
Store v. Micaller - 99 prescribed in Sec. 25 of RA 875?
Phil 762
Ruling: NO. The word “court” used in Sec. 25 of RA 875 cannot refer to the Court of Industrial
Relations for to give that meaning would be violative of the safeguards guaranteed to every
accused by our Constitution such as “no person shall be held to answer for a criminal offense
without due process of law” and that “in all criminal prosecution the accused shall enjoyed the
right to be heard by himself and counsel, against him, to have a speedy and public trial, to meet
the witnesses face to face, and to have compulsory process to secure the attendance of witnesses
in his behalf”.

The procedure laid down by law to be observed by the Court of Industrial Relations in dealing with
ULP cases negates those constitutional guarantee to the accused. And this is so because, among
other things, the law provides that "the rules of evidence prevailing in court for the courts of law
or equity cannot be controlling and it is the spirit and intention of this act that the Court of
Industrial Relations and its members and its Hearing Examiners shall be use every and all
reasonable means to ascertain the facts in each case speedily and objective and without regards
to technicalities of law of procedure." It is likewise enjoined that "the Court shall not be bound
solely by the evidence presented during the hearing but may avail itself of all other means such
as, but not limited to, ocular inspections and questioning of well-informed persons which results
must be made a part of the record" [section 5 (b), Republic Act No. 875]. All this means that an
accused may be tried without the right "to meet the witnesses face to face" and may be convicted
merely on preponderance of evidence and not beyond reasonable doubt. This is against the due
process guaranteed by the constitution. It may be contended that this gap may be subserve
strictly the rules applicable to criminal cases to meet the requirements of the Constitution, but
this would be tantamount to amending the law which is not within the province of the judicial
branch of our Government.

Thus, the power to impose the penalties provided for in Sec. 25 of RA 875 is lodged in ordinary
courts, and not in the Court of Industrial Relations, notwithstanding the definition of the word

134
"Court" contained in Sec. 2(a) of said Act. Hence, the decision of the industrial court in so far as it
imposes a fine upon petitioners is illegal and should be nullified.

C. Presumption of innocence

United States v. Luling Issue: Whether a law (Sec. 316 of Act 355) which requires the accused to prove his innocence is
- 34 Phil 725 unconstitutional?

Ruling: NO. While presumption of innocence is the rule, it is established also that certain facts
only shall constitute prima facie evidence and that then the burden is on the defendant to show
that his acts are not criminal. In case of statutory crimes (such in the case in the Philippines), no
constitutional provision is violated by a statute providing that proof by the state of some material
fact or facts shall constitute prima facie evidence of guilt, and that then the burden is shifted to
the defendant for the purpose of showing that such act or acts are innocent and are committed
without unlawful intention.

People v. Mingoa - 92 Issue: Whether Sec. 217 of the RPC which states that failure of a public officer to account any
Phil 856 public funds which he is chargeable upon demand is presumed to misappropriated the same is
unconstitutional?

Ruling: NO. The test for validity of a presumption is “there be a rational connection between the
facts proved and the ultimate fact presumed so that the inference of the one from proof of the
others is not unreasonable and arbitrary because of lack of connection between the two in
common experience.”

The statute in the present case creates a presumption of guilt once certain facts are proved. It
makes the failure of public officer to have duly forthcoming, upon proper demand, any public
funds or property with which he is chargeable prima facie evidence that he has put such missing
funds or property to personal use. The ultimate fact presumed is that officer has malversed the
funds or property entrusted to his custody, and the presumption is made to arise from proof that
he has received them and yet he has failed to have them forthcoming upon proper demand.
Clearly, the fact presumed is but a natural inference from the fact proved, so that it cannot be
said that there is no rational connection between the two. Furthermore, the statute establishes
only a prima facie presumption, thus giving the accused an opportunity to present evidence to
rebut it. The presumption is reasonable and will stand the test of validity.

D. Right to counsel

People v. Holgado - 86 Issue: Whether the failure of the trial court to inform the accused of his right to counsel when he
Phil 752 entered a plea of guilty to the information without the aid of a counsel violates the accused right
to counsel?

Ruling: YES. The right to be heard would be meaningless since without the assistance of counsel,
there can be no fair trial. “Even the most intelligent or educated man may have no skill in the
science of the law, particularly in the rules of procedure, and, without counsel, he may be
convicted not because he is guilty but because he does not know how to establish his innocence.”

When a defendant appears without attorney, the court has 4 important duties to comply with: (1)
it must inform the defendant that it is his right to have attorney before being arraigned; (2) after
giving him such information the court must ask him if he desires the aid of an attorney; (3) if he
desires and is unable to employ attorney, the court must assign attorney de oficio to defend him;
and (4) if the accused desires to procure an attorney of his own the court must grant him a
reasonable time therefor.

Not one of these duties had been complied with by the trial court. The record discloses that said
court did not inform the accused of his right to have an attorney nor did it ask him if he desired
the aid of one. The trial court failed to inquire whether or not the accused was to employ an
attorney, to grant him reasonable time to procure or assign an attorney de oficio. The question
asked by the court to the accused was "Do you have an attorney or are you going to plead
guilty?" Not only did such a question fail to inform the accused that it was his right to have an
attorney before arraignment, but, what is worse, the question was so framed that it could have
been construed by the accused as a suggestion from the court that he plead guilty if he had no
attorney. And this is a denial of fair hearing in violation of the due process clause contained in our
Constitution.

People v. Simben - 98 Issue: Whether Simben’s plea of guilty for violation of Art. 201 of RPC without the aid of counsel
Phil 138 is valid?

Ruling: YES. What transpired when the appellant was arraigned shows that his rights were fully
protected and safeguarded. The Court complied with its duty when it informed the appellant that
it was his right to have the aid of counsel. And before pronouncing the sentence the Court took
pains to ascertain whether he was aware of the consequences of the plea he had entered.
Notwithstanding this precaution and warning, he waived his right to have the aid of counsel and
entered a plea of guilty to the information.

Delgado v. Court of Issue: Whether the petitioner’s conviction of estafa thru falsification of public and/or official
Appeals - 145 SCRA documents in deceiving Rueda, a MedTech, in arranging her travel to US may be set aside on the
357 ground that his counsel (Atty. Yco) was not a member of Philippine Bar?
135
Ruling: YES. An accused person is entitled to be represented by a member of the bar in a
criminal case filed against her before the Regional Trial Court. Unless she is represented by a
lawyer, there is great danger that any defense presented in her behalf will be inadequate
considering the legal perquisites and skills needed in the court proceedings. This would certainly
be a denial of due process.

E. Right to be informed

People v. Regala - 113 Issue: Whether the conviction of the accused on the complex crime of murder with assault upon
SCRA 613 an agent of a person in authority violates his right to be informed of the crime charged?

Ruling: YES. Assault upon person in authority cannot be appreciated as a qualifying


circumstance. The information filed against appellant did not allege the essential elements of
assault that the accused then knew that, before or at the time of the assault, the victim was an
agent of a person in authority. The information in this case barely alleged that the accused "with
deliberate intent to kill, with evident premeditation and treachery and taking advantage of
nighttime, did then and there wilfully, unlawfully and feloniously attack and stab with a knife one
Sgt. Juan Desilos Jr., a member of the Philippine Constabulary while he was then in the
performance of his official duty thereby inflicting upon the latter serious stab wounds at the
midepigastric region penetrating abdominal cavity and perforating cardial and cardiac region
which injury directly caused his instantaneous death.” Consequently, the killing of Sgt. Juan
Desilos Jr. by appellant cannot be qualified as murder. It was simple homicide.

The fact that the crime of assault was established by the evidence of the prosecution without any
objection on the part of the accused cannot likewise cure the aforestated defect in the information
so as to validly convict the accused thereof; because to do so would be convicting the accused of
a crime not properly alleged in the body of the information in violation of his constitutional right
to be informed of the nature and cause of the accusation against him.

Enrile v. Salazar - 186 Issue: Whether Enrile’s constitutional right to be informed of the nature and cause of accusation
SCRA 217 against him had been violated when he was charged with the crime of rebellion with murder and
multiple frustrated murder?

Ruling: NO. The Court ruled that the information filed against the petitioner does in fact charge
an offense. Disregarding the objectionable phrasing that would complex rebellion with murder and
multiple frustrated murder, that indictment is to be read as charging simple rebellion. In Amado
vs. Hernandez, the Court held that under the allegations of the amended information against the
accused, the murders, arsons and robberies described therein are mere ingredients of the crime
of rebellion allegedly committed by said defendants, as means "necessary" for the perpetration of
said offense of rebellion; that the crime charged in the aforementioned amended information is,
therefore, simple rebellion, not the complex crime of rebellion with multiple murder, arsons and
robberies. Therefore the information filed against them is valid. Since simple rebellion only, they
are entitled to bail before final conviction as a matter of right.

People v. Ortega - 276 Issue: Whether the accused Garcia’s conviction for a crime of homicide through drowning in an
SCRA 166 information that charged murder by means of stabbing violates his right to be informed?

Ruling: YES. An accused has a right to be informed of the nature and cause of the accusation
against him. To convict him of an offense other than that charged in the complaint or information
would be a violation of this constitutional right.

Here, the Information accused Garcia of Murder by stabbing. However, he was convicted of
Homicide through drowning. Hence, it violates his constitutional right under Sec. 14 (2).

F. Right to speedy trial

Conde v. Rivera - 45 Issue: Whether the 8 arbitrary postponements of Conde’s trials without her consent violated her
Phil 650 constitutional right to speedy trial?

Ruling: YES. The Constitution guarantees that in all criminal prosecutions the accused shall enjoy
the right to have a speedy trial. Aurelia Conde, like all other accused persons, has a right to a
speedy trial in order that if innocent she may go free, and she has been deprived of that right in
defiance of law. Dismissed from her humble position, and compelled to dance attendance on
courts while investigations and trials are arbitrarily postponed without her consent, is palpably
and openly unjust to her and a detriment to the public.

Where a prosecuting officer, without good cause, secures postponements of the trial of a
defendant against his protest beyond a reasonable period of time, as in this instance for more
than a year, the accused is entitled to relief by a proceeding in mandamus to compel a dismissal
of the information, or if he be restrained of his liberty, by habeas corpus to obtain his freedom.

G. Right to impartial trial

Mateo, Jr. v. Villaluz - Issue: Whether the respondent judge is disqualified from deciding the validity of the repudiation

136
50 SCRA 18 of an extrajudicial statement signed and sworn by the accused before him?

Ruling: YES. Due process of law requires a hearing before an impartial and disinterested tribunal,
and that every litigant is entitled to nothing less than the cold neutrality of an impartial judge. A
judge should strive to be at all times wholly free, disinterested, impartial and independent. A
judge has both the duty of rendering a just decision and the duty of doing it in a manner
completely free from suspicion as to its fairness and as to his integrity.

Here, Reyes subscribed before respondent judge an extra-judicial statement, which he later
repudiated on the ground that it was a product of intimidation by government agents, he in effect
ruled that such extrajudicial statement was entered freely. With its repudiation on the ground that
it was not so at all, coercion having come into the picture there is apparent the situation of a
judge having to pass on a question that by implication had already been answered by him. For
respondent Judge was called upon to review a matter on which he had previously given his
opinion. It is this inroad in one's objectivity that is sought to be avoided by the law on
disqualification. The misgivings then as to the requirement of due process for "the cold neutrality
of an impartial judge" not being met are more titan justified.

H. Right to public trial

Garcia v. Domingo - 52 Issue: Whether the trial of cases held exclusively in the chamber of the judge and not in the
SCRA 143 court room open to public violates the right to public trial of the accused?

Ruling: NO. The constitutional provision that trial must be public possesses the character that
when anyone interested in observing the manner a judge conducts the proceedings in his
courtroom may do so. There is to be no ban on such attendance. His being a stranger to the
litigants is of no moment. But there is the well recognized exception that warrants the exclusion
of the public where the evidence may be characterized as "offensive to decency or public morals."

Here, there is no showing that the public was excluded. It is to be admitted that the size of the
room allotted the Judge would reduce the number of those who could be present. Such a fact
though is not indicative of any transgression of this right. Justice Black said that it suffices to
satisfy the requirement of a trial being public if the accused could "have his friends, relatives and
counsel present, no matter with what offense he may be charged." Also, at least 14 hearings
were already conducted in the chambers of Judge Garcia without any objection on the part of the
policemen, which implies lack of weight as to their contention.

I. Compulsory process / confrontation

Fajardo v. Garcia - 98 Issue: Whether the Judge who refused to grant the request of the accused for leave to serve
SCRA 514 written interrogatories on the doctor who treated their injuries who already left for abroad
violated the constitutional guarantee to an accused to compulsory process to secure the
production of evidence in his behalf?

Ruling: NO. The said medical testimony on the injuries they sustained was vital to their defense
can still be adduced thru other witnesses and hospital records. The fact that the petitioners were
treated in the hospital by a doctor in question could be testified to by other witnesses including
the nurses who must have been present. It cannot be assumed that there would be an
insuperable objection to the presentation of the medical certificate as to the wounds alleged to
have been inflicted, as they could very well show traces of such maltreatment. Even the length of
their stay in the hospital could be verified by its records.

People v. Miyake - 279 Issue: Whether the adoption by the Makati court of the facts of the decision of Parañaque court
SCRA 180 for estafa to constitute the basis of the subsequent conviction for illegal recruitment in large scale
is erroneous as it violates the right of the accused to confront the witness during the trial before
it?

Ruling: YES. The accused in a criminal case is guaranteed the right of confrontation. Such right
has two purposes: first, to secure the opportunity of cross-examination; and, second, to allow the
judge to observe the deportment and appearance of the witness while testifying. This right,
however, is not absolute as it is recognized that it is sometimes impossible to recall or produce a
witness who has already testified in a previous proceeding, in which event his previous testimony
is made admissible as a distinct piece of evidence, by way of exception to the hearsay rule. The
previous testimony is made admissible because it makes the administration of justice orderly and
expeditious.

The adoption by the Makati trial court of the facts stated in the decision of the Parañaque trial
court does not fall under the exception to the right of confrontation as the exception
contemplated by law covers only the utilization of testimonies of absent witnesses made in
previous proceedings, and does not include utilization of previous decisions or judgments.

Here, the evidence by the prosecution only established that said illegal acts were done to one
person. This was supported by the evidence and testimony presented by witnesses Elenita
Marasigan. Hence, Miyake is only guilty of simple illegal recruitment.

As for Generillo and Del Rosario, no concrete proof that acts were done to all complainants,
especially since the prosecution failed to present them as witnesses, therefore violating the right
of the accused-appellant to confront witnesses.
137
People v. Seneris - 99 Issue: Whether the testimony of a prosecution witness (Delos Santos) in the parricide case who
SCRA 92 died before completion of his cross-examination is admissible in evidence?

Ruling: YES. Where a party has had the opportunity to cross-examine an opposing witness but
failed to avail himself of it, he necessarily forfeits the right to cross examine and the testimony
given on direct examination of the witness will be received or allowed to remain in the record.

On the other hand, when the cross-examination is not and cannot be done or completed due to
causes attributable to the party offering the witness, the uncompleted testimony is thereby
rendered incompetent and inadmissible in evidence.

The direct testimony of a witness who dies before conclusion of the cross-examination can be
stricken only insofar as not covered by the cross-examination and absence of a witness is not
enough to warrant striking his testimony for failure to appear for further cross-examination where
the witness has already been sufficiently cross-examined or the matter on which further cross-
examination is sought is not in controversy.

Here, because the cross-examination made by the counsel of private respondent of the deceased
witness was extensive and already covered the subject matter of his direct testimony as state
witness relating to the essential elements of the crime of parricide, and what remained for further
cross-examination is the matter of price or reward allegedly paid by private respondent for the
commission of the crime, which is merely an aggravating circumstance and does not affect the
existence of the offense charged, the respondent judge gravely abused his discretion in declaring
as entirely inadmissible the testimony of the state witness who died through no fault of any of the
parties before his cross- examination could be finished.

J. Trial in absentia; right to be present

Carredo v. People - Issue: Whether an accused who, after arraignment, waives his further appearance during the
183 SCRA 273 trial can be ordered arrested by the court for non-appearance upon summons to appear for
purposes of identification?

Ruling: YES. The provision of the Constitution authorizing the trial in absentia of the accused in
case of his non-appearance after arraignment despite due notice simply means that he thereby
waives his right to meet the witnesses face to face among others. An express waiver of
appearance after arraignment, as in this case, is of the same effect. However, such waiver of
appearance and trial in absentia does not mean that the prosecution is thereby deprived of its
right to require the presence of the accused for purposes of identification by its witnesses which is
vital for the conviction f the accused. Such waiver of a right of the accused does not mean a
release of the accused from his obligation under the bond to appear in court whenever so
required. The accused may waive his right but not his duty or obligation to the court. Thus, he
can still be ordered arrested by the court for non-appearance upon summons.

Section 16

Gonzales v. Issue: Whether a delay of more than 6 years of the termination of the preliminary investigation
Sandiganbayan - 199 violates the constitutional right of the accused to speedy disposition of their cases?
SCRA 299
Ruling: NO. While there may seem to have been a delay in the termination of the preliminary
investigation involved in the case at bar, the same cannot be imputed solely to the prosecution.
In its questioned resolution, the Sandiganbayan recounted the incidents which contributed largely
to the delay and which are attributable to petitioner Gonzales and his counsel.

Here, it will be noted that it was only on Aug. 22, 1988 when the complaint was filed. The
original information charging petitioner with a violation of Sec. 3(e) of RA 3019, was filed on May
5, 1989. After the filing of said information, petitioner filed several motions before the Office of
the Ombudsman and the Sandiganbayan seeking reconsideration and re-evaluation of the case
and praying for the suspension of the proceedings in Criminal Case No. 13563 during the
pendency of the resolution of these motions. Thereafter, the Sandiganbayan ordered the conduct
of a preliminary investigation against Palanca and, subsequently, the amended information was
filed. It is, therefore, apparent and irremissible that the delay is equally chargeable to petitioner.
Hence, he cannot now seek the protection of the law to cover up for his own actuations or benefit
from what he now considers the adverse effects of his own conduct in the case.

The right to a speedy disposition of a case, like the right to speedy trial, is deemed violated only
when the proceeding is attended by vexatious, capricious, and oppressive delays; or when
unjustified postponements of the trial are asked for and secured, or when without cause or
justifiable motive a long period of time is allowed to elapse without the party having his case
tried. Equally applicable is the balancing test used to determine whether a defendant has been
denied his right to a speedy trial, or a speedy disposition of a case for that matter, in which the
conduct of both the prosecution and the defendant are weighed, and such factors as length of the
delay, reason for the delay, the defendant’s assertion or non-assertion of his right, and prejudice
to the defendant resulting from the delay, are considered.

Magante v. Issue: Whether a delay of 7 years in the termination of the preliminary investigation of
Sandiganbayan – G.R. Magante’s case amounts to a violation of his right to speedy disposition of cases?
No. 230950-51, July
138
23, 2018
Ruling: YES. 4 factors to determine whether inordinate delay attended the conduct of preliminary
investigation are:
1.Length of delay

The reckoning point when delay starts to run is the date of the filing of a formal complaint by a
private complainant or the filing by the Field Investigation Office with the Ombudsman of a formal
complaint based on an anonymous complaint or as a result of its motu proprio investigations. The
period devoted to the fact-finding investigations prior to the date of the filing of the formal
complaint with the Ombudsman shall NOT be considered in determining inordinate delay. After
the filing of the formal complaint, the time devoted to fact finding investigations shall always be
factored in.

2.Reason for the delay;

Valid reasons are but not limited to: (a) extraordinary complications such as the degree of
difficulty of the questions involved, the number of persons charged, the various pleadings filed,
and the voluminous documentary and testimonial evidence on record; and (b) acts attributable to
the respondent.

3.Defendant’s assertion of his right; and

Failure to move for the early resolution of the preliminary investigation or similar reliefs before
the Ombudsman amounted to a virtual waiver of the constitutional right.

4.Prejudice to the defendant.

Who between the two parties was placed at a greater disadvantage by the delay in the
investigation.

Here, the case against petitioner was initiated on Jan. 7, 2011, when the PACPOOMB-Visayas filed
a formal complaint against petitioner. The fact-finding investigation, having preceded the filing of
the formal complaint, is excluded in computing the duration of the delay. Thus, petitioner’s
preliminary investigation lasted from Jan. 7, 2011 until Apr. 15, 2016, or about 5 years and 3
months from the date of the filing of the formal complaint, and 5 years and 2 months from Feb.
15, 2011 when petitioner was ordered to file his counter-affidavit.

Since the duration of the preliminary investigation is excessive, it is incumbent then on the
prosecution to justify the delay. Unfortunately, no circumstance in this case warranted the
protracted period of investigation. The prosecution harps on the fact that there were 10
respondents in the complaint filed with the OMB and each of them was afforded the right to
explain themselves. Also, the records of the case were allegedly voluminous that entailed
considerable time to study and analyze. These reasons, to Our mind, do not sufficiently explain
the more than 5-year long preliminary investigation.

Verily, on May 6, 2011, the criminal complaint was then already deemed submitted for resolution.
Yet, it would only be on Apr. 15, 2016 when petitioner would once again hear about the case,
through the receipt of the adverse ruling finding probable cause to charge him with splitting of
contracts and falsification of public documents. Noticeably, the prosecution did not offer any
acceptable explanation for this gap between Feb. 15, 2011 and Apr.15, 2016. Contrary to the
finding of the Sandiganbayan, there is a hiatus on the part of the Ombudsman during this period.
Left unsatisfactorily explained, this amounts to a violation of petitioner's constitutional right to a
speedy disposition of case, corollarily warranting the dismissal of the criminal case against him.

Likewise, it is not for the petitioner to ensure that the wheels of justice continue to turn. Rather,
it is for the State to guarantee that the case is disposed within a reasonable period. Thus, it is of
no moment that petitioner herein, unlike in Angchango, did not file any motion before the
Ombudsman to expedite the proceeding. It is sufficient that he raised the constitutional infraction
prior to his arraignment before the Sandiganbayan.

Neither can petitioner be deemed to have waived his right to a speedy disposition of a case when
he filed a MR against an adverse resolution of the Ombudsman on May 31, 2015. The filing of this
singular motion cannot by itself be considered as active participation in the preliminary
investigation proceeding that amounted to a waiver of a constitutional right.

Lastly, there could have been no grave prejudice suffered by the State from the delay since the
criminal charges for falsification of public documents and splitting of contracts are offences that
chiefly rely on the presentation of documentary evidence that, at this point, has already formed
part of the records of the case. The evidence of the prosecution is then sufficiently protected and
preserved.

Section 17

C. Right against self-incrimination

Bermudez v. Castillo - Issue: Whether the complainant may be compelled to furnish new specimen of her handwriting
64 Phil 483 by copying the exhibits?
139
Ruling: NO. The protection under Sec. 17 extends to all cases — criminal, civil, or administrative.
It should be noted that the complainant, under oath, has sworn to tell the truth before the
investigator authorized to receive statements under oath; she asserted that the documents in
question were not written by her. If she were to be compelled to write and if it were proven by
means of what she might write later that said documents had really been written by her, it would
be impossible for her to evade prosecution for perjury. The purpose of the provision is to avoid
and prohibit the inhuman procedure of compelling a person, in a criminal or any other case, to
furnish the missing evidence for his conviction. In order that this constitutional provision may
prove to be a real protection, and not just a dead letter law, it must be given a liberal and broad
interpretation favorable to the person invoking it. When those present and capable of establishing
the facts are unwilling to testify because of fear of incriminating themselves, the constitutional
and statutory guarantees cannot be swept aside merely because they result in making difficult,
even impossible, the conviction of the accused. It is no invasion of the constitutional guaranty
against self- incrimination to compel the witness to answer questions relating to the truthfulness
of his previous testimony.

Cabal v. Kapunan, Jr. - Issue: Whether the order of the Committee requiring the petitioner to take the witness stand
6 SCRA 1059 violates the petitioner’s right against self-incrimination?

Ruling: YES. Although the said Committee was created to investigate the administrative charge
of unexplained wealth, it seems that the purpose of the charge against petitioner is to apply the
provisions of the Anti-Graft Law, which authorizes the forfeiture to the State of property of a
public officer or employee which is manifestly out of proportion to his salary as such public officer
or employee and his other lawful income and the income from legitimately acquired property.
However, such forfeiture has been held to partake of the nature of a penalty. As a consequence,
proceedings for forfeiture of property are deemed criminal or penal, and, hence, the exemption of
defendants in criminal case from the obligation to be witnesses against themselves are applicable
thereto.

No person shall be compelled in any criminal case to be a witness against himself. This prohibition
against compelling a person to take the stand as a witness against himself applies to criminal,
quasi-criminal, and penal proceedings, including a proceeding civil in form for forfeiture of
property by reason of the commission of an offense, but not a proceeding in which the penalty
recoverable is civil or remedial in nature.

The privilege still applies as long as the penalty or forfeiture is recoverable, or is imposed in
vindication of the public justice of the state, as a statutory fine or penalty, or violation of a
municipal ordinance. However, the privilege is simply an option of refusal, and not a prohibition of
inquiry.

Almonte v. Vasquez - Issue: Whether the issuance of subpoena duces tecum ordering the petitioners to produce
244 SCRA 286 documents relating to personal services and salary vouchers of Economic Intelligence and
Investigation Bureau employees is violative of their right against selfincrimination?

Ruling: NO. The documents required to be produced in this case are public records and the
subpoena duces tecum is directed to government officials in whose possession or custody the
documents are. Moreover, if, as petitioners claim, the disbursement by the EIIB of funds for
personal service has already been cleared by the COA, there is no reason why they should object
to the examination of the documents by respondent Ombudsman.

People v. Malimit - 264 Issue: Whether the admission as evidence of Malaki’s wallet together with its contents violates
SCRA 167 his right against self-incrimination?

Ruling: NO. The right against self-incrimination is a prohibition of the use of physical or moral
compulsion, to extort communications from the accused. It is simply a prohibition against legal
process to extract from the accused's own lips, against his will, admission of his guilt. Hence, the
right of self-incrimination only applies to testimonial compulsion. It does not apply to case where
the evidence sought to be excluded is not an incriminating statement but an object evidence.
Concededly, appellant was not informed of his right to remain silent and to have his own counsel
by the investigating policemen during the custodial investigation. Neither did he execute a written
waiver of these rights in accordance with the constitutional prescriptions. Nevertheless, these
constitutional short-cuts do not affect the admissibility of Malaki's wallet, identification card,
residence certificate and keys for the purpose of establishing other facts relevant to the crime.
Thus, the wallet is admissible to establish the fact that it was the very wallet taken from Malaki
on the night of the robbery. The identification card, residence certificate and keys found inside the
wallet, on the other hand, are admissible to prove that the wallet really belongs to Malaki.
Furthermore, even assuming arguendo that these pieces of evidence are inadmissible, the same
will not detract from appellant's culpability considering the existence of other evidence and
circumstances establishing appellant's identity and guilt as perpetrator of the crime charged.

United States v. Tan Issue: Whether the physical examination conducted to the accused was a violation of his right
Teng - 23 Phil 145 against self-incrimination?

Ruling: NO. Physical examination is not covered by the self-incrimination clause. The taking of a
substance from his body was not a violation of the said right. He was neither compelled to make
any admissions or to answer any questions. The mere fact that an object found on his person was
examined: seems no more to infringe the rule invoked, than would the introduction in evidence of
stolen property taken from the person of a thief.

The prohibition that a person shall not be compelled to be a witness against himself, is simply a
140
prohibition against legal process to extract from the defendant's own lips, against his will, an
admission of his guilt. Its purpose is to prohibit compulsory oral examination of prisoners before
trial. Or upon trial, for the purpose of extorting unwilling confessions or declarations implicating
them in the commission of a crime.

Dela Cruz vs. People – Issue: Whether taking urine sample for drug testing is covered by allowable nontestimonial
730 SCRA 655 compulsion making it non-incriminatory.

Ruling: NO. Cases where non-testimonial compulsion has been allowed reveal that the pieces of
evidence obtained were all material to the principal cause of the arrest. In the instant case, we
fail to see how a urine sample could be material to the charge of extortion. The RTC and the CA,
therefore, both erred when they held that the extraction of petitioner’s urine for purposes of drug
testing was “merely a mechanical act, hence, falling outside the concept of a custodial
investigation.”

Standard Chartered Issue: Whether petitioners, invoking their right against self-incrimination, justify their non-
Bank vs. Senate appearance in a legislative inquiry?
Committee on Banks -
541 SCRA 546 Ruling: NO. The petitioners, officers of SCB-Philippines, are not being indicted as accused in a
criminal proceeding. They were summoned by respondent merely as resource persons, or as
witnesses, in a legislative inquiry.

An accused occupies a different tier of protection from an ordinary witness. Whereas an ordinary
witness may be compelled to take the witness stand and claim the privilege as each question
requiring an incriminating answer is shot at him, an accused may altogether refuse to take the
witness stand and refuse to answer any and all questions. Concededly, this right of the accused
against self-incrimination is extended to respondents in administrative investigations that partake
of the nature of or are analogous to criminal proceedings. The privilege has consistently been held
to extend to all proceedings sanctioned by law; and to all cases in which punishment is sought to
be visited upon a witness, whether a party or not. However, here, petitioners neither stand as
accused in a criminal case nor will they be subjected by the respondent to any penalty by reason
of their testimonies. Hence, they cannot altogether decline appearing before respondent, although
they may invoke the privilege when a question calling for an incriminating answer is propounded.

Section 19

C. Cruel, degrading or inhuman punishment; excessive fines

People v. Estoista - 93 Issue: Whether the penalty of 5 to 10 years imprisonment for illegal possession of firearms
Phil 647 (read the imposed by RA 4 is cruel and unusual?
resolution dated 3
December 1953) Ruling: (Decision) NO. Confinement from 5 to 10 years for possessing or carrying firearm is not
cruel or unusual, having due regard to the prevalent conditions which the law proposes to
suppress or curb. The rampant lawlessness against property, person, and even the very security
of the Government, directly traceable in large measure to promiscuous carrying and use of
powerful weapons, justify imprisonment which in normal circumstances might appear excessive.
If imprisonment from 5 to 10 years is out of proportion to the present case in view of certain
circumstances, the law is not to be declared unconstitutional for this reason. The constitutionality
of an act of the legislature is not to be judged in the light of exceptional cases. Small
transgressors for which the heavy net was not spread are, like small fishes, bound to be caught,
and it is to meet such a situation as this that courts are advised to make a recommendation to
the Chief Executive for clemency or reduction of the penalty.

(MR) NO. It takes more than merely being harsh, excessive, out of proportion, or severe for a
penalty to be obnoxious to the Constitution. "The fact that the punishment authorized by the
statute is severe does not make it cruel and unusual." To come under the ban, the punishment
must be "flagrantly and plainly oppressive," "wholly disproportionate to the nature of the offense
as to shock the moral sense of the community." Having in mind the necessity for a radical
measure and the public interest at stake, we do not believe that five years' confinement for
possessing firearms, even as applied to appellant's and similar cases, can be said to be cruel and
unusual, barbarous, or excessive to the extent of being shocking to public conscience.

Corpuz vs. People – Issue: Whether the imposition of additional 1 year of imprisonment in excess of P22,000 for the
724 SCRA 1 crime of estafa under the RPC is a cruel and unusual punishment?

Ruling: YES. There seems to be a perceived injustice brought about by the range of penalties
that the courts continue to impose on crimes against property committed today, based on the
amount of damage measured by the value of money 80 years ago in 1932. However, this Court
cannot modify the said range of penalties because that would constitute judicial legislation. What
the legislature’s perceived failure in amending the penalties provided for in the said crimes cannot
be remedied through this Court’s decisions, as that would be encroaching upon the power of
another branch of the government.

Maturan vs. Comelec – Issue: Whether penalty of perpetual disqualification to hold public office may be properly
G.R. No. 227155, 28 imposed on a candidate for public office who repeatedly fails to submit his Statement of
March 2017 Contributions and Expenditures (SOCE) pursuant to Sec. 14 of RA 7166 amounts to cruel,
degrading and inhuman punishment?

141
Ruling: NO. The constitutional proscription under the Bill of Rights extends only to situations of
extreme corporeal or psychological punishment that strips the individual of his humanity. The
proscription is aimed more at the form or character of the punishment rather than at its severity.

Republic v. N. Dela Issue: Whether the penalty of P10,000 per day of violation in accordance with Sec. 28 of RA
Merced & Sons, Inc. – 9275 is excessive?
G.R. No. 201501,
January 22, 2018 Ruling: NO. The constitutional prohibition on the imposition of excessive fines applies only to
criminal prosecutions. Here, it involves an administrative proceeding and, contrary to the
supposition of Dela Merced, the fine imposed is not a criminal penalty. Hence, the proscription
under Sec. 19 is inapplicable to this case.

D. Effect of abolition on application of penal laws

People v. Muñoz - 170 Issue: Whether death penalty has been abolished by Sec. 19(1) thus changing the Graduated 3-
SCRA 107 Period Scheme (RT Max – RP – Death)?

Ruling: NO. A reading of Sec. 19(1) will readily show that there is really nothing therein which
expressly declares the abolition of the death penalty. The provision merely says that the death
penalty shall not be imposed unless for compelling reasons involving heinous crimes the Congress
hereafter provides for it and, if already imposed, shall be reduced to reclusion perpetua. The
language, while rather awkward, is still plain enough. And it is a settled rule of legal hermeneutics
that if the language under consideration is plain, it is neither necessary nor permissible to resort
to extrinsic aids, like the records of the constitutional convention, for its interpretation.

Sec. 19(1) does not change the periods of the penalty prescribed by Art. 248 of the RPC except
only insofar as it prohibits the imposition of the death penalty and reduces it to reclusion
perpetua. The range of the medium (reclusion perpetua) and minimum (reclusion temporal in
max period) penalties remains unchanged. The Court realizes that this interpretation may lead to
certain inequities that would not have arisen under Article 248 of the Revised Penal Code before
its modification. Thus, a person originally subject to the death penalty and another who
committed the murder without the attendance of any modifying circumstance will now be both
punishable with the same medium period although the former is concededly more guilty than the
latter. True enough. But that is the will not of this Court but of the Constitution.

E. Death Penalty

People v. Bon – 506 Issue: With the abolition of the imposition of death penalty, what penalties should be imposed?
SCRA 169
Ruling: Reclusion perpetua for the 6 counts of rape. The sentence of death imposed by the RTC
and affirmed by the CA can no longer be affirmed in view of RA 9346, titled “An Act Prohibiting
the Imposition of Death Penalty in the Philippines.” Sec. 2 of the law mandates that in lieu of the
death penalty, the penalty of reclusion perpetua shall be imposed. Correspondingly, the Court can
no longer uphold the death sentences imposed by lower courts, but must, if the guilt of the
accused is affirmed, impose instead the penalty of reclusion perpetua, or life imprisonment when
appropriate.

Prision mayor for the 2 counts of attempted rape. Since Art. 71 denominates “death” as an
element in the graduated scale of penalties, there is no question that the operation of Art. 71
involves the actual application of the death penalty as a means of determining the extent which a
person’s liberty is to be deprived. Since RA 9346 unequivocally bars the application of the death
penalty, as well as expressly repeals all such statutory provisions requiring the application of the
death penalty, such effect necessarily extends to its relevance to the graduated scale of penalties
under Art. 71. We cannot find basis to conclude that RA 9346 intended to retain the operative
effects of the death penalty in the graduation of the other penalties in our penal laws.

As to sentences not yet handed down, or affirmed with finality, the application is immediate.
Henceforth, “death,” as utilized in Art. 71 of the RPC, shall no longer form part of the equation in
the graduation of penalties. For example, in the case of appellant, the determination of his
penalty for attempted rape shall be reckoned not from 2 degrees lower than death, but 2 degrees
lower than reclusion perpetua. Hence, the maximum term of his penalty shall no longer be
reclusion temporal, as ruled by the CA, but instead, prision mayor.

Section 20.

A. Imprisonment for debt

Lozano v. Martinez - Issue: Whether BP 22 violates the Constitutional prohibition for imprisonment for debt?
146 SCRA 323
Ruling: NO. The enactment of BP 22 a valid exercise of the police power and is not repugnant to
the constitutional inhibition against imprisonment for debt.

The gravamen of the offense punished by BP 22 is the act of making and issuing a worthless
142
check or a check that is dishonored upon its presentation for payment. It is not the nonpayment
of an obligation which the law punishes. The law is not intended or designed to coerce a debtor to
pay his debt. The thrust of the law is to prohibit, under pain of penal sanctions, the making of
worthless checks and putting them in circulation. Because of its deleterious effects on the public
interest, the practice is proscribed by the law. The law punishes the act not as an offense against
property, but an offense against public order. The enactment of BP 22 is a declaration by the
legislature that, as a matter of public policy, the making and issuance of a worthless check is
deemed a public nuisance to be abated by the imposition of penal sanctions.

Section 21

A. Attachment of jeopardy

People v. Ylagan - 58 Issue: Whether the Ylagan was placed in double jeopardy?
Phil 851
Ruling: YES. Under Sec. 28 of the Code of Criminal Procedure, a defendant is in legal jeopardy
when placed on trial under the following conditions:

(1) In a court of competent jurisdiction;


(2) upon a valid complaint or information;
(3) after he has been arraigned; and
(4) after he has pleaded to the information.

Here, the counsel for the government, contends that the previous case (Serious physical injuries)
brought against the appellee was dismissed with her consent, on the theory that the phrase
"without the consent of the accused", used in Sec. 28 of the Code of Criminal Procedure, should
be construed to mean "over the objection of the accused" or "against the will of the accused". The
SC rebutted this by saying, the mere silence of the defendant or his failure to object to the
dismissal of the case does not constitute consent within the meaning of Sec. 28. The right not to
be put in jeopardy a second time for the offense is as important as the other constitutional right
of the accused in a criminal case. Its waiver cannot, and should not, be predicated on mere
silence.

People v. Balisacan - L Issue: Whether the appeal made by the prosecution placed the accused in double jeopardy?
26376, 17 SCRA 1119
Ruling: NO. The existence of a plea is an essential requisite to double jeopardy. In the present
case, it is true, the accused had first entered a plea of guilty. Subsequently, however, he testified,
in the course of being allowed to prove mitigating circumstances that he acted in complete self-
defense. Said testimony, therefore— as the court a quo recognized in its decision—had the effect
of vacating his plea of guilty and the court a quo should have required him to plead anew on the
charge, or at least direct that a new plea of not guilty be entered for him. This was not done. It
follows that in effect there having been no standing plea at the time the court a quo rendered its
judgment of acquittal, there can be no double jeopardy with respect to the appeal herein.

Furthermore, as afore-stated, the court a quo decided the case upon the merits without giving the
prosecution any opportunity to present its evidence or even to rebut the testimony of the
defendant. In doing so, it clearly acted without due process of law. And for lack of this
fundamental prerequisite, its action is perforce null and void. The acquittal, therefore, being a
nullity for want of due process, is no acquittal at all, and thus cannot constitute a proper basis for
a claim of former jeopardy.

Cudia v. Court of Issue: Whether Cudia was placed in double jeopardy?


Appeals - 284 SCRA
173 Ruling: NO. In order to successfully invoke the defense of double jeopardy, the following
requisites must be present:
1. a first jeopardy must have attached prior to the second;
2. the first jeopardy must have been validly terminated; and
3. the second jeopardy must be for the same offense or the second offense includes or
is necessarily included in the offense charged in the first information, or is an
attempt to commit the same or a frustration thereof.

In determining when the first jeopardy may be said to have attached, it is necessary to prove the
existence of the following:
1) Court of competent jurisdiction
2) Valid complaint or information
3) . Arraignment
4) Valid plea
5) The defendant was acquitted or convicted or the case was dismissed or otherwise
terminated without the express consent of the accused.

With respect to the second requisite, however, it is the Provincial Prosecutor of Pampanga, not
the City Prosecutor, who should prepare informations for offenses committed within Pampanga
but outside of Angeles City. As the fiscal had no authority to file the information, the dismissal of
the first information would not be a bar to petitioner’s subsequent prosecution. Jeopardy does not
attach where a defendant pleads guilty to a defective indictment that is voluntarily dismissed by
the prosecution. Jeopardy does not attach where a defendant pleads guilty to a defective
indictment that is voluntarily dismissed by the prosecution. As the first information was fatally
defective for lack of authority of the officer filing it, the instant petition must fail for failure to
143
comply with all the requisites necessary to invoke double jeopardy.

Lejano vs. People of Issue: Whether reconsideration based on the alleged Court’s appreciation of facts, treatment of
the Philippines , G.R. the evidence and prosecution witnesses violates the constitutional right of the accused against
No. 176389, 18 double jeopardy?
January 2011
Ruling: YES. To reconsider a judgment of acquittal places the accused twice in jeopardy of being
punished for the crime of which he has already been absolved. There is reason for this provision
of the Constitution. In criminal cases, the full power of the State is ranged against the accused. If
there is no limit to attempts to prosecute the accused for the same offense after he has been
acquitted, the infinite power and capacity of the State for a sustained and repeated litigation
would eventually overwhelm the accused in terms of resources, stamina, and the will to fight.

On occasions, a motion for reconsideration after an acquittal is possible. But the grounds are
exceptional and narrow as when the court that absolved the accused gravely abused its
discretion, resulting in loss of jurisdiction, or when a mistrial has occurred. In any of such cases,
the State may assail the decision by special civil action of certiorari under Rule 65.

Here, although complainant Vizconde invoked the exceptions, he has been unable to bring his
pleas for reconsideration under such exceptions. Private complainant wants the Court to review
the evidence anew and render another judgment based on such a re-evaluation. This is not
constitutionally allowed as it is merely a repeated attempt to secure Webb, et al.’s conviction. The
judgment acquitting Webb, et al. is final and can no longer be disturbed.

Ysidoro v. Leonardo-de Issue: Whether an appeal for acquittal of the accused is barred by double jeopardy?
Castro, G.R. No.
171513, February 6, Ruling: YES. As we explained in People v. Nazareno, the constitutional right of the accused
2012 against double jeopardy proscribes appeals of judgments of acquittal through the remedies of
ordinary appeal and a Rule 45 petition. The Constitution has expressly adopted the double
jeopardy policy and thus bars multiple criminal trials, thereby conclusively presuming that a
second trial would be unfair if the innocence of the accused has been confirmed by a previous
final judgment. Further prosecution via an appeal from a judgment of acquittal is likewise barred
because the government has already been afforded a complete opportunity to prove the criminal
defendant’s culpability; after failing to persuade the court to enter a final judgment of conviction,
the underlying reasons supporting the constitutional ban on multiple trials applies and becomes
compelling.

The second opportunity comes via an appeal does not make the effects any less prejudicial by the
standards of reason, justice and conscience.

The rule against double jeopardy cannot be properly invoked in a Rule 65 petition, predicated on
2 exceptional grounds, namely: in a judgment of acquittal rendered with grave abuse of
discretion by the court; and where the prosecution had been deprived of due process. The rule
against double jeopardy does not apply in these instances because a Rule 65 petition does not
involve a review of facts and law on the merits in the manner done in an appeal. In certiorari
proceedings, judicial review does not examine and assess the evidence of the parties nor weigh
the probative value of the evidence. It does not include an inquiry on the correctness of the
evaluation of the evidence. A review under Rule 65 only asks the question of whether there has
been a validly rendered decision, not the question of whether the decision is legally correct. In
other words, the focus of the review is to determine whether the judgment is per se void on
jurisdictional grounds.

Macapagal-Arroyo vs. Issue: Whether granting the motion for reconsideration on the dismissal of a criminal case would
People – G.R. No. violate the constitutional proscription against double jeopardy?
220598, 18 April 2017
Ruling: YES. The Court's consequential dismissal of Criminal Case No. SB-12- CRM-0174 as to
the petitioners for insufficiency of evidence amounted to their acquittal of the crime of plunder
charged against them. The demurrer to evidence in criminal cases, such as the one at bar, is
''filed after the prosecution had rested its case," and when the same is granted, it calls "for an
appreciation of the evidence adduced by the prosecution and its sufficiency to warrant conviction
beyond reasonable doubt, resulting in a dismissal of the case on the merits, tantamount to an
acquittal of the accused." Such dismissal of a criminal case by the grant of demurrer to evidence
may not be appealed, for to do so would be to place the accused in double jeopardy. The verdict
being one of acquittal, the case ends there.

The rule on double jeopardy, however, is not without exceptions. In People v. Laguio, Jr., this
Court stated that the only instance when double jeopardy will not attach is when the RTC acted
with grave abuse of discretion.

The constitutional prohibition against placing a person under double jeopardy for the same
offense bars not only a new and independent prosecution but also an appeal in the same action
after jeopardy had attached. As such, every acquittal becomes final immediately upon
promulgation and cannot be recalled for correction or amendment. With the acquittal being
immediately final, granting the State's motion for reconsideration in this case would violate the
Constitutional prohibition against double jeopardy because it would effectively reopen the
prosecution and subject the petitioners to a second jeopardy despite their acquittal.

It is cogent to remind in this regard that the Constitutional prohibition against double jeopardy
provides to the accused 3 related protections, specifically: protection against a second
prosecution for the same offense after acquittal; protection against a second prosecution for the
same offense after conviction; and protection against multiple punishments for the same offense.
144
B. Termination of jeopardy

Bulaong v. People - 14 Issue: Whether accused Bulaong can interpose the defense of double jeopardy in this case in
SCRA 746 view of the filing against him of the information for subversion which allegedly involves the same
facts obtaining in this case of rebellion?

Ruling: NO. Under Sec. 9, Rule 113 of the Rules of Court, the defense of double jeopardy is
available to the accused only where he was either convicted or acquitted or the case against him
was dismissed or otherwise terminated without his consent. Such is not the situation in this case.
For accused has not been convicted or acquitted in the case filed in the CFI Manila against him for
subversion. Neither was the said case dismissed or terminated without his consent, for as stated,
it is still pending in said court. Needless to say, it is the conviction, acquittal of the accused or
dismissal or termination of the case that bars further prosecution for the same offense or any
attempt to commit the same or frustration thereof, or for any offense which necessarily includes
or is necessarily included in the offense charged in the former complaint or information.

Bustamante v. Issue: Whether the withdrawal of the plea of guilty by the accused, could in law amount to a
Maceren - 48 SCRA 155 waiver against double jeopardy?

Ruling: NO. Under Sec. 10, Rule 117 of the Revised Rules of Court, the defense of double
jeopardy is waivable, and if not raised or set up at the proper time, is deemed waived, but the
same rule provides that “if, however, the defendant learns after he has pleaded or has moved to
quash on some other ground that the offense for which he is now charged is an offense for which
he has been pardoned, or of which he has been convicted or acquitted or been in jeopardy, the
court may in its discretion entertain at any time before judgment a motion to quash on the
ground of such pardon, conviction, acquittal or jeopardy."

Here, considering that defense counsel raised the question of double jeopardy in favor of
petitioner during the new trial and before Judge Maceren rendered judgment based on said new
trial, it is believed that the above principle can be applied to this case by analogy and that said
judge, in the exercise of his discretion, should have entertained said plea of double jeopardy in
the interest of justice, especially since at the time such plea was made, petitioner had already
fully served the one-year straight sentence imposed upon him by Judge Coquia on Dec. 14, 1970,
and was already entitled to be released from custody after such full service of his penalty under
said judgment."

People v. Obsania - 23 Issue: Whether the appeal for the dismissal of the criminal complaint violates the right of the
SCRA 1249 accused against double jeopardy?

Ruling: NO. In order that the protection against double jeopardy may inure in favor of an
accused, the following requisites must have obtained in the original prosecution: (a) a valid
complaint or information; (b) a competent court; (c) the defendant had pleaded to the charge;
and (d) the defendant was acquitted, or convicted, or the case against him was dismissed or
otherwise terminated without his express consent. The complaint filed with the municipal court in
the case at bar was valid; the court a quo was a competent tribunal with jurisdiction to hear the
case; the record shows that the accused pleaded not guilty upon arraignment. Hence, the only
remaining and decisive question is whether the dismissal of the case was without the express
consent of the accused.

The accused admits that the controverted dismissal was ordered by the trial judge upon his
motion to dismiss. The appealed order of dismissal in this case now under consideration did not
terminate the action on the merits, whereas in Cloribel and in the other related cases the
dismissal amounted to an acquittal because the failure to prosecute presupposed that the
Government did not have a case against the accused, who, in the first place, is presumed
innocent.

The application of the sister doctrines of waiver and estoppel requires two sine qua non
conditions: first, the dismissal must be sought or induced by the defendant personally or through
his counsel; and second, such dismissal must not be on the merits and must not necessarily
amount to an acquittal. Indubitably, the case at bar falls squarely within the periphery of the said
doctrines which have been preserved unimpaired in the corpus of our jurisprudence.

Rivera, Jr. v. People - Issue: Whether the verbal order of dismissal made in open court amounted to the acquittal of
189 SCRA 331 petitioner and which order is immediately final and executory which, the respondent Judge could
no longer set it aside without violating petitioner’s constitutional right against double jeopardy?

Ruling: NO. An order of dismissal must be written in the official language, personally and directly
prepared by the judge and signed by him conformably with the provisions of Rule 120 of the
Rules of Court.

Here, it is very clear that the order was merely dictated in open court by the trial judge. There is
no showing that this verbal order of dismissal was ever reduced to writing and duly signed by
him. Thus, it did not yet attain the effect of a judgment of acquittal, so that it was still within the
powers of the judge to set it aside and enter another order, now in writing and duly signed by
him, reinstating the case."

Cuison v. Court of Issue: Whether the promulgation of the new decision will violate the accused’s constitutional
Appeals - 289 SCRA right against double jeopardy?
145
159
Ruling: NO. As a rule, a criminal prosecution includes a civil action for the recovery of indemnity.
Hence, a decision in such case disposes of both the criminal as well as the civil liabilities of an
accused. Here, trial court promulgated only the civil aspect of the case, but not the criminal.

As earlier observed, the promulgation of the CA Decision was not complete. In fact and in truth,
the promulgation was not merely incomplete; it was also void. In excess of its jurisdiction, the
trial judge rendered a substantially incomplete promulgation on April 4, 1995, and he repeated
his mistake in his April 12, 1996 Order. We emphasize that grave abuse of discretion rendered
the aforementioned act of the trial court void. Since the criminal cases have not yet been
terminated, the first jeopardy has not yet attached. Hence, double jeopardy cannot prosper as a
defense. We must stress that Respondent Courts questioned Decision did not modify or amend its
July 30, 1991 Decision. It merely ordered the promulgation of the judgment of conviction and the
full execution of the penalty it had earlier imposed on petitioner.

Salcedo v. Mendoza - Issue: Whether the dismissal of the criminal case against him was equivalent to acquittal and
88 SCRA 811 reinstatement of the case will violate the constitutional right of the accused against double
jeopardy?

Ruling: YES. The dismissal of a criminal case predicated on the right of the accused to speedy
trial, amounts to an acquittal on the merits which bars the subsequent prosecution of the accused
for the same offense.

Here, the respondent Judge dismissed the case, upon the motion of the petitioner invoking his
constitutional right to speedy trial, because the prosecution failed to appear on the day of the trial
after it had previously been postponed twice.

The effect of such dismissal is at once clear following the established jurisprudence, a dismissal
predicated on the right of the accused to speedy trial upon his own motion or express consent,
amounts to an acquittal which will bar another prosecution of the accused for the same offense.
This is an exception to the rule that a dismissal upon the motion or with the express consent of
the accused win not be a bar to the subsequent prosecution of the accused for the same offense
as provided for in Sec. 9, Rule 113 of the Rules of Court.

Oriente vs. Peo - 513 Issue: Whether the act of the RTC in setting aside its first decision and promulgate another
SCRA 348 decision modifying/increasing the penalty of the crime placed the accused in double jeopardy?

Ruling: NO. Courts have the inherent power to amend their decisions to make them conformable
to law and justice. This prerogative, however, is not absolute. The rules do not contemplate
amendments that are substantial in nature. They merely cover formal changes or such that will
not affect the crux of the decision, like the correction of typographical or clerical errors. Courts
will violate due process if they make substantial amendments in their decisions without affording
the other party the right to contest the new evidence presented in a motion for reconsideration.

Here, the change in the penalty by the RTC did not involve the consideration of any new evidence
but a mere correction of the penalty imposed to conform with the RPC and ISLaw.

The trial court modified the penalty in its Decision before the petitioner could perfect his appeal
from the first Decision. Noteworthy is that it was the RTCs second Decision which the petitioner
elevated on appeal to the CA. It is well settled that when an accused appeals from the sentence
of the trial court, he waives the constitutional safeguard against double jeopardy, and, as
discussed above, throws the whole case open to the review of the appellate court, which is then
called to render judgment as the law and justice dictate, whether favorable or unfavorable, and
whether they are made the subject of assigned errors or not.

Pacoy vs. Cajigal - 534 Issue: Whether the amendment of the information from Homicide to Murder placed the accused
SCRA 338 in double jeopardy?

Ruling: NO. The change of the offense charged from Homicide to Murder is merely a formal
amendment and not a substantial amendment or a substitution.

Dismissal of the first case contemplated presupposes a definite or unconditional dismissal which
terminates the case. And for the dismissal to be a bar under the jeopardy clause, it must have the
effect of acquittal. The respondent judge's Order was for the trial prosecutor to correct and
amend the Information but not to dismiss the same upon the filing of a new Information charging
the proper offense. Homicide is necessarily included in the crime of murder; thus, the respondent
judge merely ordered the amendment of the Information and not the dismissal of the original
Information. To repeat, it was the same original information that was amended by merely
crossing out the word “Homicide” and writing the word “Murder,” instead, which showed that
there was no dismissal of the homicide case. In addition, a reading of the Order showed that the
respondent judge granted petitioner's MR, not on the ground that double jeopardy exists, but on
his realization that “disregard of rank” is a generic aggravating circumstance which does not
qualify the killing of the victim to murder. Thus, he rightly corrected himself by reinstating the
original Information for Homicide. The requisite of double jeopardy that the first jeopardy must
have attached prior to the second is not present, considering that petitioner was neither convicted
nor acquitted; nor was the case against him dismissed or otherwise terminated without his
express consent.

C. Same offense; ordinance and states


146
People v. Relova - 148 Issue: Whether the filing of the second information against the accused for the crime of theft of
SCRA 292 electric power violates his right against double jeopardy since the first case for violation of
unauthorised wiring installations had been dismissed?

Ruling: YES. A person who was charged for violating a city ordinance for having installed a
metering device to lower his electric bills which was dismissed for prescription of the offense may
not be charged again for theft of electric power under the RPC. The second sentence of Sec. 21
embodies an exception to the general proposition: the constitutional protection, against double
jeopardy is available although the prior offense charged under an ordinance be different from the
offense charged subsequently under a national statute such as the RPC, provided that both
offenses spring from the same act or set of acts. Where an offense is punished by different
sections of a statute or different statutes, the inquiry, for purposes of double jeopardy is on the
identity of offenses charged BUT where an offense is penalized by an ordinance and a statute, the
inquiry is on the identity of acts. Since the dismissal of the case against Opulencia for violation of
an ordinance already amounted to an acquittal, he can no longer charged with an offense
punishable under a statute arising from the same act.

People v. City Court of Issue: Whether the two informations (1. Violation of RA 3060; 2. Art. 201 of RPC) constitutes the
Manila - 154 SCRA 175 same offense thus violates the constitutional prohibition against double jeopardy?

Ruling: NO. The 2 informations with which the accused was charged, do not make out only one
offense. The 2 offenses are different and distinct from each other. The gravamen of the offense
defined in RA 3060 is the public exhibition of any motion picture which has not been previously
passed by the Board of Censors for Motion Pictures. The motion picture may not be indecent or
immoral but if it has not been previously approved by the Board, its public showing constitutes a
criminal offense. On the other hand, the offense punished in Art. 201(3) of the RPC is the public
showing of indecent or immoral plays, scenes, acts, or shows, not just motion pictures.

The nature of both offenses also shows their essential difference. The crime punished in RA 3060
is a malum prohibitum in which criminal intent need not be proved because it is presumed, while
the offense punished in Art. 201 (3) of the RPC is malum in se, in which criminal intent is an
indispensable ingredient.

Considering these differences in elements and nature, there is no Identity of the offenses here
involved for which legal jeopardy in one may be invoked in the other.

Evidence required to prove one offense is not the same evidence required to prove the other. The
defense of double jeopardy cannot prosper. Where 2 different laws (or articles of the same code)
define 2 crimes, prior jeopardy as to one of them is no obstacle to a prosecution of the other,
although both offenses arise from the same facts, if each crime involves some important act
which is not an essential element of the other.

D. Rule on "supervening facts"

Melo v. People - 85 Issue: Whether the filing of the amended information of consummated homicide against the
Phil 766 accused violated the constitutional prohibition against double jeopardy?

Ruling: NO. Where after the first prosecution a new fact supervenes for which the defendant is
responsible, which changes the character of the offense and, together with the facts existing at
the time, constitutes a new and distinct offense”, the accused cannot be said to be in second
jeopardy if indicted for the new offense.

Here, supervening fact is when Obillo died from his wounds on the same day of Melo’s
arraignment. Hence, the amendment of the information from Frustrated to Consummated
Homicide did not violate the prohibition against double jeopardy.

People v. Buling - 107 Issue: Whether the prosecution and conviction of the accused for less serious physical injuries a
Phil 712 bar to the second prosecution for serious physical injuries?

Ruling: YES. The rule is that "where after the first prosecution a new fact supervenes for which
the defendant is responsible, which changes the character of the offense and, together with he
facts existing at the time, constitutes a new and distinct offense", the accused cannot be said to
be in second jeopardy if indicted for the new offense.”

Here, the Court does not believe that a new fact supervened, or that a new fact has come into
existence. Where the exact nature of the injury could have been discovered, but was not, because
of the incompetence of the physician; the subsequent discovery of the real extent of the injury
would not be a supervening act. If the X-ray examination discloses the existence of a fracture on
January 17, 1957, that fracture must have existed when the first examination was made on
December 10, 1956. Therefore, no new or supervening fact that could be said to have developed
or arisen since the filing of the original action. The wound causing the delay in healing was
already in existence at the time of the first examination, but said delay was caused by the very
superficial examination then made. Had an X-ray examination taken at the time, the fracture
would have certainly been disclosed.

Section 22

147
A. Ex Post Facto Laws

People v. Jabinal - 55 Issue: Whether Jabinal should be convicted in view of the complete reversal of the Macarandang
SCRA 602 and Lucero doctrine prevailing at the time when he was appointed as Secret Agent, in Mapa case?

Ruling: NO. When appellant was appointed Secret Agent by the Provincial Government in 1962,
and Confidential Agent by the Provincial Commander in 1964, the prevailing doctrine on the
matter was that laid down by Us in People v. Macarandang (1959) and People v. Lucero (1958).
Our decision in People v. Mapa reversing the aforesaid doctrine came only in 1967.

Lucero and Macarandang was part of the jurisprudence, hence of the law, of the land, at the time
appellant was found in possession of the firearm in question and when he arraigned by the trial
court. It is true that the doctrine was overruled in the Mapa case in 1967, but when a doctrine of
this Court is overruled and a different view is adopted, the new doctrine should be applied
prospectively, and should not apply to parties who had relied on the old doctrine and acted on the
faith thereof.

Considering that appellant conferred his appointments as Secret Agent and Confidential Agent
and authorized to possess a firearm pursuant to the prevailing doctrine enunciated in
Macarandang and Lucero, under which no criminal liability would attach to his possession of said
firearm in spite of the absence of a license and permit therefor, appellant must be absolved.
Certainly, appellant may not be punished for an act which at the time it was done was held not to
be punishable.

B. Bills of Attainder

People v. Ferrer - 48 Issue: Whether the Anti-Subversion Act is a bill of attainder?


SCRA 382
Ruling: NO. A bill of attainder is a legislative act which inflicts punishment without trial. Its
essence is the substitution of a legislative for a judicial determination of guilt. The constitutional
ban against bills of attainder serves to implement the principle of separation of powers by
confining legislatures to rule-making and thereby forestalling legislative usurpation of the judicial
function.

The singling out of a definite class, the imposition of a burden on it, and a legislative intent,
suffice to stigmatize a statute as a bill of attainder.

Here, the Act applies not only to the Communist Party of the Philippines but also to "any other
organization having the same purpose and their successors." Its focus is not on individuals but on
conduct. Even assuming, however, that the Act specifies individuals and not activities, this feature
is not enough to render it a bill of attainder.

Indeed, were the Anti-Subversion Act a bill of attainder, it would be totally unnecessary to charge
Communists in court, as the law alone, without more, would suffice to secure their punishment.
But the undeniable fact is that their guilt still has to be judicially established. The contention
would be correct if the statute were construed as punishing mere membership devoid of any
specific intent to further the unlawful goals of the Party. But the statute specifically required that
membership must be knowing or active, with specific intent to further the illegal objectives of the
Party. The ingredient of specific intent to pursue the unlawful goals of the Party must be shown
by "overt acts."

People v. Issue: Whether the crime charged against the accused has already prescribed?
Sandiganbayan - 211
SCRA 241 Ruling: YES. BP 195 may not be given retroactive application to the "crime" which was
committed by Paredes in Jan. 1976 yet, for it should be prejudicial to the accused. It would
deprive him of the substantive benefit of the shorter (10 years) prescriptive period under Sec. 11,
RA 3019, which was an essential element of the "crime" at the time he committed it. To apply BP
195 to Paredes would make it an ex post facto law for it would after his situation to his
disadvantage by making him criminally liable for a crime that had already been extinguished
under the law existing when it was committed. Since an ex post facto law is proscribed by our
Constitution, the Sandiganbayan committed no reversible error in ruling that Paredes may no
longer be prosecuted for his supposed violation of RA 3019 in 1976, 6 years before BP 195 was
approved on March 16, 1982. The new prescriptive period under that law should apply only to
those offense which were committed after the approval of BP 195.

ELECTION LAW

ARTICLE V SUFFRAGE

A. Citizenship Qualification

Tan v. Crisologo Issue: Whether Tan can be considered a Filipino citizen at the time she registered as a voter?

148
Ruling: NO. Only Filipino citizens are qualified to vote and may be included in the
permanent list of voters.

Here, Tan filed her voter’s registration application on Oct. 26, 2009, and that she only took her
Oath of Allegiance to the Republic on Nov. 30, 2009, or more than a month after the ERB
approved her application. Thus, Tan is not a Filipino citizen at the time she registered as a voter.

The phrase in Sec. 2 of RA 9225 providing “deemed to have not lost their PH citizenship” does not
mean that once PH citizenship is reacquired after taking the Oath of Allegiance required in RA
9225, the effect on the citizenship status retroacts to the period before taking said oath.

Once PH citizenship is renounced because of naturalisation in a foreign country, we cannot


consider one a Filipino citizen unless and until his or her allegiance to the Republic is reaffirmed.

Here, Tan took an Oath of Allegiance to the USA on Jan. 19, 1993, prior to the enactment of RA
9225 on Aug. 29, 2003. If we were to effect as retroactive Tan’s PH citizenship to the date she
lost her PH citizenship, then the different use of the words “reacquire” and “retain” in RA 9225
would effectively be futile.

B. Residence Qualification

Romualdez v. RTC Issue: Whether Romualdez has met the one-year residency requirement thus entitled to vote?

Ruling: YES. Romualdez is entitled to vote. In election cases, the Court treats domicile and
residence as synonymous terms, thus: "the term "residence" as used in the election law is
synonymous with "domicile", which imports not only an intention to reside in a fixed place but
also personal presence in that place, coupled with conduct indicative of such intention."
"Domicile" denotes a fixed permanent residence to which when absent for business or pleasure,
or for like reasons, one intends to return. That residence, in the case of the petitioner, was
established during the early 1980's. Residence thus acquired, however, may be lost by adopting
another choice of domicile. In order, in turn, to acquire a new domicile by choice, there must
concur (1) residence or bodily presence in the new locality, (2) an intention to remain there, and
(3) an intention to abandon the old domicile. In other words, there must basically be animus
manendi coupled with animus non revertendi. The purpose to remain in or at the domicile of
choice must be for an indefinite period of time; the change of residence must be voluntary; and
the residence at the place chosen for the new domicile must be actual.

The political situation brought about by the "People's Power Revolution" must have truly caused
great apprehension to the Romualdezes, as well as a serious concern over the safety and welfare
of the members of their families. Their going into self-exile until conditions favorable to them
would have somehow stabilized is understandable. Certainly, their sudden departure from the
country cannot be described as "voluntary," or as "abandonment of residence" at least in the
context that these terms are used in applying the concept of "domicile by choice." We have
closely examined the records, and we find not that much to convince us that the petitioner had, in
fact, abandoned his residence in the Philippines and established his domicile elsewhere.

C. Biometrics

Kabataan Party List v. Issue: Whether a mandatory biometrics registration system violates the right of suffrage by the
COMELEC petitioners?

Ruling: NO. The right to vote is not a natural right but is a right created by law. Suffrage is a
privilege granted by the State to such persons or classes as are most likely to exercise it for the
public good. The State may therefore regulate said right by imposing statutory disqualifications,
with the restriction, however, that the same do not amount to, as per the second sentence of the
provision, a “literacy, property or other substantive requirement.”

Registration regulates the exercise of the right of suffrage. It is not a qualification for such right.
As a form of regulation, compliance with the registration procedure is dutifully enjoined. The
exercise of the right of suffrage, as in the enjoyment of all other rights, is subject to existing
substantive and procedural requirements embodied in our Constitution, statute books and other
repositories of law.

The act of registration is an indispensable precondition to the right of suffrage. For registration is
part and parcel of the right to vote and an indispensable element in the election process. Thus,
contrary to petitioners' argument, registration cannot and should not be denigrated to the lowly
stature of a mere statutory requirement. Proceeding from the significance of registration as a
necessary requisite to the right to vote, the State undoubtedly, in the exercise of its inherent
police power, may then enact laws to safeguard and regulate the act of voter's registration for the
ultimate purpose of conducting honest, orderly and peaceful election, to the incidental yet
generally important end, that even pre-election activities could be performed by the duly
constituted authorities in a realistic and orderly manner—one which is not indifferent, and so far
removed from the pressing order of the day and the prevalent circumstances of the times.

Here, respondents have shown that the biometrics validation requirement under RA 10367
advances a compelling state interest. It was precisely designed to facilitate the conduct of
orderly, honest, and credible elections by containing—if not eliminating, the perennial problem of
having flying voters, as well as dead and multiple registrants. The objective of the law was to
149
cleanse the national voter registry so as to eliminate electoral fraud and ensure that the results of
the elections were truly reflective of the genuine will of the people. The foregoing consideration is
unquestionably a compelling state interest.

Also, it was shown that the regulation is the least restrictive means for achieving the above-said
interest. Sec. 6 of Resolution 9721 sets the procedure for biometrics validation, whereby the
registered voter is only required to: (a) personally appear before the Office of the Election Officer;
(b) present a competent evidence of identity; and (c) have his photo, signature, and fingerprints
recorded. While registrants may be inconvenienced by waiting in long lines or by not being
accommodated on certain days due to heavy volume of work, these are typical burdens of voting
that are remedied by bureaucratic improvements to be implemented by the COMELEC as an
administrative institution. By and large, the COMELEC has not turned a blind eye to these
realities. It has tried to account for the exigencies by holding continuous registration as early as
May 6, 2014 until Oct. 31, 2015, or for over a period of 18 months. To make the validation
process as convenient as possible, the COMELEC even went to the extent of setting up off-site
and satellite biometrics registration in shopping malls and conducted the same on Sundays.
Moreover, it deserves mentioning that RA 10367 and Resolution 9721 did not mandate registered
voters to submit themselves to validation every time there is an election. In fact, it only required
the voter to undergo the validation process 1 time, which shall remain effective in succeeding
elections, provided that he remains an active voter. To add, the failure to validate did not
preclude deactivated voters from exercising their right to vote in the succeeding elections. To
rectify such status, they could still apply for reactivation following the procedure laid down in Sec.
28 of RA 8189.

That being said, the assailed regulation on the right to suffrage was sufficiently justified as it was
indeed narrowly tailored to achieve the compelling state interest of establishing a clean,
complete, permanent and updated list of voters, and was demonstrably the least restrictive
means in promoting that interest.

POLITICAL PARTIES

Liberal Party, etc. et al. Issue: Whether NP-NPC’s petition for registration as a coalition and accreditation as dominant
vs. COMELEC minority party should be denied on the ground that NP-NPC was not duly registered coalition of
political parties at the time of filing of their petition?

Ruling: YES. The freedom to coalesce or to work together in an election to secure the vote for
chosen candidates is different from the formal recognition the Constitution requires for a political
party, organization or coalition to be entitled to full and meaningful participation in the elections
and to the benefits that proceed from formal recognition. Political coalitions need to register in
accordance with the established norms and procedures, if they are to be recognized as such and
be given the benefits accorded by law to registered coalitions. Registered political parties carry a
different legal personality from that of the coalition they may wish to establish with other similarly
registered parties.

Here, the NP-PNC was not duly registered coalition of political party because their petition was
filed beyond the deadline which is on February 12, 2010. Hence, the petition for registration as a
coalition and petition for accreditation as dominant minority party should be denied.

PARTY LIST

Lokin, Jr. vs. COMELEC Issue: Whether Sec. 13 of COMELEC Resolution which provides for the withdrawal of nomination
by the party as a party-list nominee after the submission of list to the COMELEC is valid?

Ruling: NO. The Legislature thereby deprived the party-list organization of the right to change its
nominees or to alter the order of nominees once the list is submitted to the COMELEC, except
when: (a) the nominee dies; (b) the nominee withdraws in writing his nomination; or (c) the
nominee becomes incapacitated. The enumeration is exclusive, for, necessarily, the general rule
applies to all cases not falling under any of the three exceptions.

Sec. 13 of Resolution 7804 expanded the exceptions under Sec. 8 of RA 7941. It states “a party-
list nominee may be substituted only when he dies, or his nomination is withdrawn by the party,
or he becomes incapacitated to continue as such, or he withdraws his acceptance to a
nomination. In any of these cases, the name of the substitute nominee shall be placed last in the
list of nominees.” No substitution shall be allowed by reason of withdrawal after the polls. The
insertion of the new ground was invalid. The COMELEC, despite its role as the implementing arm
of the Government in the enforcement and administration of all laws and regulations relative to
the conduct of an election, has neither the authority nor the license to expand, extend, or add
anything to the law it seeks to implement thereby.

Milagros E. Amores vs. Issues: 1. Whether private respondent who is more than 30 years old on election day is qualified
HRET and Emmanuel to be a youth sector nominee of CIBAC?
Joel J. Villanueva
2. Whether private respondent who shifts from CIBAC’s youth sector to its OFWs and their
families sector within 2 months prior to election will be eligible for nomination under the new
sectoral affiliation?

Ruling:
1. NO. As the law states in unequivocal terms that a nominee of the youth sector must at least be
150
25 but not more than 30 years of age on the day of the election, so it must be that a candidate
who is more than 30 on election day is not qualified to be a youth sector nominee. Since this
mandate is contained in RA 7941, the Party-List System Act, it covers ALL youth sector nominees
vying for party-list representative seats. Here, private respondent was already 31 years old at the
time of election. Hence, he is disqualified to be a nominee of the youth.

2. NO. Sec. 15 of the Party-list System Act states that “any elected party-list representative who
changes his political party or sectoral affiliation during his term of office shall forfeit his seat,
provided, that if he changes his political party or sectoral affiliation within 6 months before an
election, he shall not be eligible for nomination as party-list representative under his new party or
organization”.

What is clear is that the wording of Sec. 15 covers changes in both political party and sectoral
affiliation. And the latter may occur within the same party since multi-sectoral party-list
organizations are qualified to participate in the Philippine party-list system. Hence, a nominee
who changes his sectoral affiliation within the same party will only be eligible for nomination
under the new sectoral affiliation if the change has been effected at least six months before the
elections.

Here, private respondent did not change his sectoral affiliation at least 6 months before May,
2007, public respondent itself having found that he shifted to CIBACs OFWs and their families
sector only on March 17, 2007.

Philippine Guardians Issue: Whether the petitioner’s delisting from the roster of registered national, regional or
Brotherhood, Inc. sectoral parties, organizations or coalitions under the party list on the ground that it failed to get
(PGBI), etc. vs. 2% of the votes cast in 2004 and did not participate in 2007 elections is valid?
COMELEC
Ruling: NO. The law is clear the COMELEC may motu proprio or upon verified complaint of any
interested party, remove or cancel, after due notice and hearing, the registration of any national,
regional or sectoral party, organization or coalition if it: (a) fails to participate in the last two (2)
preceding elections; or (b) fails to obtain at least 2% of the votes cast under the party-list system
in the 2 preceding elections for the constituency in which it has registered. The word “or” is a
disjunctive term signifying disassociation and independence of one thing from the other things
enumerated; it should, as a rule, be construed in the sense in which it ordinarily implies, as a
disjunctive word. Thus, the plain, clear and unmistakable language of the law provides for 2
separate reasons for delisting. It cannot be concluded that the non-participation of a party-list
organization is similar to a failure to garner 2% threshold party-list vote.

Sec. 6 (8) of Party-list System Act provides for two separate grounds for delisting; these grounds
cannot be mixed or combined to support delisting; and (b) the disqualification for failure to garner
2% party-list votes in 2 preceding elections should now be understood, in light of the Banat
ruling, that party-list groups or organizations garnering less than 2% of the party-list votes may
yet qualify for a seat in the allocation of additional seats, to mean failure to qualify for a party-list
seat in 2 preceding elections for the constituency in which it has registered. This, we declare, is
how law should be understood and applied. We are aware that PGBIs situation a party list group
or organization that failed to garner 2% in a prior election and immediately thereafter did not
participate in the preceding election is something that is not covered by Sec. 6(8) of RA 7941.

Atong Paglaum v. issue: Whether petitioner’s disqualification to participate in a party-list elections on the ground
COMELEC that as a political or regional parties they are not organized along sectoral lines and do not
represent the marginalized and underrepresented is valid?

Ruling: NO. In determining who may participate in a party-list elections, the COMELEC shall
adhere to the following parameters:

1. 3 different groups may participate in the party-list system: (1) national parties or
organizations, (2) regional parties or organizations, and (3) sectoral parties or organizations.

2. National parties or organizations and regional parties or organizations do not need to organize
along sectoral lines and do not need to represent any "marginalized and underrepresented"
sector.

3. Political parties can participate in party-list elections provided they register under the party-list
system and do not field candidates in legislative district elections. A political party, whether major
or not, that fields candidates in legislative district elections can participate in party-list elections
only through its sectoral wing that can separately register under the party-list system. The
sectoral wing is by itself an independent sectoral party, and is linked to a political party through a
coalition.

4. Sectoral parties or organizations may either be "marginalized and underrepresented" or lacking


in "well-defined political constituencies." It is enough that their principal advocacy pertains to the
special interest and concerns of their sector. The sectors that are "marginalized and
underrepresented" include labor, peasant, fisherfolk, urban poor, indigenous cultural
communities, handicapped, veterans, and overseas workers. The sectors that lack "well-defined
political constituencies" include professionals, the elderly, women, and the youth.

5. A majority of the members of sectoral parties or organizations that represent the "marginalized
and underrepresented" must belong to the "marginalized and underrepresented" sector they
represent. Similarly, a majority of the members of sectoral parties or organizations that lack
"well-defined political constituencies" must belong to the sector they represent. The nominees of
sectoral parties or organizations that represent the "marginalized and underrepresented," or that
151
represent those who lack "well-defined political constituencies," either must belong to their
respective sectors, or must have a track record of advocacy for their respective sectors. The
nominees of national and regional parties or organizations must be bona-fide members of such
parties or organizations.

6. National, regional, and sectoral parties or organizations shall not be disqualified if some of their
nominees are disqualified, provided that they have at least one nominee who remains qualified.

The COMELEC excluded from participating in the 13 May 2013 party-list elections those that did
not satisfy these 2 criteria: (1) all national, regional, and sectoral groups or organizations must
represent the "marginalized and underrepresented" sectors, and (2) all nominees must belong to
the "marginalized and underrepresented" sector they represent. Petitioners may have been
disqualified by the COMELEC because as political or regional parties they are not organized along
sectoral lines and do not represent the "marginalized and underrepresented." Also, petitioners'
nominees who do not belong to the sectors they represent may have been disqualified, although
they may have a track record of advocacy for their sectors. Likewise, nominees of nonsectoral
parties may have been disqualified because they do not belong to any sector. Moreover, a party
may have been disqualified because one or more of its nominees failed to qualify, even if the
party has at least one remaining qualified nominee. As discussed above, the disqualification of
petitioners, and their nominees, under such circumstances is contrary to the 1987 Constitution
and RA 7941.

DISQUALIFICATION AND PETITION TO DENY DUE COURSE TO AND CANCEL OF CANDIDACY

Jalosjos vs. COMELEC Issue: Whether the cancellation motu proprio of Certificate of Candidacy of Jalosjos as a
mayoralty candidate for Zamboanga City on the ground of conviction of 2 counts of rape and 6
counts of acts of lasciviousness which carried the accessory penalty of perpetual absolute
disqualification is valid?

Ruling: YES. The COMELEC is under a legal duty to cancel the certificate of candidacy of anyone
suffering from the accessory penalty of perpetual special disqualification to run for public office by
virtue of a final judgment of conviction. The final judgment of conviction is notice to the COMELEC
of the disqualification of the convict from running for public office. The law itself bars the convict
from running for public office, and the disqualification is part of the final judgment of conviction.
The final judgment of the court is addressed not only to the Executive branch, but also to other
government agencies tasked to implement the final judgment under the law.

While Sec. 40(a) of the LGC allows a prior convict to run for local elective office after the lapse of
2 years from the time he serves his sentence, the said provision should not be deemed to cover
cases wherein the law imposes a penalty, either as principal or accessory, which has the effect of
disqualifying the convict to run for elective office. In this relation, Art. 30 of the RPC provides that
the penalty of perpetual absolute disqualification has the effect of depriving the convicted felon of
the privilege to run for elective office.

Here, petitioner was sentenced to suffer the principal penalties of reclusion perpetua and
reclusion temporal which, pursuant to Article 41 of the RPC, carried with it the accessory penalty
of perpetual absolute disqualification and in turn, pursuant to Art. 30 of the RPC, disqualified him
to run for elective office. As discussed, Sec. 40(a) of the LGC would not apply to cases wherein a
penal provision – such as Art. 41 in this case – directly and specifically prohibits the convict from
running for elective office. Hence, despite the lapse of 2 years from petitioner’s service of his
commuted prison term, he remains bound to suffer the accessory penalty of perpetual absolute
disqualification which consequently, disqualifies him to run as mayor for Zamboanga City.

Sergio G. Amora, Jr. Issue: Whether it is proper to disqualify the petitioner who, in executing his COC, merely
vs. Commission on presented to the Notary Public his Community Tax Certificate?
Elections
Ruling: NO. A defective notarization of COC is not a ground for disqualification.

Sec. 68 of the Omnibus Election code provides the following ground for disqualification: If a party
is declared by final decision of a competent court guilty of, or found by the Commission of having:
1. Given money or other material consideration to influence, induce or corrupt the voters or
public officials performing electoral functions;
2. Committed acts of terrorism to enhance his candidacy;
3. Spent in his election campaign an amount in excess of that allowed by this Code;
4. Solicited, received or made any contribution prohibited under Sections 89, 95, 96, 97 and 104;
or
5. Violated any of Sections 80, 83, 85, 86, and 261, paragraphs d, e, k, v, and cc, subparagraph
6. It is quite obvious that the Olandria petition is not based on any of the grounds for
disqualification as enumerated in the foregoing statutory provisions.

Nowhere therein does it specify that a defective notarization is a ground for the disqualification of
a candidate. Yet, the COMELEC would uphold that petition upon the outlandish claim that it is a
petition to disqualify a candidate for lack of qualifications or possessing some grounds for
disqualification.

Sec. 40 of the LGC also provides the following grounds for disqualification:
1. Those sentenced by final judgment for an offense involving moral turpitude or for an offense
punishable by 1 year or more of imprisonment, within 2 years after serving sentence;
2. Those removed from office as a result of an administrative case;
3. Those convicted by final judgment for violating the oath of allegiance to the Republic;
152
4. Those with dual citizenship;
5. Fugitives from justice in criminal or nonpolitical cases here or abroad;
6. Permanent residents in a foreign country or those who have acquired the right to reside abroad
and continue to avail of the same right after the effectivity of this Code; and
7. The insane or feeble-minded.

Here, it is quite obvious that the Olandria petition is not based on any of the grounds for
disqualification as enumerated in the foregoing statutory provisions. Nowhere therein does it
specify that a defective notarization is a ground for the disqualification of a candidate. Yet, the
COMELEC would uphold that petition upon the outlandish claim that it is a petition to disqualify a
candidate for lack of qualifications or possessing some grounds for disqualification.

Amora complied with the requirement of a sworn COC. He readily explained that he and Atty.
Granada personally knew each other; they were not just colleagues at the League of Municipal
Mayors, Bohol Chapter, but they consider each other as distant relatives. Thus, the alleged defect
in the oath was not proven by Olandria since the presentation of a CTC turned out to be sufficient
in this instance. On the whole, the COMELEC should not have brushed aside the affidavit of Atty.
Granada and remained inflexible in the face of Amoras victory and proclamation as Mayor of
Candijay, Bohol.

Fernando V. Gonzalez Issue: Whether the petition for disqualification and cancellation of COC for the position of
v. Commission on Representative of 3rd district of the Province of Albay filed by Lim against the petitioner was filed
Elections, et al. on time?

Ruling: NO. In order to justify the cancellation of COC, it is essential that the false representation
mentioned therein pertain to a material matter for the sanction imposed by Sec. 78 of the
Omnibus Election Code would affect the substantive rights of a candidate the right to run for the
elective post for which he filed the certificate of candidacy. Although the law does not specify
what would be considered as a material representation, the Court concluded that this refers to
qualifications for elective office. Sec. 78 contemplates statements regarding age, residence and
citizenship or nonpossession of natural-born Filipino status. Furthermore, aside from the
requirement of materiality, the false representation must consist of a deliberate attempt to
mislead, misinform, or hide a fact which would otherwise render a candidate ineligible. In other
words, it must be made with an intention to deceive the electorate as to ones qualification for
public office.

The only instance where a petition questioning the qualifications of a candidate for elective office
can be filed before election is when the petition is filed under Sec. 78 of the OEC. The petition
based on the allegation that Gonzalez was not a natural-born Filipino which was filed before the
elections, is in the nature of a petition filed under Sec. 78. The recitals in the petition in said case,
however, state that it was filed pursuant to Sec. 4 (b) of COMELEC Resolution No. 8696 and Sec.
68 of the OEC to disqualify a candidate for lack of qualifications or possessing some grounds for
disqualification. The COMELEC treated the petition as one filed both for disqualification and
cancellation of COC, with the effect that Sec. 68, in relation to Sec. 3, Rule 25 of the COMELEC
Rules of Procedure, is applicable insofar as determining the period for filing the petition.

Sec. 4(B) of Resolution No. 8696 allowing a petition to disqualify a candidate based on his lack of
qualifications for elective office such as age, residence and citizenship to be filed on any day after
the last day for filing of certificates of candidacy but not later than the date of proclamation (the
period provided in Sec. 68 of the OEC), instead of the period for filing under Sec. 78 (not later
than 25 days from the filing of the certificate of candidacy) is similar to Rule 25 of the COMELEC
Rules of Procedure.

We find that Sec. 4(B) of Resolution No. 8696 represents another attempt to modify by a mere
procedural rule the statutory period for filing a petition to cancel COC on the ground of false
representation therein regarding a candidates qualifications. Like Rule 25 of the COMELEC Rules
of Procedure, Sec. 4(B) of Resolution No. 8696 would supplant the prescribed period of filing of
petition under Section 78 with that provided in Section 68 even if the latter provision does not at
all cover the false representation regarding age, residence and citizenship which may be raised in
a petition under Section 78.

Since the petition in SPA No. 10-074 (DC) sought to cancel the COC filed by Gonzalez and
disqualify him as a candidate on the ground of false representation as to his citizenship, the same
should have been filed within 25 days from the filing of the COC, pursuant to Sec. 78 of the OEC.
Gonzales filed his COC on December 1, 2009. Clearly, the petition for disqualification and
cancellation of COC filed by Lim on March 30, 2010 was filed out of time.

Villafuerte v. Comelec Issue: Whether the use of respondents’ nickname “LRAY JR. MIGS” in his COC constitutes a
material misrepresentation that would warrant the cancellation of his certificate of candidacy for
the Gubernatorial position of the province of Camarines Sur?

Ruling: NO. In order to justify the cancellation of the certificate of candidacy under Sec. 78, it is
essential that the false representation mentioned therein pertains to a material matter for the
sanction imposed by this provision would affect the substantive rights of a candidate — the right
to run for the elective post for which he filed the certificate of candidacy.

Aside from the requirement of materiality, a false representation under Sec. 78 must consist of a
"deliberate attempt to mislead, misinform, or hide a fact which would otherwise render a
candidate ineligible." In other words, it must be made with an intention to deceive the electorate
as to one’s qualifications for public office. The use of surname, when not intended to mislead, or
deceive the public as to one's identity is not within the scope of the provision.

153
Clearly, from the foregoing, for the petition to deny due course or cancel the COC of one
candidate to prosper, the candidate must have made a material misrepresentation involving his
eligibility or qualification for the office to which he seeks election, such as the requisite residency,
age, citizenship or any other legal qualification necessary to run for local elective office as
provided in the LGC. Hence, petitioner’s allegation that respondent’s nickname "LRAY JR. MIGZ"
written in his COC is a material misrepresentation is devoid of merit. Respondent's nickname
written in the COC cannot be considered a material fact which pertains to his eligibility and thus
qualification to run for public office.

That MIGZ is the son of LRAY is known to the voters of Camsur. Their relationship is shown by the
posters, streamers and billboards displayed in the province with the faces of both the father and
son on them. Thus, the voters of the Province of Camarines Sur know who respondent is.

Hayudini v. Comelec Issue: Whether a statement in a certificate of candidacy claiming that a candidate is eligible to
run for public office when in fact he is not constitute a false material representation that would
warrant the cancellation of the candidate’s COC?

Ruling: YES. Sec. 74 of the Omnibus Election Code requires the candidate to state under oath in
his CoC "that he is eligible for said office." A candidate is eligible if he has a right to run for the
public office. If a candidate is not actually eligible because he is not a registered voter in the
municipality where he intends to be elected, but still he states under oath in his certificate of
candidacy that he is eligible to run for public office, then the candidate clearly makes a false
material representation, a ground to support a petition under Sec. 78. It is interesting to note
that Hayudini was, in fact, initially excluded by the ERB as a voter. On November 30, 2012, the
ERB issued a certificate confirming the disapproval of Hayudini’s petition for registration. This is
precisely the reason why he needed to file a Petition for Inclusion in the Permanent List of Voters
in Barangay Bintawlan before the MCTC. Thus, when he stated in his CoC that "he is eligible for
said office," Hayudini made a clear and material misrepresentation as to his eligibility, because he
was not, in fact, registered as a voter in Barangay Bintawlan.

It bears stressing that one of the requirements for a mayoralty candidate is that he must be a
resident of the city or municipality where he intends to be elected. Thus, under Section 74 of the
Omnibus Election Code, it is required that a candidate must certify under oath that he is eligible
for the public office he seeks election. In this case, when petitioner stated in his CoC that he is a
resident of Barangay Bintawlan, South Ubian, Tawi Tawi and eligible for a public office, but it
turned out that he was declared to be a non-resident thereof in a petition for his inclusion in the
list of registered voters, he therefore committed a false representation in his CoC which pertained
to a material fact which is a ground for the cancellation of his CoC under Section 78 of the
Omnibus Election Code. Petitioner's ineligibility for not being a resident of the place he sought
election is not a ground for a petition for disqualification, since the grounds enumerated under
Section 68 of the Omnibus Election Code specifically refer to the commission of prohibited acts,
and possession of a permanent resident status in a foreign country.

Ongsiako Reyes vs Issue: Whether the cancellation of petitioner’s COC for the position of Representative of the lone
COMELEC district of Marinduque on the ground that he is American citizen is valid?

Ruling: YES. A Filipino citizen who becomes naturalized elsewhere effectively abandons his
domicile of origin. Upon re-acquisition of Filipino citizenship pursuant to RA 9225, he must still
show that he chose to establish his domicile in the Philippines through positive acts, and the
period of his residency shall be counted from the time he made it his domicile of choice. Here,
there is no showing whatsoever that petitioner had already re-acquired her Filipino citizenship
pursuant to RA 9225 so as to conclude that she has regained her domicile in the Philippines.
There being no proof that petitioner had renounced her American citizenship, it follows that she
has not abandoned her domicile of choice in the USA.

The only proof presented by petitioner to show that she has met the 1-year residency
requirement of the law and never abandoned her domicile of origin in Boac, Marinduque is her
claim that she served as Provincial Administrator of the province from Jan. 18, 2011 to July 13,
2011. But such fact alone is not sufficient to prove her 1-year residency. For, petitioner has never
regained her domicile in Marinduque as she remains to be an American citizen. No amount of her
stay in the said locality can substitute the fact that she has not abandoned her domicile of choice
in the USA.

Tagolino vs HRET Issue: Whether Richard Gomez was validly substituted by his wife as candidate for
Representative of 4th Legislative District of Leyte in view of the former’s failure to meet the 1
year residency requirement?

Ruling: NO. Only an official candidate can be substituted. Since Richard Gomez’s COC was not
valid, he was not an official candidate. Hence, he cannot be substituted.

There is a difference between a petition for disqualification and a petition to deny due course
to/cancel a certificate of candidacy. A disqualification case under Section 68 of the OEC is hinged
on either: (a) a candidate’s possession of a permanent resident status in a foreign country; or (b)
his or her commission of certain acts of disqualification. Anent the latter, the prohibited acts
under Section 68 refer to election offenses under the OEC, and not to violations of other penal
laws. It must be stressed that one who is disqualified under Section 68 is still technically
considered to have been a candidate, albeit proscribed to continue as such only because of
supervening infractions which do not, however, deny his or her statutory eligibility. In other
words, while the candidate’s compliance with the eligibility requirements as prescribed by law,
such as age, residency, and citizenship, is not in question, he or she is, however, ordered to
discontinue such candidacy as a form of penal sanction brought by the commission of the above-
154
mentioned election offenses.

On the other hand, a denial of due course to and/or cancellation of a CoC proceeding under
Section 78 of the OEC is premised on a person’s misrepresentation of any of the material
qualifications required for the elective office aspired for. It is not enough that a person lacks the
relevant qualification; he or she must have also made a false representation of the same in the
CoC. Pertinently, while a disqualified candidate under Section 68 is still considered to have been a
candidate for all intents and purposes, on the other hand, a person whose CoC had been denied
due course to and/ or cancelled under Section 78 is deemed to have not been a candidate at all.
The reason being is that a cancelled CoC is considered void ab initio and thus, cannot give rise to
a valid candidacy and necessarily, to valid votes. Moreover, while a person who is disqualified
under Section 68 is merely prohibited to continue as a candidate, a person who certificate is
cancelled or denied due course under Section 78 is not treated as a candidate at all, as if he/she
never filed a CoC.

A valid COC is a condition sine qua non for candidate substitution. One must have validly filed a
CoC in order to be considered a candidate. Hence, a candidate who is disqualified under Section
68 can be validly substituted pursuant to Section 77 because he remains a candidate until
disqualified; but a person whose CoC has been denied due course to and/or cancelled under
Section 78 cannot be substituted because he is not considered a candidate. Stated differently,
since there would be no candidate to speak of under a denial of due course to and/or cancellation
of a CoC case, then there would be no candidate to be substituted; the same does not obtain,
however, in a disqualification case since there remains to be a candidate to be substituted,
although his or her candidacy is discontinued.

Here, Richard’s disqualification is his failure to comply with the residency requirement under
Section 6 of the Constitution which is a ground for the denial of due course to and/or cancellation
of COC under Section 78 of OEC, misrepresentation contemplated under a Section 78 petition
refers to statements affecting one’s qualifications for elective office such as age, residence and
citizenship or non-possession of natural-born Filipino status. Hence, there is no valid substitution.

Maquiling vs. Comelec Issue: Whether Arnado is disqualified to run as mayor of Kauswagan, Lanao del Norte on the
ground of his continued use of a US passport after renouncing his foreign citizenship?

Ruling: YES. The use of foreign passport after renouncing one’s foreign citizenship is a positive
and voluntary act of representation as to one’s nationality and citizenship; it does not divest
Filipino citizenship regained by repatriation but it recants the Oath of Renunciation required to
qualify one to run for an elective position.

While the act of using a foreign passport is not one of the acts enumerated in CA 63 constituting
renunciation and loss of Philippine citizenship, it is nevertheless an act which repudiates the very
oath of renunciation required for a former Filipino citizen who is also a citizen of another country
to be qualified to run for a local elective position. In this case, when Arnado used his US passport
on 14 April 2009, or just eleven days after he renounced his American citizenship, he recanted his
Oath of Renunciation that he "absolutely and perpetually renounces all allegiance and fidelity to
the UNITED STATES OF AMERICA" and that he "divests himself of full employment of all civil and
political rights and privileges of the United States of America."

Hence, Arnado, by using his US passport after renouncing his American citizenship, has recanted
the same Oath of Renunciation he took. He is disqualified not only from holding the public office
but even from becoming a candidate in the May 2010 elections.

Ibrahim vs. Comelec Issue: Whether the COMELEC En Banc has the authority to motu proprio disqualify Ibrahim as a
G.R. No. 192289, January candidate for the position of Vice-Mayor of Datu Unsay, Maguindanao on the ground that he is not
8, a registered voter of the said municipality?

Ruling: NO. Here, the COMELEC en banc, through the herein assailed resolutions, ordered
Ibrahim’s disqualification even when no complaint or petition was filed against him yet. Let it be
stressed that if filed before the conduct of the elections, a petition to deny due course or cancel a
certificate of candidacy under Section 78 of the OEC is the appropriate petition which should have
been instituted against Ibrahim considering that his allegedly being an unregistered voter of Datu
Unsay disqualified him from running as Vice-Mayor. His supposed misrepresentation as an eligible
candidate was an act falling within the purview of Section 78 of the OEC. Moreover, even if we
were to assume that a proper petition had been filed, the COMELEC en banc still acted with grave
abuse of discretion when it took cognizance of a matter, which by both constitutional prescription
and jurisprudential declaration, instead aptly pertains to one of its divisions.

Javier v. Comelec Issue: Whether Gov. Javier’s disqualification as the governor of Antique on the ground that the
latter committed election offenses of Coercion of Subordinates and Threats, Intimidation,
Terrorism, or other forms of Coercion by suspending Mayor Roquero is invalid because RA No.
7890 effectively repeals Sec. 261(d) of the Omnibus Election Code thereby remove coercion as a
ground for disqualification under Sec. 68 of the OEC?

Ruling: YES. R.A. No. 7890 expressly repealed Section 261, paragraphs (d)(1) and (2) of the
Omnibus Election Code. The COMELEC’s reasoning that coercion remains to be a ground for
disqualification under Section 68 of the Election Code despite the passage of R.A. No. 7890 is
erroneous. To the point of our being repetitive, R.A. No. 7890 expressly repealed Section 261
d(1) and (2) of Batas Pambansa Blg. 881, rendering these provisions inoperative. The effect of
this repeal is to remove Section 261(d) from among those listed as ground for disqualification
under Section 68 of the Omnibus Election Code. Hence, Gov. Javier cannot be disqualified on the
ground not listed under Section 68 of the OEC.

155
Francisco v. COMELEC Issue: Whether a prior judgment is not a precondition to filing a Petition for Disqualification.

Ruling: YES. The doctrine in Poe was never meant to apply to Petitions for Disqualification. A
prior court judgment is not required before the remedy under Sec. 68 of the OEC can prosper.
This is highlighted by the provision itself, which contemplates of two scenarios: first, there is a
final decision by a competent court that the candidate is guilty of an election offense and second,
it is the Commission itself that found that the candidate committed any of the enumerated
prohibited acts. Noteworthy is that in the second scenario, it is not required that there be a prior
final judgment; it is sufficient that the Commission itself made the determination. The conjunction
"or" separating "competent court" and "the Commission" could only mean that the legislative
intent was for bot/1 bodies to be clothed with authority to ascertain whether or not there is
evidence that the respondent candidate ought to be disqualified. We are, therefore, constrained
to rule that the COMELEC erred when, relying on Poe, it imposed the requirement of a prior court
judgment before resolving the current controversy.

Also, the records are bereft of evidence to hold that respondent violated Secs. 261(v) and 104 of
the Omnibus Election Code. Here, petitioner merely submitted the following to support his
allegations: (1) Pictures of the asphalt-paving along Imelda Avenue of Cainta Green Park Village,
Barangay San Isidro, Cainta, Rizal; (2) Picture of the Facebook page of the respondent
acknowledging the project as one of the accomplishments of his administration; and (3)Picture of
a tarpaulin banner expressing gratitude for the asphalt-paving.

The photographs petitioner presented depicting the construction and works done on the
asphalting project would only prove the fact of paving, which is not even contested. They do not,
however, establish that respondent expended public funds or made financial contributions during
the election prohibition. On the other hand, respondent Nieto sufficiently parried the alleged
commission of the election offenses by proving that the asphalting project squarely falls under the
exception in Sec. 261 (v)(l)(b). Private respondent Nieto was able to show with competent
evidence that the bidding for and the award of the subject project were regular and done
consistent with existing laws. The charge for illegal contribution under Sec. 104 of the OEC has
even less leg to stand on. There was no contribution to speak of since it was established that the
asphalting work was a government project and not a contribution.

RECOUNT

Maliksi vs. COMELEC Issue: Whether the recount proceedings conducted by the COMELEC First Division with the use of
the printouts of the ballot images is invalid on the ground that it violates petitioner’s right to due
process by failing to give due notice on the decryption and printing of the ballot images?

Ruling: YES. The decryption of the images stored in the CF cards and the printing of the
decrypted images take place during the revision or recount proceedings. There is a good reason
for thus fixing where and by whom the decryption and the printing should be conducted. It is
during the revision or recount conducted by the Revision/Recount Committee when the parties
are allowed to be represented, with their representatives witnessing the proceedings and timely
raising their objections in the course of the proceedings. Moreover, whenever the
Revision/Recount Committee makes any determination that the ballots have been tampered and
have become unreliable, the parties are immediately made aware of such determination.

When, as in the present case, it was not the Revision/Recount Committee or the RTC exercising
original jurisdiction over the protest that made the finding that the ballots had been tampered,
but the First Division in the exercise of its appellate jurisdiction, the parties should have been
given a formal notice thereof. Maliksi was not immediately made aware of that crucial finding
because the First Division did not even issue any written resolution stating its reasons for
ordering the printing of the picture images. The parties were formally notified that the First
Division had found that the ballots had been tampered only when they received the resolution of
August 15, 2012, whereby the First Division nullified the decision of the RTC and declared
Saquilayan as the duly elected Mayor. Even so, the resolution of the First Division to that effect
was unusually mute about the factual bases for the finding of ballot box tampering, and did not
also particularize how and why the First Division was concluding that the integrity of the ballots
had been compromised.

Here, the First Division denominated the proceedings it had conducted as an "appreciation of
ballots" like in Mendoza. But unlike in Mendoza, the proceedings conducted by the First Division
were adversarial, in that the proceedings included the decryption and printing of the picture
images of the ballots and the recount of the votes were to be based on the printouts of the
picture images. The First Division did not simply review the findings of the RTC and the Revision
Committee, but actually conducted its own recount proceedings using the printouts of the picture
image of the ballots. As such, the First Division was bound to notify the parties to enable them to
participate in the proceedings.

POWER OF COMELEC

Joselito R. Mendoza vs. Issue: Whether the respondent’s election protest against the petitioner originally filed before the
COMELEC COMELEC should have been dismissed after the re-hearing of the case?

Ruling: YES. There is a difference in the result of the exercise of jurisdiction by the COMELEC
over election contests. The difference inheres in the kind of jurisdiction invoked, which in turn, is

156
determined by the case brought before the COMELEC. When a decision of a trial court is brought
before the COMELEC for it to exercise appellate jurisdiction, the division decides the appeal but, if
there is a motion for reconsideration, the appeal proceeds to the banc where a majority is needed
for a decision. If the process ends without the required majority at the banc, the appealed
decision stands affirmed. Upon the other hand, and this is what happened in the instant case, if
what is brought before the COMELEC is an original protest invoking the original jurisdiction of the
Commission, the protest, as one whole process, is first decided by the division, which process is
continued in the banc if there is a motion for reconsideration of the division ruling. If no majority
decision is reached in the banc, the protest, which is an original action, shall be dismissed. There
is no first instance decision that can be deemed affirmed.

It is easy to understand the reason for the difference in the result of the two protests, one as
original action and the other as an appeal, if and when the protest process reaches the COMELEC
En Banc. In a protest originally brought before the COMELEC, no completed process comes to the
banc. It is the banc which will complete the process. If, at that completion, no conclusive result in
the form of a majority vote is reached, the COMELEC has no other choice except to dismiss the
protest. In a protest placed before the Commission as an appeal, there has been a completed
proceeding that has resulted in a decision. So that when the COMELEC, as an appellate body, and
after the appellate process is completed, reaches an inconclusive result, the appeal is in effect
dismissed and resultingly, the decision appealed from is affirmed.

The grave abuse of discretion of the COMELEC is underscored by the fact that the protest that
petitioner Pagdanganan filed on 1 June 2007 overstayed with the COMELEC until the present
election year when the end of the term of the contested office is at hand and there was hardly
enough time for the re-hearing that was conducted only on 15 February 2010. As the hearing
time at the division had run out, and the re-hearing time at the banc was fast running out, the
unwanted result came about: incomplete appreciation of ballots; invalidation of ballots on general
and unspecific grounds; unrebutted presumption of validity of ballots.

Atty. Reynante B. Issue: Whether the COMELEC has the authority to prohibit an authorized person from bearing,
Orceo vs. COMELEC carrying or transporting firearms or other deadly weapons, including airsoft guns and their
replicas/imitation in public places during the election period?

Ruling: YES. The COMELEC had the authority to promulgate Resolution No. 8714 pursuant to
Section 35 of R.A. No. 7166. It was granted the power to issue the implementing rules and
regulations of Sections 32 and 33 of R.A. No. 7166. Under this broad power, the COMELEC was
mandated to provide the details of who may bear, carry or transport firearms or other deadly
weapons, as well as the definition of firearms, among others. These details are left to the
discretion of the COMELEC, which is a constitutional body that possesses special knowledge and
expertise on election matters, with the objective of ensuring the holding of free, orderly, honest,
peaceful and credible elections.

The inclusion of airsoft guns and airguns in the term firearm in Resolution No. 8714 for purposes
of the gun ban during the election period is a reasonable restriction, the objective of which is to
ensure the holding of free, orderly, honest, peaceful and credible elections.

Legaspi v. Comelec Issue: Whether the COMELEC’s dismissal of the electoral aspect of the case filed by petitioner on
the ground that COMELEC cannot reached the majority votes required despite rehearing of the
case is valid?

Ruling: YES. Section 7 of Article IX-A of the Constitution obliges the COMELEC, like the other
constitutional commissions, to decide all cases or matters before it by a "majority vote of all its
members." When such majority vote cannot be mustered by the COMELEC en banc, the case shall
be reheard, and if on rehearing no decision is reached, the action or proceeding shall be
dismissed if originally commenced in the Commission; in appealed cases, the decision appealed
from shall stand affirmed; and in all incidental matters, the petition or motion shall be denied.

The effects of the COMELEC en banc’s failure to decide vary depending on the type of case or
matter that is before the commission. Thus, under the provision, the first effect (i.e., the
dismissal of the action or proceeding) only applies when the type of case before the COMELEC is
an action or proceeding "originally commenced in the commission"; the second effect (i.e., the
affirmance of a judgment or order) only applies when the type of case before the COMELEC is an
"appealed case"; and the third effect (i.e., the denial of the petition or motion) only applies when
the case or matter before the COMELEC is an "incidental matter."

The COMELEC en banc did not err when it dismissed the electoral aspect of SPA No. 13-323 (DC)
when it was unable to reach a majority vote after the rehearing. Contrary to what petitioner
asserts, SPA No. 13-323 (DC) is most definitely an action that was filed originally before the
COMELEC within the contemplation of the said provision. While SPA No. 13-323 (DC) reached the
COMELEC en banc only through a motion for reconsideration of the decision of the Special First
Division, its character as an original case filed before the commission remains the same. Hence,
the failure of COMELEC en banc to decide in this case properly results in the application of the
first effect of Section 6, Rule 18 of the COMELEC Rules.

TRANSFER OF EMPLOYEES

Causing vs. COMELEC Issue: Whether the relocation of the petitioner from her office as Local Civil Registrar to the
Office of the Mayor during the election period constitutes a transfer prohibited under the Omnibus
Election Code?

157
Ruling: NO. The only personnel movements prohibited by COMELEC Resolution No. 8737 were
transfer and detail. Transfer is defined in the Resolution as "any personnel movement from one
government agency to another or from one department, division, geographical unit or subdivision
of a government agency to another with or without the issuance of an appointment;" while detail
as defined in the Administrative Code of 1987 is the movement of an employee from one agency
to another without the issuance of an appointment Having acquired technical and legal meanings,
transfer and detail must be construed as such.

Obviously, the movement involving Causing did not equate to either a transfer or a detail within
the contemplation of the law if Mayor Biron only thereby physically transferred her office area
from its old location to the Office of the Mayor "some little steps" away. We cannot accept the
petitioner's argument, therefore, that the phrase "any transfer or detail whatsoever"
encompassed "any and all kinds and manner of personnel movement," including the mere change
in office location.

Equally material is that Mayor Biron's act of transferring the office space of Causing was rooted in
his power of supervision and control over the officials and employees serving in his local
government unit, in order to ensure the faithful discharge of their duties and functions. His
explanation that he transferred Causing's work station from her original office to his office in
order to closely supervise her after his office received complaints against her could not be justly
ignored. Verily, she thereafter continued to perform her tasks, and uninterruptedly received her
salaries as the Municipal Civil Registrar even after the transfer to the Office of the Mayor.

Aquino v. Comelec Issue: Whether the reassignment of PHIC officers and employees falls within the coverage of
prohibited transfers or details during election period?

Ruling: YES. In Resolution No. 8737, the COMELEC defined the phrase "transfer or detail
whatever" found in Section 261(h) of BP 881 as including any personnel action, i.e.,
"reassignment." As used in Section 261(h) of BP 881, the term whatever should be not be read
strictly in conjunction with only either the term transfer or the term detail; nor should the phrase
transfer or detail whatever be read in isolation from the purpose of the legal prohibition. Rather,
consistent with our rules in reading provisions of law, the term - whatever - as well as the phrase
transfer or detail whatever should be understood within the broader context of the purpose of BP
881. They should likewise be understood within the context of all other laws that the COMELEC is
required to administer and enforce. This is the proper approach that anyone, including this Court,
should take when reading Section 261(h), as well as all other provisions of BP 881 and other
election laws.

Thus, it is immaterial whether or not the personnel action has in fact been actually used for
electioneering purposes or whether there has been any allegation in the complaint to this effect.
The mere existence of such plausibility for electioneering is the reason that animated the legal
prohibition against any personnel action, including transfers and re-assignments, during the
election period. To our mind, the interpretation that includes any form of personnel action, such
as reassignment, within the coverage of the phrase precisely guards against any such
electioneering and political harassment situations. This interpretation also more vigorously
enforces the aim not only of BP 881, but more importantly of the Constitution to secure free,
orderly, honest, peaceful, and credible elections.

Under Section 261(h) of BP 881, a person commits the election offense of violation of the election
transfer ban when he makes or causes the transfer or detail whatever of any official or employee
of the government during the election period absent prior approval of the COMELEC. By its terms,
Section 261(h) provides at once the elements of the offense and its exceptions. The elements
are: (1) the making or causing of a government official or employee's transfer or detail whatever,
(2) the making or causing of the transfer or detail whatever was made during the election period;
and (3) these acts were made without the required prior COMELEC approval.

As this provision operates, the making or causing of the movement of personnel during the
election period but without the required COMELEC approval is covered by the prohibition and
renders the responsible person liable for the offense. Conversely, the making or causing (of the
movement of personnel) before or after the election period even without the required COMELEC
approval, or during the election period but with the required COMELEC approval are not covered
by the prohibition and do not render the responsible person liable for this election offense.

ARTICLE XI – Accountability of Public Officers

Sec. 2

A. Impeachment

Republic v. Sereno Quo Warranto v. Impeachment

Quo warranto and impeachment are two distinct proceedings, although both may result in
the ouster of a public officer. Strictly speaking, 

 quo warranto grants the relief of "ouster", while

158
 impeachment affords "removal."

A quo warranto proceeding is the proper legal remedy to determine a person's right or


title to a public office and to oust the holder from its enjoyment.  

 It is the proper action to inquire into a public officer's eligibility  or the validity of
his appointment. 

 Under Rule 66 of the Rules of Court, a quo warranto proceeding involves a judicial


determination of the right to the use or exercise of the office.

Impeachment, on the other hand, is a political process undertaken by the legislature to


determine whether the public officer committed any of the impeachable offenses, namely,
culpable violation of the Constitution, treason, bribery, graft and corruption, other high
crimes, or betrayal of public trust.  

 It does not ascertain the officer's eligibility for appointment or election, or challenge
the legality of his assumption of office.
 Conviction for any of the impeachable offenses shall result in the removal of the
impeachable official from office. 

Quo warranto as a remedy to oust an ineligible public official may be availed of when the
subject act or omission was committed prior to or at the time of appointment or election
relating to an official’s qualifications to hold office as to render such appointment or election
invalid. Acts or omissions, even if it relates to the qualification of integrity being a continuing
requirement but nonetheless committed during the incumbency of a validly appointed and/or
validly elected official cannot be the subject of a quo warranto proceeding, but of
impeachment if the public official concerned is impeachable and the act or omission
constitutes an impeachable offense, or to disciplinary, administrative or criminal action, if
otherwise.

Impeachment Quo Warranto

Removal Ouster

Political Process undertaken by the Proceeding involving a Judicial determination


Legislature
Grounds: violation of CTB-GOB Grounds: Ineligibility or invalidity of
1. Constitution appointment or election relating to an official’s
2. Treason qualifications to hold office.
3. Bribery
4. Graft and Corruption
5. Other High Crimes
6. Betrayal of public trust

Act or omission was committed during the Act or omission was committed prior or at the
incumbency of a validly appointed or elected time of appointment/election.
official.

Discussion of Sec. 2, Art. XI

Respondent's insistence that she could not be removed from office except through
impeachment is predicated on Section 2, Article XI of the Constitution.

 By its plain language, however, Section 2 of Article XI does not preclude a quo
warranto action questioning an impeachable officer's qualifications to assume
office. These qualifications include age, citizenship and professional experience -
matters which are manifestly outside the purview of impeachment under the above-
cited provision.

 Furthermore, Section 2 of Article XI cannot be read in isolation from Section 5(1) of


Article VIII of the Constitution which gives this Court its quo warranto jurisdiction, or
from Section 4, paragraph 7 of Article VII of the Constitution which designates the
Court as the sole judge of the qualifications of the President and Vice-President.

AP: Here since Sereno’s act or omission was committed prior to her appointment to the Chief
Justice position, which was her failure to submit the required SALN as a public employee during
her employment as a faculty of UP College of Law (UP Law) and various position in the
government. SALN is a constitutional and statutory requirement for the position of Chief Justice,
hence her failure to com ply with the SALN, shows the she lacks the PROVEN INTERGRITY needed
as member of the Judiciary especially the highest position of a Chief Justice.

159
Sec. 3

A. Impeachment

Francisco v. House of One-Year Time Bar Rule


Representatives
ISSUE: Whether the impeachment proceedings initiated against the Chief Justice transgressed
the constitutionally imposed one-year time bar rule. YES.

LB: The HoR have exclusive power to initiate all cases of impeachment which simply takes place
by the act of filing of the impeachment complaint and referral to the House Committee on Justice
another interpretation is invalid.

LB: No impeachment proceedings shall be initiated against the same official more than once
within a period of one year. Thus, the second impeachment case is deemed barred

AP: Considering that the first impeachment complaint, was filed on June 2, 2003 and the second
impeachment complaint filed was on October 23, 2003, it violates the constitutional prohibition
against the initiation of impeachment proceedings against the same impeachable officer within a
one-year period.

Unconstitutionality of Sec. 16 and 17 of Rule V of the House Impeachment Rules

ISSUE 2: Whether or not Sections 15 and 16 of Rule V of the Rules on Impeachment adopted by
the 12th Congress are unconstitutional? YES.

LB: From the records of the Constitutional Commission, to the amicus curiae briefs of two former
Constitutional Commissioners, it is without a doubt that the term “to initiate” refers to the filing
of the impeachment complaint coupled with Congress’ taking initial action of said complaint

AP: The provisions of Sections 16 and 17 of Rule V of the House Impeachment Rules which
state that impeachment proceedings are deemed initiated
1. (1) if there is a finding by the House Committee on Justice that the verified
complaint and/or resolution is sufficient in substance, or
2. (2) once the House itself affirms or overturns the finding of the Committee on
Justice that the verified complaint and/or resolution is not sufficient in substance or
3. (3) by the filing or endorsement before the Secretary General of the House of
Representatives of a verified complaint or a resolution of impeachment by at least
1/3 of the members of the House
 Thus clearly contravene Section 3 (5) of Article XI as they give the term
“initiate” a meaning different from “filing.”

Determination of Impeachable Offence

ISSUE 3: Can the Court make a determination of what constitutes an impeachable offense? NO.

LB: Such a determination is a purely political question which the Constitution has left to the
sound discretion of the legislation. Although Section 2 of Article XI of the Constitution enumerates
six grounds for impeachment, two of these, namely, other high crimes and betrayal of public
trust, elude a precise definition.

In Re Gonzales ISSUE: W/N a public officer required by the Constitution to be a member of the PH bar be
disbarred during his term? NO.
F: Gonzales
(Tanodbayan/ Special LB: A public officer who under the Constitution is required to be a Member of the Philippine Bar as
prosecutor) forwarded a a qualification for the office held by him and who may be removed from office only by
letter-complaint to Justice impeachment, cannot be charged with disbarment during the incumbency of such public officer.
Fernan. Attached to the  Further, such public officer, during his incumbency, cannot be charged criminally before
letter-complaint was an the Sandiganbayan or any other court with any offense which carries with it the penalty
anonymous letter by of removal from office, or any penalty service of which would amount to removal from
Concerned Employees of office.
the SC, referring to
charges for disbarment LB: The Special Prosecutor cannot conduct an investigation into alleged misconduct of a Supreme
against Justice Fernan. Court Justice, with the purpose of filing criminal information against him with the Sandiganbayan ,
as this would violate the security of tenure of the Supreme Court Justices.
 The Supreme Court Justice should be impeached first, before a criminal action may lie
against him.

LB: Members of the Supreme Court must, under Article VIII (7) (1) of the Constitution, be
members of the Philippine Bar and may be removed from office only by impeachment (Article XI
[2], Constitution).
 To grant a complaint for disbarment of a Member of the Court during the Member’s
incumbency, would in effect be to circumvent and hence to run afoul of the constitutional
mandate that Members of the Court may be removed from office only by impeachment
160
for and conviction of certain offenses listed in Article XI (2) of the Constitution.

LB: It is important to make clear that the Court is not here saying that its Members or the other
constitutional officers we referred to above are entitled to immunity from liability for possibly
criminal acts or for alleged violation of the Canons of Judicial Ethics or other supposed
misbehaviour.

What the Court is saying is that there is a fundamental procedural requirement that must
be observed before such liability may be determined and enforced.

 A Member of the Supreme Court must first be removed from office via the constitutional
route of impeachment under Sections 2 and 3 of Article XI of the 1987 Constitution.
Should the tenure of the Supreme Court Justice be thus terminated by impeachment, he
may then be held to answer either criminally or administratively (by disbarment
proceedings) for any wrong or misbehaviour that may be proven against him in
appropriate proceedings.

Gutierrez v. The House Judicial Review over Impeachment proceedings


of Representatives
Committee on Justice LB: Impeachment proceeding may be subjected to judicial review but only limited to the
determination of whether or not there has been a grave abuse of discretion amounting to lack or
excess of jurisdiction on the part of any branch or instrumentality of the Government.

AP: In the present case, petitioner invokes the Court’s expanded certiorari jurisdiction, using the
special civil actions of certiorari and prohibition as procedural vehicles. The Court finds it well-
within its power to determine whether public respondent committed a violation of the Constitution
or gravely abused its discretion in the exercise of its functions and prerogatives that could
translate as lack or excess of jurisdiction, which would require corrective measures from the
Court.

Impeachment Rules are NOT intended to be published

LB: Impeachment Rules are not intended to be published.

 Sec. 3 (8), Article XI of the Constitution states that “The Congress shall promulgate its
rules on impeachment to effectively carry out the purpose of this section”.

 Had the Constitution intended to have the Impeachment Rules published, it could have
stated so as categorically as it did in the case of the rules of procedure in legislative
inquiries, per Neri.

 Other than "promulgate," there is no other single formal term in the English language to
appropriately refer to an issuance without need of it being published.

One-Year Bar Rule

 The one-year bar rule (No impeachment proceedings shall be INITIATED against the
same official more than once within a period of one year): The term "INITIATE" means
to file the complaint and take initial action on it.

 The initiation starts with the filing of the complaint which must be accompanied with an
action to set the complaint moving.

 It refers to the filing of the impeachment complaint coupled with Congress’ taking initial
action of said complaint.

 The initial action taken by the House on the complaint is the referral of the complaint to
the Committee on Justice.

In the case of Francisco, “initiate” refers to the filing of the impeachment complaint
coupled with congress taking initial action of the complaint.

Reckoning period is the referral

 Thus, the filing of the complaint must be accompanied by the referral of the Committee
of Justice which is the action that sets the complaint moving.

 Thus, referral is the reckoning point that ignites impeachment proceedings.

 The action of the House is already a further step in the proceeding, not its initiation or
beginning. Rather, the proceeding is initiated or begins, when a verified complaint is filed
and referred to the Committee on Justice for action. This is the initiating step which
triggers the series of steps that follow.

Corona v. Senate Nachura: Impeachment refers to the power of Congress to remove a public official for serious
crimes or misconduct as provided in the Constitution.
161
F: During his  A mechanism to check abuse of power.
impeachment proceeding,
Petitioner filed a LB: Given their concededly political character, the precise role of the judiciary in impeachment
Supplemental Petition cases is a matter of utmost importance to ensure the effective functioning of the separate
claiming that his right to branches while preserving the structure of checks and balance in our government.
due process is being
violated in the ongoing Moreover, in this jurisdiction, the acts of any branch or instrumentality of the government,
impeachment including those traditionally entrusted to the political departments, are proper subjects of
proceedings because judicial review if tainted with grave abuse or arbitrariness.
certain Senator-Judges
have lost the cold By the nature of the functions they discharge when sitting as an Impeachment Court, Senator
neutrality of impartial Judges are clearly entitled to propound questions on the witnesses, prosecutors and
judges by acting as counsel during the trial.
prosecutors.

AP: Respondents contend that the issues raised in the Supplemental Petition regarding the
Petitioner wanted to behavior of certain Senator-Judges in the course of the impeachment trial are issues that do not
nullify the impeachment concern, or allege any violation of, the three express and exclusive constitutional limitations on
complaint of the ground the Senate’s sole power to try and decide impeachment cases . They argue that unless
of the behavior of certain there is a clear transgression of these constitutional limitations, this Court may not
Senator-Judges. exercise its power of expanded judicial review over the actions of Senator-Judges during the
proceedings.

By the nature of the functions they discharge when sitting as an Impeachment Court, Senator-
Judges are clearly entitled to propound questions on the witnesses, prosecutors and counsel
during the trial.

Petitioner thus failed to prove any semblance of partiality on the part of any Senator-
Judges. But whether the Senate Impeachment Rules were followed or not, is a political
question that is not within this Court’s power of expanded judicial review.

Disposition: the impeachment trial had been concluded with the conviction of petitioner
by more than the required majority vote of the Senator-Judges. Petitioner immediately
accepted the verdict and without any protest vacated his office. MOOT AND ACADEMIC

Sec. 4

A. SANDIGANBAYAN

Azarcon v. ISSUE: Whether or not Sandiganbayan has jurisdiction over a private individual designated by
Sandiganbayan BIR as a custodian of distrained property. None.

LB: Sandiganbayan has jurisdiction over a private individual when the complaint charges the
private individual either as a co-principal, accomplice or accessory of a public officer or employee
who has been charged with a crime within its jurisdiction.

AP: Although the NIRC authorizes the BIR to effect a constructive distraint by requiring
“any person” to preserve distrained property, there is no provision in the NIRC
constituting such person a public officer by reason of such requirement.

 The BIR’ s power to appoint a private individual to act as a depositary cannot be


stretched to include the power to appoint him as public officer.

 Information does not charge petitioner Azarcon of being a co-principal, accomplice or


accessory to a public officer committing an offense under the Sandiganbayan’s
jurisdiction

 Since petitioner is not a public officer or EE, but a private individual not
charged as a co-principal, accomplice or accessory of a public officer in
committing an offense under the Sandiganbayan’s jurisdiction, it has no
jurisdiction over petitioner.

Republic v. Investa LB: Presidential Commission on Good Government v. Peña


-defined the Sandiganbayan’s jurisdiction over the PCGG in the exercise of the PCGG’s
powers under the applicable Executive Orders and the Constitution, thus:

Under section 2 of the Executive Order No. 14, all cases of the Commission regarding the
“Funds, Moneys, Assets, and Properties Illegally Acquired or Misappropriated by Former
President Ferdinand Marcos, Mrs. Imelda Romualdez Marcos, their Close Relatives,
Subordinates, Business Associates, Dummies, Agents or Nominees” whether civil or criminal, are
lodged within the “exclusive and original jurisdiction of the Sandiganbayan” and all
incidents arising from, incidental to, or related to, such cases necessarily fall likewise
under the Sandiganbayan’s exclusive and original jurisdiction, subject to review on
certiorari exclusively by the Supreme Court.

AP: Thus, the present action properly lies within the jurisdiction of the Sandiganbayan. The
present case clearly pertains to the percentage share of the Republic in Domsat as
represented by the sequestered Domsat shares.
162
The Domsat shares are properties, which the Republic claims to be illegally acquired or
misappropriated by former President Marcos, his family, cronies, or dummies. The SB
should now rule upon the propriety of the management contract, and consider the issues
raised by Investa in their memorandum before this Court.

People v. Go Disadvantageous Transactions

F: Henry T. Go, as LB: —R.A. 3019 (ANTI GRAFT AND CORRUPT PRACTICES ACT) - In addition to acts or omissions
Chairman and President of public officers already penalized by existing law, the following shall constitute corrupt practices
of the Philippine of any public officer and are hereby declared to be unlawful:
International Air
Terminals Co., Inc. Entering, on behalf of the Government, into any contract or transaction manifestly and
(PIATCO) was charged for grossly disadvantageous to the same, whether or not the public officer profited or will
violation of the Anti-Graft profit thereby.
and Corrupt Practices Act
in conspiracy with the
late Arturo Enrile, then The elements of the above provision are: (P-O-G)
Secretary of the (1) that the accused is a public officer;
Department of (2) that he entered into a contract or transaction on behalf of the government; and
Transportation and (3) that such contract or transaction is grossly and manifestly disadvantageous to the
Communications (DOTC) government.
before the
Sandiganbayan. Prior to Private persons in conspiracy with public officers
the filing of the
Information, Secretary
LB 2: Private persons, when acting in conspiracy with public officers, may be indicted
Enrile died. Private
and, if found guilty, held liable for the pertinent offenses under Section 3 of Republic Act
respondent Go questions
(R.A.) 3019, in consonance with the avowed policy of the anti-graft law to repress certain acts of
the jurisdiction of the
public officers and private persons alike constituting graft or corrupt practices act or which may
Sandiganbayan, alleging
lead thereto.
that since Secretary
Enrile had died, the
Sandiganbayan does not Extinguishment of criminal liability in anti-graft cases
have jurisdiction over
him, a private person Discussion: —It is true that by reason of Secretary Enrile’s death, there is no longer any public
officer with whom respondent can be charged for violation of R.A. 3019.
W/N the Sandiganbayan
has jurisdiction over  It does not mean, however, that the allegation of conspiracy between them can no
charges against private longer be proved or that their alleged conspiracy is already expunged.
persons when the
criminal liability of the
 The only thing extinguished by the death of Secretary Enrile is his criminal
alleged public officer
liability. His death did not extinguish the crime nor did it remove the basis of the charge
involved has been
extinguished by death? of conspiracy between him and private respondent.
YES.
 Stated differently, the death of Secretary Enrile does not mean that there was no
public officer who allegedly violated Section 3(g) of R.A. 3019.

 In fact, the Office of the Deputy Ombudsman for Luzon found probable cause to indict
Secretary Enrile for infringement of Sections 3(e) and (g) of R.A. 3019. Were it not for
his death, he should have been charged.

AP: The requirement before a private person may be indicted for violation of Section 3(g) of R.A.
3019, among others, is that such private person must be alleged to have acted in
conspiracy with a public officer.

The law, however, does not require that such person must, in all instances, be indicted
together with the public officer.

If circumstances exist where the public officer may no longer be charged in court, as in the
present case where the public officer has already died, the private person may be
indicted alone.

De Lima v. Guerrero LB: The exclusive original jurisdiction over violations of RA 9165 is not transferred to the
Sandiganbayan whenever the accused occupies a position classified as Grade 27 or higher,
regardless of whether the violation is alleged as committed in relation to office. The power of the
Sandiganbayan to sit in judgment of high-ranking government officials is not omnipotent.

LB: Sandiganbayan is without jurisdiction to hear drug-related cases.

 Even Section 4(b) of PD 1606, as amended by RA 10660, touted by the petitioner and
the dissents as a catch-all provision, does not operate to strip the RTCs of its exclusive
original jurisdiction over violations of RA 9165.

 As pointed out by Justices Tijam and Martires, a perusal of the drugs law will reveal that
public officials were never considered excluded from its scope.

 Hence, Section 27 of RA 9165 punishes government officials found to have benefited


from the trafficking of dangerous drugs, while Section 28 of the law imposes the
maximum penalty on such government officials and employees.

163
PD 1606 v. RA 9165

LB: Presidential Decree (PD) No. 1606 (Creation of Sandiganbayan), as amended by


Republic Act (RA) No. 10660, is the general law on jurisdiction of the Sandiganbayan over
crimes and offenses committed by high-ranking public officers.

Section 90, RA No. 9165 (Comprehensive Dangerous Drugs Act) is the special law
excluding from the Sandiganbayan’s jurisdiction violations of RA No. 9165 committed by such
public officers.

AP: Republic Act (RA) No. 9165 specifies the Regional Trial Court (RTC) as the court with the
jurisdiction to “exclusively try and hear cases involving violations of [RA No. 9165].” This is an
exception, couched in the special law on dangerous drugs, to the general rule under Section 4(b)
of Presidential Decree (PD) No. 1606, as amended by RA No. 10660.

Sec. 7

A. OMBUDSMAN AND TANODBAYAN

Zaldivar v. OMBUDSMAN
Sandiganbayan
LB: Duty of the Ombudsman.—Under the 1987 Constitution, the Ombudsman (as distinguished
Does the Tanodbayan from the incumbent Tanodbayan) is charged with the duty to: “Investigate on its own, or on
still have the authority complaint by any person, any act or omission of any public official, employee, office or
to file such criminal agency, when such act or omission appears to be illegal, unjust, improper, or
cases with the SB in inefficient.”
light of the 1987
Constitution’s INCUMBENT TANODBAYAN (IYAK)
reorganization? NO

LB: Incumbent Tanodbayan lost his right to conduct preliminary investigation and to direct the
filing of criminal cases with the Sandiganbayan effective February 2, 1987.
 Now then, inasmuch as the aforementioned duty is given to the Ombudsman.

 The incumbent Tanodbayan (called Special Prosecutor under the 1987 Constitution and
who is supposed to retain powers and duties NOT GIVEN to the Ombudsman) is clearly
without authority to conduct preliminary investigations and to direct the filing
of criminal cases with the Sandiganbayan, except upon orders of the Ombudsman.

 This right to do so was lost effective February 2, 1987. From that time, he has
been divested of such authority.

AP: Under the present Constitution, the Special Prosecutor (Raul Gonzalez) is a mere
subordinate of the Tanodbayan (Ombudsman) and can investigate and prosecute cases only
upon the latter’s authority or orders.

 The Special Prosecutor cannot initiate the prosecution of cases but can only conduct the
same if instructed to do so by the Ombudsman.
 Even his original power to issue subpoena, which he still claims under Section 10(d)
of PD 1630, is now deemed transferred to the Ombudsman. (POWER TO ISSUE
SUBPOENA)

NO RIGHT TO HOLD OVER

The Special Prosecutor has no right to hold over the position of Ombudsman.
 It is not correct either to suppose that the Special Prosecutor remains the Ombudsman
as long as he has not been replaced, for the fact is that he has never been the
Ombudsman.

 The Office of the Ombudsman is a new creation under Article XI of the


Constitution different from the Office of the Tanodbayan created under PD1607
although concededly some of the powers of the two offices are identical or similar.

 The Special Prosecutor cannot plead that he has a right to hold over the position of
Ombudsman as he has never held it in the first place.

Acop v. Ombudsman LB: While the constitution did not explicitly provides that OMBUDSMAN has the power to conduct
preliminary investigation to the violation of laws by government official, it was given such
F: 11 suspected members power pursuant to a law enacted by the Congress.
of the Kuratong Baleleng
Gang were allegedly  Such was allowed by the Constitution since Sec 13 of Art XI provides that
killed in a shootout. OMBUDSMAN may perform such other function as the law may provide.

164
However, SPO2 de los AP: Hence it can investigate the alleged summary killing of the Police to 11 members of the
Reyes made an expose, Kuratong Baleleng Gang.
stating that there was no
shootout, the 11 Tanodbayan powers to investigate may also be limited by the congress since its power or
suspected members of authority emanates from the existing laws that the Congress may provide. Hence, as of the
the Kuratong Baleleng current law now, Tanodbayan is under the supervision of the OMBUDSMAN.
Gang were victims of
summary execution. The  Congress has the power to place the Office of the Special Prosecutor under the Office of
Ombudsman, his deputy the Ombudsman, and it may remove some of the powers granted to the Tanodbayan
for military affairs, the under P.D. No. 1630 and transfer them to the Ombudsman; Through R.A. No. 6770, the
Senate Committee on Office of the Special Prosecutor was made an organic component of the Office of the
Justice, the CHR Ombudsman.
conducted investigations
on such claim. The
Deputy Ombudsman for Epal doctrine: The Deputy Ombudsman for Military Affairs is not prohibited from performing
Military affairs took over other functions or duties affecting nonmilitary personnel—the Ombudsman may refer cases
the investigation and involving non-military personnel for investigation by the Deputy for Military Affairs.
ordered petitioners to
submit counter-affidavits.
Petitioners are now
refuting the power of
Ombudsman to conduct
PI, because under Sec. 3
Rule II of Adm. Order 07
does not include the
Ombudsman to conduct
PI.

I: Whether the
Ombudsman has the
power to conduct PI. YES.

Calingin v. Desierto LB: Under the Constitution, the Special Prosecutor is a mere subordinate of the Ombudsman and
can investigate and prosecute cases only upon the latter’s authority or orders

LB: The Special Prosecutor has the power and authority under the supervision and
control of the Ombudsman, to conduct preliminary investigation and prosecute criminal
cases before the Sandiganbayan and perform such other duties assigned to him by the
Ombudsman.

LB: —The Office of the Special Prosecutor is but a mere subordinate of the Ombudsman and is
subject to his supervision and control.
 In Perez v. Sandiganbayan, 503 SCRA 252 (2006), this Court held that control means
“the power of an officer to alter or modify or nullify or set aside what a subordinate
officer had done in the performance of his duties and to substitute the judgment of the
former for that of the latter.”

AP: Clearly, in disapproving the recommendation of the Office of the Special Prosecutor to
dismiss all the charges against petitioner and his co-accused, respondent Ombudsman did not act
with grave abuse of discretion.

Article X
Local Government

Section 2

A. Local autonomy

Lina. Jr. v. Pano D: (1) While a policy statement expressing the local government’s objection to the lotto is valid,
as it is part of the local government’s autonomy to air its views which may be contrary to that of
the national government’s, this freedom to exercise contrary views does not mean that
local governments may actually enact ordinances that go against laws duly enacted by
Congress.

(2) What the national legislature allows by law, such as lotto, a provincial board may
not disallow by ordinance or resolution.

AP: The game of lotto is a game of chance duly authorized by the national government through
an Act of Congress. Republic Act 1169, as amended by Batas Pambansa Blg. 42, is the law which
grants a franchise to the PCSO and allows it to operate the lotteries, x x x This statute remains
valid today. While lotto is clearly a game of chance, the national government deems it wise and
proper to permit it. Hence, the Sangguniang Panlalawigan of Laguna, a local government unit,
cannot issue a resolution or an ordinance that would seek to prohibit permits.

Other Doctrines:

(1)In our system of government, the power of local government units to legislate and enact
165
ordinances and resolutions is merely a delegated power coming from Congress.
(2) Sections 2 (c) and 27 (consultation requirement) of the Local Government Code (Republic Act
7160) apply only to national programs and/or projects which are to be implemented in a
particular local community—lotto is neither a program nor a project of the national
government, but of a charitable institution, the PCSO, and it is far fetched to say that lotto falls
within the contemplation of aforesaid legal provisions.

Dadole v. COA D: Although our Constitution guarantees autonomy to local government units, the exercise of
local autonomy remains subject to the power of control by Congress and the power of
Petitioner judges argue supervision by the President.
that LBC 55 is void for
infringing on the local The President can only interfere in the affairs and activities of a local government unit
autonomy of Mandaue if he or she finds that the latter has acted contrary to law.
City by dictating a - This is the scope of the President’s supervisory powers over local government units.
uniform amount that a - Hence, the President or any of his or her alter egos cannot interfere in local affairs as long as
local government unit can the concerned local government unit acts within the parameters of the law and the Constitution.
disburse as additional - Any directive therefore by the President or any of his or her alter egos seeking to alter
allowances to judges the wisdom of a law—conforming judgment on local affairs of a local government unit is a
stationed therein. They patent nullity because it violates the principle of local autonomy and separation of
maintain that said powers of the executive and legislative departments in governing municipal
circular is not supported corporations.
by any law and therefore
goes beyond the AP: DBM’s LBC 55 is void. It provides that the additional monthly allowances to be given by a
supervisory powers of the local government unit should not exceed P1,000 in provinces and cities and P700 in
President. They further municipalities. Section 458, par. (a)(l)(xi), of RA 7160, the law that supposedly serves as the
allege that said circular is legal basis of LBC 55, allows the grant of additional allowances to judges “when the finances of
void for lack of the city government allow.” The said provision does not authorize setting a definite
publication. maximum limit to the additional allowances granted to judges.

Thus, the finances of a city government may allow the grant of additional allowances higher than
P1,000 if the revenues of the said city government exceed its annual expenditures.

Setting a uniform amount for the grant of additional allowances is an inappropriate way of
enforcing the criterion found in Section 458, par. (a)(l)(xi), of RA 7160.

The DBM over-stepped its power of supervision over local government units by
imposing a prohibition that did not correspond with the law it sought to implement. In
other words, the prohibitory nature of the circular had no legal basis.

San Juan v. CSC D: WHERE A LAW IS CAPABLE OF TWO INTERPRETATIONS, ONE IN FAVOR OF
CENTRALIZED POWER IN MALACAÑANG AND THE OTHER BENEFICIAL TO LOCAL
Vacant Prov’l Budget AUTONOMY, THE SCALES MUST BE WEIGHED IN FAVOR OF AUTONOMY.
Officer post in Rizal –
Gov. nominated Dalisay,
but DBM Regional APP: When the Civil Service Commission interpreted the recommending power of the Provincial
Director Abella did not Governor as purely directory, it went against the letter and spirit of the constitutional provisions
endorse Dalisay and on local autonomy. If the DBM Secretary jealously hoards the entirety of budgetary powers and
recommended Cecilia. ignores the right of local governments to develop self- reliance and resoluteness in the handling
Cecilia was approved by of their own funds, the goal of meaningful local autonomy is frustrated and set back.
DBM -- DBM’s act is ultra
vires, it can only appoint
those recommended by DBM Circular is ultra vires and is, accordingly, set aside. The DBM may appoint only from
the Gov. the list of qualified recommendees nominated by the Governor. If none is qualified, he must
return the list of nominees to the Governor explaining why no one meets the legal requirements
and ask for new recommendees who have the necessary eligibilities and qualifications.

Madramang Doctrine:

The value of local governments as institutions of democracy is measured by the degree of


autonomy that they enjoy. Local assemblies of citizens constitute the strength of free nations. A
people may establish a system of free government but without the spirit of municipal institutions,
it cannot have the spirit of liberty. Our national officials should not only comply with the
constitutional provisions on local autonomy but should also appreciate the spirit of liberty upon
which these provisions are based.

Laguna Lake D: The LLDA, by virtue of its special charter, obviously has the responsibility to protect the
Development Authority inhabitants of the Laguna Lake region from the deleterious effects of pollutants emanating from
v. Court of Appeals the discharge of wastes from the surrounding areas. In carrying out the aforementioned declared
policy, the LLDA is mandated, among others, to pass upon and approve or disapprove all plans,
Tala Estate, Caloocan programs, and projects proposed by local government offices/agencies within the region, public
open garbage dumpsite corporations, and private persons or enterprises where such plans, programs and/or projects are
was complained to LLDA related to those of the LLDA for the development of the region.
due to harmful effects;
LLDA issued cease and While it is a fundamental rule that an administrative agency has only such powers as are
desist order which expressly granted to it by law, it is likewise a settled rule that an administrative agency has also
Caloocan city objected to; such powers as are necessarily implied in the exercise of its express powers.26 In the exercise,
therefore, of its express powers under its charter, as a regulatory and quasi-judicial
Cease and desist order is body with respect to pollution cases in the Laguna Lake region, the authority of the
LLDA to issue a “cease and desist order” is, perforce, implied. Otherwise, it may well be
166
valid, LLDA has mandate reduced to a “toothless” paper agency.
under National Law, CDO
is an implied power

AP: The cease and desist order issued by the LLDA requiring the City Government of Caloocan
to stop dumping its garbage in the Camarin open dumpsite found by the LLDA to have been done
in violation of Republic Act No. 4850, as amended, and other relevant environment laws is valid.

The issuance, therefore, of the cease and desist order by the LLDA, as a practical matter of
procedure under the circumstances of the case, is a proper exercise of its power and authority
under its charter and its amendatory laws. Had the cease and desist order issued by the LLDA
been complied with by the City Government of Caloocan as it did in the first instance, no further
legal steps would have been necessary.

Magtajas v. Pryce D: The rationale of the requirement that the ordinances should not contravene a statute is
Properties obvious as municipal governments are only agents of the national government and that
the delegate cannot be superior to the principal or exercise powers higher than those of
(Cagayan de Oro PAGCOR the latter.
Casino; LGU, women’s
and youth groups - It is a heresy to suggest that the local government units can undo the acts of Congress, from
objected. LGU passed an which they have derived their power in the first place, and negate by mere ordinance the
ordinance prohibiting mandate of the statute.
issuance of business
permits to casinos. AP: the ordinances violate P.D. 1869, which has the character and force of a statute, as well as
Ordinance not valid) the public policy expressed in the decree allowing the playing of certain games of chance despite
the prohibition of gambling in general.

Other Doctrines:

(1)Congress retains control of the local government units although in significantly reduced degree
now than under the previous Constitutions.
- The power to create still includes the power to destroy. The power to grant still includes the
power to withhold or recall. True, there are certain notable innovations in the Constitution, like
the direct conferment on the local government units of the power to tax, which cannot now be
withdrawn by mere statute. By and large, however, the national legislature is still the principal of
the local government units, which cannot defy its will or modify or violate it.

(2) The tests of a valid ordinance are well established. A long line of decisions has held that
to be valid, an ordinance must conform to the following substantive requirements:
1) It must not contravene the constitution or any statute;
2) It must not be unfair or oppressive;
3) It must not be partial or discriminatory;
4) It must not prohibit but may regulate trade;
5) It must be general and consistent with public policy;
6) It must not be unreasonable.

League of Provinces of D: The Court has clarified that the constitutional guarantee of local autonomy in the
the Philippines v. Constitution [Art X, Sec. 2] refers to the administrative autonomy of local government
DENR units or, cast in more technical language, the decentralization of government authority.
- It does not make local governments sovereign within the State .
Applications for Quarry - Administrative autonomy may involve devolution of powers, but subject to limitations like
Permits by several following national policies or standards, and those provided by the Local Government
individuals with PENRO Code, as the structuring of local governments and the allocation of powers, responsibilities, and
were granted for the resources among the different local government units and local officials have been placed by the
same area that Atlantic Constitution in the hands of Congress under Section 3, Article X of the Constitution.
Mine and Trading also
applied for. Bulacan Gov. AP: DENR’s act is valid. The Local Government Code did not fully devolve the enforcement of the
issued the permits for small-scale mining law to the provincial government, as its enforcement is subject to the
individuals, upon protest supervision, control and review of the DENR, which is in charge, subject to law and higher
of AMTC, DENR favoured authority, of carrying out the State’s constitutional mandate to control and supervise the
AMTC; DENR’s act is exploration, development, utilization of the country’s natural resources.
valid, LGU’s autonomy is
merely administrative, a
decentralization of
government authority;
although it may involve
devolution of powers, it is
subject to limitations
under national laws and
LGC.
Rama v. Moises D: In Navarro v. Ermita, the Court has pointed out that the central policy considerations in the
creation of local government units are economic viability, efficient administration, and capability
(b)  Appointing authority. to deliver basic services to their constituents. These considerations must be given importance as
—The person empowered they ensure the success of local autonomy. It is accepted that the LGUs, more than the
to appoint National Government itself, know the needs of their constituents, and cater to such
needs based on the particular circumstances of their localities. Where a particular law
the members of the or statute affecting the LGUs infringes on their autonomy, and on their rights and
Board of Directors of a powers to efficiently and effectively address the needs of their constituents, we should
local water district, lean in favor of their autonomy, their rights and their powers.
depending upon the
geographic coverage and AP: Sec. 3(b) P.D. No. 198 (see facts) should be partially struck down for being repugnant to the
167
population make-up of local autonomy granted by the 1987 Constitution to LGUs, and for being inconsistent with R.A.
the particular district. In No. 7160 (1991 Local Government Code) and related laws on local governments.
the event that more
than seventy- five At the time of the enactment of P.D. No. 198, Cebu City was still a component city of Cebu
percent of the total Province. Section 328 of B.P. Blg. 51 reclassified the cities of the Philippines based on well-
active water service defined criteria. Cebu City thus became an HUC, which immediately meant that its inhabitants
connections of a local were ineligible to vote for the officials of Cebu Province. Later on, Cebu City, already an HUC, was
water district are further effectively rendered independent from Cebu Province .
within the boundary of
any city or
municipality, the Water and its efficient supply are among the primary concerns of every LGU. Issues that tend to
appointing authority reduce or diminish the authority of the boards of directors to manage the water districts are
shall be the mayor of imbued with public interest. Bearing this in mind, and recalling that the MCWD had been
that city or established from the erstwhile Osmeña Waterworks Systems (OWS) without any investment or
municipality, as the contribution of funds and material from the Province of Cebu towards the creation and
case may be; maintenance of OWS and the MCWD, and considering that it had always been the City
otherwise, the Mayor of the City of Cebu who appointed the members of the MCWD Board of Directors
appointing authority regardless of the percentage of the water subscribers, the provision on making the
shall be the governor provincial governor the appointing authority in the event that the LGU failed to meet the 75%
of the province within connection requirement is a violation of local autonomy, Cebu City being already independent
which the district is from the province of Cebu.
located. Xxx

Section 3

A. Local Government Code

Garcia v. Comelec D: There is nothing in the Constitution that will remotely suggest that the people have
the “sole and exclusive right to decide on whether to initiate a recall proceeding.” The
Recall of Bataan Gov. Constitution did not provide for any mode, let alone a single mode, of initiating recall elections.
Garcia – 1 year after his Neither did it prohibit the adoption of multiple modes of initiating recall elections.
election, about 146 - The mandate given by section 3 of Article X of the Constitution is for Congress to “enact a local
persons constituted government code which shall provide for a more responsive and accountable local government
themselves as structure through a system of decentralization with effective mechanisms of recall, initiative, and
Preparatory Recall referendum x x x” By this constitutional mandate, Congress was clearly given the power
Assembly complying with to choose the effective mechanisms of recall as its discernment dictates.
LGC requirements. Gov
said it is unconstitutional; AP: Using its constitutionally granted discretion, Congress deemed it wise to enact an alternative
PRAC is valid mode of initiating recall elections to supplement the former mode of initiation by direct action of
the people. Congress provided for a second mode of initiating the recall process through a
Note: Petitioners have preparatory recall assembly (PRAC) which in the provincial level is composed of all mayors,
embraced the view that vice-mayors and Sanggunian members of the municipalities and component cities. Alternative
initiation by the PRAC is mode of initiating the recall process thru an assembly was adopted (a) to diminish the difficulty of
not initiation by the initiating recall thru the direct action of the people; and (b) to cut down on its expenses
people. This is a
misimpression for Congress has made its choice as called for by the Constitution and it is not the prerogative of this
initiation by the PRAC is Court to supplant this judgment. The choice may be erroneous but even then, the remedy against
also initiation by the a bad law is to seek its amendment or repeal by the legislative.
people, albeit done
indirectly through their The alternative mode of initiating recall proceedings thru a preparatory recall assembly is,
representatives. It is not however, an innovative attempt by Congress to remove impediments to the effective exercise by
constitutionally the people of their sovereign power to check the performance of their elected officials . The
impermissible for the power to determine this mode was specifically given to Congress and is not proscribed
people to act through by the Constitution. Hence, the preparatory recall assembly is valid.
their elected
representatives. Nothing
less than the paramount
task of drafting our
Constitution is delegated
by the people to their
representatives, elected
either to act as a
constitutional convention
or as a congressional
constituent assembly

Section 4

A. General supervision

Ganzon v. Court of D: Since local governments remain accountable to the national authority, the latter
Appeals may, by law, and in the manner set forth therein, impose disciplinary action against
local officials;
Iloilo City Mayor Ganzon
was suspended by DILG (1) in spite of autonomy, the Constitution places the local governments under the general
due to complaints of supervision of the Executive. It is noteworthy finally, that the Charter allows Congress to include
misconduct and in the local government code provisions for removal of local officials, which suggest that Congress
misfeasance in office; may exercise removal powers, and as the existing Local Government Code has done, delegate its

168
Mayor invoked local exercise to the President.
autonomy; Local
autonomy is not self- (2) The petitioners are under the impression that the Constitution has left the President mere
executing, based on LGC, supervisory powers, which supposedly excludes the power of investigation, and denied her
President’s control, which allegedly embraces disciplinary authority. It is a mistaken impression because
supervisory powers legally, “supervision” is not incompatible with disciplinary authority. x x x “Control” has been
includes power to defined as “the power of an officer to alter or modify or nullify or set aside what a
discipline; but subordinate officer had done in the performance of his duties and to substitute the
suspension may only judgment of the former for test of the latter.” “Supervision” on the other hand means
be for 60 days max. “overseeing or the power or authority of an officer to see that subordinate officers
perform their duties.” however, “investigating” is not inconsistent with “overseeing”,
although it is a lesser power than “altering”.

AP: The Secretary of Local Government, as the alter ego of the president, in suspending Ganzon
is exercising a valid power. He however overstepped by imposing a 600 day suspension.

Note: The sole objective of a suspension, is simply “to prevent the accused from hampering the
normal cause of the investigation with his influence and authority over possible witnesses”60 or
to keep him off “the records and other evidence.” It is a means, and no more, to assist
prosecutors in firming up a case, if any, against an erring local official. Under 62 the Local
Government Code, it can not exceed sixty days, which is to say that it need not be exactly sixty
days long if a shorter period is otherwise sufficient, and which is also to say that it ought to be
lifted if prosecutors have achieved their purpose in a shorter span.

Section 5

A. Sources of Revenue

Manila Electric v. D: Under the now prevailing Constitution, where there is neither a grant nor a prohibition
Province of Laguna by statute, the tax power (of LGUs) must be deemed to exist although Congress may
provide statutory limitations and guidelines.
Laguna franchise tax to - The basic rationale for the current rule is to safeguard the viability and self-sufficiency of local
MERALCO’ paid on protest government units by directly granting them general and broad tax powers. Nevertheless, the
claiming LGU’s franchise fundamental law did not intend the delegation to be absolute and unconditional; the
tax contravenes PD 551; constitutional objective obviously is to ensure that, while the local government units
Local franchise tax is are being strengthened and made more autonomous, the legislature must still see to it
Valid that
- (a) the taxpayer will not be over-burdened or saddled with multiple and unreasonable
impositions;
- (b) each local government unit will have its fair share of available resources;
- (c) the resources of the national government will not be unduly disturbed; and
- (d) local taxation will be fair, uniform, and just.

AP: The Local Gov 1991 Code explicitly authorizes provincial governments, notwithstanding “any
exemption granted by any law or other special law, x x x (to) impose a tax on businesses
enjoying a franchise.

Here, since the LGC expressly provides that LGUs may impose tax on business enjoying franchise
notwithstanding any exemption granted by any law or other special law, the Province of Laguna
may validly impose a franchise tax on MERALCO.

Yamane v. BA Lepanto D: The power of LGUs to impose taxes within its territorial jurisdiction is derived from the
Constitution itself, which recognizes the power of these units “to create its own sources of
City Treasurer of Makati revenue and to levy taxes, fees, and other charges subject to such guidelines and limitations as
assessed BA Lepanto the Congress may provide, consistent with the basic policy of local autonomy.
Condominium
Corporation for business GR: Condominium corporations are exempt from local business taxation under the Local
taxes based on dues Government Code, irrespective of any local ordinance that seeks to declare otherwise.
collected from unit
owners to defray Exception: If unit owners of a condominium band together to engage in activities for profit under
expenses for common the shelter of the condominium corporation.
areas.
BA Lepanto protested,
claiming that it was not AP: The City Treasurer has not posited the claim that the Corporation is engaged in
engaged in business, business activities beyond the statutory purposes of a condominium corporation but
trading or any based the assessment solely on the Corporations’ collection of assessments from unit owners,
commercial activity. being utilized to defray necessary expenses for its project and common areas.
That it was not organized
for profit but rather Hence, the assailed tax assessment has no basis under the LGC or the Makati Revenue
merely to hold title over Code, and the insistence of the void tax constitute an attempt to the deprivation of property
common areas, manage without due process of law.
the condominium and
hold title to parcels of
land.
PFDA v. CA D: As a rule, Philippine Fisheries Development Authority (PFDA), being an
instrumentality of the national government, is exempt from real property tax but the
Navotas LGU assessed exemption does not extend to the portions of the Fishing Port Complex that it manages
and operates that were leased to taxable or private persons and entities for their
169
Philippine Fisheries beneficial use.
Development Authority
for real estate taxes;
since taxes were not
paid, notice of sale by AP: Here, Exemption does not apply to the portions of the IFPC, which PFDA leased to private
public auction were entities. With respect to these properties, PFDA is liable to pay property tax. Nonetheless, the
delivered to PFDA IFPC, being a property of public dominion cannot be sold at public auction to satisfy the tax
delinquency.
Smart Communications D: LGC defines:
v. Municipality of “Charges” as referring to pecuniary liability, as rents or fees against persons or property.
Malvar “Fee” means “a charge fixed by law or ordinance for the regulation or inspection of a
business or activity.
Smart Telco Tower was
charged with fees by the AP: Ordinance 18 imposed fees (not taxes) and it is valid.
LGU based on Ordinance
No. 18, Smart didn’t pay, Since the primary purpose of Ordinance No. 18 is to regulate the “placing, stringing, attaching,
hence LGU ordered installing, repair and construction of all gas mains, electric, telegraph and telephone wires,
closure of the Tower; conduits, meters and other apparatus” listed therein, which included Smart’s telecommunications
Ordinance 18 is valid tower, the purpose is to regulate the enumerated activities.
It is primarily regulatory in nature. – Not revenue-raising.
Therefore, they are fees. – Not taxes.
City of Cagayan de Oro D: The term "taxes" has been defined by case law as "the enforced proportional
v. Cagayan Electric contributions from persons and property levied by the state for the support of
Power & Light Co, Inc. government and for all public needs."
- While, under the Local Government Code, a "fee" is defined as "any charge fixed by law or
- CEPALCO assailed an ordinance for the regulation or inspection of a business or activity.”
ordinance which
imposed an annual In Smart Comm. vs. Malvar, the Court was able to address the issue after a simple reading of
Mayor's Permit Fee of the ordinance's whereas clauses, which revealed that the primary purpose of the
Five Hundred Pesos ordinance was to regulate cell sites or telecommunications towers, including Smart's. Thus, since
(P500.00) on every the whereas clauses showed that the ordinance served a regulatory purpose, it was ruled that the
electric or case involved a fee and not a tax.
telecommunications
post belonging to public
utility companies
operating in the city. From jurisprudential and statutory definitions, it can be gleaned that the purpose of an
imposition will determine its nature as either a tax or a fee.
- If the purpose is primarily revenue, or if revenue is at least one of the real and
- It argued that, substantial purposes, then the exaction is properly classified as an exercise of the
assuming the imposition power to tax.
was a valid regulatory - On the other hand, if the purpose is primarily to regulate, then it is deemed an exercise of police
fee, it violated the power in the form of a fee, even though revenue is incidentally generated. 
legislative franchise that - Stated otherwise, if generation of revenue is the primary purpose, the imposition is a
specifically exempted tax but, if regulation is the primary purpose, the imposition is properly categorized as
the electricity distributor a regulatory fee.
from taxes or fees
assessed by Cagayan de
Oro City. AP: the ordinance in this case serves a regulatory purpose and is, hence, an exercise of
police power. Nowhere in the text of the ordinance is it shown that it was enacted to raise
revenue. As in Smart Communications, the fee is not imposed on the structure itself, but on the
activity subject of government regulation, which is the installation and establishment of utility
posts. Thus, the ordinance imposes a fee since it was enacted pursuant to the city's police power
and serves to regulate, not to raise revenue.

****
As to whether or not the amount of P500.00 collected annually on a per post basis violated Sec.
147 of the Local Government Code, which provides that fees must be commensurate with the
cost of regulation, inspection, and licensing

The presumption of constitutionality, in its most basic sense, only means that courts, in passing
upon the validity of a law, will afford some deference to the statute and charge the party assailing
it with the burden of showing that the act is incompatible with the constitution.

AP: CEPALCO never pointed out the particulars of the fee's unreasonableness. While it stated that
the ordinance only ordered the inspection and inventory of electric poles erected in the city, it
never even bothered to allege, much less prove, the cost of such inspection and inventory. 

Without evidence indicating that the amount of the Mayor's Permit Fee is disproportionate to the
cost of regulation, inspection, and licensing of utility poles located in Cagayan de Oro City, the
ordinance is thus valid.

Metropolitan D: (1) A government instrumentality exercising corporate powers is not liable for the
Waterworks Sewerage payment of real property taxes on its properties unless it is alleged and proven that the
System v. Local beneficial use of its properties been extended to a taxable person.
Government of Quezon
City (2) The Local Government Code provides two (2) specific limitations on local
government units' power of taxation.  
- MWSS received several
Final Notices of Real - The first limitation provides a general rule, that is, that local government units cannot levy
Property Tax any taxes, fees, or charges of any kind on the national government or its agencies and
Delinquency from the instrumentalities. The provision, however, also provides for an exception: "[u]nless
Local Government of otherwise provided herein." The implication, therefore, is that while a government agency
Quezon City. or instrumentality is generally tax-exempt, the Local Government Code may provide
170
for instances when it could be taxable.
- Local Government of - The second limitation is provided for under Section 234 of the Local Government Code. Under
Quezon City warned it Section 234(a), the general rule is that any real property owned by the Republic or its political
that failure to pay would subdivisions is exempt from the payment of real property tax "except when the beneficial
result in the issuance of use thereof has been granted, for consideration or otherwise, to a taxable person."
warrants of levy against The implication is that real property, even if owned by the Republic or any of its political
its properties (na subdivisions, may still be subject to real property tax if the beneficial use of the real property
ginawa nga Nya was granted to a taxable person.
eventually)
AP: The Executive and Legislative Branches have already categorized MWSS not as a
- MWSS filed a TRO on government-owned and controlled corporation but as a Government Instrumentality with
the auction sale of its Corporate Powers/Government Corporate Entity
properties. It claims
that its real properties - Privileges enjoyed by these Government Instrumentalities with Corporate Powers/Government
in Quezon City were Corporate Entities should necessarily also extend to petitioner.
exclusively devoted to - Hence, petitioner's real property tax exemption under Republic Act No. 623481  is still
public use, and thus, valid as the proviso of Section 234 of the Local Government Code is only applicable to
were exempt from real government-owned and -controlled corporations.
property tax
Thus, petitioner is not liable to respondent Local Government of Quezon City for real property
taxes, except if the beneficial use of its properties has been extended to a taxable person.

Respondents have not alleged that the beneficial use of any of petitioner's properties was
extended to a taxable person. In the absence of any allegation to the contrary, petitioner's
properties in Quezon City are not subject to the levy of real property taxes.

Section 6

A. Share in National Taxes

Pimentel v. Aguirre D: A basic feature of local fiscal autonomy is the automatic release of the shares of
LGUs in the national internal revenue. This is mandated by no less than the Constitution.
President’s AO 372 -
required government The Local Government Code specifies further that the release shall be made directly to the LGU
departments and concerned within five (5) days after every quarter of the year and "shall not be subject to any lien
agencies, including state or holdback that may be imposed by the national government for whatever purpose."
universities and colleges,
government-owned and The provision in the Local Government Code providing for such release uses the word “ shall” and
controlled corporations as a rule, the term “shall” is a word of command that must be given compulsory meaning.
and local governments
units to identify and
implement measures in
FY 1998 to reduce total AP: In this case, Sec. 4 of AO 372 is unconstitutional for it orders the withholding of the
expenditures for the year LGU’s IRA (Internal Revenue Allotment).
by at least 25% of
authorized regular
The withholding of 10% of the LGU’s IRA pending assessment and evaluation by the Development
appropriations for non-
Budget Coordinating Committee, although temporary is equivalent to a holdback, which means
personal services items
“something held back or withheld, often temporarily,” contravenes the Constitution.
and to withhold 10% of
the LGU IRA; AO 372 is
unconstitutional The president may not withhold or alter any authority or power given to them by the Constitution
and the law.
Batangas v. Executive D: Section 6, Article X of the Constitution when parsed, it would be readily seen that this
Secretary provision mandates that
(1) the LGUs shall have a “just share” in the national taxes;
Local Government (2) the “just share” shall be determined by law; and
Service Equalization (3) the “just share” shall be automatically released to the LGUs.
Fund, through EO 48 –
1999, 2000, 2001 GAA - The LGUs are not required to perform any act to receive the “just share” accruing to
them from the national coffers—the “just share” of the LGUs shall be released to them
P5 Billion of the Internal “without need of further action”;
Revenue Allotment (IRA)
was uniformly earmarked “Automatic” means “involuntary either wholly or to a major extent so that any activity of the
for the LGSEF; and will is largely negligible; of a reflex nature; without volition; mechanical; like or suggestive of an
automation.
2) It imposed conditions
and guidelines for its AP: the Local Government Service Equalization Fund (LGSEF) is part of the IRA or "just
release: share" of the LGUs in the national taxes.

a.) LGSEF could not be To subject its distribution and release to the vagaries of the implementing rules and regulations,
released without the including the guidelines and mechanisms unilaterally prescribed by the Oversight Committee from
Oversight Committees time to time, as sanctioned by the assailed provisos in the GAAs of 1999, 2000 and 2001 and the
prior approval; OCD resolutions, makes the release not automatic, a flagrant violation of the constitutional and
b) The LGUs should statutory mandate that the "just share" of the LGUs "shall be automatically released to them.”
identify the projects
eligible for funding based Note:
on the criteria laid down
by the Oversight
Committee; The only possible exception to the mandatory automatic release of the LGUs’ IRA is if the
171
c) The LGUs should national internal revenue collections for the current fiscal year is less than 40 percent
submit their project of the collections of the preceding third fiscal year, in which case what should be
proposals to the DILG for automatically released shall be a proportionate amount of the collections for the current fiscal
appraisal; year. The adjustment may even be made on a quarterly basis depending on the actual collections
d) The project proposals of national internal revenue taxes for the quarter of the current fiscal year.
that passed the appraisal
of the DILG to be AP: In the instant case, however, there is no allegation that the national internal revenue tax
submitted to the collections for the fiscal years 1999, 2000 and 2001 have fallen compared to the preceding three
Oversight Committee for fiscal years.
review, evaluation and
approval. EO 48 is
unconstitutional
Mandanas v. Ochoa D: In the computation of the LGU's IRA the basis should include all collections
considered as national taxes.

AP: Thus, the Bureau of Customs collection on excise taxes, VAT and DSTs should form part of
the base from which the IRA should be computed because they constitute Natl Internal Revenue
Taxes. IRA being a just share of the LGU from the national taxes.

Section 7

A. Share in the Utilization of National Wealth

Republic v. Provincial D: “An LGU’s territorial jurisdiction is not necessarily co-extensive with its exercise or
Government of assertion of powers.
Palawan - To hold otherwise may result in condoning acts that are clearly ultra vires.
- It may lead to, the words of the Republic, LGUs ‘rush(ing) to exercise its powers and
Province of Palawan is functions in areas rich in natural resources even if outside its boundaries) with the
not entitled to share in intention of seeking a share in the proceeds of its exploration’ – a situation that
the proceeds of the ‘would sow conflict not only among the local government units and the national
Camago-Malampaya government but worse, between and among local government units.’”
natural gas project.
Palawan premised its AP: The Court did not subscribe to Palawan’s argument posited by the Province of Palawan that
claim on the ground that the national wealth, the proceeds from which the State is mandated to share with the LGUs, shall
it has territorial be wherever the local government exercises any degree of jurisdiction.
jurisdiction over the
Camago-Malampaya Existing laws do not include the Camago-Malampaya reservoir within the area or territorial
reservoir. jurisdiction of the Province of Palawan. It stressed that “As defined in its organic law, the province
of Palawan comprises merely of islands. The continental shelf, where the Camago-Malamapaya
reservoir is loated, was clearly not included in its territory.

Palawan never alleged in which of its municipalities or component cities and barangays the
Camago-Malampay reservoir is located, militating against its claim that the area form part of its
territory.

The Court further held that


1)estoppel does not lie against the Republic as previous acknowledgments of Palawan’s
share were based on the mistaken assumption that it it is entitled to the said
allocation,
2)Section1, Article X of the 1987 Constitution did not apportion the entire Philippine territory
among the LGUs such that at any one time, a body of water or a piece of land should belong to
some province or city,
3)the United Nations Convention on the Law of the Seas (UNCLOS) did not confer on LGUs their
own continental shelf as this pertains to the coastal state.

Section 8

A. Term of Office

Borja v. Comelec - 295 D: The three-term limit on a local official is to be understood to refer to terms for which
SCRA 157 the official concerned was elected.

Capco, VM in 1988, but Conversely, if he is not serving a term for which he was elected because he is simply continuing
when Mayor Borja, Capco the service of the official he succeeds, such official cannot be considered to have fully
served as Mayor by served the term notwithstanding his voluntary renunciation of office prior to its
succession til 1992; expiration.
Capco run for Mayor in
1992, and reelected in AP: Thus, a person who was elected Vice Mayor in 1988 and who, because of the death of the
1995; Petitioner Borja Mayor, became Mayor in 1989, may still be eligible to run for the position of Mayor in
opposed Capco’s 1998 1998, even if elected as such in 1992 and 1995.
candidacy for violating 3
consecutive term rule.
Capco’s 1989 succession
to Borja cannot be
considered a full term,

172
hence eligible to run.
Socrates v. Comelec - D: The intent in Section 8, Article X of the Constitution and under Section 43 (b) of RA No.
391 SCRA 453 7160 is that the first part provides that an elective local official cannot serve for more than
three consecutive terms. The clear intent is that only consecutive terms count in
Recall of Socrates in determining the three-term limit rule.
Puerto Princesa,
Hagedorn who served The second part states that voluntary renunciation of office for any length of time does not
from 1992-2001 run interrupt the continuity of service. The clear intent is that involuntary severance from office
during recall and won. for any length of time interrupts continuity of service and prevents the service before
Socrates invoked 3 and after the interruption from being joined together to form a continuous service or
consecutive terms rule consecutive terms.

After three consecutive terms, an elective local official cannot seek immediate reelection for a
fourth term. The prohibited election refers to the next regular election for the same
office following the end of the third consecutive term.
- Any subsequent election, like a recall election, is no longer covered by the prohibition
for two reasons.
- First, a subsequent election like a recall election is no longer an immediate reelection after three
consecutive terms.
- Second, the intervening period constitutes an involuntary interruption in the continuity of
service.

The Constitution, however, does not prohibit a subsequent reelection for a fourth term as long as
the reelection is not immediately after the end of the third consecutive term.
- A recall election mid-way in the term following the third consecutive term is a
subsequent election but not an immediate reelection after the third term. The winner in
the recall election cannot be charged or credited with the full term of three years for purposes of
counting the consecutiveness of an elective official’s terms in office.

AP: In the case of Hagedorn, his candidacy in the recall election on September 24, 2002 is
not an immediate reelection after his third consecutive term which ended on June 30,
2001. The immediate reelection that the Constitution barred Hagedorn from seeking
referred to the regular elections in 2001. Hagedorn did not seek reelection in the 2001
elections.

In Hagedorn’s case, the nearly 15-month period he was out of office, although short of a full term
of three years, constituted an interruption in the continuity of his service as mayor. The
Constitution does not require the interruption or hiatus to be a full term of three years. The clear
intent is that interruption for any length of time, as long as the cause is involuntary, is
sufficient to break an elective local officials continuity of service.

Rivera v. Comelec - D: For the three-term limit for elective local government officials to apply, two conditions or
523 SCRA 41 requisites must concur
(1) that the official concerned has been elected for 3 consecutive terms in the same local
Mabalacat, Pampanga government post, and
Mayor Boking filed (2) that he has fully served 3 consecutive terms.
candidacy in 2004; this is
after serving as mayor Where a proclaimed candidate had served the full term of office but was either disqualified or
from 1995- 1998, de his/her proclamation voided only after the term of the contested office had expired, such service
facto mayor from 1998- is counted and is legally taken as service for a full term in contemplation of the three
2001 and as elected (3)-term rule
mayor from 2001-2004,
3-consecutive term rule
was invoked; 3-
consecutive term rule AP: The fact that he was ousted as Mayor on his 2nd term in the electoral protest does
was validly applied not constitute an interruption in serving the full term.
- He was proclaimed elected in 1998, assumed position and served as mayor until 2001.
- He was mayor for the entire period notwithstanding the decision of the RTC in the electoral case
ousting him as mayor.
- Whether as caretaker or as de facto officer, he exercised the powers and enjoyed the
prerequisites of the office.
Aldovino, Jr. v. D: Preventive suspension or temporary inability or disqualification to exercise the
COMELEC - GR No. functions of an elective post, even if involuntary, should not be considered an effective
184836, December 23, interruption of a term because it does not involve the loss of title to office or at least an
2009 effective break from holding office—the office holder, while retaining title, is simply barred
from exercising the functions of his office for a reason provided by law
Lucena City Councilor
Asilo served from 1998 to The best indicator of the suspended official’s continuity in office is the absence of a
2007 [3 terms], on this permanent replacement and the lack of the authority to appoint one since no vacancy
third term, he was exists.
suspended for 90 days;
he filed for candicacy in AP: Here, Asilo’s 90 day suspension did not interrupt his term. Hence, the 3-term rule is validly
2007; 3 consecutive term invoked.
rules was invoked, but he
countered, the
suspension was an
interruption to his full-
term. Preventive
suspension did not
interrupt the term,
hence, Asilo is
disqualified
173
Datu Alas kida vs. D: Neither the court nor the Congress can create a new term, shorten or extend the
Senate - G.R. No. term of office for local elective officials because it would be a violation of the constitution,
196271, February 28, which, as provided under Article X Section 8.
2012 The term of office of elective local officials, except barangay officials, which shall be determined
by law, shall be three years and no such official shall serve for more than three consecutive
- Pursuant to the terms. Neither the congress can extend the term of office through a holdover.
constitutional mandate
of synchronization of AP: Holdover Option is Unconstitutional.
elections, RA 10153 - the first option holdover for those who were elected in executive and legislative positions in the
postponed the regional ARMM during the 2008-2011 term as an option that Congress could have chosen because a
elections in the ARMM holdover violates Section 8, Article X of the Constitution.
and recognized the
President’s power to - Since elective ARMM officials are local officials, they are covered and bound by the three-year
appoint OICs to term limit prescribed by the Constitution ; they cannot extend their term through a
temporarily assume holdover.
these positions upon the
expiration of the terms
of the elected officials. If it will be claimed that the holdover period is effectively another term mandated by Congress,
- Constitutionality of RA the net result is for Congress to create a new term and to appoint the occupant for the new term.
10153 was questioned. This view like the extension of the elective term is constitutionally infirm because Congress
- Court issued a TRO cannot do indirectly what it cannot do directly, i.e., to act in a way that would effectively extend
enjoining the the term of the incumbents. This is effectively an act of appointment by Congress and an
implementation of RA unconstitutional intrusion into the constitutional appointment power of the President.
No. 10153 and ordering
the incumbent elective Hence, holdover whichever way it is viewed is a constitutionally infirm option that
officials of ARMM to Congress could not have undertaken.
continue to perform
their functions should
these cases not be
decided by the end of
their term

Abundo v. COMELEC- D: When due to an incorrect proclamation, the true winner of an election is not able to
G.R. No. 201715 , hold office while his election protest is pending, the service of such term will be
January 8, 2013 considered involuntarily interrupted, and shall not be counted as one of the terms
contemplated in the three-consecutive term limit under the Constitution.

***

· Interruption means temporary cessation, intermission or suspension. To interrupt is to


obstruct, thwart or prevent.
o Obstruction to the continuance of the service by the concerned elected official
by effectively cutting short the service of a term or giving a hiatus in the
occupation of the elective office.
· Renunciation” connotes the idea of waiver or abandonment of a known right.
o To renounce is to give up, abandon, decline or resign.
o Voluntary renunciation of the office by an elective local official would thus mean
to give up or abandon the title to the office and to cut short the service of the
term the concerned elected official is entitled to.

AP: In this case, Abundo had served as Mayor of Viga from 2001-2004.
- However, his opponent in the 2004 mayoral election was incorrectly proclaimed as the winner.
He filed an election protest, which was eventually successful.
- However, because of the time it took for his election protest to be decided, Abundo was only
able to serve 1 year and 1 month in the 2004-2007 term.
- After which he won and served again as mayor from 2007-2010. When he filed his certificate of
candidacy for Mayor for the 2010-2013, his opponent sought his disqualification.

The Supreme Court held that Abundo’s service of the 2004-2007 term had been
involuntarily interrupted, excluding such term from the 3-consecutive years term-limit
under the Constitution.

Other doctrines:

Involuntary Interruption/ Breaks continuity

1. When a permanent vacancy occurs in an elective position and the official merely assumed
the position pursuant to the rules on succession under the LGC, then his service for the
unexpired portion of the term of the replaced official cannot be treated as one full term as
contemplated under the subject constitutional and statutory provision that service cannot be
counted in the application of any term limit (Borja, Jr.). If the official runs again for the same
position he held prior to his assumption of the higher office, then his succession to said position
is by operation of law and is considered an involuntary severance or interruption (Montebon).

2. An elective official, who has served for three consecutive terms and who did not seek the
elective position for what could be his fourth term, but later won in a recall election, had an
interruption in the continuity of the official’s service. For, he had become in the interim, i.e., from
the end of the 3rd term up to the recall election, a private citizen (Adormeo and Socrates).

3. When a candidate is proclaimed as winner for an elective position and assumes office, his
174
term is interrupted when he loses in an election protest and is ousted from office, thus
disenabling him from serving what would otherwise be the unexpired portion of his term of office
had the protest been dismissed (Lonzanida and Dizon). The break or interruption need not be for
a full term of three years or for the major part of the 3-year term; an interruption for any length
of time, provided the cause is involuntary, is sufficient to break the continuity of service
(Socrates, citing Lonzanida).

Voluntary or Non-Interruption/ Does not break continuity

1. The abolition of an elective local office due to the conversion of a municipality to a city does
not, by itself, work to interrupt the incumbent official’s continuity of service (Latasa).

2. Preventive suspension is not a term-interrupting event as the elective officer’s continued


stay and entitlement to the office remain unaffected during the period of suspension, although he
is barred from exercising the functions of his office during this period (Aldovino, Jr.).

3. When an official is defeated in an election protest and said decision becomes final after said
official had served the full term for said office, then his loss in the election contest does not
constitute an interruption since he has managed to serve the term from start to finish. His full
service, despite the defeat, should be counted in the application of term limits because the
nullification of his proclamation came after the expiration of the term (Ong and Rivera).

The intention behind the three-term limit rule was not only to abrogate the “monopolization of
political power” and prevent elected officials from breeding “proprietary interest in their position”
but also to “enhance the people’s freedom of choice.” In the words of Justice Vicente V. Mendoza,
“while people should be protected from the evils that a monopoly of power may bring about, care
should be taken that their freedom of choice is not unduly curtailed.”

Halili v. Comelec – GR. D: The conversion of a municipality into a city does not constitute an interruption of the
No. 231643, January incumbent official's continuity of service. (Latasa v. COMELEC)
15, 2019 - to be considered as interruption of service, the "law contemplates a rest period during which the
local elective official steps down from office and ceases to exercise power or authority over the
Petitioner Morales was inhabitants of the territorial jurisdiction of a particular local government unit. “
elected as Mayor of the
Municipality of Mabalacat AP: the provisions of RA 10164 mean that the delineation of the metes and bounds of
for 3 consecutive terms: Mabalacat City did not change even by an inch the land area previously covered by the
1st term (2007 to 2010), Municipality of Mabalacat.
2nd term (2010 to 2013),
3rd term (2013 to 2016). Consequently, the inhabitants are the same group of voters who elected Morales to be their
On his 2nd term, the mayor for three consecutive terms, and over whom he held power and authority as their mayor.
Municipality of Mabalacat
became a component
city. On the 2016 Accordingly, Morales never ceased from acting and discharging his duties and
elections, he filed his CoC responsibilities as chief executive of Mabalacat, despite the conversion of the
for the position of Mayor Municipality of Mabalacat into Mabalacat City.
of Mabalacat City

Respondent alleged that


Morales was disqualified
to run for mayor, since
he was elected and had
served three consecutive
terms prior to the 2016
elections.

WON the conversion of


the Municipality of
Mabalacat into Mabalacat
City interrupted Morales'
service for the full term
for which he was elected.
NO

Section 10

A. Creation of Political Units

Tan v. Comelec - 142 D: A plebiscite for creating a new province should include the participation of the
SCRA 727 residents of the mother province for the plebiscite to conform to the constitutional
requirements.
- There was a plebiscite - It can be plainly seen that the cited constitutional provision makes it imperative that there be
conducted for the first obtained “the approval of a majority of votes in the plebiscite in the unit or units
creation of the province affected” whenever a province is created, divided or merged and there is substantial
of Negros del Norte. alteration of the boundaries.
- BP 885 was enacted,
creating the province of AP: The plebiscite is declared null and void.
Negros del Norte.
- Tan questioned the The boundaries of the existing province of Negros Occidental would necessarily be substantially
175
validity of the plebiscite altered by the division of its existing boundaries in order that there can be created the proposed
on the ground that: new province of Negros del Norte.
- Plain and simple logic will demonstrate than that two political units would be affected.
- In the plebiscite, the - The first would be the parent province of Negros Occidental because its boundaries would
voters of the remaining be substantially altered.
areas of Negros - The other affected entity would be composed of those in the area subtracted from the
Occidental (the mother mother province to constitute the proposed province of Negros del Norte.
province) were excluded
from participating. The phrase “subject to the approval by a majority of the votes in a plebiscite in the unit or units
- The province did not affected” must be construed to mean that the remaining areas in the province of Negros
comply with the area Occidental should have been allowed to participate in the said plebiscite. The reason is that cities
criterion prescribed in belonging to Negros Occidental will be added to Negros del Norte, thus Negros Occidental’s land
the LGC.- Negros del area will be dismembered. Certainly, the people of Negros Occidental should have been
Norte failed to meet the allowed to vote in the plebiscite as they are directly affected by the diminution in land
required land area of size of their province.
3,500 sq. km for it to
become a province.
Umali v. Comelec - G.R. D: In the conversion of a component city into a highly urbanized city, the registered
No. 203974, April 22, voters of the entire province should participate in the plebiscite, not just the residents
2014 of the concerned city.
- Sec. 453 of the LGC must be construed in harmony with Sec.10, Art.X of the Constitution thus,
- A plebiscite was to be the phrase “qualified voters herein” in the LGC means not only the qualified voters of the city
conducted for proposed to be converted to a HUC but also the voters of the political units directly affected by
conversion of the such conversion.
Cabanatuan city (a
component city of the AP: In this case, the plebiscite called for the conversion of Cabanatuan City to a HUC must be
province of Nueva Ecija) voted upon by the all the qualified voters of the entire province of Nueva Ecija.
to a highly urbanized
city.
- COMELEC issued a
resolution, providing
that only registered
residents of Cabanatuan
city should participate in
the plebiscite.
- The Governor of Nueva
Ecija (Umali) objected,
arguing that since the
conversion would affect
the whole province, the
whole province should
be allowed to
participate.

Section 11

A. Special Metropolitan Subdivisions

MMDA v. Bel-Air D: MMDA is not an LGU empower with police power, nor its charter allows it to enact
Village - 328 SCRA 836 laws hence it cannot issue order pursuant to police power absent an ordinance from a
local legislative body.
MMDA was ordered Bel-
Air Village Association - Unlike the legislative bodies of the local government units, there is no provision in RA 7924 that
(BAVA) to open Jupiter empowers the MMDA or its Council to enact ordinances, approve resolutions and appropriate
Street to the public funds for the general welfare of the inhabitants of Metro Manila.
because Jupiter Street is
owned by BAVA. AP: Thus, in the absence of an ordinance from the City of Makati, its own order to open
the street was invalid.

- The MMDA is a development authority.


- It is an agency created for the purpose of laying down policies and coordinating with the various
national government agencies, people’s organizations and the private sector for the efficient and
expeditious delivery of basic services in the vast metropolitan area.
- All its functions are administrative in nature.
- It is not a municipal corporation endowed with police power. Its creation was never submitted to
a plebiscite.
MMDA v. Garin - April D: The MMDA is only allowed to confiscate or suspend driver’s licenses if there is a
15, 2005 traffic law or regulation enacted by the legislature or those agencies to whom
legislative power have been delegated).
- The MMDA issued a - MMDA cannot legislate that it can confiscate driver’s licenses, only when the Congress
traffic violation receipt or LGU enacts a law or ordinance providing such, can it do so as an exercise of its
(TVR) to and confiscated power to implement traffic laws.
the driver’s license of
Atty. Dante O. Garin for AP: Sec. 5(f) of RA 7924 provides that the MMDA shall… “confiscate and suspend or revoke
illegal parking. driver’s licenses in the enforcement of such traffic laws and regulations…”
- Garin questions the
constitutionality of Sec. Where there is a traffic law or regulation validly enacted by the legislature or those agencies to
5(f) of RA 7924 (creating whom legislative powers have been delegated (the City of Manila in this case), the MMDA is not
the MMDA), which precluded to confiscate and suspend or revoke driver’s licenses in the exercise of its mandate of
authorizes it to confiscate transport and traffic management, as well as the administration and implementation of all traffic
and suspend or revoke enforcement operations, traffic engineering services and traffic education programs.
176
driver’s licenses in the
enforcement of traffic Therefore, by itself, the MMDA cannot legislate that it can confiscate driver’s licenses, only when
laws and regulations. the Congress or LGU enacts a law or ordinance providing such, can it do so as an exercise of its
power to implement traffic laws.

Other Doctrine:

A license to operate a motor vehicle is a privilege that the state may withhold in the exercise of
its police power. Police power is primarily lodged in the National Legislature.
- Through the Local Government Code of 1991, Congress delegated police power to the LGUs.
- Local Government is a political subdivision of a nation or state which is constituted by law and
has substantial control of local affairs.
- LGUs are provinces, cities, municipalities and barangays. MMDA is a body composed of
several LGUs. -- not a political unit of the Government.
- Its powers are limited to formulation, coordination, regulation, implementation, preparation,
management, monitoring, setting of policies, installation of a system and administration. Unlike
legislative bodies of LGUs, the MMDA is not empowered to enact ordinances, approve resolutions
and appropriate funds for the general welfare of the inhabitants of Metro Manila. All its functions
are administrative in nature.
MMDA v. Trackworks - D: Metropolitan Manila Development Authority (MMDA) has no power on its own to
GR No. 179554, dismantle, remove, or destroy the billboards, signages and other advertising media
December 16, 2009 installed on the Metro Rail Transit-3 (MRT3) structure by Trackworks
- MMDA’s powers are limited to the formulation, coordination, regulation, implementation,
- The Philippine preparation, management, monitoring, setting of policies, installing a system, and
government entered administration.
into a build-lease- - Nothing in Republic Act No. 7924 granted MMDA police power, let alone legislative power.
transfer (BLT)
agreement with the The power to enforce the provisions of the Building Code is lodged in the Department of
Metro Rail Transit Public Works and Highways (DPWH), not in MMDA.
Corporation (MRTC) for
the construction of MRT
3
- Metro Rail entered into AP: The right to install billboards is from the BLT agreement, which right was expressly
a Contract for recognized already by this Court. This must therefore be respected.
Advertising Services
with TRACKWORKS, Also, the MDDA regulation does not apply to Metro Rail because it only applies to public areas.
giving it exclusive right MRT 3 was then privately owned.
to undertake advertising
and promotional
activities within the
interior and exterior of
the MRT structure.
- MMDA requested
TRACKWORKS to
dismantle its billboards
in conformity with
MMDA Reg 96-009.
- TRACKWORKS refused
to comply and filed a
petition for injunction
with the trial court.

Does the MMDA have the


power to dismantle,
remove, or destroy the
billboards, signages and
other advertising media
installed on the MRT3
structure by Trackworks?
NO.
Gancayco v. City D: The MMDA does not have the power to enact ordinances.
Government of Quezon - It is an agency created for the purpose of laying down policies and coordinating with the various
City - G.R. No. 177807, national government agencies, people's organizations, nongovernmental organizations and the
October 11, 2011 private sector for the efficient and expeditious delivery of basic services in the vast metropolitan
area.
- MMDA then sent a - All its functions are administrative in nature and these are actually summed up in the charter
notice of demolition to itself.
Justice Gancayco
alleging that a portion of AP: Thus, the demolition of the petitioner’s property through its Resolution No. 02-28 is
his building violated the INVALID for it cannot supplement the provisions of Quezon City Ordinance No. 2904
National Building Code merely through such resolution.
of the Philippines in - The claims of the MMDA that the City Government of Quezon City may be considered to have
relation to Ordinance approved the demolition of the structure, simply because then Quezon City Mayor Feliciano R.
No. 2904. Belmonte signed MMDA Resolution No. 02-28 is also WITHOUT MERIT.
- He did not comply with - There was no valid delegation of powers to the MMDA. Contrary to the claim of the MMDA,
the notice. the City Government of Quezon City washed its hands off the acts of the former. In its
- Thelma then proceeded Answer, the city government stated that the demolition was undertaken by the MMDA
to demolish the party only, without the participation and/or consent of Quezon City.
wall of the ground floor - Therefore, the MMDA acted on its own and should be held solely liable for the destruction of the
structure. portion of Justice Gancaycos building.
- The City Government of
Quezon City claimed
that the ordinance was
a valid exercise of police
177
power, regulating the
use of property in a
business zone.
- Justice Gancayco filed a
Petition with prayer for
a TRO and/or WPI.

Did MMDA Resolution


No. 02-28, validly
empower the MMDA to
demolish Justice
Gancayco’s property? NO

Section 17

Sema v. COMELEC - D: There is no provision in the Constitution that conflicts with the delegation to regional
July 16, 2008 legislative bodies of the power to create municipalities and barangays, provided Sec 10
(plebiscite) is followed.
In Sec 19 RA 9054, - However, the creation of provinces and cities is another matter. Sec 5(3), Art. VI provides that
Congress delegated to each city with a population of at least 250,000 or each province shall have 1 representative.
the ARMM the power to - Sec 3 of the ordinance appended to the Constitution provides that any province that is hereafter
create provinces, cities, created or any city whose population may increase to more than 250,000 shall be entitled in the
municipalities and immediately following election to at least 1 members in the HR. Thus, a province cannot be
barangays within the created without creating a legislative district, nor can a city with a population of 250,000 or
ARMM. - Unconsti more be created without a legislative district.

AP: Thus, the power to create a province or city with a population of 250,000 or more
requires the power to create a legislative district. The provision is therefore
unconstitutional because Congress cannot validly delegate the power to create
legislative districts for the HR since the power to increase the allowable membership in
the HR and to reapportion legislative districts, is vested exclusively in Congress.

Section 18

A. Creation of Autonomous Region

Abbas v. Comelec - 179 D: Under the Constitution and R.A. No. 6734, the creation of the autonomous region shall take
SCRA 287 effect only when approved by a majority of the votes cast by the constituent units in a plebiscite,
and only those provinces and cities where a majority vote in favor of the Organic Act shall be
The legality of RA 6734, included in the autonomous region. The provinces and cities wherein such a majority is not
the Organic Act of attained shall not be included in the autonomous region.
Mindanao is challenged - The creation of the autonomous region is made to depend, not on the total majority
and the plebiscite called vote in the plebiscite, but on the will of the majority in each of the constituent units.
in the thirteen provinces - The ascertainment by Congress of the areas that share common attributes is within the
of Mindanao for the exclusive realm of the legislature’s discretion.
ratification of the Organic
Act is challenged - Administrative regions are not territorial and political subdivisions like provinces,
cities, municipalities and barangays [see Art. X, sec. 1 of the Constitution]. While the
Is the Organic Act valid? power to merge administrative regions is not expressly provided for in the
Constitution, it is a power which has traditionally been lodged with the President to
facilitate the exercise of the power of general supervision over local governments.
YES
AP: RA 6734, the organic act establishing the Autonomous Regional Government of
Muslim Mindanao was held valid. It was passed pursuant to the mandate of Art. X.
Leonor v. Cordillera D: Since the Cordillera Autonomous Region did not come into legal existence, the
Bodong Administration Maeng Tribal Court was not constituted into an indigenous or special court under R.A.
- 194 SCRA 101 No. 6766. Hence, the Maeng Tribal Court is an ordinary tribal court existing under the
customs and traditions of an indigenous cultural community.
In the January 23, 1990
plebiscite, the creation of AP: Such tribal courts are not a part of the Philippine judicial system which consists of the
the Cordillera Supreme Court and the lower courts which have been established by law (Sec. 1, Art. VIII, 1987
Autonomous Region was Constitution). They do not possess judicial power. Like the pangkats or conciliation panels created
rejected by all the by P.D. No. 1508 in the barangays, they are advisory and conciliatory bodies whose principal
provinces and the city of objective is to bring together the parties to a dispute and persuade them to make peace, settle,
the Cordillera region, and compromise.
except Ifugao province.
Later on, the tribal court
of the Cordillera Bodong
Administration rendered a
valid executory decision
in a land dispute.

Is such decision valid?

NO
Cordillera Broad D: The creation of autonomous regions in Muslim Mindanao and the Cordilleras, which
Coalition v. COA - 181 is peculiar to the 1987 Constitution, contemplates the grant of political autonomy and
SCRA 495 not just administrative autonomy to these regions. Thus, the provision in the Constitution

178
for an autonomous regional government with a basic structure consisting of an executive
The President, issued EO department and a legislative assembly and special courts with personal, family and property law
220, in the exercise of jurisdiction in each of the autonomous regions [Art. X, sec. 18].
her legislative powers
prior to the convening of AP: E.O. 220, which created the Cordillera Administrative Region (CAR) is a mere
the 1st Congress under transitory coordinating agency the purpose of which is to prepare the stage for political
the 1987 Constitution. autonomy.
Petitioners argue that the - It fills in the resulting gap in the process of transforming a group of adjacent territorial and
President has virtually political subdivisions already enjoying local or administrative autonomy into an autonomous
pre-empted Congress region vested with political autonomy. The creation of such administrative region is not prohibited
from its mandated task of by the Constitution, including Article 10, Section 18 thereof.
enacting an organic act
and created an A reading of E.O. No. 220 will easily reveal that what it actually envisions is the consolidation and
autonomous region in the coordination of the delivery of services of line departments and agencies of the National
Coridlleras. Government in the areas covered by the administrative region as a step preparatory to the grant
of autonomy to the Cordilleras.
Issue I: W/N EO 220 pre- - It does not create the autonomous region contemplated in the Constitution. It merely
empted Congress? NO provides for transitory measures in anticipation of the enactment of an organic act
and the creation of an autonomous region.

- it did not create a new territorial and political subdivision or merge existing ones into a larger
subdivision.
- 1. Firstly, the CAR is not a public corporation or a territorial and political subdivision. It does not
have a separate juridical personality, unlike provinces, cities and municipalities. x x x
- Then, considering the control and supervision exercised by the President over the CAR and the
offices created under E.O. No. 220, and considering further the indispensable participation of the
line departments of the National Government, the CAR may be considered more than anything
else as a regional coordinating agency of the National Government, similar to the regional
development councils which the President may create under the Constitution [Art. X, sec. 14]. x
x x In this wise, the CAR may be considered as a more sophisticated version of the regional
development council.
Datu Abas Kida vs. D: Only amendments to, or revisions of, the Organic Act constitutionally-essential to the
Senate - G.R. No. creation of autonomous regions—i.e., those aspects specifically mentioned in the
196271, February 28, Constitution which Congress must provide for in the Organic Act – require ratification
2012 through a plebiscite.
- These amendments to the Organic Act are those that relate to:
Pursuant to the - (a) the basic structure of the regional government;
constitutional mandate of - (b) the region’s judicial system, i.e., the special courts with personal, family, and property law
synchronization, RA jurisdiction; and,
10153 postponed the - (c) the grant and extent of the legislative powers constitutionally conceded to the regional
regional elections in the government under Section 20, Article X of the Constitution.
ARMM (which were
scheduled to be held on
thes econd Monday of AP: The date of the ARMM elections does not fall under any of the matters that the
August 2011 to the Constitution specifically mandated Congress to provide for in the Organic Act. Therefore,
second Monday of May even assuming that the supermajority votes and the plebiscite requirements are valid, any
2013 and recognized the change in the date of elections cannot be construed as a substantial amendment of the Organic
President’s power to Act that would require compliance with these requirements.
appoint OICs to
temporarily assume these
positions upon the
expiration of the terms of
the elected officials.

Whether Section 3,
Article XVII of RA No.
9054 is unconstitutional
for excessively enlarging
the plebiscite
requirement in Section
18, Article X of the
Constitution?

YES
Philippine Constitution D: The Congress has the authority to create autonomous regions under Sec. 8 of Art.X of the
Association vs. Constitution.
Philippine Government
- G.R. No. 218406, The (Comprehensive Agreement on the Bangsamoro) CAB, including the (Framework
November 29, 2016 Agreement on the Bangsamoro) FAB did not preempt such power of the Congress
because they are just mere preliminary framework agreements that serves as a guide
Petitioners contend that for the formulation of the Bangsamoro Basic Law.
CAB and the FAB
(entered into by the PH AP: It is not the CAB or the FAB that will establish the Bangsamoro but the Bangsamoro
GOV and the MILF) are Basic Law enacted by Congress and ratified in a plebiscite in accordance with the
unconstitutional since the Constitution.
agreements seek to
create a virtual sub-state
known as the
Bangsamoro Political
Entity (BPE) to replace
the Autonomous Region
of Muslim Mindanao
(ARMM), hence pre-
179
empting the power of the
Congress to enact an
organic act.

ARTICLE XII NATIONAL ECONOMY AND PATRIMONY

SECTION 2

A. Alienation Of Natural Resources

Santa Rosa Mining Co. Issue: Whether PD 1214 which requires holders of subsisting and valid patentable mining claims
vs. Leido, Jr. located under the provisions of the Philippine Bill of 1902 to file a mining lease application within
1 year from the approval of the Decree is constitutional?

Ruling: YES. It is a valid exercise of the sovereign power of the State, as owner, over lands of
the public domain, of which petitioner's mining claims still form a part, and over the patrimony of
the nation, of which mineral deposits are a valuable asset. It may be underscored, in this
connection, that the Decree does not cover all mining claims located under the Phil. Bill of 1902,
but only those claims over which their locators had failed to obtain a patent And even then, such
locators may still avail of the renewable 25-year lease prescribed by PD 463, the Mineral
Development Resources Decree of 1974.

Mere location does not mean absolute ownership over the affected land or the mining claim. It
merely segregates the located land or area from the public domain by barring other would-be
locators from locating the same and appropriating for themselves the minerals found therein. To
rule otherwise would imply that location is all that is needed to acquire and maintain rights over a
located mining claim. This, we cannot approve or sanction because it is contrary to the intention
of the lawmaker that the locator should faithfully and consistently comply with the requirements
for annual work and improvements in the located mining claim.

San Miguel Corporation Issue: Whether the property had been acquired through prescription thus making the property
vs. CA private?

Ruling: NO. Open, exclusive and undisputed possession of alienable public land for the period
prescribed by law creates the legal fiction whereby the land, upon completion of the requisite
period ipso jure and without the need of judicial or other sanction, ceases to be public land and
becomes private property. Such open, continuous, exclusive and notorious occupation of the
disputed properties for more than 30 years must, however, be conclusively established. This
quantum of proof is necessary to avoid the erroneous validation of actually fictitious claims of
possession over the property in dispute.

Tax declarations and receipts are not conclusive evidence of ownership or right of possession over
a piece of land. They are merely indicia of a claim of ownership. Tax declarations only become
strong evidence of ownership of land acquired by prescription, a mode of acquisition of ownership
relied upon by petitioner in this case, when accompanied by proof of actual possession.

Such proof of actual possession was sought to be provided by the testimony of vendor Silverio
Perez that he had been in possession of the property since 1933 until he sold it to SMC in 1975;
that the property was given to him by his parents when he got married; that no document
evidenced that transfer; that it had been in the possession of his parents since 1925; that he had
declared the property in his name for taxation purposes; that he had paid taxes therefor, and that
he was in peaceful, continuous and exclusive possession of the property until its sale to SMC.
Petitioner did not present other witnesses to corroborate Perez’ testimony.

We hold that there is paucity of evidence of actual, notorious and exclusive possession of the
property on the part of vendor Silverio Perez so as to attach to it the character of an express
grant from the government.

B. Utilization Of Natural Resources

Miners Association vs. Issue: Whether AO Nos. 57 which unduly preterminates existing mining leases and other mining
Factoran, Jr. agreements and automatically converts them into production-sharing agreements within 1 year
from its effectivity date and AO 82 which declares that failure to submit Letters of Intent and
Mineral Production Sharing Agreements within 2 years from the date of effectivity shall cause the
abandonment of their mining, quarry and sand gravel permits are valid?

Ruling: YES. What the Constitution says is that whenever natural resources are involved,
particularly in the case of inalienable natural resources, the State must always have some control
of the exploration, development and utilization even if the individual or corporation engaged is a
Filipino. Under the text of EO211, there is a reservation clause which provides that the privileges
and terms and conditions of all existing mining leases or agreements granted after the effectivity
of the Constitution, shall be subject to any and all modifications or alterations which Congress
may adopt pursuant to Article XII Sec 2. The president validly modified or altered the privileges
granted, as well as the terms and conditions of mining leases and agreements under EO211 after
the effectivity of the Constitution by authorizing the DENR Secretary to negotiate and conclude
joint venture, utilization of minerals. Well-settled is the rule, however, that regardless of the
180
reservation clause, mining leases or agreements granted by the State are subject to alterations
through a reasonable exercise of the police power of the State.

La Bugal vs. DENR Issues: (a) Whether a Filipino corporation is not allowed by the Constitution to enter into an
FTAA with the government?
(b) Whether FTAA is unconstitutional as it grants the foreign corporation control to the extraction
of minerals?
(c) May the state enter into a service contract with foreign owned corporations?

Ruling:
a. NO. A textual analysis of the first paragraph of Section 2 of Article XII does not support
petitioners’ argument. The pertinent part of the said provision states: “Sec. 2. x x x The
exploration, development and utilization of natural resources shall be under the full control and
supervision of the State. The State may directly undertake such activities, or it may enter into
coproduction, joint venture, or production-sharing agreements with Filipino citizens, or
corporations or associations at least 60% of whose capital is owned by such citizens. x x x.”
Nowhere in the provision is there any express limitation or restriction insofar as arrangements
other than the three aforementioned contractual schemes are concerned.

Sec. 40 expressly applies to the assignment or transfer of the FTAA, not to the sale and transfer
of shares of stock in WMCP. When the transferee of an FTAA is another foreign corporation, there
is a logical application of the requirement of prior approval by the President of the Republic and
notification to Congress in the event of assignment or transfer of an FTAA. In this situation, such
approval and notification are appropriate safeguards, considering that the new contractor is the
subject of a foreign government. On the other hand, when the transferee of the FTAA happens to
be a Filipino corporation, the need for such safeguard is not critical; hence, the lack of prior
approval and notification may not be deemed fatal as to render the transfer invalid. Besides, it is
not as if approval by the President is entirely absent in this instance.

At bottom, we find completely outlandish petitioners’ contention that an FTAA could be entered
into by the government only with a foreign corporation, never with a Filipino enterprise. Indeed,
the nationalistic provisions of the Constitution are all anchored on the protection of Filipino
interests. How petitioners can now argue that foreigners have the exclusive right to FTAAs totally
overturns the entire basis of the Petition—preference for the Filipino in the exploration,
development and utilization of our natural resources. It does not take deep knowledge of law
and logic to understand that what the Constitution grants to foreigners should be
equally available to Filipinos.

b. NO. The 1987 commissioners did not effectively ban foreign service contracts, but instead
simply wanted to minimize dubious and biased agreements by limiting exploration, development
and utilization of natural resources to Filipino citizens. According to this new interpretation, FTAAs
are in fact service contracts and Paragraph 4 in the 1987 constitution was inserted precisely as an
exception to the 60-40 ratio norm. The Court supports this by arguing that in reality the
Philippines despite its natural resource abundance has no capacity to finance extraction
operations, which underscores the necessity of service contracts as supposedly recognized by the
framers of the constitution. As such, “The Court believes that it is not unconstitutional to allow
wide degree of discretion to the Chief Executive, given the nature and complexity of such
agreements, the humongous amounts of capital and financing required for large-scale mining
operations, the complicated technology needed, and the intricacies of international trade, coupled
with the State's need to maintain flexibility in its dealing, in order to preserve and enhance our
country's competitiveness in world markets.”

c. YES. But subject to the strict limitation in the last 2 paragraphs of Sec. 2. Financial and
technical agreements are a form of a service contract. Such service contracts may be entered into
only with respect to minerals, petroleum and other mineral oils. The grant of such service
contracts is subject to several safeguards, among them:

1. Crafted in accordance with a general law setting standard or uniform terms, conditions and
requirements;
2. President be the signatory for the government; and
3. President report the executed agreement to Congress within 30 days.

Alvarez vs. PICOP Issue: Whether the contract is enforceable under the non-impairment clause of the Constitution,
so as to make the signing of the IFMA a ministerial duty?

Ruling: NO. The 1969 Document is not a contract recognized under the nonimpairment clause,
much less a contract specifically enjoining the DENR Secretary to issue the IFMA. IFMAs are
production-sharing agreements concerning the development and utilization of natural resources.
As such, these agreements "may be for a period not exceeding 25 years, renewable for not more
than 25 years, and under such terms and conditions as may be provided by law." Any superior
"contract" requiring the State to issue TLAs and IFMAs whenever they expire clearly circumvents
Sec. 2, Art. 12 of the Constitution, which provides for the only permissible schemes wherein the
full control and supervision of the State are not derogated: co-production, joint venture, or
production-sharing agreements within the time limit of 25 years, renewable for another 25 years.

The TLA was meant to expire on 26 Apr 2002, which states that: We confirm your tenure over the
area and exclusive right to cut, collect and remove sawtimber and pulpwood on April 26, 1977; to
be renewable for another 25 years subject to compliance with constitutional and statutory
requirements as well as with existing policy on timber concessions.

Initiative for Dialogue Issue: Whether the sale of Angat Hydro-Electric Power Plant (AHEPP) to Korea Water Corporation
and Empowerment vs. violates Sec. 2 of Art. 12?
181
Power Sector Assets
and Liabilities Ruling: NO. Sale of Government-Owned AHEPP to a Foreign Corporation Not Prohibited But Only
Management Filipino Citizens and Corporations 60% of whose capital is owned by Filipinos May be Granted
Corporation Water Rights

The State's policy on the management of water resources is implemented through the regulation
of water rights. PD 1067, otherwise known as "The Water Code of the Philippines". The NWRB is
the chief coordinating and regulating agency for all water resources management development
activities.

It is clear that the law limits the grant of water rights only to Filipino citizens and juridical entities
duly qualified by law to exploit and develop water resources, including private corporations with
sixty percent of their capital owned by Filipinos. In the case of Angat River, the NWRB has issued
separate water permits to MWSS, NPC and NIA.

Foreign ownership of a hydropower facility is not prohibited under existing laws. The construction,
rehabilitation and development of hydropower plants are among those infrastructure projects
which even wholly-owned foreign corporations are allowed to undertake under the Amended
Build-Operate-Transfer (Amended BOT) Law (RA 7718).

With respect to foreign investors, the nationality issue had been framed in terms of the character
or nature of the power generation process itself, i.e., whether the activity amounts to utilization
of natural resources within the meaning of Sec. 2, Art. XII of the Constitution. If so, then foreign
companies cannot engage in hydropower generation business; but if not, then government may
legally allow even foreign-owned companies to operate hydropower facilities.

Appropriation of water, as used in the Water Code refers to the "acquisition of rights over the use
of waters or the taking or diverting of waters from a natural source in the manner and for any
purpose allowed by law."

On the other hand, "water right" is defined in the Water Code as the privilege granted by the
government to appropriate and use water. Black’s Law Dictionary defined "water rights" as "a
legal right, in the nature of a corporeal hereditament, to use the water of a natural stream or
water furnished through a ditch or canal, for general or specific purposes, such as irrigation,
mining, power, or domestic use, either to its full capacity or to a measured extent or during a
defined portion of the time," or "the right to have the water flow so that some portion of it may
be reduced to possession and be made private property of individual, and it is therefore the right
to divert water from natural stream by artificial means and apply the same to beneficial use."

As the new owner of the AHEPP, K-Water will have to utilize the waters in the Angat Dam for
hydropower generation. Consistent with the goals of the EPIRA, private entities are allowed to
undertake power generation activities and acquire NPC’s generation assets. But since only the
hydroelectric power plants and appurtenances are being sold, the privatization scheme should
enable the buyer of a hydroelectric power plant in NPC’s multi-purpose dam complex to have
beneficial use of the waters diverted or collected in the Angat Dam for its hydropower generation
activities, and at the same time ensure that the NPC retains full supervision and control over the
extraction and diversion of waters from the Angat River.

Narra Nickel Mining vs. Issue: Whether control test should be used to determine the nationality of the corporation
Redmont
Ruling: NO, Grandfather Rule should be used in cases where corporate layering is present.

Based on the said SEC Rule and DOJ Opinion, the Grandfather Rule or the second part of the SEC
Rule applies only when the 60-40 Filipino-foreign equity ownership is in doubt (i.e., in cases
where the joint venture corporation with Filipino and foreign stockholders with less than 60%
Filipino stockholdings [or 59%] invests in other joint venture corporation which is either 60-40%
Filipino-alien or the 59% less Filipino). Stated differently, where the 60-40 Filipino-foreign equity
ownership is not in doubt, the Grandfather Rule will not apply.

Republic vs. Jabson Issue: Whether the grant of Jabson’s application for registration of title to the subject property
was proper under the law and jurisprudence?

Ruling: NO. Any application for registration of title to land derived through a public grant must
sufficiently establish 3 things: (a) the subject’s land alienable and disposable nature; (b) his
predecessor’s adverse possession thereof; and (c) the reckoning date from which such adverse
possession was under a bona fide claim of ownership, that is, June 12, 1945 or earlier.

That land has been removed from the scope of the Regalian Doctrine and reclassified as part of
public domain’s alienable and disposable portion cannot be assumed or implied. The applicant
must establish the existence of a positive act of the government, such as the Presidential
proclamation or EO; an administrative action; investigation reports of Bureau of Lands
investigators; and a legislative act or statute to prove the alienable and disposable nature of the
land.

Here, We cannot give probative value to the DENR Certification submitted by Jabson.

First, Jabson’s belated submission of a supposed vital document tending to prove the subject
properties’ alienability is fatal to their cause.

182
The general rule is that an applicant must formally offer evidence supporting his application
before the trial court to duly prove the documents' genuineness and due execution. As an
exception to this rule, in Llanes v. Republic as cited by the CA, the Court admitted in evidence a
corrected CENRO certification not formally offered in the trial court and only presented on appeal.
However, Llanes is not on all fours with the present petition. There are special circumstances
justifying the Court's ruling in Llanes that are not present in the case at bar.

What was belatedly filed in Llanas was merely a corrected or amended certification, the unedited
version of which had been earlier presented in the trial court as evidence of the alienable and
disposable nature of the land. And the correction or amendment pertained merely to the
statement of the reckoning date of adverse possession. Unlike in this case, Jabson failed to
present during trial any evidence establishing the subject properties' alienable and disposable
nature.

Second, a DENR Sr. Forest Management Specialist, was not authorized to issue certifications as to
land classification, much less order for the release of lands of the public domain as alienable and
disposable. (President and DENR Secretary is authorized to classify lands).

Third, a certification alone is not sufficient in proving the subject land's alienable and disposable
nature. We have already ruled that a PENRO and/or CENRO certification must be accompanied by
a copy of the original classification, certified as a true copy by the legal custodian of the official
records, which: (a) released the subject land of the public domain as alienable and disposable,
and (b) was approved by the DENR Secretary.

Fourth, even assuming arguendo that the DENR Certification dated February 19, 2009 does not
suffer the aforementioned shortcomings, the same only served to prove the land classification of
one of the subject properties-Bagong Katipunan. To recall, Jabson filed their application in
relation to 2 properties, viz.: San Jose and Bagong Katipunan properties. However, the DENR
Certification dated February 19, 2009 covers the Bagong Katipunan property only.

To this day, Jabson have not established the alienable and disposable nature of the San Jose
property.

All told, from the foregoing, it is clear that Jabson did not overcome the presumption that the
parcels of land sought to be registered still formed part of the public domain. Thus, there was
absolutely no basis for the CA to approve Jabson's application pertaining to the Bagong Katipunan
property, and much less the San Jose property.

C. Reversion

Republic vs. Espinosa Issue: Whether reversion of the land is proper?

Ruling: NO. Reversion is the remedy where the State, pursuant to the Regalian doctrine, seeks
to revert land back to the mass of the public domain. It is proper when public land is fraudulently
awarded and disposed of to private individuals or corporations. There are also instances when we
granted reversion on grounds other than fraud, such as when a "person obtains a title under the
Public Land Act which includes, by oversight, lands which cannot be registered under the Torrens
system, or when the Director of Lands did not have jurisdiction over the same because it is of the
public domain."

Here, the State, through the Solicitor General, alleges neither fraud nor misrepresentation in the
cadastral proceedings and in the issuance of the title in Espinosa's favor. The argument for the
State is merely that the property was unlawfully included in the certificate of title because it is of
the public domain.

Since the case is one for reversion and not one for land registration, the burden is on
the State to prove that the property was classified as timberland or forest land at the
time it was decreed to Espinosa. To reiterate, there is no burden on Caliston to prove that the
property in question is alienable and disposable land. At this stage, it is reasonable to presume
that Espinosa, from whom Caliston derived her title, had already established that the property is
alienable and disposable land considering that she succeeded in obtaining the OCT over it. In this
reversion proceeding, the State must prove that there was an oversight or mistake in the
inclusion of the property in Espinosa' s title because it was of public dominion. This is consistent
with the rule that the burden of proof rests on the party who, as determined by the pleadings or
the nature of the case, asserts the affirmative of an issue..

We stress that our ruling is not inconsistent with the doctrine that forest lands are outside the
commerce of man and unsusceptible of private appropriation. Neither are we changing the rule on
imprescriptibility of actions for reversion. We are merely deciding on the facts as proved by the
record. To allow a reversion based on a classification made at the time when the property was
already declared private property by virtue of a decree would be akin to expropriation of land
without due process of law.

The result would have been different had the State proved that the property was already
classified as part of forest land at the time of the cadastral proceedings and when title was
decreed to Espinosa in 1962. However, it failed to discharge this burden; the grant of title which
carries with it the presumption that Espinosa had already proved the alienable character of the
property in the cadastral proceedings stands. To grant the reversion based on a subsequent
reclassification, more so on lack of evidence, would amount to taking of private property without
just compensation and due process of law. This, however, is not what our Constitution envisions;
183
fairness and due process are paramount considerations that must still be observed.

D. Ownership Of The State Vs. Local Government Unit

Sangguniang Issue: Whether the subject lands are patrimonial properties of the province of bataan which
Panlalawigan of cannot be taken without due process of law and without just compensation?
Bataan vs. Garcia
Ruling: NO. Local government property devoted to governmental purposes, such as local
administration, public education, and public health, as may be provided under special laws, is
classified as public.

Here, RA 8562 was not intended to expropriate the subject lots titled in the name of the Province
of Bataan, but to confirm their character as communal land of the State and to make them
available for disposition by the National Government.

SECTION 3

A. Lands Of The Public Domain

Director of Lands vs. Issue: Whether the lower court erred in ruling that the ownership of the land was acquired
Judge Aquino through prescription?

Ruling: YES. Forest lands or forest reserves are incapable of private appropriation and possession
thereof, however long, cannot convert them into private properties. This ruling is premised on the
Regalian doctrine. Pursuant to this constitutional provision, the land must first be released from
its classification as forest land and reclassified as agricultural land in accordance with the
certification issued by the Director of Forestry as provided for by Sec. 1827 of the Revised
Administrative Code. This is because the classification of public lands is an exclusive prerogative
of the executive department of the government and not of the courts. Moreover, a positive act of
the government is needed to declassify a forest land into alienable or disposable land for
agricultural or other purposes.

It should be emphasized, however, that the classification of the land as forest land is descriptive
of its legal nature or status and does not have to be descriptive of what the land actually looks
like. Hence, the fact that the contested parcels of land have long been denuded and actually
contains rich limestone deposits does not in any way affect its present classification as forest
land.

Republic vs. Court of Issues: (1) Can a land have a mixed classification (partly mineral, partly agricultural)?
Appeals (2)Can the owner of agricultural lands in which minerals are discovered extract or utilize the said
minerals without the permission of the State?

Ruling:
(1) NO. The rights over the land are indivisible and that the land itself cannot be half agriculture
and half mineral. The classification must be categorical: the land must be either completely
mineral or completely agricultural. In the instant case as already observed, the land, which was
originally classified as forestland ceased to be so and became mineral and completely mineral,
once the mining claims were perfected. As long as mining corporations were being undertaken
thereon, or underneath, it did not cease to be so, and become agricultural, even if only partly so,
because it was enclosed with a fence and was cultivated by those who were unlawfully occupying
the surface.

(2) NO. This is an application of the Regalian doctrine. Thus, once minerals are discovered in the
land, whatever the use to which it is being devoted at the time, such use may be discontinued by
the State to enable it to extract the minerals therein in the exercise of its sovereign prerogative.
The land is thus converted to mineral land. Thus, if a person is the owner of agricultural land in
which minerals are discovered, his ownership of such land does not give him the right to extract
or utilize the said minerals without the permission of the State to which such minerals belong.
However, Benguet and Atok have exclusive rights to the property in question by virtue of their
respective mining claims which they validly acquired before the Constitution of 1935 prohibited
the alienation of all lands of the public domain except agricultural lands, subject to vested rights
existing at the time of its adoption.

In Re: Application for Issue: Whether the CA committed a reversible error in ruling that Dumo failed to prove that the
Land Registration land applied for has already been declared alienable and disposable?
Dumo vs. Republic
Ruling: NO. To summarize the discussion and reiterate the guidelines set by this Court in Heirs of
Malabanan v. Republic of the Philippines, we state:

1. If the applicant or his predecessors-in-interest have been in open, continuous, exclusive and
notorious possession and occupation of the land sought to be registered under a bona fide claim
of ownership since 12 June 1945 or earlier, the applicant must prove that the land has been
classified by the Executive department as alienable and disposable land of the public domain. This
is covered by Section 14(1) of PD No. 1529 in relation to Section 48(b) of CA No. 141.

184
While it is not necessary that the land has been alienable and disposable since 12 June 1945 or
earlier, the applicant must prove that the President or DENR Secretary has classified the land as
alienable and disposable land of the public domain at any time before the application was made.

2. If the occupation and possession of the land commenced at any time after 12 June 1945, the
applicant may still register the land if he or his predecessors-in-interest have complied with the
requirements of acquisitive prescription under the Civil Code after the land has been expressly
declared as patrimonial property or no longer needed for public use, public service or the
development of national wealth. This is governed by Section 14(2) of PD No. 1529 in relation to
the Civil Code.

Under the Civil Code, acquisitive prescription, whether ordinary or extraordinary, applies only to
private property. Thus, the applicant must prove when the land sought to be registered was
expressly declared as patrimonial property because it is only from this time that the period for
acquisitive prescription would start to run.

Based on the foregoing, we find that the CA committed no reversible error in finding that Dumo
had no registerable title over the land she seeks to register. She failed to prove her right under
either Section 14(1) or Section 14(2) of PD No. 1529. She failed to prove that the land she seeks
to register was alienable and disposable land of the public domain. She failed to prove her and
her predecessors-in-interest's possession and occupation since 12 June 1945 or earlier. Thus, she
has no right under Section 14(1) of PD No. 1529. While she argues that she and her
predecessors-in-interest have been in possession and occupation of the land for 56 years, she
failed to prove that the and has been expressly declared as patrimonial property. Therefore, she
also has no right under Section 14(2) of PD No. 1529.

B. Reclaimed Lands

Chavez vs. PEA Issue: Whether AMARI, a private corporation, can acquire and own a reclaimed foreshore and
submerged areas in Manila Bay?

Ruling: NO. Under Sec. 2, Art. 12 of the 1987 Constitution, the foreshore and submerged areas
of Manila Bay are part of the "lands of the public domain, waters and other natural resources" and
consequently "owned by the State." As such, foreshore and submerged areas "shall not be
alienated," unless they are classified as "agricultural lands" of the public domain. The mere
reclamation of these areas by PEA does not convert these inalienable natural resources of the
State into alienable or disposable lands of the public domain. There must be a law or presidential
proclamation officially classifying these reclaimed lands as alienable or disposable and open to
disposition or concession.

The 1987 Constitution prohibits private corporations from acquiring alienable lands of the public
domain. AMARI, being a private corporation, is barred from such acquisition. The PEA is not an
end user agency with respect to the reclaimed lands under the amended JVA, and PEA may
simply turn around and transfer several hundreds of hectares to a single private corporation in
one transaction.

Moreover, these reclaimed lands cannot be classified as alienable or disposable if the law has
reserved them for some public or quasi-public use.

The Amended JVA covers not only the Freedom Islands, but also an additional 592.15 hectares
which are still submerged and forming part of Manila Bay. There is no legislative or
Presidential act classifying these submerged areas as alienable or disposable lands of
the public domain open to disposition. These submerged areas are not covered by any
patent or certificate of title. There can be no dispute that these submerged areas form part of
the public domain, and in their present state are inalienable and outside the commerce of man.
Until reclaimed from the sea, these submerged areas are, under the Constitution, "waters owned
by the State," forming part of the public domain and consequently inalienable. Only when actually
reclaimed from the sea can these submerged areas be classified as public agricultural lands,
which under the Constitution are the only natural resources that the State may alienate. Once
reclaimed and transformed into public agricultural lands, the government may then officially
classify these lands as alienable or disposable lands open to disposition. Thereafter, the
government may declare these lands no longer needed for public service. Only then can these
reclaimed lands be considered alienable or disposable lands of the public domain and within the
commerce of man.

Chavez vs. National Issue: Whether the reclaimed foreshore and submerged parcels of land acquired by the
Housing Authority respondent are declared alienable and disposable land of public domain.

Ruling: YES. The reclaimed lands across R-10 were classified alienable and disposable lands of
public domain of the State for the following reasons:

First, there were 3 presidential proclamations classifying the reclaimed lands across R-10 as
alienable or disposable hence open to disposition or concession, to wit:

1. MO 415 issued by President Aquino, Section 4 of which states that the land covered by the
Smokey Mountain Dumpsite is conveyed to the NHA as well as the area to be reclaimed across R-
10. The directive to transfer the lands once reclaimed to the NHA implicitly carries with it the
declaration that said lands are alienable and disposable. Otherwise, the NHA cannot effectively
use them in its housing and resettlement project.

185
2. Proclamation No. 39 issued by then President Ramos by which the reclaimed lands were
conveyed to NHA for subdivision and disposition to qualified beneficiaries and for development
into a mixed land use (commercial/industrial) to provide employment opportunities to on-site
families and additional areas for port-related activities. Said directive carries with it the
pronouncement that said lands have been transformed to alienable and disposable lands.
Otherwise, there is no legal way to convey it to the beneficiaries.

3. Proclamation No. 465 likewise issued by President Ramos enlarged the reclaimed area to 79
hectares to be developed and disposed of in the implementation of the SMDRP. The authority put
into the hands of the NHA to dispose of the reclaimed lands tacitly sustains the conversion to
alienable and disposable lands.

Secondly, Special Patents Nos. 3591, 3592, and 3598 issued by the DENR anchored on
Proclamations Nos. 39 and 465 issued by President Ramos, without doubt, classified the
reclaimed areas as alienable and disposable. Thus, MO 415 and Proclamations Nos. 39 and 465
cumulatively and jointly taken together with Special Patent Nos. 3591, 3592, and 3598 more than
satisfy the requirement in PEA that there must be a law or presidential proclamation officially
classifying these reclaimed lands as alienable or disposable and open to disposition or concession.

Even if it is conceded that there was no explicit declaration that the lands are no longer needed
for public use or public service, there was however an implicit executive declaration that the
reclaimed areas R-10 are not necessary anymore for public use or public service when President
Aquino through MO 415 conveyed the same to the NHA partly for housing project and related
commercial/industrial development intended for disposition to and enjoyment of certain
beneficiaries and not the public in general and partly as enabling component to finance the
project.

MO 415 and Proclamations Nos. 39 and 465 are declarations that proclaimed the nonuse of the
reclaimed areas for public use or service as the Project cannot be successfully implemented
without the withdrawal of said lands from public use or service. Certainly, the devotion of the
reclaimed land to public use or service conflicts with the intended use of the Smokey Mountain
areas for housing and employment of the Smokey Mountain scavengers and for financing the
Project because the latter cannot be accomplished without abandoning the public use of the
subject land. Without doubt, the presidential proclamations on SMDRP together with the issuance
of the special patents had effectively removed the reclaimed lands from public use.

C. Right Of Corporations To Acquire Land

JG Summit vs. CA Issue: Whether KAWASAKI may buy shares of PHILSECO in excess of 40% of the shares of the
corporation.

Ruling: YES. Nothing in the JVA prevents KAWASAKI from acquiring more than 40% of
PHILSECOs total capitalization

First of all, the right of first refusal is a property right of PHILSECO shareholders, KAWASAKI and
NIDC, under the terms of their JVA. This right allows them to purchase the shares of their co-
shareholder before they are offered to a third party. The agreement of co-shareholders to
mutually grant this right to each other, by itself, does not constitute a violation of the
provisions of the Constitution limiting land ownership to Filipinos and Filipino
corporations. As PHILYARDS correctly puts it, if PHILSECO still owns land, the right of first
refusal can be validly assigned to a qualified Filipino entity in order to maintain the 60%-40%
ratio. This transfer, by itself, does not amount to a violation of the Anti-Dummy Laws, absent
proof of any fraudulent intent. The transfer could be made either to a nominee or such other
party which the holder of the right of first refusal feels it can comfortably do business with.
Alternatively, PHILSECO may divest of its landholdings, in which case KAWASAKI, in exercising its
right of first refusal, can exceed 40% of PHILSECOs equity. In fact, it can even be said that if the
foreign shareholdings of a landholding corporation exceeds 40%, it is not the foreign stockholders
ownership of the shares which is adversely affected but the capacity of the corporation to own
land that is, the corporation becomes disqualified to own land. This finds support under the basic
corporate law principle that the corporation and its stockholders are separate juridical entities. In
this vein, the right of first refusal over shares pertains to the shareholders whereas the capacity
to own land pertains to the corporation. Hence, the fact that PHILSECO owns land cannot deprive
stockholders of their right of first refusal. No law disqualifies a person from purchasing
shares in a landholding corporation even if the latter will exceed the allowed foreign
equity, what the law disqualifies is the corporation from owning land. This is not a case of
transfer of ownership of land, this involves a right of first refusal over shares of stock, there is a
distinction between ownership of shares and the corporation’s ownership of land as they are
separate juridical personalities.

SECTION 5

Ancestral Lands

Cruz vs. Secretary Issue: Whether the IPRA is unconstitutional on the ground that it deprives the States of its
ownership over ancestral domains.

186
Ruling: The court was not able to obtain the required majority votes to declare the law
unconstitutional. The vote of the SC on the subject was equally divided, 7-7.

OPINION UPHOLDING THE CONSTITUTIONALITY OF THE LAW


The provisions of the IPRA affirming the ownership by the indigenous peoples of their ancestral
lands and domains by virtue of native title do not diminish the State’s ownership of lands of the
public domain, because said ancestral lands and domains are considered as private land, and
never to have been part of the public domain.

The constitutional provision vesting ownership over minerals, mineral lands and other natural
resources in the State is not violated by IPRA which grant certain rights to the indigenous peoples
over the natural resources found within the ancestral domains, e.g., to benefit from and share in
the profits from the allocation and utilization of the same, as well as priority rights in the
harvesting, extraction, development or exploitation thereof. The State retains full control over the
exploration, development and utilization of natural resources even with the grant of said rights to
the indigenous peoples, through the imposition of requirements and conditions for the utilization
of natural resources under existing laws, such as the Small-Scale Mining Act of 1991 and the
Philippine Mining Act of 1995. Moreover, the rights granted to indigenous peoples for the
utilization of natural resources within their ancestral domains merely amplify what has been
earlier granted to them under the aforesaid laws.

While the IPRA recognizes the rights of indigenous peoples with regard to their ancestral lands
and domains, it also protects the vested rights of persons, whether indigenous or non-indigenous
peoples, who may have acquired rights of ownership lands or rights to explore and exploit natural
resources within the ancestral lands and domains.

OPINION DECLARING THE LAW UNCONSTITUTIONAL


IPRA declares that ancestral domains, including the natural resources found therein, are owned
by ICCs/IPs and cannot be sold, disposed or destroyed. This provision directly contravene Section
2, Article XII of the Constitution, more specifically the declaration that the State owns all lands of
the public domain, minerals and natural resources – none of which, except agricultural lands, can
be alienated.

IPRA law relinquishes the constitutional power of the State of full control on the exploration,
development, and utilization of natural resources in favor of ICCs/IPs, insofar as natural resources
found within their territories are concerned. Pursuant to their rights of ownership and possession,
they may develop and manage the natural resources, benefit from and share in the profits from
the allocation and the utilization thereof. ICCs/IPs are given the right to negotiate directly the
terms and conditions for the exploration of natural resources, a right vested by the Constitution
only in the State. Congress, through IPRA, has in effect abdicated in favor of a minority group the
State's power of ownership and full control over a substantial part of the national patrimony, in
contravention of our most fundamental law.

SECTION 8

Aliens And Private Lands

Ramirez vs. Vda. De Issue: Whether the grant of usufruct over real property in the Philippines in favor of an alien
Ramirez (Wanda—Austrian) by virtue of a will is unconstitutional?

Ruling: NO. The Constitutional provision which enables aliens to acquire private lands does not
extend to testamentary succession. The Court also upheld the usufruct in favor of Wanda because
a usufruct, albeit a real right, does not vest title to the land in the usufructuary and it is the
vesting of title to land in favor of aliens which is proscribed by the Constitution.

Republic vs. Court of Issue: Can a foreign national apply for registration of title over a parcel of land which he
Appeals acquired by purchase while still a citizen of the Philippines, from a vendor who has complied with
the requirements for registration under the Public Land Act (CA 141)?

Ruling: YES. In the case at bar, private respondents were undoubtedly natural-born Filipino
citizens at the time of the acquisition of the properties and by virtue thereof, acquired vested
rights thereon, tacking in the process, the possession in the concept of owner and the prescribed
period of time held by their predecessors-in-interest under the Public Land Act. In addition,
private respondents have constructed a house of strong materials on the contested property, now
occupied by respondent Lapiñas mother.

But what should not be missed in the disposition of this case is the fact that the Constitution itself
allows private respondents to register the contested parcels of land in their favor. Section 8 of
Article XII of the Constitution provides that “notwithstanding the provisions of Section 7 of this
Article, a natural-born citizen of the Philippines who has lost his Philippine citizenship may be a
transferee of private lands, subject to limitations provided by law.” From the adoption of the 1987
Constitution up to the present, no other law has been passed by the legislature on the same
subject. Thus, what governs the disposition of private lands in favor of a natural-born Filipino
citizen who has lost his Philippine citizenship remains to be BP 185.

It is undisputed that private respondents, as vendees of a private land, were naturalborn citizens
of the Philippines. For the purpose of transfer and/or acquisition of a parcel of residential land, it
is not significant whether private respondents are no longer Filipino citizens at the time they
187
purchased or registered the parcels of land in question. What is important is that private
respondents were formerly natural-born citizens of the Philippines, and as transferees of a private
land, they could apply for registration in accordance with the mandate of Section 8, Article XII of
the Constitution. Considering that private respondents were able to prove the requisite period and
character of possession of their predecessors-in-interest over the subject lots, their application for
registration of title must perforce be approved.

Cheesman vs. Issue: Whether an alien has a capacity to question the subsequent sale of the property by his
Intermediate Appellate Filipino wife to another person?
Court
Ruling: NO. The fundamental law prohibits the sale to aliens of residential land. Sec. 14, Art. 14
of the 1973 Constitution ordains that, "Save in cases of hereditary succession, no private land
shall be transferred or conveyed except to individuals, corporations, or associations qualified to
acquire or hold lands of the public domain.” Cheesman was, of course, charged with knowledge of
this prohibition. Thus, assuming that it was his intention that the lot in question be purchased by
him and his wife, he acquired no right whatever over the property by virtue of that purchase; and
in attempting to acquire a right or interest in land, vicariously and clandestinely, he knowingly
violated the Constitution; the sale as to him was null and void.

In any event, he had and has no capacity or personality to question the subsequent sale of the
same property by his wife on the theory that in so doing he is merely exercising the prerogative
of a husband in respect of conjugal property. To sustain such a theory would permit indirect
controversion of the constitutional prohibition. If the property were to be declared conjugal, this
would accord to the alien husband a not insubstantial interest and right over land, as he would
then have a decisive vote as to its transfer or disposition. This is a right that the Constitution does
not permit him to have. Land purchased by a Filipino does not extend to the foreigner spouse,
even if the purchase was made with the latter’s money or by conjugal funds.

Halili vs. Court of Issue: Whether the transfer of an interest in a piece of land to an alien is unconstitutional even
Appeals after the entire parcel has been sold to a qualified citizen?

Ruling: NO. Jurisprudence is consistent that if land is invalidly transferred to an alien who
subsequently becomes a citizen or transfers it to a citizen, the flaw in the original transaction is
considered cured and the title of the transferee is rendered valid.

The rationale behind this principle is that if the ban on aliens from acquiring not only agricultural
but also urban lands is to preserve the nations lands for future generations of Filipinos, that aim
or purpose would not thwarted but achieved by making lawful the acquisition of real estate by
aliens who became Filipino citizen by naturalization. Accordingly, since the disputed land is now
owned by Private Respondent Cataniag, a Filipino citizen, the prior invalid transfer can no longer
be assailed. The objective of the constitutional provision -- to keep our land in Filipino hands --
has been served.

SECTION 10

National Economy And Patrimony

Manila Prince Hotel vs. Issue: Whether Manila Hotel Corporation is part of the Philippine national patrimony?
GSIS
Ruling: YES. In its plain and ordinary meaning, the term patrimony pertains to heritage. When
the Constitution speaks of national patrimony, it refers not only to the natural resources of the
Philippines, as the Constitution could have very well used the term natural resources, but also to
the cultural heritage of the Filipinos.

Manila Hotel has become a landmark - a living testimonial of Philippine heritage. While it was
restrictively an American hotel when it first opened in 1912, it immediately evolved to be truly
Filipino. Formerly a concourse for the elite, it has since then become the venue of various
significant events which have shaped Philippine history. It was called the Cultural Center of the
1930s. It was the site of the festivities during the inauguration of the Philippine Commonwealth.
Dubbed as the Official Guest House of the Philippine Government it plays host to dignitaries and
official visitors who are accorded the traditional Philippine hospitality. During World War II the
hotel was converted by the Japanese Military Administration into a military headquarters. When
the American forces returned to recapture Manila the hotel was selected by the Japanese together
with Intramuros as the two (2) places for their final stand. Thereafter, in the 1950s and 1960s,
the hotel became the center of political activities, playing host to almost every political
convention. In 1970 the hotel reopened after a renovation and reaped numerous international
recognitions, an acknowledgment of the Filipino talent and ingenuity. In 1986 the hotel was the
site of a failed coup detat where an aspirant for vice-president was proclaimed President of the
Philippine Republic.

For more than 8 decades Manila Hotel has bore mute witness to the triumphs and failures, loves
and frustrations of the Filipinos; its existence is impressed with public interest; its own historicity
associated with our struggle for sovereignty, independence and nationhood. Verily, Manila Hotel
has become part of our national economy and patrimony. For sure, 51% of the equity of the MHC
comes within the purview of the constitutional shelter for it comprises the majority and controlling
stock, so that anyone who acquires or owns the 51% will have actual control and management of
the hotel. In this instance, 51% of the MHC cannot be disassociated from the hotel and the land
on which the hotel edifice stands. Consequently, we cannot sustain respondents claim that the

188
Filipino First Policy provision is not applicable since what is being sold is only 51% of the
outstanding shares of the corporation, not the Hotel building nor the land upon which the building
stands.

A. Public Utilities

Albano vs. Reyes Issue: Whether MICT needs a legislative franchise before it can legally operate as a public utility.

Ruling: NO. PD 857 empowers the PPA to provide services within the port districts, on its own or
by contract, pursuant to EO 30, mandating it to manage and operate the Manila Intl Port
Complex. Even if it is a public utility, its operation would not necessarily call for a franchise from
Congress. Such is not required before each and every public utility may operate. Thus, the law
has granted certain administrative agencies the power to grant licenses or authorize operation of
certain public utilities. That in Sec. 11, the issuance of franchise or other forms of authorization
for the operation of a public utility shall be subject to amendment, alteration, or repeal by
Congress does not mean that only Congress has the power to grant such authorization, there are
any laws granting specified agencies of the Executive the power to issue such authorization for
certain classes of public utilities. By virtue of the power granted to PPA by PD 857, it chose to
contract out the operation to a private corporation. Also, since the contract was approved by the
Pres., it enjoys the legal presumption of validity and regularity of official action. There is no
evidence showing constitutional infirmity of such action.

Associated Issue: Whether the operation of a television station requires a congressional franchise?
Communications vs.
NTC Ruling: YES. A congressional franchise is necessary for petitioner to operate television Channel
25. Even assuming that Act No. 3846 applies only to radio stations and not to television stations
as petitioner adamantly insists, the subsequent PD 76-A clearly shows in Sec. 1 that a franchise is
required to operate radio as well as television stations.

There is nothing in PD 576-A that reveals any intention to do away with the requirement of a
franchise for the operation of radio and television stations. Sec. 6 of PD 576-A merely identifies
the regulatory agencies from whom authorizations, in addition to the required congressional
franchise, must be secured after December 31, 1981. With the creation of the Board of
Communications under the Plan, it was no longer sufficient to secure authorization from the
Secretary of Public Works and Communications as provided in Act No. 3846. The Boards
authorization was also necessary. Thus, P.D. No. 576-A provides in Section 6 that radio and
television station operators must secure authorization from both the Secretary of Public Works
and Communications and the Board of Communications.

A textual interpretation of Section 6 of P.D. No. 576-A yields the same interpretation that after
December 31, 1981, a franchise is still necessary to operate radio and television stations. Were it
the intention of the law to do away with the requirement of a franchise after said date, then the
phrase thereafter, irrespective of any franchise, grant, license, permit, certificate or other forms
of authority to operate granted by any office, agency or person would not have been necessary
because the first sentence of Section 6 already states that all franchises, grants, licenses,
permits, certificates or other forms of authority to operate radio or television broadcasting
systems shall terminate on December 31, 1981. It is therefore already understood that these
forms of authority have no more force and effect after December 31, 1981. If the intention were
to do away with the franchise requirement, Section 6 would have simply laid down after the first
sentence the requirements to operate radio and television stations after December 31, 1981, i.e.,
no radio or television station shall be authorized to operate without the authority of the Board of
Communications and the Secretary of Public Works and Communications. Instead, however, the
phrase irrespective of any franchise, was inserted to emphasize that a franchise or any other form
of authorization from any office, agency or person does not suffice to operate radio and television
stations because the authorizations of both the Board of Communications and the Secretary of
Public Works and Communications are required as well.

Where there is a law such as P.D. No. 576-A which requires a franchise for the operation of radio
and television stations, that law must be followed until subsequently repealed. As we have earlier
shown, however, there is nothing in the subsequent E.O. No. 546 which evinces an intent to
dispense with the franchise requirement.

Tatad vs. Garcia, Jr. Issue: Whether the agreement between EDSA LRT Corp (HK Corp) and DOTC is constitutional?

Ruling: YES. What private respondent owns are the rail tracks, rolling stocks like the coaches,
rail stations, terminals and the power plant, not a public utility. While a franchise is needed to
operate these facilities to serve the public, they do not by themselves constitute a public utility.
What constitutes a public utility is not their ownership but their use to serve the public.

The Constitution, in no uncertain terms, requires a franchise for the operation of a public utility.
However, it does not require a franchise before one can own the facilities needed to operate a
public utility so long as it does not operate them to serve the public. In law, there is a clear
distinction between the "operation" of a public utility and the ownership of the facilities and
equipment used to serve the public.

The right to operate a public utility may exist independently and separately from the ownership of
the facilities thereof. One can own said facilities without operating them as a public utility, or
conversely, one may operate a public utility without owning the facilities used to serve the public.
The devotion of property to serve the public may be done by the owner or by the person in
control thereof who may not necessarily be the owner thereof.
189
While private respondent is the owner of the facilities necessary to operate the EDSA. LRT III, it
admits that it is not enfranchised to operate a public utility. In view of this incapacity, private
respondent and DOTC agreed that DOTC shall as a common carrier and private respondent shall
provide technical maintenance and repair services to DOTC. Technical maintenance consists of
providing (1) repair and maintenance facilities for the depot and rail lines, services for routine
clearing and security; and (2) producing and distributing maintenance manuals and drawings for
the entire system.

In sum, private respondent will not run the light rail vehicles and collect fees from the riding
public. It will have no dealings with the public and the public will have no right to demand any
services from it.

B. Definition Of Capital

Gamboa vs. Teves Issue: Whether the term “capital” in Sec. 11, Article XII, refers to the total common shares only,
or to the total outstanding capital stock (combined total of common and non-voting preferred
shares.

Ruling: The term capital refers to the combined total of common and non-voting preferred
shares.

Since the constitutional requirement of at least 60 percent Filipino ownership applies not only to
voting control of the corporation but also to the beneficial ownership of the corporation, it is
therefore imperative that such requirement apply uniformly and across the board to all classes of
shares, regardless of nomenclature and category, comprising the capital of a corporation. Under
the Corporation Code, capital stock consists of all classes of shares issued to stockholders, that is,
common shares as well as preferred shares, which may have different rights, privileges or
restrictions as stated in the articles of incorporation.

Since a specific class of shares may have rights and privileges or restrictions different from the
rest of the shares in a corporation, the 60-40 ownership requirement in favor of Filipino citizens in
Section 11, Article XII of the Constitution must apply not only to shares with voting rights but
also to shares without voting rights. Preferred shares, denied the right to vote in the election of
directors, are anyway still entitled to vote on the eight specific corporate matters mentioned
above. Thus, if a corporation, engaged in a partially nationalized industry, issues a mixture of
common and preferred nonvoting shares, at least 60 percent of the common shares and at least
60 percent of the preferred non-voting shares must be owned by Filipinos. Of course, if a
corporation issues only a single class of shares, at least 60 percent of such shares must
necessarily be owned by Filipinos. In short, the 60-40 ownership requirement in favor of Filipino
citizens must apply separately to each class of shares, whether common, preferred non-voting,
preferred voting or any other class of shares. This uniform application of the 60-40 ownership
requirement in favor of Filipino citizens clearly breathes life to the constitutional command that
the ownership and operation of public utilities shall be reserved exclusively to corporations at
least 60 percent of whose capital is Filipino-owned. Applying uniformly the 60-40 ownership
requirement in favor of Filipino citizens to each class of shares, regardless of differences in voting
rights, privileges and restrictions, guarantees effective Filipino control of public utilities, as
mandated by the Constitution.

SECTION 12

Filipino First Policy

Tañada vs. Angara Issue: Whether the WTO is constitutional?

Ruling: YES. Secs. 10 and 12 of Art. 12, apart from merely laying down general principles
relating to the national economy and patrimony, should be read and understood in relation to the
other sections in said article, especially Secs. 1 and 13 thereof.

As pointed out by the Solicitor General, Sec. 1 lays down the basic goals of national economic
development, as follows:

1. A more equitable distribution of opportunities, income and wealth;


2. A sustained increase in the amount of goods and services provided by the nation for the benefit
of the people; and
3. An expanding productivity as the key to raising the quality of life for all especially the
underprivileged.

With these goals in context, the Constitution then ordains the ideals of economic nationalism (1)
by expressing preference in favor of qualified Filipinos in the grant of rights, privileges and
concessions covering the national economy and patrimony and in the use of Filipino labor,
domestic materials and locally-produced goods; (2) by mandating the State to adopt measures
that help make them competitive; and (3) by requiring the State to develop a self-reliant and
independent national economy effectively controlled by Filipinos. In similar language, the
Constitution takes into account the realities of the outside world as it requires the pursuit of a
trade policy that serves the general welfare and utilizes all forms and arrangements of exchange

190
on the basis of equality and reciprocity; and speaks of industries which are competitive in both
domestic and foreign markets as well as of the protection of Filipino enterprises against unfair
foreign competition and trade practices.

All told, while the Constitution indeed mandates a bias in favor of Filipino goods, services, labor
and enterprises, at the same time, it recognizes the need for business exchange with the rest of
the world on the bases of equality and reciprocity and limits protection of Filipino enterprises only
against foreign competition and trade practices that are unfair. In other words, the Constitution
did not intend to pursue an isolationist policy. It did not shut out foreign investments, goods and
services in the development of the Philippine economy. While the Constitution does not encourage
the unlimited entry of foreign goods, services and investments into the country, it does not
prohibit them either. In fact, it allows an exchange on the basis of equality and reciprocity,
frowning only on foreign competition that is unfair.

Constitution Does Not Rule Out Foreign Competition


Furthermore, the constitutional policy of a self-reliant and independent national economy does
not necessarily rule out the entry of foreign investments, goods and services. It contemplates
neither economic seclusion nor mendicancy in the international community.

SECTION 16

Formation Of Private Corporation

National Development Issue: Whether PD No. 1717 which was enacted to rehabilitate Agrix and created New Agrix is
Co. vs. Philippine unconstitutional?
Veterans Bank
Ruling: YES. New Agrix, Inc. was created by special decree notwithstanding Sec. 16 that the
Congress shall not, except by general law, provide for the formation, organization, or regulation
of private corporations, unless such corporations are owned or controlled by the Government or
any subdivision or instrumentality thereof.

The new corporation is neither owned nor controlled by the government. The National
Development Corporation was merely required to extend a loan of not more than P10,000,000.00
to New Agrix but with the obligation of making periodic reports to the Agrix board of directors.
After payment of the loan, it can manage on its own. Therefore, it is entirely private owned and
thus, should have been organized under the Corporation Law in accordance with Sec. 16.

Thus, PD 1717 is an invalid exercise of the police power, not being in conformity with the
traditional requirements of a lawful subject and a lawful method. The extinction of the mortgage
and other liens and of the interest and other charges pertaining to the legitimate creditors of
AGRIX constitutes taking without due process of law, and this is compounded by the reduction of
the secured creditors to the category of unsecured creditors in violation of the equal protection
clause. Moreover, the new corporation, being neither owned nor controlled by the Government,
should have been created only by general and not special law.

Liban vs. Gordon Issue: Whether RA 95 which created PNRC Charter is unconstitutional?

Ruling: NO. The office of the Chairman of the Philippine National Red Cross is not a government
office or an office in a government-owned or controlled corporation for purposes of the prohibition
in Sec. 13, Art. 6 of the 1987 Constitution.

A closer look at the nature of the PNRC would show that there is none like it not just in terms of
structure, but also in terms of history, public service and official status accorded to it by the State
and the international community. There is merit in PNRCs contention that its structure is sui
generis.

In the Decision, the Court, citing Feliciano v. Commission on Audit, explained that the purpose of
the constitutional provision prohibiting Congress from creating private corporations was to
prevent the granting of special privileges to certain individuals, families, or groups, which were
denied to other groups. Based on the above discussion, it can be seen that the PNRC Charter
does not come within the spirit of this constitutional provision, as it does not grant special
privileges to a particular individual, family, or group, but creates an entity that strives to serve
the common good.

The Republic of the Philippines, adhering to the Geneva Conventions, established the PNRC as a
voluntary organization for the purpose contemplated in the Geneva Convention of 27 July 1929.

So must this Court recognize too the country’s adherence to the Geneva Convention and respect
the unique status of the PNRC in consonance with its treaty obligations. The Geneva Convention
has the force and effect of law. Under the Constitution, the Philippines adopts the generally
accepted principles of international law as part of the law of the land. This constitutional provision
must be reconciled and harmonized with Article XII, Section 16 of the Constitution, instead of
using the latter to negate the former.

The PNRC, as a National Society of the International Red Cross and Red Crescent Movement, can
neither be classified as an instrumentality of the State, so as not to lose its character of neutrality
as well as its independence, nor strictly as a private corporation since it is regulated by

191
international humanitarian law and is treated as an auxiliary of the State

Based on the above, the sui generis status of the PNRC is now sufficiently established. Although it
is neither a subdivision, agency, or instrumentality of the government, nor a government-owned
or -controlled corporation or a subsidiary thereof, such a conclusion does not ipso facto imply that
the PNRC is a private corporation within the contemplation of the provision of the Constitution,
that must be organized under the Corporation Code. As correctly mentioned by Justice Roberto A.
Abad, the sui generis character of PNRC requires us to approach controversies involving the PNRC
on a case-to-case basis. The structure of the PNRC is sui generis being neither strictly
private nor public in nature. R.A. No. 95 remains valid and constitutional in its entirety.

SECTION 17

Temporary Turn-Over

David vs. Arroyo Issue: Whether by issuing PP 1017, declaring a state of national emergency, the president has
the power to take over private properties that would not follow the standards set by the
government?

Ruling: NO. Under the Proclamation, President Arroyo, during the state of national emergency
under PP 1017, can call the military not only to enforce obedience "to all the laws and to all
decrees " but also to act pursuant to the provision of Section 17, Article XII. A distinction must be
drawn between the President’s authority to declare "a state of national emergency" and to
exercise emergency powers. The exercise of emergency powers, such as the taking over of
privately owned public utility or business affected with public interest requires a delegation from
Congress. Sec. 17, Art. XII must be understood as an aspect of the emergency powers clause.
The taking over of private business affected with public interest is just another facet of the
emergency powers generally reposed upon Congress. Thus, Sec. 17 refers to Congress, not the
President. Whether or not the President may exercise such power is dependent on whether
Congress may delegate it to him pursuant to a law prescribing the reasonable terms. Thus, such
Proclamation does not authorize Arroyo during the emergency to temporarily take over or direct
the operation of any privately owned public utility or business affected with public interest without
authority from Congress.

Agan vs. PIATCO Issue: Whether PIATCO can, by mere contractual stipulation, obligate the government to pay
“reasonable cost for the use of the Terminal and/or Terminal Complex.” during the temporary
take over?

Ruling: NO. PIATCO cannot, by mere contractual stipulation, contravene Sec. 17 and obligate the
government to pay in this manner. The Constitutional provision envisions a situation wherein the
exigencies of the times necessitate the government to “temporarily take over or direct the
operation of any privately owned public utility or business affected with public interest.” It is the
welfare and interest of the public, which is of paramount consideration in determining whether or
not to temporarily take over a business.

The temporary takeover by the government extends only to the operation of the business and not
to the ownership thereof. As such, government is not required to compensate the private
entity/owner of the said business as there is no transfer of ownership, whether permanent or
temporary. The private entity-owner affected by the temporary takeover cannot claim just
compensation for the use of said business and its properties, as the temporary takeover by the
government is in exercise of police power and not the power of eminent domain.

SECTION 18

Telephone Interconnections

Republic vs. PLDT Issue: Whether Bureau has the right to compel PLDT to allow it to use the latter’s trunklines and
facilities?

Ruling: NO. PLDT cannot be compelled to enter into any contract as such would be violative of
the freedom to stipulate and enter into contracts as protected by law. However, recourse can be
had by the Government in its exercise of eminent domain, wherein it can compel PLDT to require
interconnection of the government lines with its own lines, as the needs of the government may
require, as long as just compensation is given to PLDT for such services and accommodation.

There is great public need for such services and for such interconnection of lines between the
government system and that of PLDT. The condemnation in this case would be proper, as long as
just compensation would be paid by the Government to PLDT for the use of the latter’s trunklines
and for the burden of such usage. Thus, a government subsidy may exercise the sovereign power
of eminent domain given that the purpose of such corporation is for public interest and welfare.

SECTION 19

192
Monopoly

Tatad vs. Secretary of Issue: Whether RA No. 8180 which deregulates the downstream oil industry providing for 4%
Energy tariff differential between crude and refined oil, 10% minimum inventory requirement and
prohibition on predatory pricing violates the constitutional prohibition against monopolies,
combinations in restraint of trade and unfair competition.

Ruling: YES. A monopoly is a privilege or peculiar advantage vested in one or more persons or
companies, consisting in the exclusive right or power to carry on a particular business or trade,
manufacture a particular article, or control the sale or the whole supply of a particular
commodity. It is a form of market structure in which one or only a few firms dominate the total
sales of a product or service. On the other hand, a combination in restraint of trade is an
agreement or understanding between two or more persons, in the form of a contract, trust, pool,
holding company, or other form of association, for the purpose of unduly restricting competition,
monopolizing trade and commerce in a certain commodity, controlling its, production, distribution
and price, or otherwise interfering with freedom of trade without statutory authority. Combination
in restraint of trade refers to the means while monopoly refers to the end.

Again, we underline in scarlet that the fundamental principle espoused by section 19, Article XII
of the Constitution is competition for it alone can release the creative forces of the market. But
the competition that can unleash these creative forces is competition that is fighting yet is fair.
Ideally, this kind of competition requires the presence of not one, not just a few but several
players. A market controlled by one player (monopoly) or dominated by a handful of players
(oligopoly) is hardly the market where honest-to-goodness competition will prevail. Monopolistic
or oligopolistic markets deserve our careful scrutiny and laws which barricade the entry points of
new players in the market should be viewed with suspicion.

Here, it cannot be denied that our downstream oil industry is operated and controlled by an
oligopoly, a foreign oligopoly at that Petron, Shell and Caltex stand as the only major league
players in the oil market. All other players belong to the lilliputian league.

We come to the final point. We now resolve the total effect of the untimely deregulation, the
imposition of 4% tariff differential on imported crude oil and refined petroleum products, the
requirement of inventory and the prohibition on predatory pricing on the constitutionality of R.A.
No. 8180. The question is whether these offending provisions can be individually struck down
without invalidating the entire R.A. No. 8180.

R.A. No. 8180 contains a separability clause. This separability clause notwithstanding, we hold
that the offending provisions of R.A. No. 8180 so permeate its essence that the entire law has to
be struck down. The provisions on tariff differential, inventory and predatory pricing are among
the principal props of R.A. No. 8180. Congress could not have deregulated the downstream oil
industry without these provisions. Unfortunately, contrary to their intent, these provisions on
tariff differential, inventory and predatory pricing inhibit fair competition, encourage monopolistic
power and interfere with the free interaction of market forces. R.A. No. 8180 needs provisions to
vouchsafe free and fair competition. The need for these vouchsafing provisions cannot be
overstated. Before deregulation, PETRON, SHELL and CALTEX had no real competitors but did not
have a free run of the market because government controls both the pricing and nonpricing
aspects of the oil industry. After deregulation, PETRON, SHELL and CALTEX remain unthreatened
by real competition yet are no longer subject to control by government with respect to their
pricing and non-pricing decisions. The aftermath of R.A. No. 8180 is a deregulated market where
competition can be corrupted and where market forces can be manipulated by oligopolies.

Eastern Assurance vs. Issue: Whether the assailed LTFRB circulars with their implementing circulars violate the
LTFRB constitutional proscription against monopoly, combination in restraint of trade and unfair
competition.

Ruling: NO. While embracing free enterprise as an economic creed, the Constitution does not
totally prohibit the operation of monopolies. However, it mandates the State to regulate them
when public interest so requires.

Intense competition has led insurance companies/agents offering insurance policies for public
utility vehicles to resort to ruinous tactics to sell their services. Moreover, different persons or
operators were issued certificates of cover (COC) or policies bearing the same number. Thus,
claims under these policies were not paid, or payments were unreasonably delayed, resulting in
prejudice to the riding public.

The present case shows a clear public necessity to regulate the proliferation of such insurance
companies. It found that in order to protect the interests of the riding public and to resolve
problems involving the passenger insurance coverage of PUVs, it had to issue Memorandum
Circular No. 2001-001 authorizing the two-group system. Subsequently, it promulgated
Memorandum Circular No. 2001-010 accrediting PAMI and PAIC II as the two groups allowed to
participate in the program.

Indeed, in authorizing and regulating the two insurance monopolies, the LTFRB acted within its
prerogatives in promoting public interest and protecting the riding public. After all, the consortia
are open to all insurance companies, including petitioner. There is no discrimination against any
legitimate insurer. On the whole, the public is given protection without unfair competition or
undue restraint of trade. As the Court of Appeals pointed out, the two consortia are not engaged
in the insurance business; they merely serve as service arms of their respective members.

193
ARTICLE XIII SOCIAL JUSTICE AND HUMAN RIGHTS

Right To Strike

SSS Employees v. CA Issue: Whether SSS Employees have the right to strike?

Ruling: NO. A reading of the proceedings of the Constitutional Commission that drafted the 1987
Constitution would show that in recognizing the right of government employees to organize, the
commissioners intended to limit the right to the formation of unions or associations only, without
including the right to strike. SSS is one such GOCC with an original charter, its employees are
part of the civil service and are covered by the CSC's memorandum prohibiting strikes. This being
the case, the strike staged by the employees of the SSS was illegal.

Therefore, government employees may, therefore, through their unions or associations, either
petition the Congress for the betterment of the terms and conditions of employment which are
within the ambit of legislation or negotiate with the appropriate government agencies for the
improvement of those which are not fixed by law. If there be any unresolved grievances, the
dispute may be referred to the Public Sector Labor - Management Council for appropriate action.
But employees in the civil service may not resort to strikes, walk-outs and other temporary work
stoppages, like workers in the private sector, to pressure the Government to accede to their
demands.

Agrarian Reform

Association of Small Issue: Whether the CARP or CARL is unconstitutional on the ground that it violates the
Landowners v. constitutional provision that no private property shall be taken without due process or just
Secretary of Agrarian compensation?
Reform
Ruling: NO. The cases before us present no knotty complication insofar as the question of
compensable taking is concerned. To the extent that the measures under challenge merely
prescribe retention limits for landowners, there is an exercise of the police power for the
regulation of private property in accordance with the Constitution. But where, to carry out such
regulation, it becomes necessary to deprive such owners of whatever lands they may own in
excess of the maximum area allowed, there is definitely a taking under the power of eminent
domain for which payment of just compensation is imperative. The taking contemplated is not a
mere limitation of the use of the land. What is required is the surrender of the title to and the
physical possession of the said excess and all beneficial rights accruing to the owner in favor of
the farmer-beneficiary. This is definitely an exercise not of the police power but of the power of
eminent domain.

It bears repeating that the measures challenged in these petitions contemplate more than a mere
regulation of the use of private lands under the police power. We deal here with an actual taking
of private agricultural lands that has dispossessed the owners of their property and deprived them
of all its beneficial use and enjoyment, to entitle them to the just compensation mandated by the
Constitution. Where the State itself is the expropriator, it is not necessary for it to make a deposit
upon its taking possession of the condemned property, as "the compensation is a public charge,
the good faith of the public is pledged for its payment, and all the resources of taxation may be
employed in raising the amount."

With these assumptions, the Court hereby declares that the content and manner of the just
compensation provided for in the aforequoted Section 18 of the CARP Law is not violative of the
Constitution. We do not mind admitting that a certain degree of pragmatism has influenced our
decision on this issue, but after all this Court is not a cloistered institution removed from the
realities and demands of society or oblivious to the need for its enhancement. The Court is as
acutely anxious as the rest of our people to see the goal of agrarian reform achieved at last after
the frustrations and deprivations of our peasant masses during all these disappointing decades.
We are aware that invalidation of the said section will result in the nullification of the entire
program, killing the farmer's hopes even as they approach realization and resurrecting the
spectre of discontent and dissent in the restless countryside. That is not in our view the intention
of the Constitution, and that is not what we shall decree today.

The recognized rule, indeed, is that title to the property expropriated shall pass from the owner to
the expropriator only upon full payment of the just compensation. Title to property which is the
subject of condemnation proceedings does not vest the condemnor until the judgment fixing just
compensation is entered and paid, but the condemnor's title relates back to the date on which the
petition under the Eminent Domain Act, or the commissioner's report under the Local
Improvement Act, is filed.

It is true that P.D. No. 27 expressly ordered the emancipation of tenant-farmer as October 21,
1972 and declared that he shall "be deemed the owner" of a portion of land consisting of a
family-sized farm except that "no title to the land owned by him was to be actually issued to him
unless and until he had become a full-fledged member of a duly recognized farmers' cooperative."
It was understood, however, that full payment of the just compensation also had to be made
first, conformably to the constitutional requirement.

The CARP Law, for its part, conditions the transfer of possession and ownership of the land to the
government on receipt by the landowner of the corresponding payment or the deposit by the DAR
194
of the compensation in cash or LBP bonds with an accessible bank. Until then, title also remains
with the landowner. 57 No outright change of ownership is contemplated either. Hence, the
argument that the assailed measures violate due process by arbitrarily transferring title before
the land is fully paid for must also be rejected.

Maddumba v. GSIS Issue: Whether under Sec. 85 of RA 3844, the GSIS may be compelled to accept Land Bank
bonds at their face value in payment for a residential house and lot purchased by the bondholder
from GSIS?

Ruling: YES. The GSIS may be compelled to accept at face value land bank notes or receipts
payment of expropriated land under the land reform law because it will be unfair to compel the
land owner to discount such notes.

The nature of a Land Bank bond itself fortifies our view that the respondent may be compelled to
accept those bonds at their face value. True, the statute does not explicitly provide that Land
Bank bonds shall be accepted at their face value. There can be no question, however, that such is
the intendment of the law particularly in the absence of any provision expressly permitting
discounting, as differentiated from RA 304, or the Backpay Law, as amended by RAs 800 and
897, which expressly allows it.

Land Bank bonds are certificates of indebtedness, approved by the Monetary Board of the Central
Bank. They are fully negotiable and unconditionally guaranteed by the Government of the
Republic of the Philippines. These bonds are deemed contracts and the obligations within the
purview of the non-impairment clause of the Constitution. Thus, the value of these bonds cannot
be diminished by any direct or indirect act, particularly, since said bonds are fully guaranteed by
the Government of the Philippines.

Luz Farms v. Secretary Issue: Whether a law including the raising of livestock, poultry and swine in the coverage of
of Agrarian Reform agrarian reform law is unconstitutional?

Ruling: YES. The inclusion of land devoted to the raising of livestock, poultry and swine, in
agrarian reform law is void. The transcripts of the deliberations of the Constitutional Commission
of 1986 on the meaning of the word "agricultural," clearly show that it was never the intention of
the framers of the Constitution to include livestock and poultry industry in the coverage of the
constitutionally-mandated agrarian reform program of the Government.

The intention of the Committee is to limit the application of the word "agriculture." Commissioner
Jamir proposed to insert the word "ARABLE" to distinguish this kind of agricultural land from such
lands as commercial and industrial lands and residential properties because all of them fall under
the general classification of the word "agricultural". This proposal, however, was not considered
because the Committee contemplated that agricultural lands are limited to arable and suitable
agricultural lands and therefore, do not include commercial, industrial and residential lands.

It is evident from the foregoing discussion that Sec. 2 of RA 6657 which includes "private
agricultural lands devoted to commercial livestock, poultry and swine raising" in the definition of
"commercial farms" is invalid, to the extent that the aforecited agroindustrial activities are made
to be covered by the agrarian reform program of the State. There is simply no reason to include
livestock and poultry lands in the coverage of agrarian reform.

Land Bank of the Issue: What is the correct amount of provisional compensation which the LBP is required to
Philippines vs. Heir of deposit in the name of the landowner if the latter rejects the DAR/LBPs offer?
Trinidad S. Vda. De
Arieta Ruling: It should be the initial valuation of the land subject of Voluntary Offer to Sell (VOS). We
find the foregoing as a strained interpretation of a simple and clear enough provision on the
procedure governing acquisition of lands under CARP, whether under the compulsory acquisition
or VOS scheme. Indeed, it would make no sense to mention anything about the provisional
deposit in sub-paragraphs (a) and (b) the landowner is sent a notice of valuation to which he
should reply within a specified time, and in sub-paragraph (c) when the landowner accepts the
offer of the DAR/LBP as compensation for his land. Sub-paragraph (d) provides for the
consequence of the landowners rejection of the initial valuation of his land, that is, the conduct of
a summary administrative proceeding for a preliminary determination by the DARAB through the
PARAD or RARAD, during which the LBP, landowner and other interested parties are required to
submit evidence to aid the DARAB/RARAD/PARAD in the valuation of the subject land. Sub-
paragraph (e), on the other hand, states the precondition for the States taking of possession of
the landowners property and the cancellation of the landowners title, thus paving the way for the
eventual redistribution of the land to qualified beneficiaries: payment of the compensation (if the
landowner already accepts the offer of the DAR/LBP) or deposit of the provisional compensation
(if the landowner rejects or fails to respond to the offer of the DAR/LBP). Indeed, the CARP Law
conditions the transfer of possession and ownership of the land to the government on receipt by
the landowner of the corresponding payment or the deposit of the compensation in cash or LBP
bonds with an accessible bank.

It was thus erroneous for the CA to conclude that the provisional compensation required to be
deposited as provided in Section 16 (e) is the sum determined by the DARAB/PARAD/RARAD in a
summary administrative proceeding merely because the word deposit appeared for the first time
in the sub-paragraph immediately succeeding that sub-paragraph where the administrative
proceeding is mentioned (sub-paragraph d). On the contrary, sub-paragraph (e) should be
related to sub-paragraphs (a), (b) and (c) considering that the taking of possession by the State
of the private agricultural land placed under the CARP is the next step after the DAR/LBP has
complied with notice requirements which include the offer of just compensation based on the
initial valuation by LBP. To construe sub-paragraph (e) as the appellate court did would hamper
the land redistribution process because the government still has to wait for the termination of the
195
summary administrative proceeding before it can take possession of the lands. Contrary to the
CAs view, the deposit of provisional compensation is made even before the summary
administrative proceeding commences, or at least simultaneously with it, once the landowner
rejects the initial valuation (offer) by the LBP. Such deposit results from his rejection of the DAR
offer (based on the LBPs initial valuation). Both the conduct of summary administrative
proceeding and deposit of provisional compensation follow as a consequence of the landowners
rejection under both the compulsory acquisition and VOS. This explains why the words rejection
or failure to reply and rejection or no response from the landowner are found in sub-paragraphs
(d) and (e). Such rejection/no response from the landowner could not possibly refer to the award
of just compensation in the summary administrative proceeding considering that the succeeding
sub-paragraph (f) states that the landowner who disagrees with the same is granted the right to
petition in court for final determination of just compensation. As it is, the CAs interpretation
would have loosely interchanged the terms rejected the offer and disagrees with the decision,
which is far from what the entire provision plainly conveys.

SECTION 11

Health

City of General Santos Issue: Whether the grant and release of the hospitalization and health care insurance benefits
v. COA given to General Santos City officials and employees are valid?

Ruling: YES. Consistent with the constitutional mandate for a comprehensive approach to health
development, with priority for the needs of the sick, the cash gift for the sickly employees,
lifetime free medical consultation in petitioner’s city hospital, and other similar benefits under
Sec. 6 of the ordinance are valid.

The text of the ordinance indicates its purpose of encouraging employees, especially those who
are unproductive due to health reasons, to avail of the program even before they reach the
compulsory retirement age. It provides for a form of severance pay to those who availed of
GenSan SERVES, which was executed in good faith.

The benefits provided in Section 6 serve its purpose of inducting petitioner city’s employees, who
are unproductive due to health reasons, to retire early. Furthermore, the benefits under GenSan
SERVES were only given to a select few – the sickly and unproductive due to health reasons.
Certainly, it negates the position that the benefits provide for supplementary retirement benefits
that augment existing retirement laws.

Petitioner city is authorized by the LGC to approve ordinances to provide for the care of the sick.
This is also consistent with the constitutional mandate for a comprehensive approach to health
development, with priority for the needs of the sick.

SECTION 17

Commission On Human Rights

Cariño v. CHR Issue: Whether the CHR has the power try and decide human rights cases?

Ruling: NO. The CHR cannot try and decide or hear and determine cases as courts of justice, or
even quasi-judicial bodies do. To investigate is not to adjudicate or adjudge.

The most that may be conceded to the Commission in the way of adjudicative power is that it
may investigate, i.e., receive evidence and make findings of fact as regards claimed human rights
violations involving civil and political rights. But fact finding is not adjudication, and cannot be
likened to the judicial function of a court of justice, or even a quasi-judicial agency or official. The
function of receiving evidence and ascertaining therefrom the facts of a controversy is not a
judicial function, properly speaking. To be considered such, the faculty of receiving evidence and
making factual conclusions in a controversy must be accompanied by the authority of applying
the law to those factual conclusions to the end that the controversy may be decided or
determined authoritatively, finally and definitively, subject to such appeals or modes of review as
may be provided by law. This function, to repeat, the Commission does not have.

Thus, CHR, having merely the power to “investigate” cannot and should not “try and resolve on
the merits” the matters involved in Striking Teachers HRC Case No. 90-775, as it has announced
it means to do; and it cannot do so even if there be a claim that in the administrative disciplinary
proceedings against the teachers in question, initiated and conducted by the DECS, their human
rights, or civil or political rights had been transgressed. These are matters clearly within the
original jurisdiction of the Secretary of Education, being within the scope of the disciplinary
powers granted to him under the Civil Service Law, and also, within the appellate jurisdiction of
the Civil Service Commission.

To “investigate” is to inquire or examine; To “adjudicate” is to rule, judge, decide, determine or


resolve.

Export Processing Issue: Whether CHR has jurisdiction to issue a writ of injunction or restraining order against
196
Zone Authority v. CHR supposed violators of human rights, to compel them to cease and desist from continuing the acts
complained of?

Ruling: NO. CHR is not a court of justice nor even a quasi- judicial body. The constitutional
provision directing the CHR to "provide for preventive measures and legal aid services to the
underprivileged whose human rights have been violated or need protection" may not be
construed to confer jurisdiction on the Commission to issue a restraining order or writ of
injunction for, if that were the intention, the Constitution would have expressly said so.

Evidently, the "preventive measures and legal aid services" mentioned in the Constitution refer to
extrajudicial and judicial remedies including a preliminary writ of injunction which the CHR may
seek from the proper courts on behalf of the victims of human rights violations. CHR is not a
court of justice nor even a quasi-judicial body. Not being a court of justice, the CHR itself has no
jurisdiction to issue the writ, for a writ of preliminary injunction may only be issued "by the judge
of any court in which the action is pending within his district, or by a Justice of the Court of
Appeals, or of the Supreme Court. It may also be granted by the judge of RTC in any action
pending in an inferior court within his district.”

Hence, the orders of injunction issued by CHR against EPZA, the 125th PNP Company and
Governor Remulla and their subordinates are hereby annulled and set aside.

Simon, Jr. v. CHR Issue: Whether the CHR has jurisdiction to investigate the alleged violations of the “business
rights” of the private respondents?

Ruling: NO. Art. 13, Sec. 18 empowers the CHR to "investigate, on its own or on complaint by
any party, all forms of human rights violations involving civil and political rights.”

Civil Rights are those rights that belong to every citizen of the state or country, or, in a wider
sense, to all its inhabitants, and are not connected with the organization or administration of
government.

The deliberations of the Constitutional Commission is readily apparent that the delegates
envisioned a CHR that would focus its attention to the more severe cases of human rights
violations. Delegate Garcia, for instance, mentioned such areas as the “(1) protection of rights of
political detainees, (2) treatment of prisoners and the prevention of tortures, (3) fair and public
trials, (4) cases of disappearances, (5) salvagings and hamletting, and (6) other crimes
committed against the religious.”

While the enumeration has not likely been meant to have any preclusive effect, more than just
expressing a statement of priority, it is, nonetheless, significant for the tone it has set. In any
event, the delegates did not apparently take comfort in peremptorily making a conclusive
delineation of the CHR’s scope of investigatorial jurisdiction. They have thus seen it fit to resolve,
instead, that “Congress may provide for other cases of violations of human rights that should fall
within the authority of the Commission, taking into account its recommendation.”

Here, there is no cavil that what are sought to be demolished are the stalls, sari-sari stores and
carinderia, as well as temporary shanties, erected by private respondents on a land which is
planned to be developed into a "People's Park". More than that, the land adjoins the North EDSA
of Quezon City which, this Court can take judicial notice of, is a busy national highway. The
consequent danger to life and limb is not thus to be likewise simply ignored. It is indeed
paradoxical that a right which is claimed to have been violated is one that cannot, in the first
place, even be invoked, if it is, in fact, extant. Be that as it may, looking at the standards
hereinabove discoursed vis-a-vis the circumstances obtaining in this instance, we are not
prepared to conclude that the order for the demolition of the stalls, sari-sari stores and carinderia
of the private respondents can fall within the compartment of "human rights violations involving
civil and political rights" intended by the Constitution.

On its contempt powers, the CHR is constitutionally authorized to "adopt its operational guidelines
and rules of procedure, and cite for contempt for violations thereof in accordance with the Rules
of Court." Accordingly, the CHR acted within its authority in providing in its revised rules, its
power "to cite or hold any person in direct or indirect contempt, and to impose the appropriate
penalties in accordance with the procedure and sanctions provided for in the Rules of Court." That
power to cite for contempt, however, should be understood to apply only to violations of its
adopted operational guidelines and rules of procedure essential to carry out its investigatorial
powers. To exemplify, the power to cite for contempt could be exercised against persons who
refuse to cooperate with the said body, or who unduly withhold relevant information, or who
decline to honor summons, and the like, in pursuing its investigative work. The "order to desist"
in the instance before us, however, is not investigatorial in character but prescinds from an
adjudicative power that it does not possess.

ARTICLE XIV EDUCATION, SCIENCE AND TECHNOLOGY, ARTS, CULTURE, AND SPORTS

SECTION 1

Quality And Accessibility Of Educational System

197
Department of Issue: Whether a person who has thrice failed the National Medical Admission Test is entitled to
Education, Culture and take it again based on his constitutional right to academic freedom and quality education?
Sports vs. San Diego
Ruling: NO. The subject of the challenged regulation is certainly within the ambit of the police
power. It is the right and responsibility of the State to insure that the medical profession is not
infiltrated by incompetents to whom patients may unwarily entrust their lives and health. The
method employed by the challenged regulation is not irrelevant to the purpose of the law nor is it
arbitrary or oppressive. The threeflunk rule is intended to insulate the medical schools and
ultimately the medical profession from the intrusion of those not qualified to be doctors.

While every person is entitled to aspire to be a doctor, he does not have a constitutional right to
be a doctor. This is true of any other calling in which the public interest is involved; and the closer
the link, the longer the bridge to one's ambition. The State has the responsibility to harness its
human resources and to see to it that they are not dissipated or, no less worse, not used at all.
These resources must be applied in a manner that will best promote the common good while also
giving the individual a sense of satisfaction.

The right to quality education invoked by the private respondent is not absolute. The Constitution
also provides that "every citizen has the right to choose a profession or course of study, subject
to fair, reasonable and equitable admission and academic requirements.” The Court feels that it is
not enough to simply invoke the right to quality education as a guarantee of the Constitution: one
must show that he is entitled to it because of his preparation and promise. The private
respondent has failed the NMAT 5 times. No depreciation is intended or made against the private
respondent. It is stressed that a person who does not qualify in the NMAT is not an absolute
incompetent unfit for any work or occupation. The only inference is that he is a probably better,
not for the medical profession, but for another calling that has not excited his interest.

SECTION 4

Academic Freedom Of “Institutions Of Higher Learning”

Board of Medical Issue: Whether the closure of the respondent College is valid?
Education v. Judge
Alfonso and Philippine Ruling: YES. The Secretary of Education, Culture and Sports exercises the power to enjoin
Muslim-Christian compliance with the requirements laid down for medical schools and to mete out sanctions where
College of Medicine he finds that violations thereof have been committed, it was a grave abuse of discretion for the
respondent judge to issue the questioned injunction and thereby thwart official action, in the
premises correctly taken, allowing the College to operate without the requisite government
permit. A single ocular inspection, done after the College had been pre-warned thereof, did not, in
the circumstances, warrant only the findings of more qualified inspectors about the true state of
the College, its faculty, facilities, operations, etc. The, members of the evaluating team came
from the different sectors in the fields of education and medicine, and their judgment in this
particular area is certainly better than that of the respondent Judge whose sole and only visit to
the school could hardly have given him much more to go on than a brief look at the physical plant
and facilities and into the conduct of the classes and other school activities. Respondent Judge
gravely abused his discretion in substituting his judgment for theirs.

Here, the recorded facts quite clearly fail to support the College’s claim of grave abuse of
discretion tainting the order of closure, and on the contrary convincingly show the challenged
decision to be correct. From 1985, no less than 5 surveys were conducted of respondent
institution to determine its compliance with the minimum standards established for a medical
college. The first survey, that undertaken by the Commission on Medical Education, disclosed
such various and significant deficiencies in the school as to constrain the inspectors to
recommend its closure. 4 other surveys were thereafter made by as many different committees
or teams, at the school’s instance or otherwise, all of which basically confirmed the results of that
first survey. Moreover, the findings of all 5 surveys were affirmed by the Office of the President.

It is well-settled doctrine that courts of justice should not generally interfere with purely
administrative and discretionary functions; that courts have no supervisory power over the
proceedings and actions of the administrative departments of the government; involving the
exercise of judgment and findings of facts, because by reason of their special knowledge and
expertise over matters falling under their jurisdiction, the latter are in a better position to pass
judgment on such matters and their findings of facts in that regard are generally accorded
respect, if not finality, by the courts. There are, to be sure, exceptions to this general rule but
none of them obtains in this case.

The claim of denial of due process likewise holds no water, as the record clearly shows that the
College was given every opportunity to so improve itself as to come up to requirements, but
remained sadly sub-standard after the inspections conducted by the evaluating teams. It had, in
fact, admitted its failure to have up to the desired standards when it proposed its gradual phase-
out in its letter dated June 27, 1988 to Secretary Quisumbing. It was also precisely because of its
complaints of bias and prejudice that the Board of Medical Education dispatched new teams to
survey and reevaluate its performance. It had even gone all the way up to the Office of the
President to seek a reversal of the order of closure. There is thus no reason for it to complain of a
lack of opportunity to be heard and to explain its side as well as to seek reconsideration of the
ruling complained of.

Garcia v. Faculty Issue: Whether a mandamus proceeding filed by Garcia against the Faculty Admission
Admission Committee Committee violates the latter’s academic freedom?
198
— MA in Theology
Ruling: YES. There is the recognition in the Constitution of institutions of higher learning
enjoying academic freedom. It is more often identified with the right of a faculty member to
pursue his studies in his particular specialty and thereafter to make known or publish the result of
his endeavors without fear that retribution would be visited on him in the event that his
conclusions are found distasteful or objectionable to the powers that be, whether in the political,
economic, or academic establishments.

Academic freedom is the freedom of professionally qualified persons to inquire, discover, publish
and teach the truth as they see it in the field of their competence. It is subject to no control or
authority except the control or authority of the rational methods by which truths or conclusions
are sought and established in these disciplines.

The internal conditions for academic freedom in a university are that the academic staff should
have de facto control of the following functions: (i) the admission and examination of students;
(ii) the curricula for courses of study; (iii) the appointment and tenure of office of academic staff;
and (iv) the allocation of income among the different categories of expenditure. It would be a
poor prospect for academic freedom if universities had to rely on the literal interpretation of their
constitutions in order to acquire for their academic members control of these four functions, for in
one constitution or another most of these functions are laid on the shoulders of the law governing
body.

It is not an easy matter then to disregard the views of persons knowledgeable in the field, to
whom cannot be imputed lack of awareness of the need to respect freedom of thought on the part
of students and scholars. Moreover, it could amount to minimizing the full respect that must be
accorded the academic freedom expressly granted by the Constitution "to institutions of higher
learning." It is equally difficult to yield conformity to the approach taken that colleges and
universities should be looked upon as public utilities devoid of any discretion as to whom to admit
or reject. Education, especially higher education, belongs to a different, and certainly higher,
category. It only remains to be added that the futility that marked the persistence of petitioner to
continue her studies in the Loyola School of Theology is the result solely of a legal appraisal of the
situation before us. The decision is not to be construed as in any way reflecting on the scholastic
standing of petitioner. There was on the part of respondent due acknowledgment of her
intelligence. Nonetheless, for reasons explained in the letter of Father Lambino, it was deemed
best, considering the interest of the school as well as of the other students and her own welfare,
that she continue her graduate work elsewhere.

Lupangco v. CA — CPA Issue: Whether a resolution issued by the PRC which prohibits reviewees from review classes 3
Boards days before the board examination is valid?

Ruling: NO. Resolution No. 105 is not only unreasonable and arbitrary, it also infringes on the
examinees' right to liberty guaranteed by the Constitution. Respondent PRC has no authority to
dictate on the reviewees as to how they should prepare themselves for the licensure
examinations. They cannot be restrained from taking all the lawful steps needed to assure the
fulfillment of their ambition to become public accountants. They have every right to make use of
their faculties in attaining success in their endeavors. They should be allowed to enjoy their
freedom to acquire useful knowledge that will promote their personal growth.

Another evident objection to Resolution No. 105 is that it violates the academic freedom of the
schools concerned. Respondent PRC cannot interfere with the conduct of review that review
schools and centers believe would best enable their enrollees to meet the standards required
before becoming a full-fledged public accountant. Unless the means or methods of instruction are
clearly found to be inefficient, impractical, or riddled with corruption, review schools and centers
may not be stopped from helping out their students.

University of San Issue: Whether University of San Carlos may be compelled to grant graduation honors to the
Carlos v. CA — petitioner?
Architecture to
Commerce, with failing Ruling: NO. It is an accepted principle that schools of teaming are given ample discretion to
marks formulate rules and guidelines in the granting of honors for purposes of graduation. This is part of
academic freedom. Within the parameters of these rules, it is within the competence of
universities and colleges to determine who are entitled to the grant of honors among the
graduating students. Its discretion on this academic matter may not be disturbed much less
controlled by the courts unless there is grave abuse of discretion in its exercise.

Here, the petitioner's bulletin of information provides all students and all other interested parties
advise on the University policies and rules on enrollment and academic achievements. Therein it
is provided, among others, that a student may not officially withdraw from subjects in the
curriculum if he does not have the written permission of his parents or guardian. For an
incomplete grade, there must be an application for completion or removal within the period
announced by the school calendar and when not removed within one (1) year, it automatically
becomes final. 16 A "DR" (Dropped) subject which is in the same category, as a "5" disqualifies a
student from receiving honors. A candidate for honors should have earned no less than 18 units
per semester but a working student should earn no less that 12 units. A failure in any subject
disqualifies a student from honors. Good moral character and exemplary conduct are as important
criteria for honors as academic achievements.

Here, Private respondent have 2 failing marks and one Incomplete grade when she was still in
the College of Architecture. Hence, she is disqualified to graduate with honors.

Reyes v. CA — UP Med Issue: Whether the act of BOR in directing the petitioner to admit the students in UP College of

199
70 to 90 cut-off by the Medicine is violative of the petitioner’s academic freedom?
Faculty
Ruling: NO. The individual faculty member has the freedom to pursue his studies in his particular
specialty and thereafter to make known or publish the result of his endeavors without fear that
retribution would be visited on him in the event that his conclusions are found distasteful or
objectionable to the powers that be, whether in the political, economic, or academic
establishments. In contrast, the University has the academic freedom to determine for itself on
academic grounds who may teach, what may be taught, how it shall be taught, and who may be
admitted to study. As a corporate body, the University has entrusted to its academic staff the de
facto control of its function of admission and examination of students.

Petitioners now claim to be in charge of that function with respect to fixing the admission
requirements in the college. We disagree. Under the UP Charter, the power to fix admission
requirements is vested in the University Council of the autonomous campus which is composed of
the President of the University of the Philippines and of all instructors holding the rank of
professor, associate professor or assistant professor. Consequently, the UC alone has the right to
protest against any unauthorized exercise of its power. Petitioners cannot impugn these BOR
directives on the ground of academic freedom inasmuch as their rights as university teachers
remain unaffected.

Academic freedom may be asserted by the University Council or by the Board of Regents or both
in so far as it relates to the functions vested in them by law which are essential to institutional
academic freedom. The academic freedom claimed by the faculty to have been violated by the
Board of Regents when it issued the questioned order is related to the right of the University to
fix admission requirements. This right and power to fix admission requirements is clearly vested
by law in the University Council. The College Faculty was merely empowered by the Board of
Regents under Art. 324 of the University Code to initially determine the admission requirements,
subject to the approval of the University Council and the President of the University. The
questioned order of the Board of Regents in upholding the admission requirement approved by
the University Council in 1986 is supportive of right of the University Council to fix or approve
admission requirements, against the UPCM faculty and Dean who changed the admission
requirements approved by the University Council without following the prescribed rules and
procedures of the University.

Hence, the UP Faculty cannot raise the cut-off score to 90% since the UC provides that the cut off
score is 70%.

Miriam College v. CA Issue: Whether Sec. 7 of RA 7079 which provides that a student shall not be expelled or
suspended solely on the basis of articles he or she has written precludes Mirriam College of its
right to discipline its students?

Ruling: NO. Miriam College Discipline Board has jurisdiction to hear and decide cases filed
against its students. Sec. 5 (2), Art. 14 of the Constitution guarantees all institutions of higher
learning academic freedom. This institutional academic freedom includes the right of the school or
college to decide for itself, its aims and objectives, and how best to attain them free from outside
coercion or interference save possibly when the overriding public welfare calls for some restraint.
The essential freedoms subsumed in the term "academic freedom" encompasses the freedom to
determine for itself on academic grounds: (1) Who may teach; (2) What may be taught; (3) How
it shall be taught; and (4) Who may be admitted to study. The right of the school to discipline its
students is at once apparent in the third freedom, i.e., "how it shall be taught." A school certainly
cannot function in an atmosphere of anarchy.

Thus, there can be no doubt that the establishment of an educational institution requires rules
and regulations necessary for the maintenance of an orderly educational program and the
creation of an educational environment conducive to learning. Such rules and regulations are
equally necessary for the protection of the students, faculty, and property. Moreover, the school
has an interest in teaching the student discipline, a necessary, if not indispensable, value in any
field of learning. By instilling discipline, the school teaches discipline. Accordingly, the right to
discipline the student likewise finds basis in the freedom "what to teach.”

The right of students to free speech in school is not always absolute. The court upheld the right of
students for the freedom of expression but it does not rule out disciplinary actions of the school
on the conduct of their students. Further, the of the Campus Journalism Act provides that the
school cannot suspend or expel a student solely on the basis of the articles they write EXCEPT
when such article materially disrupts class work involving substantial disorder or invasion of the
rights of others.

The power of the school to investigate is an adjunct of its power to suspend or expel. It is a
necessary corollary to the enforcement of rules and regulations and the maintenance of a safe
and orderly educational environment conducive to learning. That power, like the power to
suspend or expel, is an inherent part of the academic freedom of institutions of higher learning
guaranteed by the Constitution. The court held that Miriam College has the authority to hear and
decide the cases filed against respondent students.

Cudia v. Issue: Whether PMA enjoys the academic freedom which authorizes it to impose disciplinary
Superintendent of the measures and punishment as it deems fit and consistent with the peculiar needs of the academy?
PMA — Late Dismissal
Ruling: YES. To determine for itself on academic grounds (1) who may teach, (2) what may be
taught, (3) how it shall be taught, and (4) who may be admitted to study. An educational
institution has the power to adopt and enforce such rules as may be deemed expedient for its
government, this being incident to the very object of incorporation, and indispensable to the
successful management of the college. It can decide for itself its aims and objectives and how
200
best to attain them, free from outside coercion or interference except when there is an overriding
public welfare which would call for some restraint.

The schools' power to instill discipline in their students is subsumed in their academic freedom
and that "the establishment of rules governing university-student relations, particularly those
pertaining to student discipline, may be regarded as vital, not merely to the smooth and efficient
operation of the institution, but to its very survival.

Every citizen has a right to select a profession or, course of study, subject to fair, reasonable, and
equitable admission and academic requirements. The PMA is not different. As the primary training
and educational institution of the AFP, it certainly has the right to invoke academic freedom in the
enforcement of its internal rules and regulations, which are the Honor Code and the Honor
System in particular.

SECTION 15-14

Historical And Cultural Heritage

Knights of Rizal v. Issue: Whether the construction of Torre de Manila is unconstitutional?


DMCI — Torre De Manila
Construction Ruling: NO. There is no law prohibiting the construction of Torre de Manila due to its effect on
the background view, vista, sightline, or setting of the Rizal Monument.

Sec. 15, Art. 14 of the Constitution, which deals with the subject of arts and culture, provides
that the State shall conserve, promote, and popularize the nation’s historical and cultural heritage
and resources. Since this provision is NOT self-executory, the Congress passed laws dealing with
the preservation and conversation of our cultural heritage.

RA No. 10066, or the National Cultural Heritage Act of 2009, which empowers the National
Commission for Culture and Arts and other cultural agencies to issue a cease and desist order
when the physical integrity of the national and cultural treasures or important cultural
properties is found to be in danger of destruction or significant alteration from its original
state. This law declares that the State should protect the “physical integrity” of the heritage
property or building if there is danger of destruction or significant alteration from its original
state. Physical integrity refers to the structure itself – how strong and sound the
structure is. The same law does not mention that another project, building, or property, not
itself a heritage property or building, may be the subject of a cease and desist order when it
adversely affects the background view, vista, or sightline of a heritage property or building. Thus,
RA No. 10066 cannot apply to Torre de Manila condominium project.

Also, Torre de Manila is not a nuisance per se because it cannot be considered as a “direct
menace to public health or safety.” Not only is a condominium project commonplace in the City of
Manila, DMCI-PDI has, according to the proper government agencies, complied with health and
safety standards set by law.

201

You might also like